Você está na página 1de 549

Antonio Caminha Muniz Neto

FU DAMENTOS
DECÁLCULO

Sociedade B asile1ra de Matemática


Antonio Caminha Muniz Neto

FUNDAMENTOS
DECÁLCULO

1• edição
2015
Rio de Jan Iro
(2• impres o)

Sociedade Brasileira de Matemática


Prefácio XI

De que Trata o Câlculo? 1

1 Funções 13
1.1 Definições e exemplos . 14
1.2 Monotonicidade, extremos e imagem 25
1.3 Composição de funções 35
1.4 Inversão de funções . . . 44
1.5 Gráficos de funções ... 48
1.6 Funções trigonométricas 60

2 Sequências e continuidade 67
2.1 Supremo e ínfimo .... 68
2.2 Limites de sequências .. 76
2.3 O conceito de continuidade . 92
2.4 Continuidade sequencial . . 103
2.5 O teorema do valor intermediário 112

3 Limites e Derivadas 121


3.1 Limites de funções . . . . . . . . 122
3.2 Propriedades básicas de derivadas 139
3.3 Regras de derivação . . . . . . . . 150
3.4 O teorema de Rõlle e aplicações . 161
3.5 A primeira variação de uma função 16
3.6 A segunda variação de uma função 179
3.7 Construindo gráficos . . . . 192
3.8 Algumas aplicações à Física 201

4 A Integral de Riemann 213


4.1 O conceito de integral 214
4.2 O Teorema de Riemann e algumas observações . 224
4.3 Operações com funções integráveis . . . . 230

VII
"1 ' 1 \ 1; 1e ,

4.4 O teorema fundamental do Câlculo 245


4.5 Algumas aplicações à Geometria . 262
4.6 Logaritmos e exponenciais .... 276
4.7 'f\1aissobre técnicas de integração 289
4.8 Integração imprópria . . 301
4.9 Mais aplicações à Física . . . 312

5 Séries Numéricas e de Funções 325


5.1 Séries de números reais 326
5.2 Algumas aplicações 341
5.3 Séries de Taylor . . 350
5.4 Séries de funções 359
5.5 Séries de potências 369
5.6 Mais aplicações .. 381

A Alguns Pré-Requisitos 393


A .1 Números reais . . . . 394
A.1.1 Aritmética em 1R . . . . . 395
A .1.2 A relação de ordem em 1R 401
A.1.3 A completude do conjunto dos reais . 406
A.1.4 A representação geométrica 409
A.2 Álgebra elementar . . . . . . . . . . . 413
A.2.1 Identidades algébricas .... 413
A.2.2 Módulo e equações modulares 418
A.2.3 A desigualdade triangular 421
A.2.4 Equações polinomiais . . . 424
A.3 Sequências e indução ...... . 429
A.3.1 Recorrências elementares . 429
A.3.2 Somatórios e produtórios . 435
A.3.3 Indução finita . . . . . . . 440
A.3.4 A fórmula do binômio . . 448
A.4 Geometria Analítica e Trigonometria 454
A.4.1 O plano cartesiano . . . . 454
A.4.2 Retas no plano cartesiano 460
A.4.3 Seções cônicas . 467
A.4.4 Trigonometria . -!75

VIII
B Sugestões aos Problemas

Bibliografia 551

Índice Remissivo 553

IX
Os próximos capítulos apresentarão, em detalhe, os temas centrais concernentes a uma das
maiores sínteses intelectuais da história da civilização, o Cálculo Diferencial e Integral.
Como frequentemente ocorre com as grandes ideias científicas, a cronologia do desenvolvi-
mento do Cálculo revelou-se completamente errática, enredando-se na ordem exatamente con-
trária à que utilizaremos em nossa exposição. Após um incrível lampejo de genialidade de um
dos maiores sábios da antiguidade clássica grega, foi necessário esperar cerca de 20 séculos até
que dois outros luminares da ciência dessem o próximo passo; a este, por sua vez, seguiu-se um
século e meio de intensas pesquisas e maturação, após o qual o Cálculo pôde, finalmente, ser
colocado em bases sólidas.
Mas, de que trata o Cálculo Diferencial e Integral? esta breve introdução, tentamos res-
ponder a essa pergunta apresentando, por meio de argumentos heurísticos, os dois conceitos
que o nomeiam, quais sejam, a integral e a derivada de uma função. Em seguida, criticamos
brevemente nossas abordagens de tais conceitos, enfatizando como as deficiências que elas en-
cerram apontam para a necessidade de uma melhor compreensão do conceito de número real e
da introdução do conceito de função contínua.
Nossa história começa no século III a.C., com o grande Arquimedes de Siracusa (cf. Figura 1).
Por essa época, Euclides de Alexandria já havia sistematizado toda a geometria conhecida (à qual

Figura 1: Arquimedes de Siracusa, que viveu no século III a.C., foi o


maior matemático de seu tempo. Dentre outras contribuições à Mate-
mática, suas ideias sobre o cálculo da área sob um segmento parabólico
anteciparam, em 2000 anos, o desenvolvimentodo Cálculo Integral.

nos referimos modernamente como Euclidiana) em sua magistral obra Elementos. Arquimedes,
então, considerou (e resolveu) o problema de calcular a área sob um arco de parábola, utilizando
para tanto o método da exaustão.

1
Em linguagem atual e em uma situação ligeiramente mais geral, a heurística por trás de tal
método é a seguinte: dada uma função não negativa J : [a, b]---+IR (figura 2), queremos calcular
a área da região n do plano cartesiano, situada sob o gráfico de f e acima do ei.xo das abscissa ,
de sorte que
'R, = {(x, y) E IR2 ; a< X< b e Ü < y < J(x)}.

a b X

Figura 2: a região sob o gráfico de f.

Para tanto, dividimos o intervalo [a, b] em k intervalo iguais com o auxílio dos ponlos a =
t 0 < t1 < · · · < tk = b, de tal modo que ti - t,_ 1 = b·t, pnrn 1 < t $ k. Em seguida (Figura 3),

consideramos a porção de n contida na fai.xa vertical delimitada pelas reta x = t, 1 e :r = l" e


aproximamos sua área, por falta, pela área do mnior retângulo nela contido e tendo o intervalo
[ti-l, t,] como um de seus lados; aproximamo n área da porção em questão também por excesso,
pela área do menor retângulo que a contém e que também tem o intervalo [t,_1, t,] como um de
seus lados.

Figura 3: aproximações por falta e por excesso para a área .

.
Agora, aproximamos a área de n por falta e por excesso, calculando, em cada caso, as sornas
das áreas dos k retângulos descritos no parágrafo anterior e que aproximam a área de cada
porção de 'R, também por falta e por excesso, respectivamente. Assumindo que J atinge valores
tnfnimo e máximo em cada intervalo [t,_11 t,], e denotando (também respectivamente) por m, e
M, tais valores mínimo e máximo (de modo que m 1 e J\l, sejam os comprimentos das alturas dos

2
retângulos considerados anteriormente), obtemos como resultado as somas

(1)

Sendo A(R) a área de


A(!: k) =

n, temos, então,
k

~ AI,(t, -
que
t,_ 1) = ea)~
k
k

M,. (2)

A(f; k) < A('R) < A(f; k),

e esperamos que A(J; k) e A(f; k) aproximem A(R) cada vez melhor, à medida que aumentamos
mais e mais o número k de intervalos utilizados.
Conforme frisamos anteriormente, Arquimedes levou tal procedimento a cabo em um caso
bem particular. qual seja, aquele em que f é a porção da parábola xi--+ x 2 situada entre a= O
e b > O.
A fim de ilustrar as dificuldades em·olvidas, consideremos a situação mais geral do emprego
do método da exaustão para o cálculo da área de n quando f : [O,b] ➔ IR é a função definida
por f(x) = xn, onde n é um natural dado. Como antes, sendo k EN e O= t 0 < t 1 < • •• < tk = b
a partição de [O,b] tal que i 1 - t,_ 1 = ~ para 1 < i ~ k, temos t 1 = ~ para O < i < k. Também,
como xi--+ xn é crescente, segue que m, = f (t,-1) = t~_1 e A,f. = /(t 1) = t~. Assim,

b k 11 b I.· (bi)n b)n+l k


A(J,k) =
kL-• "'t 1 --kL-
- ·"' k (k Lin (3)
1 1 1 1 t=l

e, analogamente,
n+l k

A(!; k) = ( ~) ~(i - !)" (4)

Segue facilmente daí que


A(f; k) (5)

portanto, a diferença entre A(f; k) e A(J; k) é cada vez menor à medida que k aumenta e, pelo
menos nesse caso, concluímos que A(J; k) e A(f; k) aproximam A(R) cada vez melhor, à medida
que k aumenta.
Para calcular o valor de A(R), utilizemos o resultado do item (b) do exemplo 31, o qual
garante que
k (k l)n+l k
"'(i- ir< - < I:in.
Li=l n+l .
1-l

3
Combinando tais desigualdades com (3) e (4), obtemos

bn+l ( 1 )n+l
À(J; k) < n+
1
1- k < A(J;k). (6)

Então, fazendo k aumentar mais e mais em (6) e levando em conta que 1 - ¼aproxima-se cada
vez mais de 1 e (cf. (5)) A(J; k) - A(f; k) fica cada vez mais próximo de O à medida que k
aumenta, concluímos que
bn+l
A(n) = n + 1.
Conforme antecipamos no início desta introdução, o passo seguinte no desenvolvimento do
Cálculo teve de esperar cerca de 2000 anos, mais precisamente até o final do século XVII, para
ser dado. Isso não é de estranhar, se levarmos em conta a complexidade dos cálculos e notaçõc
que utilizamos até aqui, juntamente com o fato de que eles não estavam disponíveis a ninguém
nesse interregno de dois milênios, simplesmente porque não existiam. De fato, o hiato entre o
tempo de Arquimedes e o final do século XVII foi o tempo necessário para. que a civilização
desenvolvesse as notações e ferramentas algébricas e geométricas que possibilitaram os trabalhos
seminais de nossas próximas duas personagens, su Isaac . cwt?n e Gottfned \V1lhem Leibmz.

Figura 4: Gottfried \Vilhem Leibniz, matemãtico e filósofoalemão do


século XVII, é considerado, juntamente com sir Isaac ewton, um dos
criadores do Cálculo Diferenciale Integral. Algumas das notações que
utilizamos até hoje no Cálculo foram criadas jã por Leibniz, tendo resis-
tido ao implacável teste do tempo.

A fim de apreciar o grande passo adicional dado por ewton e Leibniz, consideremos o
problema de encontrar a reta tangente ao gráfico de uma função f : (a, b) ➔ R em um ponto
A(xa, /(x 0)) sobre tal gráfico (de modo que xo E (a, b)). Para tanto, tomando x 1 E (a, b) \ {xo}
-.+
e sendo B 1(x1, J(xi}), dizemos que a reta AB1 é uma secante ao gráfico de f passando por A.
-.+ t--+
Na Figura 6, consideramos ru; secantes AB1 e AB 2 ao gráfico de J.

4
Figura 5: O matemático e físico inglês Isaac ewton é considerado um
dos maiores cientistas que a humanidade já conheceu, sendo difícil men-
surar sua contribuição para o desenvolvimento da ciência. Considerado
o pai da Física moderna. ewton criou, juntamente com G. W. Leib-
niz, os fundamentos do Cálculo Diferencial e Integral, pedra angular do
desenvolvimento científico e tecnológico vivenciado desde sua época, no
final do século XVII, até os dias de hoje. Sua obra-prima, Philosophiae
Naturali.s Pnnctpta Mathcmattc.a (em português, Os Princípios Matemá-
ticos da Ftlosofia Natural - à época de • ewton, a Física era conhecida
como Filo:,ofia Natural). é considerada por muitos o livro que mais influ-
enciou o de.,envolvimcnto da ci\'ilizac;ão ocidental, por conter os alicerces
do Cálculo e da Fbica modcrnl\..,.

+--+
Um pouco de intuição gcométricn tornn rnzoA\'el supor que uma secante genérica AB deve
aproximar-se cada vez mai da tangente ao gráfico de f em A, à medida que B se aproxima de
,..\ao longo do gráfico (ou, o que é o mesmo, à medida que x se aproxima de x 0 em (a, b)).
+--+
Ob~crvc que. se .r.1 E (a, b) \ {.ro} e B(i 1, J(xi)). a secante AB ao gráfico de f é a reta de
equação
y- J(.ro) = (!(xi) - J(xo)) (x - xo); (7)
X1 - Xo

por outro lndo, a reta tangente ao gráfico de f em A, se não for vertical, deve ter equação da
fo1ma
Y - f (.,,,ui= m(x - xo), (8)
para algum m E JR.
Comparando (7) e ( ) e tendo em conta a discussão do parágrafo anterior, somos levados
a concluir que os quocientes /(zi)-/(:ro)
:r1 -zo
devem aproximar m cada vez melhor à medida que x 1
aproxima x 0 cada vez melhor. Assim. concluímos que a reta tangente ao gráfico de J em A é
- ( 8) , onde m e, o va l or limi t e d os quoc1en
a reta de equaçao • t es J(xixi>- f(xo) , à med'd
-xo 1 a que x 1 se
aproxima de x 0 mais e mais.
l\1odernamente, dizemos que tal valor limite dos quocientes f(:r;1=~~xo), à medida que x 1 ae

5
n f

1'
11

7 pr

li 1 'li

li u
(J [,, /,)

~
onde A({xo. x]) denota a área da porção da região em questão, situada entre as retas verticais de
abscissas x 0 e x.
É razoáYel supor que. para x bem próximo de x 0 . uma boa aproximação para A([xo, x]) seja a
área do trapézio com bases de comprimentos J(xo) e J(x) e que tem o segmento [xo,x] como um
de seus lados não paralelos (o trapézio hachurado, na Figura 7). Ademais, também é razoável
supor que tal aproximação seja tanto melhor quanto mais próximo .r estiver de xo, de maneira
que
A([xo. x)) ~ ( /(xo); f(x)) (x - ro). (10)

para r - x 0 pequeno.
Combinando {9) e (10), concluímo que

A(x) - A(xo) ~ J(xo) + J(x)


X -Xo 2

para , - .ro pequeno, ~cndo tal Rprox.imnçiiotanto melhor quanto mnis próximo x estiver de x 0.
f\J,t~,i-ic•
o gráfico dt• / for umn cur\':n oontúma {i.l'.. 'l'lll interrupções), então, à medida que x se
aproximn mais e mab d<.> x 0• l ...,Jh'!"lUno quC' \1\lor • /(.r) "iC aproximem mais e mais de f (x 0 ),
ele fo11111\ que o quocil•nt' A(:r-J::!z:> fiqlh'lll cnd , •z nuu::,prox1mos de f(.ro>;t<xo) = f (x 0). Em
1csumo, concluímo~ qut>, n ,-..-..l.' e so dt•,,• ~r

i\(x) - • \(xo) 0t f( . )
------ :lo' (11)
~.t- o

pmn .r - .r0 pcqUl'llO,!:>Cndotnl nprox.im ção ranto melhor quanto mais próximo x estiver de x 0 .
Em pnlm 1 n~. o nrgument o heurist ico acim, garante que podemos reobter a função f : [a, b] ---+
R qll(' d<'finc um grAfiro conlfnuo, contnnto que conheçamos a função A : [a, b] ➔ R, tal que
A(x) cnlculn n fü-cndn região ...itut da sob o gráfico de J. acima do eixo das abscissas e entre as
n•t o.-.de nb~c1·sn~ a n .r..
1\o ca..--o l'll\ que /(.r) = x".
por exemplo, ,imos que A(x) =::;,de forma que, para x - x 0
pl,qucno (e rom o nux1lio do problema 5, ç.:~:na 417),
1
A(x) - A(xo) = _l _ (x"- 1 - x0+ )
x - Xo n + 1 x - Xo

= n: (x" + x"- 1xo + x"- 2 x~ + · · · + xx 0- 2 + x 0- 1)


1
,.,_,
= -- 1 (x n0 , , x n-1
0 x0
+ x 0n-2 x 02 + ••• + xox n-2
0
+ x n-1)
0
n+l
1
= - -(n + l)xô =xô= J(xo)-
n+l

7
m linguag m mod rna dad nção n -o • f : [a b] . cujo gráfico ' um ur
cemínua diz mo qu função- •r : [a, b] ➔ • um int gral ind fimda p f d no am

A(x) = J.x J(t)dt. (12)

Aqui, o ímbolo J:r orda o fa o d qu ár o gTáfi o d f, d a i i


calculada com o auxílio d oma qu apr ximavam a ár d z m lh r; ímb 1
f(t)dt, por ua vez r orda o fato d qu d om 1 t rm Tni(t, - t,_ 1 )
ond m, era o m nor valor d f no alo [t,_1 ti] (logo mi = J(t) 1 um t E [t, 1 t,])
e ti - ti-I = D.ti era a i- 'sima dit r nça ntr r alo [a b] (L ibniz
utilizava istematicament a 1 ra gr ga xpr ar dif r n fini o qu
até o dias d hoje).
Por outro lado vimo im qu qu x-xo
pr ximam- m m m di
que x aproxima d x 0 da d ri da da funç -

(1)

para x - x 0 p qu no ndo a proxima mai próxim r d . O·


Sub tituindo (12) no prim iro m mbr d und m mbro d ( 1 ) r
( 11) e tran formando proxim m iguald m

d 1x
d. ª J(t)dt = J( ·) (1 )

qu é, e ncialment , o ont údo do Ti o~ ma Fundam ntal do Cálculo ug r qu


de int gração e d rivação (i. . d cál ulo d b um gráfi o d r a ng n gr'fico
em um de seus ponto ) guardam um r laçã ín irna uma om a ou r

A apr ntação qu fiz mo acima das d u n r • do álculo car u propo i ad -


mente de precisão, por p lo m no duas razõ di tin
Do ponto de vi ta p dagógico o o d ar h ur' ainda qu impr ci , po -
sibilitou-no apresentar o cone i o de int gral d rivada com um mínimo d pr' -r qui i o .
Por sua vez, uma tal apr en ação por forn r ao 1 i or uma vi ão p norâmica mínima do
.
conteúdos que discutir mo fun ion rá como fio ondu or d boa p do livro .
Do ponto de vista histórico a impr i ão d no o gum nto il impr i ão análoga,
-io maior, dos raciocínio utiliz do ' 'po
.. 1111 d on L ibniz. D f to por aqu la 'po
o próprioconceito de função não ta ab l ido!
ima análise, as difi uldad argum n o qu d lin amo r id m n
abordagens adequad do on i completude do conjun o do núm ro r ai
e de função contínua (cf. Capítulo 2) e de limites de funções (cf. Capítulo 3). &tas, por sua
vez. foram gestadas ao longo de todo o século X.IX e do primeiro terço do século XX, graças
principalmente aos esforços de Cauchy. \\"eierstra.s.5.Cantor e seus discípulos.

Figura Augustin Loub Cauchy um dos maiores matemáticos do


!'-éculoXIX e tah l7. da História. Cauchy foi um dos precursores no estudo
da Análk;e Matemática, érca d p\: qui,a extremamente importante em
matcmãtica "'UJ>l)rior.Ele taml,:'>m tem -..'ll nome ~ociado n muitos
r • ultado dt• F:bic:t-Mat11.
m tica.

\e·~ < intc111•gno,o conct ito de int<-grnJt ml> m gnnhou contorno bem definidos, inicialmente
1

com os t 1nbnlho d1~13. íli1111nu11. o qu • forrun p ,t riorim•ntt' c:-.lcndidos, por II. Lebesgue, a
um < 011tt•xto mnis gt:mi.

Figmn 9: Quando olh mos pMa o lcg elo de Bernhard Riemann à Geo-
rin, concluímo qu tah Cl a m<'nor de suas contribuições à Mate-
m.:>t
mnt ic- t t>nha sido uma formalização adequa.da do conceito de integral
qu~•.hojt.",lc"n ~ u nome. (Conforme o leitor atento deve ter notado, o
Cnpít ulo 6 e intitulado A Int de Riemann.) Outra das criações de
Rl •mru.m 1
o. geometria hoje conhecida como riemanniana, forneceu a A.
Em:-.tem o arcabouço teórico adequado ao desenvolvimento da teoria da
rclt\tl\ idade geral.

E te lhTo tem o duplo propo ito de colocar os conceitos de derivada e integral em bases
sólidas e fazer JUS às motivaçõe; históricas para o desenvolvimento do Cálculo, apresentando
vá.nas aplica~ões interessantes do mesmo à Física e à Geometria.

9
Desde sua criação, a grande razão para o sucesso do Cálculo como corpo de conhecimento
tem sido sua aplicabilidade a um sem-número de problemas de vários ramos do conhecimento.
De fato, o próprio Principia 2 , publicado em 1687, deixa claro que a grande motivação de 1 ewton
para a desenvolvimento dos métodos do Cálculo residiu nas aplicações dos mesmos à Física.
Após ewton e Leibniz, o século XVIII presenciou uma miríade de aplicações e extensões
dos métodos do Cálculo, capitaneada pelos igualmente geniais L. Euler e J. L. Lagrange.

Figura 10: o suíço Leonhard Euler é, até hoje, considerado o matemá-


tico que mais publicou trabalhos relevantes. Suas contribuições variam,
impressionantemente, da Geometria à Combinatória, passando pela Te-
oria dos Números e Física. Em cada uma dessas âreas do conhecimento
há pelo menos um celebrado teorema de Euler.

Figura 11: Joseph Louis Lagrange, matemático e físico francês do sé-


culo XVIII. Lagrange foi um dos maiores cientistas de sua época, dando
contribuições notáveis ao Cálculo das Variações e à Mecânica Celeste.

Hoje, olhando em retrospecto, é difícil subestimar o papel do Cálculo para o desenvolvimento


científico e tecnológico da sociedade moderna. Suas ideias e resultados mostraram-se e conti-
nuam sendo fundamentais ao desenvolvimento de áreas tão díspares quanto Física, Economia,
Computação e Biologia, para não falm de aplicações, diretas ou indiretas, às várias engenharias.
Folheando as páginas desta introdução, é quase irresistível parafrasear sir Isaac ewton para
concluir que, se há tamanha pervasívidade do Cálculo no mundo moderno, é porque sua criação
2Alcunhado PJulo1JophiaeNalurnlts Prmcipia Malhematica, obra prima de Newton.

10
d nvolvim n o apoiaram- m ombro d gigant .

11
t capitulo inicial apr
quai rã.o utilizado livr m nt ao longo do t xto. Para a di cu ão d m
ug rimo 1 itor on ul ar mpr qu achar conv ni n , o rnat rial e n • ant d

1.1 Definições e exemplo


jam dado conjun o n- . Informalm n . um fun - J J • m 1• m11t
regra qu ocia a cada x E um único y E . Por " z p d m h:zcr um fm -
J : ➔ Y d uma man ira m n or m io d di ame igurc 1.1 ond
cada a indi a qu á • d a d .r.E X

1gur 1.1: x mpl d funç- / d X m }'.

E cr vemo f : ➔ para d no ar qu J • wn funç- o d X m e ,o l m n o


y E Y as o iado x E X por f , d no ado por y = J(x), ndo d nomin o a imag m d
x E p la função f. o xemplo da Figura 1.1, mo X= {1 2. 3}, = {a, b, e, d} f (1) = a,
f (2) = a, J(3) = . 1m a a 1mag m 1 d 2 por J. e • a imagem d 3 por f.
orno a ad p lo x mplo acima, a d finição d função p rmit qu . no diagrama orr -
pondente, um ou mai lem n o d Y não recebam etas ou, ainda. qu um ou mai el m n o
d Y r cebam mai d uma ta (ob rv qu amb po ibilidad ã pr nt na 1gura
1.1). ot , ontudo, qu o diagram d figur 1.2 1.3 não orr pond m a funçõ .
A i uaç- da Figura 1.2 • proibida porqu nã há n nhuma ta partindo do l m nto 1 E
A ituação da Figura 1.3 , proibid porqu do l m nto 1 E X parte mai d uma ta.
A tr~ d finiç- gu1r ·pli it m lgun 1p x r mam n út i d funçõ

1-1
igur 1.2· m l m nt d qu

UI l.:.,'tu lmnt qu i 1, rt m i d um

\O 1. 1.
} fix do um 1 m nto e E Y, a função constante e
\ l 1ll) /(x) = para todo x E X.

d fun -o igu 1 , d finid ima odo x E X tâ ciado


lJ E ) •, qual . j . y = . ondiç- impo na d finição" d função
ll IH 11 nt tl i 1t , 1. .. todo I E r iado a um único y E Y.

1.2.
njunto não vazio X. a função identidade de X, denotada por ldx .
d d por Id. •(x) = x. para todo x E X.

n r. ndiçõ xigid p la definição" d


fm - qu Id • • r um função.

1
Para o que segue, fixado arbitrariamente n EN, denotamos por ln o conjunto dos n primeiro
número· naturais, i.e., ln= {1,2, ... ,n}. Por exemplo, 11 = {l}, h = {1,2}, / 3 = {1,2,3} e
assim por diante.

ÇÃO 1.3.
Uma eequência (infinita) de números reais é uma função f : N ➔ R. Uma sequência
(lnlta) de númerosreais é uma função f: ln ➔ R, para algum n EN.
--------------~
Dada uma sequência f : N ➔ R (resp. 1 : ln ➔ R) é costume denotar, para k > 1 inteiro,
ak = f(k). Desta forma, obtemos a notação usual para sequências, qual seja,
a1 = f(l), a2 = 1(2), a3 = 1(3), ....
Por vezes, denotamos a sequência em questão simplesmente por (adk>1 (re p. (ad1<k<n).
De um ponto de vista matematicamente mai rigoro o, uma função é um caso particular de
uma relaçãoentre dois conjuntos, de acordo com a definição a egmr.

DEFINIÇÃO 1.4.
Dados conjuntos não vazios X e Y, uma relação de X em Y (ou entre X e Y, nessa ordem)
é um subconjunto R do produto cartesiano X x Y, i.e.. R ê um conjunto de pares ordenados do
tipo (x, y), com x E X e y E Y. Se Ré uma relação de X em X, diremos simplesmente que R
é uma relação em X.

EXEMPLO 1.5.
Se X= {l, 2, 3} e Y = {2, 3, 4, 5}, o conjunto

R = {(x, y) E X x Y; x > y}

é a relação de X em Y dada por R =


{{2,2), (3, 2), (3, 3)}; de fato, esses são os únicos pares
ordenados(x, y), com x E { 1, 2, 3}, y E {2, 3, 4, 5} e tais que x > y.

O exemplo anterior é um caso particular de um procedimento óbvio a que podemos recorrer


para construirmos relações específicas R entre conjuntos não vazios X e Y: basta especificarmos,
de alguma maneira, um subconjunto do produto cartesiano X x Y. Aqueles pares ordenados de
X x Y que satisfizerem a especificação prescrita serão os elementos de R. Sendo (x, y) um tal
par, diremos que x e y são relacionadospor R.
Se R é uma relação de X cm Y, enlão R e X x Y, por definição. Reciprocamente, escolhido
um par ordenado (:r, y) E X x Y, pode ocorrer que (x, y) E R ou (x, y) <tR (i.e., que x e y

16
sejam relacionados por R, ou não). No primeiro caso, escrevemos x Ry; no segundo, zJ(71.
símbolos,
xRy ~ (x,y) E R.
Assim é que, para a relação do exemplo anterior, temos 3 R 2 mas 2,R3, uma vez que 2 > 3 é
falso.
Dentre todos os tipos de relação que podemos considerar entre dois conjuntos não vazios, o
principal é aquele isolado a seguir.

DEFINIÇÃO 1.6.
Dados conjuntos não vazios X e Y, uma relação/ de X em Y é uma função
condição for satisfeita:
\/x E X, 3 um único y E Y; xfy.
--------~~~---------
Como antes, escrevemos / : X ➔ Y para denotar que f é uma função de X em Y e f (x) = y
para denotar que o par (x, y) E X x Y é relacionado por/, i.e., satisfaz xfy. Observe que tal
notação faz sentido> uma vez que a definição de função garante que, se (x, y 1) e (x, Y2)são pares
ordenados em X x Y tais que xfy1 e xf y2 1 então y 1 = y2 . Por outro lado, é imediato verificar
que a definição formal acima coincide com a definição informal dada no início desta seção.
O mais das vezes, trabalharemos com funções f: X ➔ Y tais que X, Y e IR..Em tais casos,
geralmente indicaremos quem é o elemento f(x) E Y associado a um elemento genérico x E X
por meio de uma f 6rmula em x, a qual explicita uma regra que a função deve satisfazer. Por
exemplo, podemos dizer: considere a função f : IR. ➔ IR.,dada por f(x) = x 2 . Isto quer dizer
que a função associa, a cada x E IR.,seu quadrado x 2 . Veja que os requisitos definidores de
uma função estão satisfeitos, uma vez que, a cada x E IR.,temos associado um único outro real
f(.--c),qual seja, x 2 . Assim é que, ainda em relação a esse exemplo, temos /(./2) = (./2) 2 = 2,
f (3) = 32 = 9 etc.
Quando X, Y e IR.e / : X ➔ Y é uma função tal que o elemento f(x) E Y associado a
x E X é dado por uma fórmula em x, denotamos por vezes tal correspondência escrevendo

f:X ~ y
X 1-----t f(x)

A im, a função do parãgrafo anterior, que associa a cada x E IR.seu quadrado x 2 , poderia ser
denotada da seguinte maneira:
/ :IR. ~ IR.
X 1-----t X 2

Vejamos mais um exemplo.

17
1.7.
Considere a função J: Q ➔ lR, dada por

f (x) = { J x 2 + 1, se x <O
x+l. sex>0

Certamenteque temos uma função, pois as expressões que definem J(x) têm sentido cm lR e.
apesar de devermos aplicar fórmulas diferentes, conforme o racional I satisfaça .r < O ou .r > O.
cada racional x tem uma única imagem f(x) bem definida. posto que os cn!--os.r < O e .r > O
cobrem todos os racionais. Assim, por exemplo. f ( -1) = v (-1 ) 2 + 1 - -.,/2 ( umn v--z que
-1 < O), mas J(2) = 2 + 1 = 3 (uma vez que 2 > O).
Observe que poderíamos ter definido J(.r:) escrevendo

f (I) - { J.r2 + 1, S .7: < Ü


.r;-1, e.r:>0

Nesse caso, as condições x < O e .r. > O cobrem todo:-. o:-.racionnis 11uts não são mais disjuntas:
x = O satisfaz ambas; no entanto, as fórmula.., que d \ mo:s nplicnr n um e a ont1 o ca~o dão
um mesmo resultado quando x = O (umn \·cz que /O.!.+ 1 = O+ 1), afa tnnclo assim qunlqm•1
possibilidade de inconsistência.

Por vezes, dizemos que uma função J como a do exemplo anterior esta definida por partes
em alusão ao fato de que há uma fórmula para calcular J(x) quando .r E (- , Oj, e outr,1 quc1ndo
x E (O,+ ) (ou x E (O,+ ), conforme e queira).
Ao lidarmos com uma função f : X ➔ Y, é frequentemente útil denominarmos os conjuntos
X e Y, respectivamente, de domínio e contradomínio da função: nesse contexto, denotaremos
X = Dom (J). Assim é que, para a função f do exemplo 7. o domínio e o contradomínio são,
respectivamente, Q e JR.
O mais das vezes. vamos trabalhar com funçõe f : X ➔ IR ta1. que X e R. Em tais casos,
diremos que f é uma função real (em alu ão ao fato de que f assume valores reais - i.e .. de que
seu contradomínio é JR)de uma variâvel real (em alusão ao fato de que um elemento genérico
.e do domínio X de f a variável da função - é um número real).
Ainda em tais ca os, quando 05 valore f(x) forem dado por uma fórmula. como f (x) =
J :c(:c
1
, por exemplo, alvo menção· em contrário, com·encionamo tomar X como endo igual
1)
ao maior· domínio possível. De outro modo. tomamo X como igual ao maior subconjunto de R
no qual w; operações matemáticas que definem a e,X.-preão f(x) têm entido. Diremos. então.
que X é o domínio maximal de definição. ou imple mente o domínio maximal de J.

1
EXEMPLO 1.8.
A título de ilustração. encontremos o domínio maximal X e 1Rda função / : X --,
por J(x) = Jx<~-I). Temos

X = {xJR; Jx(x l - 1)
E E IR}

= {x E 1R;x(x - 1) > O}.

Como, para um produto ser positivo, ambos os fatores devem ter um mesmo sinal, devemos ter,
x > O ex - 1 >O.ou então x < O ex - 1 < O, i.e., x > 1 ou então x < O. Portanto,

X= (-oo, O)U (1. +oo).

o contexto de funções reais de variável real, temos maneiras padrão de construir novas
funções a partir de outras já conhecidas, utilizando as operações aritméticas do contradomínio
1Rdas mesmas. Mais precisamente, dados um conjunto não vazio X e IR, um número real e e
funções reais de uma variável real f, g: X ➔ IR(de mesmo domínio!), definimos as funções

f + g, f · g e e· f : X ➔ IR

pondo
(J + g)(x) J(x) + g(x),
(J. g)(x) - f(x) • g(x),
(e· J)(x) - e· f (x),
para todo x E X.
Algumas observações são pertinentes. Em primeiro lugar, veja que os sinais de adição na
igualdade
(! + g)(x) = J(x) + g(x)
têm significados distintos: no primeiro membro temos a definição da função f + g, ao passo que,
no segundo membro, f(x) + g(x) representa a adição usual dos números reais f(x) e g(x). Uma
observação análoga é válida para os sinais de multiplicação utilizados nas definições das funções
J. g e e. f. Em segundo lugar, assim como com números reais, omitiremos em geral o sinal de
multiplicação, escrevendo f g e cf em vez de J · g e e· J, mas isto não deve causar confusão.
É evidente que f + g, f g e cf são realmente funções de X em IR. As funções f + g e fg
são denominadas, respectivamente, a soma e o produto das funções f e g. Por outro lado, o
produto cf do número real e pela função J pode ser visto como caso particuhu· do produto de

19
1 1 '' 1 11 -

duas funçÕ<'s:tomando g como a função constante e igual a e, temos f g = cf; ainda para tal g,
denotaremos f + g simplesmente por f + e, de sorte que

(! + c)(x) = f (x) + e,
para todo x E X.

ExEMPLO 1.9.
Sendof e g as funçõesde IRem IRdadas por f(x) = r.l+i e g(x) = -x + 3, temos
X
(! + g)(x) = f(x) + g(x) = x 2 + 1 + (-x + 3)
x + (x 2 + 1)(-x + 3) -x 3
+ 3x 2 + 3
-
x2 + 1 x2 + 1
2
x -x + 3r
(Jg)(x) = f(x)g(x) = 2
• · (-x+3) = 2

X +1 X +1
e
( V3J)(x) = V3/(.r) = ../3. 2x - _.r_/3_3
.r +1 .z;2 T 1.

Deixamos ao leitor a verificação de que as operaçõr de adiçao e multiplicação para funções,


definidas como acima, satisfazem propriedades análogas àquelas das operações correspondentes
com números reais. Mais precisamente, para f, g. h : X ➔ IR e a, b, e E IR, temos:

• Comutatividade: J + g = g + f; fg = gf.
• Associatividade: f + (g + h) = (! + g) + h; J(gh) = (Jg)h.
• Distributividade: f (g + h) = f g + f h.
Note que a associatividade da adição e da multiplicação de funções permite definir, de modo
inteiramente análogo, a soma ou o produto de um número finito qualquer de funções de X em
IR. Por exemplo, dadas funções f 1, h, ÍJ : X ➔ IR, definimos /1 + h , h = J1 + (f 2 + h) (ou, o
que é o mesmo, (/1 + h) + h) e !ihh = fi{hh) (ou, o que é o mesmo, (J1 h)h) etc.
Por fim, sugerimos ao leitor consultar o problema 1. para uma e>,,,'tensãoadicional da dis-
cussão sobre operações com funções.
Voltemos ao estudo do contradomínio de uma função f : X ➔ Y, é importante notar que Y
em geral nao coincide com o conjunto formado pelas imagens dos elementos de X. Ilustremos e.. a
diferc11çautilizando novamente a função f do exemplo 7. Já ob ervamos que o contradomínio
da mesma é o conjunto IRdo5 número5 reais. Por outro lado, o conjunto formado pelas imagens

20
f(x) dos elementos do domínio Q de f certamente não contém números reais menores que 1. De
fato, para um racional x qualquer, temos x 2 + 1 > 1 e, daí,

por outro lado, para um racional x > O, temos f(x) = x + l > 1, de maneira que

{f(x); x E Q} e [1, +oo).

ote, por fim, que o intervalo [1, + ) é um subconjunto próprio do contradomínio IRde f.
Mais geralmente, dada uma função f : X ➔ Y, o conjunto imagem, ou simplesmente a
imagem, de f é o conjunto Im (/), cujos elementos são as imagens f(x) E Y dos elementos
XE X:
lm (/) = {f(x) E Y; x E X}.

Em particular, temos sempre Im (!) e Y, e a discu ão acima mostra que pode ocorrer Im (!) =J
Y.
o exemplo discutido no penúltimo parágrafo acima, mostramos que a imagem da função era
um subconjunto próprio do contradomínio da m~ma. No entanto, não chegamos a explicitar pre-
cisamente tal conjunto-imagem. Em situações específicas essa tarefa pode ser consideravelmente
difícil, como atesta o exemplo a eguir.

EXEMPLO 1.10.
Explicitemos a imagem da função f: Q \ {O}➔ Q dada por /(r) = ¼,se o racional r estiver
escrito na. formar= i, onde a E Z, b E N e mdc (a, b) = l.
Certamente a função f está definida de maneira não ambígua, uma vez que todo racional
não nulo admite uma representação única como quociente de dois inteiros primos entre si, sendo
o denominador um número natural. Por exemplo, -1 = -/ e, daí, f (-1) = ½-Por outro lado,
como b EN, vemos imediatamente que

Ademais, é imediato que todos os números do último conjunto acima pertencem à •


f, uma vez que f (}) = ¼para todo b E N. Portanto,

21
( \l'l 11 1 ( 1 1 Fl'\"CJ>t-:S

Infelizmente não existe um algoritmo 1 que nos permita encontrar explicitamente a imagem
de uma função qualquer dada. Entretanto, ao longo dos capítulos subsequentes, resolveremos o
problema de encontrar o conjunto-imagem para várias funções importantes.
Terminemos esta seção discutindo o conceito de igualdade de funções. Em relação à função
J do exemplo 7, não faz sentido considerarmos J(../2), uma vez que ../2 (/. Q e f tem domínio
Q. O que poderíamos fazer seria considerar, em vez de f, a função g: 1R➔ lR, dada por

_ { J x 2 + 1, se x < O
g (X ) - .
x + l, se x > O

Apesar de as fórmulas que definem f(x) e g(x) serem as mesmas, para f elas só podem ser
aplicadas a x E Q, ao passo que para g elas podem ser aplicadas a todo x real; assim, não faz
sentido pensarmos em f e g como funções iguais, as quais apenas se encontrariam denotadas de
duas maneiras distintas.
1'.laisgeralmente nesse sentido, temos a definição a seguir.

DEFINIÇÃO 1.11.
Duas funções/: X ➔ Y e g: W ➔ Z são iguais se X= lV, Y =Z e f(x) = g(.c), para
todo x E X.
-~~-~~-----------------~--~-~----

Se duas funções f : X ➔ Y e g : lV ➔ Z forem iguais, escrevemos f = g. Frisamos que,


de acordo com a definição acima, tal notação encerra mais significado do que se, simplesmente,
escrevermos f(x) = g(x): ela significa a igualdade dos domínios, X= iv, e dos contradomínios,
- Y = Z, assim como a validade da igualdade f(x) = g(x). para todo x E X. Se duas funções f e
g como acima não forem iguais, escreveremos / =/g e diremos que f e g são funções diferentes
ou distintas.

Problemas - Seção 1.1


1.1. Encontre o domínio maximal de definição da função J, tal que J(x) = ~~=~-

1.2. Encontre o domínio maximal de definição da função J, tal que

----------------:--
·J(x) =
) Um algoritmo l! uma sequência finita e bem determinada
h~ ✓~ _✓3- vx
de procedimentos executáveis que, rigorosamente
t>eguídos, fo, 11ec·t•111 it soluçito de um certo problema Um exemplo familiar é o algoritmo que utilizamos para
divíclii dois 11ú1111•rn~ 01Jtc11clo
11aL11111is, o quociente e o resto da divisão.

22
1.3. Seja f : Q~ ➔ Q+ a função definida por J (%)= 1::~~:1, se a, b E N são primos entre si.
(a) Calcule /(1), J(lü) e f (~!)-
(b) Dentre os racionais ~~, ; 527 e ~i,quais pertencem
1
à imagem de f.

1.4. Considere a função f : lR ➔ lR definida por J(x) = x 3 - 2x2 + 5x. Prove que f(x) tem o
mesmo sinal de x, para todo real x -=/=
O.

1.5. A função f : lR ➔ lR é tal que f(l) = 2, J('1i.) = 4 e f(x + y) = J(x)f(y), para todos
x, y E lR. Calcule o valor de f (3 + '1i,).

1.6. Seja f : lR ➔ lR uma função tal que f (x + y) = J(x)J(y)


para todos x, y reais. Se (ak)k?,I.,,
é uma PA de razão r e f (ai) -=/=
O, prove que a sequência (f(ak))k?,l é uma PG de razão
J(r).

1. 7. Seja f : lR ➔ lR uma função tal que f(x+y) = f(x) + f(y), para todos x, y E lR. Se (ak)k?,1
é uma PA de razão r, prove que a sequência (J(ak))k?,l é uma PA de razão J(r).

1.8. * Dadas funções reais de variável real J,g : X ➔ lR, estenda


a discussão do texto, apre-
sentando definições apropriadas para a diferença f - g e o quociente ; de f e g.

1.9. * A parte inteira de um real x ê definida como o maior inteiro menor ou igual a x, sendo
denotada lxJ. Por exemplo, l1rJ = 3, l-~J = -2 e llJ = 1. Explicite a imagem da
função parte inteira
l·J : lR --+ lR
(1.2)
x ~ lxJ '
que associa a cada x E lR sua parte inteira lx J.

1.10. * A parte fracionária de um real x ê o real {x}, definido por {x} - x - l.rJ, onde
lx J denota a parte inteira de x ( veja o problema anterior). Por exemplo, {1r} = 1r - 3,
{-~} = -~ - (-2) - ½ e llJ = 1 - 1 = O. Explicite a imagem da função parte
fracionária
{·}: R --+ lR
1 (1.3)
T ~ {x}
que associa a cada x E lR sua parte fracionária {x}.

1.11. * Seja f: Q ➔ Q uma função tal que J(x + y) = J(x) + f(y), para todos x, y E Q. Prove
os seguintes itens:

(a) J(O) = O e J(-x) = - J(x), para todo x E Q.

23
( \ 1 111 1 1 1 1 FL\<. : •
1.2 Monotonicidade, extremos e imagem
Recorde que a imagem de uma função / : X -t Y é o conjunto

lm (/) = {J(x) E Y; x E X}.


Se a função / é real de uma variável real, i.e.. se f : X -t lR, com X e lR, uma maneira
particularmente útil de declararmos sua imagem é escrevermos

1m (/) = {y E Y; y = J(x) para algum x E X}.

Desta forma, se os valores f(x) forem dados por uma expressão (i.e., uma fórmula) que dependa
de x E X, poderemos encarar o problema de encontrar a imagem de f como aquele de encontrar
os y E 1Rpara os quais a equação f(x) = y tenha pelo menos uma solução x E X. Vejamos
alguns exemplos.

EXEMPLO 1.12.
Uma função afim é uma função f: 1R-t R tal que f(x) = ax + b para todo x real, onde a
e b são números reais dados, com a =/:O. Uma função linear é uma função afim/ como acima,
tal que b - O.
De acordo com a discussão anterior a este exemplo, a imagem de uma função afim /, dada
como acima, pode ser encontrada procurando-se o conjunto dos y E 1Rtais que a equação
a.e + b = y tenha alguma solução x E lR. Mas. como tal equação sempre admite a solução
:r = ~, concluímos que todo y E 1Rpertence à imagem de /, de sorte que Im (/) = JR.
-~~~-----'

EXEl\tPLO 1.13.
A função de proporcionalidade inversa é a função / : 1R\ {O} -t 1R\ {O} dada por
J(::r) - ¼,para todo x E 1R\ {O}.
Para encontrar sua imagem, é suficiente encontrar os y E 1Rpara os quais exista um real
x =/:O (i.e., x pertencente ao domínio de /), tal que f(x) = y, i.e., tal que ~ = y. Se y = O, tal
equação claramente não admite solução: por outro lado, se y # O, a mesma equação admite a
solução x = l,'li de sorte que Im(/) = 1R\ {G1.

Antes de continuar, precisamos de uma definição importante.

DEFINIÇÃO 1.14.
Uma função quadrática ou de segundo grau é uma função / : R -+ R tal que /~
ax 2 +bx+c, para todo x real, onde a, b e e são números reais dados, com a=/:O. O diacr
~ da função / é o discriminante do trinômio de segundo grau ax 2 + bx + e, i.e., A

25 Â
<. \1·111 1( 1 1 FL,<;01-:s

O problema da determinação da imagem de uma função quadrática é suficientemente impor-


tante para ser colecionado como o seguinte resultado.

PROPOSIÇÃO 1.15.
Em relação à função quadrática J(x) = ax21+
bx + e, temos que:

(a) Se a> O, então Im (J) = [-!, +oo).

(b) Se a< O, então Im (!) = ( -00 -!] .


1

Ademais, em qualquer um dos casos acima, OS f(x) =-ti# X= -t.


PROVA.
Seguindo a ideia geral esboçada anteriorment para obter a imagem de f. basta cncontnu 111m,
os~ E IRpara os quais a equação ax2 + b.r+ e = y. 1.c.. n equação de segundo grnu a.r2 + b.r+ (e
y) = tenha solução.
O, proposição
A garan que umn condiçiio nccc~stum e uficicntc pnm
16

a existência de raízes reais para tal equação é discriminante


cn1c não ncgal1,·o,
seu SCJn 1.c , que

2 2
b - 4a(c - y) > O. Mas, como b - 4ac = ~. m;y que procuramos siio c.xatnmcntc ns soluções
da inequação de primeiro grau (em y) ~ + 4ay O.
Agora, consideramos separadamente os C8SOI n > O e a < O Se a > O, então •lnlJ + ó. 2::O#
y >- t
e segue, daí, que

Im (f) ~ } = [- 4a'
= {y E IR; y > 4a ó. +oo) •
'

se a < O, então 4ay + ó > O# y < - t. de ID11Deiraque


- ó.]
' 4a •

Para o que falta, veja que, para y E 1m(/), as soluções da equação ax2 + bx + e _ y
(# ax2 + bx + (e - y) = O) são
-b ± Jb2 - 4a(c-y) -b ± J& + 4ay
x = 2a = 2a • (1. 4)
Portanto, a equação J(x) = y admite uma so~o única se, e só se, ó.+ 4ay = O, ou, 0 que é 0
mesmo, se, e só se, y = -t; sendo ~sse o caso, mos. a partir de (1.4), que x = - :.
2

Para o que segue, convencionamos dizer quela função quadrática f(x) = ax2 + bx + e tem
srnal constante quando J(x) > O para todo x E ou J(x) < O para todo x E IR.

26
COROLÁRIO 1.16.
A função quadrática J(x) = ax 2 + bx + e tem sinal constante se, e s6 se, ~ < O.
temos af(x) > O, para todo x E IR. De outro modo:

(a) Se D. < O e a> O, então J(x) > O para todo x E IR.

(b) Se t:::.< O e a< O, então f(x) < O para todo x E IR.


--------~~--- ......

PROVA
Analisemos o caso a> O, sendo a análise do outro caso totalmente anâloga.
Sendo a> O e t:::.< O, segue da proposição anterior que
D.
f(x) > - a > O. V x E IR.
4
Reciprocamente, suponha que a > O e que f tem sinal constante. Pelo item (a) da proposição
anterior, a imagem de f contém números positivos, de forma que a constância de sinal de /
garante que devemos ter f(x) > O, para todo x E IR. Em particular, devemos ter
6
- =f (-.!!_) > o.
-!a 2a
Logo, t:::.< O.

OBSERVAÇÃO 1. 1.
Uma pequena modificação do argumento apresentado no corolário anterior permite concluir
que

i St t:::.< O e a > O, então J(x) > O para todo x E IR.

u. Se D. < O e a < O, então f(x) < O para todo x E IR.


-----------~--_,____.,
O corolário anterior pode ser utilizado para dar uma demonstração da famosa desigualdade
de Cauchy, conforme ensina o próximo exemp10.

EXEMPLO 1.17.
Dados um inteiro n > 1 e números reais a1, a2, ... , an não todos nulos e b1,i>J,... , ,
não todos nulos, considere a função quadrática

f(x) = (a1x - bi)2 + (a2x - ~) 2 + · · • + (anx - bn)2


= Ax 2 - 2Bx + C,

27
onde A= ªi +a~+···+ a~, B = a 1b1 + a2~ + · · · + anbn, C = bi + b~+ .. · + b~.
Uma vez que f(x) é uma soma de quadrados, devemos ter f(x) > O para todo .x E IR. Por
outro lado, como os números a 1 , a2, ... , an não são todos nulos, temos A > O, e o corolário 16
garante que 6. = 4{B2 - AC) < O. Portanto, B 2 < AC e, substituindo os valores de A, B e C
em tal desigualdade, obtemos a desigualdade de Cauchy:

De acordo com a discussão acima, a igualdade na desigualdade de Cauchy equivale à igualdade


6. = O para a função f, que, por sua vez, equivale à existência de um único a E IR tal que
J(o:) = O. Mas, como /(o:) é uma soma de quadrados, temos /(a)= O se, e só se, cndn um de
tais quadrados for zero quando x = o, i.e., se, e só se,

Por fim, como ao menos um dos bi é não nulo, temos a -1-O e, escrevendo >.= ¾,obtemo~

como condição necessária e suficiente para a igunldndc.


-~~~~~~--~--------------
A fim de prosseguirmos nosso estudo de funçõc~, precisamo de mais alguns conceitos, elcn-
cados na definição a seguir.

!)EFINIÇÃO 1.18.
Seja/ e Rum intervalo. Uma função f: / ➔ 1Ré dita:

(a) Crescente se, para todos X1 < x 2 em /, tivermos f (xi) < f (x2).

{b) Decrescente se, para todos X1 < X2 em I, tivermos f(x1) > J(x2).
(c) Não decrescente se, para todos x 1 < x 2 em/, tivermos f(xi) < J(x 2).
{d) Não crescente se, para todos x 1 < x 2 em/, tivermos /(xi) > f(x 2 ).

Ademais, em um qualquer dos casos acima, dizemos que a função f é monótona em / 2 .

A respeito dessa definição, um problema interessante é o de encontrar os interoalos de mo-


notoniczdade de uma função, i.e., dada uma função f : I ➔ IR, onde J e 1Ré um intervalo,
2 Nas notações desta definição, vale observar que, para alguns autores, uma função J satisfazendo a condição do
item (a) (rcsp (b), (e), (d)) é dita estritamente crescente (resp. estntamente decrescente, crescente, decrescente).

28
pede-se investigar em que inten-alo J e I tem- e J crescente (resp. decrescente). Vejamos
alguns exemplos elementares. postergando uma análise mai geral para a seção 3.5.

EXEMPLO 1.19.
A função afim / : B ➔ JR. dada para x E R por J(x) = ax + b, é crescente se a > O ~
decrescent~ se a < O.
Verifiquemos tal afirmação quando a >O.sendo a análise do caso a < O totalmente anéloga.
Para números reais quaisquer x 1 < x 2 . segue de a > O que

/(x-i) - /(xi) = (a.x2+ b) - (ax 1 + b) = a(.r-2- .ri) > O,

e f é cr~cenh•.
~~-~------------------~--------~

EXJ;;MPLO 1.20.
A função f : (O,+ ) -, IR'.,dada por /(x) = ;:.2 , " crescente cm R. Para verificar tal
aíi, wação, to111~ número r<?Bi O < o < b. Ent - o,

1,2 a' l 2 2
J(b) - /(a)= -b - 2 - n+.?
-, = (a )(b + ) [b (n + 2) - a (b + 2)]
2 2

, 1'1.q,w (a+ 2)(b


,,, 111111, + 2) > O, l,n..,.l m tmrmos qut• /J(n + 2) - a2 (b + 2) > O. Para tanto,
, 11 jn q11c•

ll(a + 2) - a2 (b + 2) = b2a - a2 b + 2(b 2 - a2 )


= ab(b - a)+ 2(b - a)(b + a)
= (b - u)[ab + 2(b + a)].
Como O < a < b. ~egu que arnbo o fatores do último produto acima são positivos e, daí,
2
i>(a + ~) - oJ(b + 2) > O.

ExgMPLO 1.21.
A fun,ão f : ➔ R. dada por J(x) = x 3 + 2x. é crescente. De fato, para números reais
qnnbq\l\'r a < b, t •mos
f(b) - f(a) = (b3 + 2b) - (a 3 + 2a)
= (b3 - a3 ) + (2b - 2a)
= (b - a)(b 2 + ba + a2 ) + 2(b - a)
= (b - a)(b2 + ab + a2 + 2).

29
(. \1'111 1<> 1 Fl':,(,'ÔES

Como b - a > O, basta mostrarmos que a 2 + ab + b2 + 2 > O. Uma possibilidade é usar a


desigualdade entre as médias aritmética e geométrica para dois números reais (cf. (A.5), com a
no lugar de x e b no lugar de y):

a2 + b2 + ab+ 2 > 2labl+ ab+ 2 > labl+ 2 > O,

onde, na penúltima passagem, utilizamos o fato de que !ai+ a: > O, para todo a E IR.

A proposição a seguir resolve, para funções quadráticas, o problema de encontrar os inLcrvaJos


de monotonicidade da função.

PROPOSIÇÃO 1.22.
Sejam a, b,c E IR, com a# O, e f(x) = ax2 + bx + e.
(a) Se a> O, então J é decrescente em (-00 1 - 2:] e crescente em [- ~., +oo).

(b) Se a < O, então f é crescente em (-oo, - 2~] e decrescente cm [-,!, +oo).

PROVA.
Façamos a prova do item (a), sendo a prova do ilcm (b) completamente análoga. Para
x2 > xi > - 2:, temos

f(x2) - f (x1) = (ax~+ bx2+ e) - (ax~+ bx1+ e)


= a(x~- Xi)+ b(x2- x1)
= a(x2 - x,) ( x2 + x 1 + ~) > O,

uma vez que x2 > x1 > - 2: implica X2 - x1 > O e x2 + x1 + ~ > O. Logo, f é crescente em
[- 2:,+oo) quando a> O. De maneira análoga, mostramos que fé decrescente em (-oo,-
2
:J
(i.e., que J(xi) > J(x2) para X1 < X2 < - 2:).

A próxima definição é, de certa maneira, complementar à definição 18.

DEFINIÇÃO 1.23.
.
Sejam / C R um intervalo e f : / ➔ R uma função dada. Dizemos que y 0 E IR é o valor
m(nimo de f em I se as duas condições a seguir forem satisfeitas:

(a) Im (!) e [Yo,+oo).

30
(b) Yo E lm (/).

Nesse caso, os reais xo E I tais que / (x 0) = y0 são denominados os pontos de DlllDlllM:Ji


função/.

De outra forma (e nas notações da definição anterior), y0 E lR é o valor mínimo de / se


f(x) > Yo, para todo x E I, e existe x0 E I tal que /(x 0) = y0. Por outro lado, observe que a
condição (a) sozinha não é suficiente para garantir que y0 E lR seja o valor mínimo de f. Por
exemplo, se J(x) = x 2 , para x E JR,então J(x) > -1, para todo x E JR;entretanto, -1 não é o
valor mínimo de/, uma vez que não existe x 0 E lR tal que f(x 0 ) = -1, isto é, tal que xi= -l.
Analogamente, definimos o que se entende por valor máximo e ponto de máximo de uma
função f : I ➔ lR (onde I e lR é, novamente, um intervalo).
Genericamente, os pontos de máximo (resp. mínimo) de uma função são denominados seus
pontos extremos; da mesma forma, os valores que a função assume em tais pontos são seus
valores extremos.
1a seção 3.5, estudaremos um procedimento sistemático para procurar pontos extremos de

funções de um tipo muito importante, ditas deriváveis. Por ora, contentamo-nos com alguns
exemplos elementares, o primeiro dos quais sendo uma consequência imediata da proposição 15.

PROPOSIÇÃO 1.24.
Em relação à função quadrãtica f(x) = ax 2 + bx + e, se a> O (resp. a< O), então éo -:a
único ponto de mínimo (resp. máximo) de /. Ademais, o valor mínimo (resp. máximo) de fé
~
-10·

A proposição acima tem vãrias aplicações interessantes, duas das quais colecionadas abaixo
à guisa de ilustração.

EXE~IPLO 1.25.
Temos um semicírculo de diâmetro AB, centro O e raio 1cm (cf. Figura 1.4). O retângulo
PQRS tem o lado PQ situado sobre o diâmEltrA#Íosemicírculo e os vértices R e S situados sobre
o mesmo. Calcule o maior valor possível para sua área.

SOLl'Ç\O .
. Ião é difícil o leitor convencer-se de que OP = OQ, de forma que, sendo OQ = x e QR = y,
temos que a ãrea de PQRS é igual a 2xy. Por outro lado, aplicando o teorema de Pitágoras ao
triangulo OQR, obtemos x 2 + y 2 = 1 e, daí,
2
2xy = 2xJl - x 2 = 2Jx 2 (l - x 2 ) = 2Jx - x4 .

31
A p Q X Q B
Figura 1.4: maximizandoa ârea de PQRS.

essa última expressão, fazendo a substituição z = x 2 , concluímos ser suficiente maxünizar a


função quadrática f(z) = z - z 2 , com a condição de que O< z < 1 (uma vez que x < OR = 1).
Pela proposição 24, tal função admite z = ½como seu único ponto de má.ximo. Como ½E (O,1),
segue que o valor mâximo de fé f (½)= ¼- Portanto, o valor má.ximo para a área de PQRS é
2J¼= 1.

Outra estratégia elementar, por vezes útil, para abordar o problema de encontrar os valores
máximo e/ou mínimo de uma função dada é a utilização de desigualdades. A seguir, vemos um
exemplo nesse sentido.

EXEMPLO 1.26.
Seja/: [O,+oo) ➔ R a função dada por /(z) = Z:: Qual o valor núnimo que f
1
1 . assume?
A função / assume um valor máximo?

SOLUÇÃO.
Inicialmente, note que

x2 + 1 (x 2 - x2 - 1
1) + 2 2
J(x) ----- - --
+ --
x+l x+l x+l x+l
2 2
= x - l + x + 1 = (x + 1) + x + 1 - 2.
Portanto, aplicando a desigualdade (A.6) entre as médias aritmética e geométrica para dois
/ números reais positivos, obtemos

• 2
(x + 1) + -- ✓ + 1) · -- 2 = 2v'2,
> 2(x
x+l x+l
ocorrendo a igualdade se, e só se, x + 1 = x!i, i.e., se, e só se, x2 + 2x - 1 = O. Mas, como
.r > O, couclufmos que haverá igualdade na desigualdade acima se, e só se, x = J2 - -1. Assim,

32
para x > O temos
2
f(x) = (x + 1) + x + - 2 > 2\1'2 - 2,
1
de sorte que 2 ~ - 2 é o valor minimo de f, o qual é atingido se, e só se, x = ~ - 1.
Para o que falta observe que, para n E N, temos f(n) = n - l + n!i > n - 1. Daí, J não
assume valor máximo.

Problemas - Seção 1.2


2.1. Para reais dados a e b, com a # O, considere a função afim f : IR -+ IR, definida por
f(x) = ax + b. Se (ak)k~1 é uma PA de razão r, prove que a sequência (bk)k~1, dada p,a ~
k > 1 inteiro por b, = f(a,), é uma PA de razão ar. "j.,\-J..--- -- q-
2.2. Ache a imagem da função J: IR-+ IR dada por J(x) = x2~ 1 . 1,
2.3. * Ache a imagem da função f: IR• -+ 1R,dada por J(x) = x + ~-
2.4. Sejam I e IR um intervalo, a E I e f : I -+ IR urna funçâo dada. Se f ê crescente (resp.
decrescente) em (-oo, a] n I e decrescente (resp. crescente) em [a, +oo) n I, mostre que
a é o único ponto de máximo (resp. de mínimo) para f em[.

2.5. * Sejam X e
IR wn conjunto não vazio, f : X -+ IR uma função dada e e E IR também
dado. Relacione as imagens das funções f e f + e. Mais precisamente, se Y = Im (!),
prove que Im (J +e)= Y + e, onde Y + e denota o conjunto

Y + e = {y + e; y E Y}.
2.6. * Sejam X e IR um conjunto não vazio, J : X -+ IR uma função dada e e E IR*também
dado. Relacione as imagens das funções f e cf. Mais precisamente, se Y = Im (!), prove
que Im (e/)= cY, onde cY denota o conjunto

cY = {cy; y E Y}.

2. 7. Motivados pela forma canônica do trinômio de segundo grau ax 2 + bx + e (cf. lema 15),

~i
diremos doravante que

J(x) = a, [(x+ .!:.)2


2a
-
4a2
(1.5)

é a forma canônica da função quadrática J (x) = ax 2 + bx + e. V tilize tal formacanônica


para dar uma outra demonstração da proposição 15.

33
a.r2 + b.r + e uma [unção quadrática tal que ti > O, e x 1 <
2.'-'. Sl'J,,m .f ( r) X2 as raízes de
J(:r) = O ProYc os ~cguintcs itens:

(n) e a > O, então f(.r) <O<=>x E (.r1, x2).


(b) e a< O, então J(x) <O<=>.r r/:.[x1,x2].

2.9. eja /(.r.) = a.1;2 + bx + e uma função quadrática. Se existe um real x 0 tal que af (xo) < O,
prove que~> O e :t:0 E (x 1,x 2), onde x 1 < x2 são as raízes da equação f(x) = O.

2.10. Dentre todos os retângulos de mesmo perímetro, prove que o de maior área é o quadrado.

2.11. A seção reta de um túnel tem o formato de um semicírculo de raio Sm, e o túnel está
dividido em duas faixas de trânsito, de sentidos contrários, separadas por um canteiro
muito estreito. Os caminhões de uma companhia de transportes têm que atravessar o
túnel para levar mercadorias de uma cidade a outra. Se o comprimento máximo permitido
de um caminhão é 18m, quais devem ser sua largura e altura a fim de que a companhia
transporte o máximo possível de carga em cada caminhão?

2.12. Calcule o valor máximo da função f : lR ➔ JR,dada por f (x) = ;1~i


·
2.13. Sejam o 1 < a2 < · · · < O'.nreais dados e f : lR ➔ lR a função dada por

Prove que f assume um valor mínimo e calcule tal valor mínimo em função de a 1 , a 2 , ... , an.

2.14. Em cada um dos itens a seguir, use a desigualdade entre as médias para calcular o valor
máximo da função dada:

(a) f: IR-, lR dada por J(x) = 2xf+ 3 ·


2
(b) f: IR-+ lRdada por J(x) = x::!~+ 2 .

2.15. Em c;;ufa um dor,;it.cms a Hcguir, use a desigualdade entre as médias aritmética e geométrica
1>ara c&.lc11lar
o valor rnf11irno ela função dada:

( r+JO)(J+2)
(a) f : (O,+oo) ➔ lR dadA.por f (.r) ,,• 11
.,.:J
(h) f ; (O,-i oo) -; lR dada t)(Jr f (.r) :;:r.;:;;.
2.16. Jf_,numt,,, <>Hv:ilu1c•i,;J'l'aÍH d,, k pnrn os q11niHn l'unçno f : IR\ {-1} ➔ JR, dada pm
J(;.,)- 4 "'J;;;11.., f.c•J1l111
c·o1110 i111111.v•11111 rot,11 111e11oi:i 11m intcrvnlo da formn (-L, L).
1.3 Composição de funções
Dadas as funções f : X --+ Y e g : Y --+ z temos, em úliima análise, regras bem definidu
para, partindo de x E X via J, obter y = J(x) E Y e, via g, obter z = g(y) E Z. Parece, então,
razoável que possamos formar uma função que nos permita sair de X diretamente para Z, com
o auxílio de J e g. Este é de fato o caso, e a função resultante é denominada a função composta
de f e g, de acordo com a definição a seguir.

DEFINIÇÃO 1.27.
Dadas as funções J: X--+ Y e g: Y--+ Z, a função composta de Jeg (nessa ordem
função g o J : X --+ Z, definida, para cada x E X, por

(g o f)(x) = g(J(x)).
--------------~----- -------------~
Grosso modo, a definição acima significa que, para encontrarmos a imagem de x E X por
g o f, basLa encontrarmos a imagem de J(x) E Y por g. É fácil verificar que g o J, como definida
acima, é de fato uma função. Observe também que, para formarmos a composta de J e g,
devemos ter o domínio de g igual ao contradomínio de J. Vejamos alguns exemplos.

EXEMPLO l. 28.
Se f : X --+ Y é uma função arbitrária e ldx : X --+ X e ldy : Y --+ Y são, respectivamente,
as funções identidade de X e Y, então

f o ldx = f e ldv o f = f.

Verifiquemos a igualdade f o ldx = J, sendo a verificação da outra igualdade totalmente


análoga. Para tanto, basta notarmos que f o ldx é uma função de X em Y tal que, para todo
X E X,
(! o ldx )(x) = J( ldx(x)) = f (x).

EXEMPLO 1.29.
Considere as funções /, g : IR--+ IR, dadas por J(x) = x 2 e g(x) = z2~1. Temos g o/ e/ o
funções de lR.em lR.,com
1 1 1
(g o J)(x) = g(f(x)) = /(x) 2 +1 ------
(x2)2 + 1 x4 +1
e 2
1 ) 1
(! o g)(x) = f(g(x)) = g(x)2 = ( x2 +1 = x4 + 2x2 + 1•

35
O exemplo anterior mostra algo interessante: podemos ter g o f =/-f o g. Bem entendido,
pode mesmo acontecer que possamos formar g o f, mas não f o g (ou vice-versa); basta termos,
por exemplo, f: X ➔ Y e g: Y ➔ Z, com X=/- Z. Contudo, ainda que possamos formar ambas
as funções foge g o f, o exemplo mostra que podemos ter g o f =I-f o g.

EXEMPLO 1.30.
Sejam f, g : (O,+oo) ➔ (O,+oo) as funções tais que

f (X) = x:; 1 e (/ o g) (X) = X ; 2.

Encontre a expressão da função g.

SOLUÇÃO.
Segue da definição de função composta que

x+2 g(x) 2 +1
-3- = (J o g)(x) = J(g(x)) = 3g(x)2 ,

d e mo do que g(x)2+1
g(x) 2 = x + 2 ou, am
. d a,

g(x) 2 + 1 = (x + 2)g(x) 2 .

Olhando essa expressão como uma equação do primeiro grau em g(x) 2 , obtemos g(x) 2 = x:i
e, daí, g(x) = ± ✓;+ 1 , para cada x > O. Mas, como g deve ter imagem não negativa, devemos
- ter g(x) = ✓;+l' para todo x > O.

Apesar de não ser comutativa, a operação de composição de funções é associativa, conforme


ensina o resultado a seguir.

PROPOSIÇÃO 1.31.
Dadasfunções / : X ➔ Y, g : Y ➔ Z e h : Z ➔ W, temos

h o (g o f) = (h o g) o j.

PHOVA.
Veja primeiro que ambas h o (g o f) e (h o g) o f são funções de X em l-V. Portanto, para tais
funções Hercrn iguais, é suficiente que elas associem, a cada x E A, um mesmo elemento de iv.

36
Para ver isto, basta notar que
(h o (g o J))(x) = h((g o J)(x)) = h(g(J(x)))
= (h o g)(J(x)) = ((h o g) o J)(x).

A proposição anterior é muito importante. na medida em que nos assegura que, se tivermos
funções f, g e h e pudermos compô-las (nessa ordem), podemos denotar a função composta
simplesmente por h o g o f, não nos preocupando com qual composição efetuar primeiro. É
também claro que vale uma observação análoga para mais de três funções.
Como caso particular da discus"ão do parágrafo anterior. sejam dados n E N e uma função
J X ➔ X Doravante, escre\·eremos JCn) para denotar a composta 3
J'">= f o f o • •o f. (1.6)

a qual também é uma função de X em 1 m ~mo.

EXC\1PLO 1.32.
SeJn J: IR\ {-1, l} ➔ IR\ {-1, 1} n funç.ãodada por J(x) = ~~;.Para cada n EN, encontre
a e.xpr~são qm• define a fuuçiio J<n).

Sot l'Ç1\0.
\'PJH pri11H\iroque j<11> : lR \ {- l 1 1} ➔ \ {-1, l}. Agora,

2
1- f (x) 1- 1-x
/ >(.r) =(/o J)(.r) = J(J(x)) = -- = l+x = x
1 + /(x) 1+ !~:
hto é. J<2> = Id,, n função identidade de X= R \ {-1, 1}. Segue daí que

J<3J = f o j<2 l = f o Idx =f e / 4


> = f o J<3> = f o f = Idx.
Em geral. ~e já tivermo mostrado que J<2k-l) = f e JC2k) = ldx, teremos

J<2k+1) =f o j<2k) =f o ldx =f


e
j<2k+2) =f o J'2k+1) = f of = ldx.

Logo, segue por indução que j<11> = J quando n for ímpar e J<n)= Idx quando n for par.

3Apartir do Capítulo 3. / n) também denotará a n-ésima derivada de uma função n vezes derivável. Ambas
as notações sã.o padrão, e o contex1:o encarregar-se-á de evitar potenciais confusões.

37
Dada uma função f : X ➔ Y, já vimos exemplos que mostram que nem sempre a imagem de
f é igual ao contradomínio Y. Por outro lado, também podemos ter dois elementos distintos do
domínio X com uma mesma imagem; para um exemplo, considere a função quadrática f(x) = x 2 :
para todo x E R, temos J(x) = x 2 = (-x) 2 = J(-x). A próxima definição empresta nomes
especiais às funções cujos contradonúnios coincidem com suas imagens, ou que associam imagens
distintas a elementos distintos do domínio.

DEFINIÇÃO 1.33.
Uma função f : X ➔ Y é dita:

(a) Injetora, ou injetiva, ou uma injeção, se, para todo y E Y. existir no máximo um .l~

tal que J(x) = y.

(b) Sobrejetora, ou sobrejetiva, ou uma sobrejeção. se sua imagem for todo o conju
Y, i.e., se, para todo y E Y, existir pelo menos um .r E X tal que y = J(.r).

{c) Bijetora, Oli bijetiva, ou uma bijeção. se for ao mesmo tempo injetora e sobrejetorn.

Um modo eficiente de verificar se uma função f X ~ Y é injetora é verificar se a implicação

(1.7)

é satisfeita, para todos xi, X2 E X. Da mesma forma, para garantirmos que f é sobrejetora,
_devemos ser capazes de, para cada y E Y. obter pelo menos uma solução x E X para a equação
f(x) = y. Vejamos alguns exemplos.

EXEMPLO 1.34.
Se X e Rê um conjunto não vazio e / : X ➔ X é uma função tal que J(f(x)) = x, para
todo x E X, mostre que / é bijetiva.

PROVA.
Sejam xi e X2 números reais tais que f (xi) = f(x 2 ). De acordo com (1.7), para mostrarmos
que f é injetiva é suficiente provar. que x1 = x 2 . Para tanto, observe que f (xi) = f (x2 ) implica
f (f(xi)) = J(J(x2)) e, daí, em Xi = x2, pelas hipóteses sobre f.
A sobrejetividade de J é imediata: fixado y E X e tomando x = J(y) E X, temos J(x) ==
J(J(y)) = y, de sorte que y E Im (!).

38
Segue, em particular, do exemplo anterior que a função de proporcionalidade inversa (d.
exemplo 13) é uma bijeção de R \ {O} em si mesmo.

EXEMPLO 1.35.
Seja/ : (O,1] ➔ [O.1] uma função sobrejetora, tal que 1/(xi) - f (x2)I < lx1 - x2l, para
Xi, X2 E (O.l]. Prove que há somente duas possibilidades: ou J(x) = x, para todo x E (O,
f(x) = 1 - x, para todo x E (O,1].
___ .___._._::__ ___ ~----~~-----~------~Da

PROVA.
Sejam a, b E [O,1] tais que /(a)= O e J(b) = 1 (tais a e b existem por estarmos supondo que
f é sobrejetora). Então, temos por hipótese que

1 = 11- OI= IJ(b) - /(a)I < lb - ai < 1,

de modo que lb - ai = 1. ~las, os únicos a, b E [O,1) tais que lb - ai = 1 são a = O e b = 11 ou


vice-versa. Suponha que a = O e b = 1 (o outro caso pode ser analisado de modo análogo), e
lome e E (O,1) arbitrário. Então, segue da desigualdade triangular {A.11) e da hipótese sobre f
que

1 = lf{l) - f(O)I ~ 1/(1) - f(c)I + IJ(c) - f(O)I


< 11- cl + lc - OI= (1 - e)+ e= 1,
de modo que deve ser lf(c) - f(O)I = lc - OI ~Ias. como e, J(c) > O, segue que f(c) = e. Por
fim, a arbitrariedade do e E (O.1) escolhido garante que f(x) = x, para todo x E (O,l].

A proposição a seguir ensina como se comportam funções injetoras, sobrejetoras e bijetoras


cm relação à composição.

PROPOSIÇÃO 1.36.
Sejam f : X ➔ Y e g : Y ➔ Z funções dadas. Então:

(a) g o f injetora ⇒ f injetora, mas a recíproca nem sempre é verdadeira.

(b) g o J sobrejetora ::::}g sobrejetora, mas a recíproca nem sempre é verdadeira.

(c) g, f injetoras ⇒ g o f injetora.

(d) g, f sobrejetoras ⇒ g o f sobrejetora.

(e) g, f bijetoras ⇒ g o f bijetora.


--~~-~--~-~~

39
, 11 1 1 1 1 1 , 1 ri '\1,·111 "

PRO\\.
(a) Suponha que g o f é injetora. Então, para x 1 e x 2 em X, temos que

J(xi) = f(x2) ⇒ g(f (xi)) = g(J(x2))


⇒ (g o J)(xi) = (g o J)(x2)

de sorte que f também é injetora.


Temos, agora, que dar um exemplo no qual f seja injetora mas g o f não o seja. Para tanto,
basta tomarmos X= Y = Z = IR, f(x) = x e g(x) = x 2.

(b) Suponha que g o f é sobrejetora. Então, dado z E Z, a sobrejetividade de g o f garante a


exi~tência de pelo menos um x E X tal que z = (g o J)(x). las, daí, z = g(J(x)), de sorte que
g também é sobrejetora.
Para o exemplo necessário à segunda parte, tomemos X= Y = Z = IR,g(x) = x e J(x) = x 2.
Então, g o f não é sobrejetora, apesar de g o ser

(c) Suponha que f e g são injetoras, e tome x 1 , x 2 E X tais que (g o J)( x 1 )


Utilizando sucessivamente as injetividades de g e f, temos

(g o J)(x1) = (g o J)(x2) ⇒ g(f(x1)) = g(f (x2))


⇒ J(xi) = J(x2)

de modo que g o J também é injetora.

(d) Suponha que f e g são sobrejetoras. Para z E Z, a sobrejetividade de g garante a existência


de y E Y tal que z = g(y). Por outro lado, a sobrejetividade de f assegura a existência de x E X
tal que J(x) = y. Então, temos

.
(g o J)(x) = g(J(x)) = g(y) = z,

de modo que g o f também é sobrejetora.

10
(e) A partir dos itens (c) e (d). temos

g e f bijetoras =} g e f injetoras e sobrejetoras


=} g o f injetora e obrejetora
⇒ g o f bijetora.

Revisitemos o exemplo 34 à luz da proposição anterior.

EXEMPLO 1.37.
Se X é um conjunto não vazio e f: X ➔ X é uma função tal que f o f = Idx, enQMJ
bijetora.

SOLUÇÃO.
De fato, como a função identidade Idx • X-+ X é uma bijeção, aplicando os itens (a) e (b)
da proposição anterior à igualdade f o f = Idx. concluímos que J é injetora e sobrejetora, logo
bijelora.

Proble111as - Seção 1.3


3.1. Sejam f e g funçõe::,reais de uma variável real, dadas por f(x) = x - ~ e g(x) = x 2 - ¼-
Encontre o conjunto-i:,olução da mequação l(g o J)(x)I > (g o J)(x).

3.2. Sejam J e g funções reai!:,de uma variável real, tais que f(x) = 2x+ 7 e (Jog)(x) = x 2 -2x+3
. Encontre a expre ão que define a função g.

3.3. Sejam J,g: IR ➔ IR funções tais que g(x) = 2x - 3 e(! o g)(x) = 2x2 - 4x + 1. Encontre
a expre ão que define a função J.

3.4. Sejam f e g as funções reais de uma variável real dadas por J(x) = ax + b, g(x) =ex+ d,
com ac -::/:,O. ~Iostre que
f og =g o f <=>(a - l)d = (e - l)b.

3.5. Seja f : IR ➔ IR a função definida por


x+a se X -1..-b
f(x) ={ x+b' r
-1, sex=-b

Se J(f (x)) = x para todo x real, calcule os possíveis valores de b.

41
3.6. * Se I e IR é um intervalo e f : / -+ IR é uma função crescente ou decrescente, prove que
/ é injetiva.

3.7. Sejam/, J e IRintervalos e/:/-+ J, g: J-+ IR funções dadas. Se f e g forem crescentes


(resp. se f for crescente e g decrescente, ou vice-versa), prove que go f também é crescente
(resp. decrescente).

3.8. * Seja / e IR uma união de intervalos, simétrica em relação a O E IR. Dizemos que uma
função J : / -+ IR é par (resp. ímpar) se f(x) = J(-x) (resp. f(x) = - J(-x)), para
todo x E /. Prove que toda função f : IR -+ IR pode ser escrita, de uma única maneira,
como a soma de uma função par com uma função ímpar.

3.9. Seja/ : IR\ {O} -+ IR uma função tal que f (V = f(a) - J(b), para todos os reais não
nulos a e b. Prove que f é uma função par.

3.10. Seja J: IR-+ IR uma função ímpar. Decida se n função f o f ê par, ímpar, ou nem par nem
ímpar. ..
3.11. Seja g : IR -+ IR uma função ímpar, tal que g(.r) > O para x > O Mostre que existe uma
função f : IR-+ IR tal que g = J o f.

3.12. * Uma função f : IR -+ IR é periódica se existe um menor real positivo p, denominado


o período de f, tal que J(x + p) = f(x), para todo x E R. Dada uma função periódica
/ : IR -+ IR, de período p > O, faça os seguintes itens:

(a) Seja g: IR-+ IR também periódica de período p. Se J(x) = g(x) para todo x E (O,p),
prove que J = g.
(b) Dado a E IR\ {O},prove que a função g: IR-+ IR, dada por g(x) = f (ax), é periódica
de período ~.

3.13. Seja/ : R-+ R uma função tal que, para todo real x, tenhamos J(lO + x) = f (10 - x) e
/(20 + x) = - f(20 - x). Prove que J é ímpar e encontre p > O tal que f(x + p) = J(x),
para todo x E R.

3.14. Seja f: R-+ (O,1) uma função-tal que, para um certo a E IR, tenhamos

1
f(x +a)=
2 + J J(x) - J(x) 2 ,

para todo x E IR. Prove que f (x + 2a) = J(x), para todo x E IR.

42
3.15. Dê um exemplo de uma função sobrejetora J: N ➔ N tal que, para todo n EN, o conjunto
{x EN; f(x) = n} seja infinito.

3.16. Sejam X um conjunto não yazio e/ : X ~ ..\" uma função dada. Dizemos que um elemento
Xo E X é um ponto fixo de J se f(x 0) = x 0. Se J e IR é um intervalo, prove que uma
função decrescente f : / ➔ / admite no máximo um ponto fixo.

3.17. *Se/ C Ré um interi;alo e f. g: / ➔ /são funções tais que/ é decrescente e g é crescente,


prove que existe no máximo um x 0 E / tal que f (x 0 ) = g(xo).

3.18. Ache todos os reais positiYos x para os quais J2 + /'x = :r·

43
1.4 Inversão de funções
Genericamente, dentre todas as funções f : X ➔ Y, o caso de uma bijeção é o melhor
possível. Realmente, nesse caso os elementos de X e Y estão em correspondência biunívoca, ou
seja, a cá.da elemento de X corresponde um único elemento de Y via f, e vice-versa. Quando
isso ocorre, podemos obter uma outra função g : Y ➔ X, simplesmente exigindo que

j(x) = y <=} g(y) = X.

Uma pergunta natural a esta altura é a seguinte: por que não podemos usar a declaração
acima para definir a g se J não for bijetiva? De um ponto de vista intuitivo, se f não fosse
sobrejetiva, existiria um elemento y de Y que não seria imagem por f de nenhum elemento de
X; assim, não teríamos uma maneira natural de definir g(y) a partir de f. Por outro lado, se J
não fosse injetiva, existiriam elementos distintos xi e x 2 em X com uma mesma imagem y E Y
via f; quando tentássemos definir g por meio de J, também não haveria maneira natural de
decidirmos qual, dentre Xi e x 2 , deveria ser igual a g(y).
Voltando ao caso em que J é bijetiva, não é difícil ver que g, definida como acima, é de fato
uma função, ademais tal que (g o J)(x) = x, para todo x E X, e (J o g)(y) = y, para todo
y E Y. De outro modo, temos g o f = ldx e f o g = ldy Reciprocamente, se J : X ➔ Y e
g : Y ➔ X são funções tais que g o f = Idx e f o g = ldy, então a proposição 36 garante que
f deve realmente ser uma bijeção, e o problema 4.1 garante que g ê a única função que satisfaz
tais igualdades de composição.
Resumimos a discussão anterior na definição a seguir.

D~
Seja / : X ➔ Y uma bijeção dada. A função inversa de f é a função g : Y ➔ X tal que,
J>II'&
~EX, 11E Y, temos
g(y) =X<=} y = f(x).

Daqui em diante, denotaremos a inversa de uma bijeção f : X ➔ Y por 1-1 : Y ➔ X.


Observe que o expoente -1 na notação de função inversa não tem nenhum significado aritmético;
ele simplesmente chama atenção para o fato de que 1- 1 faz o caminho inverso de J, i.e., aplica
Y em X em vez de X em Y, revertendo as setas das associações feitas por f.
Surge, agora, naturalmente, a questão de como calcular efetivamente a inversa de uma bijeção.
Um tal cálculo é, em geral, mais complicado do que o de funções compostas. Entretanto, para
funções reais de uma variável real J : X ➔ Y, podemos raciocinar da seguinte maneira: fixado
1/ E Y, como 1- 1 (y) = x se, e só se, f(x) = y, a fim de e>-.-plicitar
J- 1 (y) = x basta resolvermos,
para x E X, a equação J(x) = y. Vejamos, inicialmente, alguns exemplos relevantes.

44
EXEMPLO 1.39.
Sejam a e b reais dados, sendo a =f:.O, e considere a função afim / :
f(x) = ax + b. Mostre que fé uma bijeção e calcule sua inversa.
~~~----~---

PROVA.
ote inicialmente que

y-b
J(x) = y {==}ax + b = y {==}x = --.a
Esses cálculos mostram que, para cada y E IR,existe um único x E 1R(x = y:b) tal que f(x) = y;
portanto, J é sobrejetiva (pois x existe) e injetiva (pois x ê único), logo, bijetiva. Por outro lado,
a definição de 1- 1 exige que tal valor de x deve ser exatamente igual a J- 1(y), de maneira que

EXEMPLO 1.40.
Uma discussão análoga à do exemplo anterior garante que a inversa da função identidade
Iclv do conjunto X =f:.0 é ela mesma, i.e, que ( ldx )- 1 = ldx. No entanto, a inversa de uma
função pode ser ela mesma, sem que a função seja a identidade; um exemplo é fornecido pela
função de proporcionalidade inversa / : R \ {O}➔ 1R\ {O} (a qual já sabemos ser bijetiv.a).
fato, como
1 1
f(x) = y # - = y {::}X= -,
X y
argumentando como no exemplo anterior, concluímos que

-1/ '
1
/ \,7 /
X=-
y

e, daí, que 1- 1 = /.

os dois exemplos a seguir, assumimos sem demonstração a existência de raízes quadradasde


números reais não negativos. Justificaremos esse fato rigorosamente no teorema 9 e no exemplo
62.

45
e são reai dados, com a > O as propo iço 15 22 garant m qu a funçã
/(x) = ax 2 + bx + e, vi ta como funçã

bijeção. Calc"ulea expr ão d ua mv ra.


----- -- --------------~

D acordo com di u - qu pr
d u x mpl 3 , fim d
t
1- i : [- + ) ➔ [ - 2~ + ) d J, d v m fixar y E [- t , ) lcfc .l' E

[-~, + ) a quaçã J(x) = y qu ç- a.r.2 + b.r + - y - fcz -1 ( l vanclo


m conta qu a> O) ndiçã >
-
_.k. g r nt qu
2a

-b + Jb 2 - a(c- y)
- 2a
onde 6 = b2 - 4ac é di rimin n d J. L

11:'tJ:.42.
cuo particular do exemplo anterior consid r a fun ão f : [ , ) [O,+ ) dada
:;s z2. Temos que f é uma bijeção, tendo como inv r a a função raiz quadrada

1- 1 : [O,+ ) ----+ [O,+ )


X f-----► VI

Terminemo ta çao xpli i ando uma r laç- o útil n r op r ç d ompo ição


in~ rsão de funçõ

bij tor , ntão g o f : X ➔ Z • bij tora e

-1
PROVA.
Já sabemos, pelo item (e) da proposição 36, que g o f é bijetora. Por outro lado, COIDQ
(g o J)- 1 e J- 1 o g- 1 são ambas funções de Z em X, a fim de verificar que (g o J)- 1 = 1-1 o g- 1
é suficiente, pela unicidade da inversa (cf. problema 4.1), mostrar que
1
(/- o g- 1
) o (g o J) = Idx e (g o J) o (J- 1 o g- 1) = Idz.
Tal verificação é imediata e será deixada a cargo do leitor (cf. problema 4.2).

Problemas - Seção 1.4


4.1. * Seja f : X ➔ Y uma função dada.

(a) Se g: Y ➔ X é uma função tal que g o f = Idx e f o g = ldy, prove que fé uma
bijeção.
(b) Prove que existe no máximo uma função g: Y ➔ X tal que go f = Idx e f og = Idy.
4.2. * Complete a prova da proposição 43, mostrando, nas notações do enunciado da mesma,
que
(J- 1 o g- 1
) o (g o J) = Idx e (g o!) o (J- 1 o g- 1 ) = Idz.

4.3. Seja f : [½,+oo) ➔ [¾,+oo)


a função definida por f (x) = x2 - x + l. Mostre que fé uma
bijeção e obtenha a expressão para sua inversa.

4.4. Seja f : R \ {2} ➔ IR\ {3} a função definida por J(x) = 3:~ 25 . Mostre que fé uma bijeção
e obtenha a expressão para sua inversa.

4.5. Sejam a, b,e,d E R* e f : R \ {-~} ➔ IR a função dada por J(x) = Generalize :!!-
o problema anterior, mostrando que f define uma bijeção de IR \ {- ~} sobre IR \ {~}.
Ademais, conclua que, se d= -a, então 1- 1 = f.

4.6. * Sejam n um inteiro positivo e J : :" +oo) ➔ [O,+oo) a função dada por f(x) = xn.
Admitindo que Im(f) = [O,+oo) (fato que será demonstrado no teorema 9 e no exemplo
62), prove que f é uma bijeção crescente e obtenha a expressão para 1-1 , definindo a
função raiz n -ésima.

4. 7. Dê exemplo de uma bijeção f : R ➔ R tal que f + 1-1 e f - 1-1 também sejam bijeções.

4.8. * Sejam /, J e R intervalos e f : J ➔ J uma bijeção. Se J é crescente (resp. decrescente),


prove que 1- 1 : J ➔ 1 também é crescente (resp. decrescente).

47
< • \1 • 1 1 , 1 1 1 1 F 1•:x<J>1-:s

1.5 Gráficos de funções


Para o que segue, lembre-se de que, dados conjuntos não vazios X e Y, seu produto carte-
~iano é o conjunto
X x Y = {(x, y); x E X e y E Y}.

Quando X = Y = R, usualmente vemos X e Y como retas numeradas e X x Y como um plano,


munido de um sistema cartesiano de coordenadas fixado4 .
Temos a seguinte definição importante.

DEFINIÇÃO 1.44.
Dada uma função / : X ➔ Y, o gráfico de f é o subconjunto G1 do produto cartesiano
X x Y, definido por
G1 = {(x,y) E X x Y; y = f(x)}. (1.8)
~----------
Quando f : X ➔ R é uma função real de variável real, com X e R uma união finita
de intervalos (possivelmente X - R), o gráfico de f reveste-se de significativa importância
geométrica, uma vez que
G1 cXxYcRxR

e, como vimos acima, esse último conjunto pode ser identificado com um plano, munido com um
sistema cartesiano de coordenadas.
Por outro lado, nem todo subconjunto do plano (munido de um sistema cartesiano xOy) pode
ser visto como gráfico de uma função. De fato, suponha dada uma função real de uma variável

(-3, O) X

Figura 1.5: subconjuntos do plano que não são gráficos de funções.

4 J{ef1•1 í1111Js o Jt•ítol' H.OCapfLulo 6 de IJOI pnm umn revisão sobre sistemas cartesianos de coordenadas.

48
real f : X ➔ 1R,tal que X é uma união finita de intervalos. Se (x0 , Yo) E GJ, então :coE X,
pela definição de gy-áfico;por outro lado (e mais importante), fixado Xo E X, se A1(:co,111)e
A2(xo, Y2)são pontos sobre o gráfico de J, então, novamente pela definição de gráfico, temos

Yi = f (xo) = Y2,
de maneira que A1 = A2. Em resumo, para x 0 E IR, a reta vertical x = x 0 do sistema cartesiano
em questão intersecta o gráfico de f se. e somente se, x 0 E X; ademais, nesse caso tal reta
intersecta o gráfico em exatamente um ponto. Assim, o subconjunto C do plano cartesiano,
esboçado na Figura 1.5. não representa o gráfico de função alguma f : [-3, 3] ➔ IR, uma vez que
toda reta vertical paralela às retas tracejadas e situada na faixa cinza intersecta C em mais de
um ponto.
a direção positiva (e conforme ficará claro à medida que avançarmos nesta seção), a linha
contínua da Figura 1.6 é (cf. lá indicado) o gráfico de uma função f: [a,b) ➔ IR.

1
1

-:-- Y = Yo
1 1

x'o X

Figura 1.6: imagem x gráfico.

Ainda ne e cntido, há uma interpretação geométrica bastante simples para a imagem de


uma função real de uma variável real em termos de seu gráfico. Para exibi-la, marquemos, no
plano cartesiano da Figura 1.6, os pontos ~.., interseção do gráfico da função f : [a, b) ➔ R
com uma reta horizontal r, de ordenada y = Yo- (Há três desses pontos na Figura 1.6, cujas
ab cissas denotamos como x 0 , x~ e x~.) Seja (x, Yo) um ponto comum à reta e ao gráfico. Por
pertencer ao gráfico de f, o ponto (x, Yo) deve ser tal que x E [a, b) e f(x) = y0 . Reciprocamente,
dado um ponto (x, y0 ) no plano cartesiano, com x E [a, b), temos claramente (x, y0) E r; além
disso, se f (x) = y 0 , então também teremos (x, Yo) E G J. Assim, a reta horizontal de ordenada
Yo intersecta o gráfico exatamente quando Yo pertence à imagem de f. O raciocínio para uma
função genérica J : X ➔ IR, com X e IR, é inteiramente análogo e nos permite concluir que

49
< • , 1•1 1 t 1 < 1 1 F 1•:--:<,·rn-:s

.,..\nnagcm de f é precisamente o conJunto dos y0 E lR tais que a reta horizontal de ordenada y0


intersecta o gráfico de f.
Se I C IR é um intervalo, a monotonicidade de uma função f : / -+ IR também nos diz
muito sobre o comportamento de seu gráfico. Por exemplo, supondo que f seja crescente (re p.
decre cente) em I, concluímos que, à medida que a variável x aumenta em J, os valore f(x)
aumentam (resp. d~minuem) em IR, de maneira que o gráfico de f sobe {resp. desce).
Por outro lado {cf. Figura 1.7), se y0 E IRé o valor mínimo de f: J-+ IRe x 0 E I ê um ponto

Y = Yo

Figura 1.7: ponto de mínimo de J: /~IR.

de mínimo de f (cf. definição 23), então, para cada x E 1, o ponto (x, J(x)) não está abaixo
do ponto (x, y0). De outra forma, o gráfico de f está contido no semiplano superior fechado
determinado pela reta horizontal y = y0, tocando tal reta no ponto (x0, y0).
Observe que os conceitos de valor máximo e ponto de máximo de uma função J : / -+ lR
admitem interpretações geométricas análogas às discutidas no parágrafo anterior.
A seguir, reunimos alguns exemplos elementares, mas muito importantes, de gráficos de
funções.

EXEMPLO 1.45.
Seja f : R ➔ R a função constante e igual a e. O gráfico de f é o conjunto

G 1 = {(x,y); x e R e 11=e}= {(x,c); x E R},

i.e., o gráfico de / é a reta paralela ao eixo das abscissas e passando pelo ponto (O,e) do ei..xo
das ordenada (cf. Figura 1.8).

50
o

Figura 1. : gráfi o da funç-o ons an f(x) = e.

M L 1.46.
L mbr (cf. d finiç- 2) d qu a função id ntidad Id : IR ---+IR é tal que Ida(x) =x
par t do E . u gráfi o portanto, o onjunto

G1d = {(x y); x E y =X} = {(x, x)· X E }.

fá il d G m tri Eu lidi n ,. rificar qu o ponto da forma (x, x) e (-x, x)


itu do· br bi triz do ângulo formados pelos eix~
co r n d ndo qu qu 1 d form (x. x) p rt nc m ao primeiro ou terceiro quadrant .
P r nto o gráfico d fun - Id , r t da Figur 1.9 a qual é d nominada. a bissetriz dos
qu drant ímpar . Ob rv qu o onjunto do ponto da forma (x, -x) i.e., a bissetriz dos
quadmnt dada por / (x) = -x.

função modular a função f : IR ---+ dada por f(x) = lxl. Segue imediat,amente
d finição d módulo d um número real (cf. ub eção A.2.2) que

G1 ={(x lxl);xE }
= {(x lxl); x E IR+}U {(x lxl); x E -}
= {(x, x); X E IR+}u {(x, -x); X E ]R_}.
Como os ponto (x, -x) e (x x) ão imétrico em relação ao eixo das&Dlc:l8118a, o
função modular • obtido refl tindo, em relação ao eixo das a.bsci , a porção do
função Ida ituada no terceiro quadrante (cf. Figura 1.10).
----------------~·~~-_.....

1
<' \ 1• 1 1 1 1 1 > I F L' <:o•·s

Glda

(O,a) ---

Figura 1.9: gráfico da fun~ão identidade Ida.

1
1 /
/ .,.
1 /
1 /
(x,xiJ/
/
/

Figura 1.10: gráfico da função modular /(x) = lxl.

EXEMPLO 1.48.
Se f(x) = ax + b é uma função afim, então seu gráfico é o subconjunto do plano cartesiano
dado por
G1 = {(x,y);x,y ERey = ax + b}.
A geometria analítica básica {cf. seção A.4) garante que o gráfico de f é a reta de equação
y - ax - b = O, com coeficiente angular a e pas.5ando pelos pontos A(-~. O) e B = (O,b). A
Figura 1.11 esboça o gráfico de f no caso em que a. b > O.
--------------

Para o que segue, recorde (cf. problema 6.3.13 de [lOJ. por exemplo, ou. ainda. a discussão
da subseção A.4.3) que, dados um ponto F e uma reta d no plano. com F fJ..d. a parábola de

52
(;li\llCll"l>I 11\((>I" 1

y
G1

Figura 1.11: grâfico da função afim f(x) = ax + b, para a, b > O.

foco F e diretriz d (cf. Figura 1.12) é o lugar geométrico dos pontos P do plano tais que

PF = dist(P, d).

O eixo da parábola é a reta que passa por F e é perpendicular a d, e o vértice da parábola é


seu ponto V de interseção com o eixo.

_______________ .._____ d

Figura 1.12: parábola de foco F e diretriz d.

A seguir, mostraremos que o gráfico de toda função quadrática é uma parábola (cf. subseçfio
A.4.3). Mais precisamente, temos o seguinte resultado.

53
TEOREMA 1.49.
Parn a, b, e E JR,com a# O, o gráfico da função quadrática J(x) = ax 2 + bx + e é a parábola
de dxo {:r::= - 2:} e vértice V ( 0 , - -! ! ),
"aberta para cima" se a > O, e "aberta para baixo"
se a < O.

PRO\A.
De acordo com (A.46) - com € = 1 - e (A.48), procuremos x 0 , y 0 , k E IR tais que Yo # k e,
sendo F(x 0 , y0 ) e d a reta {y = k}, tenhamos

P E G1 <=} PF = dist(P,d).

Sendo P(x, y), segue da definição de gráfico que

P E Gf <=} y = ax 2 + bx + e;
por outro lado, a fórmula para a distância entre dois pontos do plano cartesiano (cf. proposição
55) garante que
P F = dist(P, d) <=} J(x - xo) 2 + (y - Yo)2 = IY - kl.

Portanto, queremos que

y = ax 2 + bx + e <=} (x - xo)2 + (y - Yo)2 = (y - k) 2


1 2 Xo Xõ + Yõ- k2
<=} y= ---.1: - --x + -----
2(yo - k) Yo-k 2(yo-k)

Para o que falta, basta resolvermos, em x 0 , y 0 e k, o sistema de equações

1 = a _ x0 = b Xõ 1( k)
2{yo- k) ' Yo - k ' 2(y0 - k) + 2 Yo + ~ = e,
o que é imediato: dividindo membro a membro as duas primeiras igualdades, obtemos x 0 = 0 ; -!
em seguida, substituindo a primeira igualdade e o valor de x 0 na terceira igualdade, segue que

por fim, resolvendo o sistema


1 ~
Yo - k = -2a ' Yo + k = - 2a,
obthu-s,, /''i>
1 = !=.é
4"
ek = -~.,1tt

5-1
Finalmente, uma vez que o vértice V da parábola é a interseção da reta x = -/ã com o
gráfico, sua ordenada y,, é

y,, = a (-_!!__)
2a
2
+b (-.!!_)
2a
+e= - b..
4a

O próximo resultado estabelece uma importante relação entre os gráficos de uma bijeção e
de sua inversa. Em seguida, ilustramos tal resultado na construção dos gráficos de duas funções
importantes.

PROPOSIÇÃO 1.50.
Se os conjuntos não vazios I, J e IRsão uniões finitas de intervalos e f: J ➔ J é uma bijeção,
então os gráficos de f e f 1 são simétricos em relação à bissetriz dos quadrantes ímpares do plãno
cartesiano.

PROVA.
Fixe a E / e b E J. Pela definição de função inversa, temos que

b = f (a) <=>a= f- 1(b) <=>


(a, b) E GJ <=> (b,a) E G1-1.

i\Ias, como os pontos (a, b) e (b, a) são simétricos em relação à reta y =x (cf. Figura 1.13 e
problema 1, página 465), nada mais há a fazer.

EXEMPLO 1. 51.
Esboce o gráfico da função raiz quadrada

f: [O,+oo) ~ [O,+oo)
X lJ.....-t vÍX

OLCÇÃO.
o exemplo 42, vimos que fé a inversa da função g : [O,+oo) ➔ [O,+oo) dada por g(x) = xi.
Como já conhecemos o gráfico de g (graças ao teorema 49), segue da proposição anterior que o
gráfico de fé obtido como o simétrico do gráfico de g em relação à reta y = 1· (cf. Figura 1.14).

55
< • , 1•1 1 1 1 ( , 1 F 1 :\ <:rn-:s

Figura 1.13: simetria de (a, b) e (b,a) em relação a y = x.

y IX H X
2
I
I
I
I ,,'
I /
I /
I ,'
II / "
I /

Figura 1.14: gráfico de x H ..fx.

EXEMPLO 1.52.
Recorde que a função de proporcionalidade inversa é a função / : R \ {O}➔ R \ {O},tal que
/(x) =!,para todo x E R \ {O}.
De posse da discussão desenvolvida até o momento, podemos esboçar muito acuradamente
seu gráfico. De fato,/ é claramente decrescente em (O,+oo). Também como

/(-x) = _.!_= -~ = -/(x),


-X X
temos que/ 6 únpar (cf. problema 3.8, página 42), de sorte que, pelo problema 5.5, seu gráfico

56
é simétrico em relação à origem do plano cartesiano. Por outro lado, / ê a
e a proposição anterior garante que seu gráfico também é simétrico em relaçti)
quadrantes ímpares. Mostraremos na seção 3.6 que seu gráfico é emborcado
(O,+oo). Por fim, intuitivamente. J(x) = ~ se aproxima cada vez mais de zero à
aumenta (tal intuição será tornada precisa na seção 3.1).
Utilizando as observações acima e marcando, sobre o gráfico de f, os pontos auxiliares(n,
para 1 < n < 4 inteiro, chegamos ao esboço da Figura 1.15. A esse respeito, veja ~
problema 5.13.

o
X

"""
"
"""
"

Figura 1.15: gráfico da função /(x) = ½-

Problem~= - Seção 1.5

5.1. Se J : [2, + ) ➔ [l. +oo) é dada por J(x) = x 2 - 4x + 5, mostre que / é uma bijeção e,
em seguida, encontre os pontos comuns aos gráficos de f e 1-1 .

Para o próximo problema, dados um conjunto não vazio X e uma função f : X ➔ R,


dizemos que fé limitada se existe A1 > O tal que 1/(x)I ~ M, para todo x E X.

57
5.2. * Sejam I C IR um intervalo e f : I -+ IR uma função dada. Prove que, se f é limitada
então o gráfico de f está contido em uma faixa horizontal do plano cartesiano, delimitadé
por duas retas paralelas

5.3. * Se I e IR é um intervalo e f : J -+ I é uma função dada, mostre que os pontos fixos de


J são, precisamente, as abscissas dos pontos de interseção do gráfico de f com a bissetriz
dos quadrantes ímpares.

5.4. * Se I C IR é um intervalo e f 1 g : J -+ IR são funções dadas, explique como identificar os


pontos comuns aos gráficos de f e g, traçados em um mesmo sistema cartesiano.

Para o próximo problema, sugerimos ao leitor reler o enunciado do problema 3.8, página
42.

5.5. * Sejam I e IR uma união de intervalos, simétrica em relação a O E IR, e f : J -+ IR uma


função dada. Prove que:

(a) Se J é par, então GI é simétrico em relação ao eixo das ordenadas.


(b) Se fé ímpar, então G1 é simétrico em relação à origem.

5.6. Em cada um dos itens a seguir, esboce, num mesmo sistema cartesiano, os gráficos das
funções reais de uma variável real listadas:

(a) f1(x) = x2 , h(x) = x 4 e h(x) = x 3.


(b) fi(x) = x, h(x) = x 3 e !J(x) = x 5 .
5. 7. Esboce o gráfico da função f : IR -+ IR tal que f (x) = ~, para todo x E IR.

5.8. Esboce, com justificativa, o gráfico da função parte inteira, l J : IR-+ IR (cf. problema 1.9,
página 23).

Para o próximo problema sugerimos ao leitor reler o enunciado do problema 3.12, página
42.

5.9 . .- Faça os seguintes itens:

como construir o gráfico de f


(a) se f : IR -+ IR é periódica de período p > O, e:>..1)lique
conhecendo a porção do mesmo para O < x < p;

58
(b) use o item (a) para construir o gráfico da função parte fracionária, { } : R ➔ R (cl.
problema 1.10, página 23).

5.10. * Sejam J : lR➔ lR uma função dada e ai: O um real dado. Prove que o gráfico de:

(a) g(x) = J(x + a) é obtido transladando o gráfico de f de -a, paralelamente ao eixo


das abscissas.
(b) g(x) = f(x) + a é obtido transladando o gráfico de f de a, paralelamente ao eixo das
ordenadas.
(c) g(x) = - J(x) é obtido refletindo o gráfico de f ao longo do eixo das abscissas.
(d) g(x) = J(-x) é obtido refletindo o gráfico de f ao longo do eixo das ordenadas.
(e) g(x) = af(x) é obtido alongando5 o gráfico de f verticalmente do fator a, se a> O.
(f) g(x) = J(ax) é obtido alongando o gráfico de f horizontalmente do fator a, se a> O.

5.11. * Esboce o gráfico da função f: lR\ {2} ➔ lRdada por f(x) = 2 ~x·
5.12. Sejam I um intervalo da reta e f : I ➔ lR urna função dada. Que relação existe entre
os gráficos de f e da função g : I ➔ lR dada por g(x) = 1/(x)I? Utilize suas conclusões,
juntamente com o resultado do problema anterior, para esboçar os gráficos das funções
abaixo listadas:

(a) g(x) = lx 2 - 41, para x E lR.

(b) g(x) = lx!ii, para x E lR\ {-1}.

(c) g(x) = lx 2 - lx + 21+ 21, para x E lR.


Para o próximo problema, sugerimos ao leitor recordar o material da subseção A.4.3 sobre
hipérboles.

5.13. :t-.Iostreque o gráfico da função de proporcionalidade inversa é obtido pela rotação trigo-
nométrica da hipérbole de equação x 2 ,,-2= 2, do ângulo trigonométrico de i radiano.5.

50 leitor deve ter cuidado com o sentido em que a palavra. alongando é utiliza.da. aqui; para tanto, compare
oscasos0<a<lea>l.

59
< • \1·111 1< 1 1 Ft·:-.:cJ>1-:s

1.6 Funções trigonométricas


Esta seção recorda alguns fatos elementares sobre funções trigonométricas e seus gráfico , os
quais são importantes para desenvolvimentos posteriores. Para mais detalhes, referimos o leitor
ao Capítulo 7 de [10).
A função seno é a função sen : IR-+ IR, que associa a cada x E IR o eno de um arco de .r
radianos:
sen : IR ~ IR
x t-------tsen x

Analogamente, definimos a função cosseno por

cos : lR. ~ lR.


X 1-----t CO T

As propriedades básicas das funções seno e cosseno são eslabelecidru na propo içã.o a ~cgu1r,
para a qual o leitor pode achar útil recordar os enunciados do problema 3 e 3 12, pógma
página 42.

PROPOSIÇÃO 1.53.
As funções seno e cosseno têm como imagem o intervnlo (-1, l) e são periódicas ele pC'ríoclo
21r. Ademais, a função seno é ímpar e a função cosseno é par.

PROVA.
Segue imediatamente de nossas discussões anteriores, juntamente com o fato (cf. Capítulo 7
de [10] ou, ainda, problema 1, página 482) de que sen (-x) = -sen x eco (-x) = cos x.

De acordo com a proposição acima e a discussão contida no problema 3.12, página 42, a fim
de esboçarmos o gráfico da função seno, é suficiente fazê-lo no intervalo [-1r, 1r), copiando em
seguida essa porção do gráfico em cada um dos intervalos da forma [-1r + 2k1r,1r + 2k1r], onde
k E Z.
Por outro lado, uma vez que a função seno é ímpar, a fim de obtermos seu gráfico no intervalo
[-1r, 1r], é suficiente construí-lo no intervalo [O,1r); feito isto. ao refleti-lo em torno da origem do
plano cartesiano obtemos (de acordo com o problema 3.8. página 42, uma vez que a função seno
é ímpar) o gráfico no intervalo [-1r, 1r).
Provaremos na seção 2.3 (cf. exemplo 37) que o gráfico da função seno é uma curva contínua,
i.e., sem interrupções. Por outro lado, na seção 3.6 mostraremos {cf. item (c) do exemplo 62)

60
que tal gráfico é" emborcado paro baixo" no intervalo [O,1r).Assumindo por enquanto a validade
dessas afirmações, podemos finalmente esboçar o grãfico da função seno.

EXEMPLO 1.54.
Reunindo as informações de que dispomos até o momento sobre a função seno no
[O,1r),juntamente com o fato de que a mesma é crescente em [O,i] 1 decrescente em {f.
e satisfaz sen (1r - x) = sen x, a fim de esboçarmos razoavelmente o gráfico da mesma DeaN
intervalo basta tabelarmos alguns valores de senx para x E [O,i], o que fazemos a seguir:

o 7r
6
7r
4
7r
3
7r
2
o 1/2 ../2/2 /3/2 1

De posse das informações acima e utilizando a periodicidade do seno, obtemos a linha con-
tínua da Figura 1.16, primeiro no intervalo [O,1r]e, em seguida, no intervalo [-1r, 1r).
Observemos agora que, a partir das fórmulas de adição de arcos (cf. proposição 71), temos

cos x = sen ( x + ; ) .
Portanto, o item (a) do problema 5.10, página 59, garante que, uma vez esboçado o gráfico
da função seno, obtemos o esboço correspondente ao gráfico da função cosseno transladando o
gráfico da função seno de - ~ paralelamente ao eixo das abscissas. Na Figura 1.16, o gráfico
da função cosseno no intervalo [-1r, 1r]é representado pela curva tracejada, situada na faixa do
plano cartesiano entre as retas y = -1 e y = 1.

- ...1

-7r X

--------------

Figura 1.16: gráfico das funções seno e cosseno.

Vejamos, agora, um exemplo relevante de aplicação das fórmulas de adição de arcos (cf.
proposição 7.18 de (10)), o qual nos diz como proceder para esboçar o gráfico de uma soma de
múltiplos constantes das funções seno e cosseno.

61
< • \ 1•1 1 1 1 < 1 1 F L'- <JH-:s

b
Figura 1.17: definindo o ângulo a.

EXEMPLO 1.55.
Dados reais positivos a e b, seja f : IR➔ lR a função dada por

f(:r,,)= acosx + bsenx.

Escrevendo

acosx + bsenx = Ja 2 + b2 (
J a2 ª+ b2cosx +
✓a2
b
+ b2
senx),

observemos que

Portanto, o ponto P ( Ja¼+b'J.,✓)+b2 ) pertence à porção do ciclo trigonométrico r situada no


primeiro quadrante e, daí (cf. Figura 1.17), existe um real o E (O,~2 ) tal que coso = ✓a2+b2
f e
seno = J}+CJ. Assim, temos, das fórmulas de adição de arcos, que

f (x) = a cos x + b sen x


= Ja 2 + b2(cosacosx + senosenx)
= Ja 2 + b2cos(x - o).

Em particular, segue de I cos(x - o)I < 1 que

lf(x)I = Ja 2 + b2 I cos(x - a)I < Ja 2 + b2 ,


e não é difícil provar, a partir daí, que a imagem de f é precisamente o intervalo [-e, e], onde
e• .Ja2+ b2(veja o problema 6.3).
-------~~~----~~---~----
Voltemo-nos, por fim, ao estudo do gráfico da função tangente. Como tg x está definida
se, e só se, x =I=~ + k1r, para k E Z, o domínio (maximal) da função tangente é o conjunto

62
1-'1 '\I, 1 >I ..._ 1 Hll ,1 l'\l >\li I HI< \-._ 1

D = 1R\ { i + br; k E Z}. Portanto, a função desejada é

tg : D -4 1R
X f-----1 tg X

e segue de (A.51) que tg (x + 7r)= tgx, para todo x E D. Por outro lado, é imediato verificar
que não existe um real O < p < 1f tal que tg (x + p) = tg x, para todo x E D, de sorte que
a função tangente é periódica de período 7f. Ademais, uma vez que D é um subconjunto de 1R
simétrico em relação a O e

sen(-x) -senx
tg ()-x = --- = --- = -tgx
cos( -x) cos x

para todo x E D, concluímos que a função tangente também é ímpar.


Provaremos na seção 2.3 (cf. problema 3.5, pâgina 101) que o gráfico da função tangente,
restrita ao intervalo ( -~, i), é uma curva contínua, i.e., sem interrupções. Por outro lado, na
seção 3.6 mostraremos (cf. item (d) do exemplo 62) que tal gráfico é "emborcado para cimà' no
intervalo [ O,~) . Assumindo por enquanto a validade dessas afirmações, e de posse da discussão
y

7f 7f 7f 7f
-2 ,-4, X
4 12
1 1 1
1 1
1 1
1 1
1 1
1 1
1 1
1 1
1 1

Figura 1.18: gráfir,n.i,(iafunção tangente.

dos parágrafos anteriores, podemos esboçar o gráfico da função tangente no intervalo ( -1, ~)
de forma análoga ao feito no exemplo 54, obtendo aproximadamente a Figura 1.18.
Por outro lado, uma vez esboçado o gráfico no intervalo (-J, J), a periodicidade da função
tangente nos permite esboçá-lo em todos os intervalos da forma ( -J + k1f,J + k7f), com k E Z:
basta transladar o gráfico no ( -J, J) de k7f unidades paralelamente ao eixo das abscissas, para
todo k E Z.

63
(. \l'l 11 1 ( 1 1 Ft':\(_'01·:S

Problemas - Seção 1.6


6.1. * O propósito deste problema é introduzir as funções trigonométricas inversas. Para.
tanto, faça os itens a seguir:

(a) Mostre que a restrição da função seno ao intervalo [-;, ~] define uma bijeção cres-
cente (q~e ainda denotaremos por sen, sempre que não houver perigo de confusão)
sen : [-~, ~] ➔ (-1, l]; sua inversa é a função arco-seno arcsen . (-1, 1) ➔
[- ~, ~] , a qual também é crescente. Em seguida, calcule os valore· de arc~en ½,
arcsen 1 e arcsen ( -1).
(b) Mostre que a restrição da função cosseno ao intervalo (O,1r)define uma bijeção clccrc~-
cente (que ainda denotaremos por cos, sempre que não houver perigo de confusão)
cos: (O,1r]➔ [-1, l]; sua inversa é a função arco-cosseno arccos: [-1, 1) ➔ [O,1r], a
qual também é decrescente. Em seguida. calcul arccos ½,arcscn 1 e arcscn (-1).
(c) Mostre que a restrição da função tangente ao intervalo (-i, ~) define uma biJcção
crescente (que ainda denotaremos por tg I emprc que não houver perigo de confusao)
tg : (- ~, ~) ➔ JR;sua inversa é a funçiio arco-tangente arctg : 1R➔ (- i, í), a
qual também ê crescente. Em seguidn, calcule nrctg 1. nrctg J3 e arctg 73.
=~.
Para o item (a) do próximo problema. recorde que a cotangente
número real ctg x, definido por ctg .-z:=
de x f:. k1r (k E Z) é o

6.2. Em cada um dos itens abaixo, esboce o gráfico da função dada.

(a) f: 1R\ {k1r;k E Z} ➔ 1Rtal que f(x) = ctgx.


(b) f: (-1, l] ➔ [-~, ~] tal que f(x) = arcsenx.
(c) f: (-1, l] ➔ (O,1r]tal que f(x) = arccosx.
(d) f: 1R➔ [-~, ~] tal que f(x) = arctg x.
6.3. * Sejam a e b reais não ambos nulos, e f : R ➔ R a função dada por

f(x) = a cos x + bsenx.


(a) Obtenha, com justificativa, o conjunto 1m (/).
{b) Prove que f é periódica de período 2Tt.
(e) Quando a= 1 e b = 2, esboce os gráficos de feda função seno em um mesmo sistema
cartesiano.

64
6.4. * Sejam a e breais não ambos nulos, e J: 1R➔ 1Ra função dada por

f(x) = acos(.Xx) + bsen (.Xx),


onde À é um real não nulo. Mostre que J é periódica e calcule seu período.
6.5. Seja f: 1R➔ IR a função dada por f(x) = 2senx + cos{2x).
{a) Calcule os valores máximo e mínimo de f, bem como os números reais x para os quais
/ assume tais valores.
(b) Mostre que f é periódica de período 21r.
(e) Esboce o gráfico de f.

6.6. Calcule o número de soluções reais da equação senx = to·

6.7. Encontre o valor máximo assumido pela função f : [-1, l] ➔ 1Rdefinida por f(x) -
3x +4J5- x2.

6.8. Seja o um número irracional dado. Prove que a função f : 1R➔ lR, definida por f(x) =
cosx + cos(ox), para x E JR,não é periódica .

65
SEQUÊNCIAS E CONTI-
NUIDADE
(' \l 'Í 1 1 1.<> '2 SEQt·f::--:cl.\S E C'O:\Tl:\l"ID.-\DE

Neste capítulo, após discutir brevemente a completudo do conjunto dos números reais. apre-
sentamos a importante noção de limite de uma sequência (infinita). Em seguida, formalizamos o
conceito de função contínua, pensado intuitivamente como uma função cujo grãfico é uma curva
sem interrupções. Como corolário de nossas discussões, apresentamos critérios suficientes para
garantir a continuidade de uma função e, dentre outros resultados importantes. mostramos que
toda função contínua possui a propriedade do valor intermediário.

2.1 Supremo e ínfimo


Começamos este capítulo discutindo rigorosamente a completude de R. Parn tanto. ptcc1Hn-
mos de alguns conceitos preliminares.
Um conjunto não vazio X e lRé limitado superiormente e existir um número rcnl \/ tnl
que
X e (-co, .\/].

Nesse caso, dizemos que 1'1 é uma cota superior pnrn X Analogamente, um co11Jtllllonõo
vazio X e lR é limitado inferiormente se e.x1stirum numero real m tnl que

X e [m,+ )

e, sendo esse o caso, dizemos que m é uma cota inferior para X Por fim, um conjunt.o não
vazio X e lR é limitado se X for simultaneamente limitado superior e inferiormente.
Dito de outra forma, um conjunto não vazio X e IRé limitado superiormente (resp limitado
inferiormente, limitado) se existir um real positi\·o a tal que

x < a (resp. x > -a. -a < x < a), 'v x E X.

O exemplo a seguir analisa alguns subconjuntos não vazios de ll em relação à noção de ser
limitado superior e/ou inferiormente.

EXEMPLOS 2.1.
(a) O conjunto N dos naturais é ilimitado (i.e., não limitado) superiormente. De fato, para
todo M > O,existem naturais maiores que lU.

(b) O conjunto X= {1, ½,½,¾,... }é limitado superior e inferiormente. Por exemplo, 1 é uma
cota superiore O uma cota inferior para X.

68
(c) Intenralos limitados {cf. definição 6 e discussão subsequente) são conjun
sentido da discussão acima. Realmente. por definição, intervalos limitados são
tos Ide IR tais que J = (a. b), I = (a. b], I = [a. b) ou I = [a, b], para certos a,b E
um qualquer desses casos, temos, claramente, Ilimitado superior e inferiormente.

EXEMPLO 2.2.
Se um conjunto não vazio X e lR é limitado superiormente, então o subconjunto Y de R dado
por Y = {-x; x E X} é limitado inferiormente, e reciprocamente. Realmente, se X é limitado
superiormente e a > Oé tal que x < a, para todo x E X, então -x > -a, para todo x E X; mas,
como y = -x é um elemento típico do conjunto Y, concluímos que y > -a, para todo y E Y,
de sorte que Y é limitado inferiormente. Também. a ,-alidade da implicação recíproca (i.e., Y
limitado inferiormente implica X limitado superiormente) pode ser verificada de modo análogo.

Fixe um conJunlo X C IR.não ,az10 e limitado superiormente. Se Aí E lRé uma cota superior
pata X, então X e ( . J{ ~o entanto. pode ocorrer que exista Aí'< Aí que ainda seja uma
cola superior para X, 1.e., tal que X e ( .\/']. Por exemplo, se X= (1, 2), então Aí= 3 é
c:ota ~upe1ior para X. mas. tomando J\/' = ~1 temo!>que 1\/ 1 < 1\1 e Aí' ainda é cota superior
para X Po1 outro lado. se .r ( X. cntiio nenhum real JH' <. :r é cota superior para X, uma vez
que .r E X\ (- , AI' e. dai, X rt.(- H'J.
O íato íundamental acerca do conJtmto IR dos números reais é que todo conjunto X e IR,
nno , nzio e lnmtado superiormente, po~m uma cola superior mínima. Esse é o conteúdo do
axion1a da con1pletude de IR. conforme enunciado a seguir.

.\ \.10\I 2.3.
\
e.\ e IR~ não ,11zioe limitado superiormente. então X possui uma menor cota superior.

eX e
1Ré não \'azio e limitado superiormente, e 111é a menor cota superior de X I dizemos
que 1'1 é o supremo de X, e denotamos

Aí= supX.

EXE~1PLOS 2.4.
(a) Jã sabemos que o conjunto X = {1. ½,½,¼,... } é limitado superiormente, tendo 1
cota superior. Por outro lado. como 1 E X. nenhum número real menor que 1 ~
cota superior para X e. daí, sup X = 1.

69
(b) Se X = (1, 2), então 2 é cota superior para X, mas nenhum número real menor que 2 o
é. De fato, se 1 < a < 2, então o número 1 + ; ainda é maior que 1 e menor que 2, de
maneira que 1 + ; E X. Mas, como a < 1 + ~ E X, o número a não pode ser cota superior
para X. Então, segue que sup X = 2.

O exemplo anterior mostra, ainda, que podemos ter sup X E X ou sup X f/:.X. No item (a),
temos sup X = 1 E X, urna vez que 1 é o maior elemento de X. Por outro lado, no item (b).
temos sup X = 2 ff. X, refletindo o fato de que X não possui um maior elemento.
Se Y e IR é não vazio e limitado inferiormente, urna consequencia simples do exemplo 2 e do
axioma 3 (cf. problema 1.4), garante que Y admite uma maior cota inferior 1111 n q\ml ~cm
denominada o ínfimo de X. Denotamos, nesse caso.

m = inf X.

EXEMPLO 2.5.
Se X = {1, ½,½,¼,... }, então inf X = O. Renlmentc. jã ohscrvnmos anteriormente qm' O <'
uma cota inferior para X. Portanto, para mostrnr que O é n nmior cota inferior pnrn X, bnst n
mostrar que nenhum real positivo a é cota inferior pnrn X. Argumentando por contrndição,
suponhamos que, para algum a > O, fosse n < ¾,pnrn todo n E N. Entã.01 deveríamos ler
n < ¼,para todo n E N, de forma que o conjunto N dos naturais seria limitado superiormente.
Chegamos, pois, a uma contradição, o que garante que nenhum a > O é cota inferior para X.
Logo, o número O é a maior cota inferior para X.
----~--~---------

Para uso futuro, colecionamos, nos resultados a seguir, algumas propriedades úteis das noções
de supremo e ínfimo.

PROPOSIÇÃO 2.6.
Seja X C R um conjunto não vazio e limitado superiormente, com lví = sup X. Se n E N,
então existe x E X tal que
1
AI - - < x < AI.
n -

PROVA.
Como M é a menor cota superior para X e J/ - ¼< J\J, o nú.mero AI - ¼não é uma cota
superior para X. Portanto, existe x E X tal que x > .\J -¼- l\Ias, como X e (-oo, J\I],devemos
ter x < M.

70
O problema 1. traz um resultado análogo ao anterior para o ínfimo de um conjunto de
números reais não vazio e limitado infenormente.

PROPOSIÇÃO 2. 7.
Sejam X. Y e 1Rconjuntos não ,-azios. Se x < y para todos x E X e y E Y, então ~
limitado superiormente. Y é limitado inferiormente e Yale

supX < inf Y.


----------~~~~---- ........

PROVA.
Fixe y E Y qualquer Como x < y para todo x E X. temos X e (- , y], i.e., y é uma cota
superior para X. Portanto, X é limitado superiormente e, sendo 1'1 = supX (a menor cota
superior de X), temo ,\/ < y.
Observe agora que, como escolhemo y E Y arbitrariamente, o argumento do parágrafo
anlcnor mostra que. para todo y E Y temo J/ ~ y. Portanto. Y e [AI,+ ), i.e., AI é uma
cota mfonor para}' Logo. } é linutado inC')riormcnte e, endo m = inf Y (a maior cota inferior
ele Y), temos /1/ < m.

osso próximo r~ ultado fornec' umn condição suficiente para a ocorrência da igualdade na
d ~1gunlcladeda proposição antrrior.

I) HOPOSIÇ.\O- -·') .
Sl'Jnm .\,} • e 1Rconjunto!> nÃo vazios, :--cndoX limitado superiormente e Y limitado infe-
rionnentr. Suponha, ademais, que sup X ~ inf }'. Se, para todo n E N, existirem Xn E X e
.tln E } • tab que Yn - :i·n < t.
então sup X = inf Y.

PHO\ \
eJfü11 1\f = !:,Up X. m = inf Y e suponha que fosse A,f < m. Como x < M < m < y, para
todos I E X e y E Y, teríamo
y-x > m-M,
para todo!:>I E X, y E } '. ~Ias. escolhendo um natural n > m~M ( o que é possível, uma vez que
N é ilimitado uperiormente), nossas hipóteses garantiriam a existência de números reais Xn E X
e Yn E Y tais que
1
Yn - Xn < - < m - /11,
n
o que no dá uma contradição!

71
Por fim, como a suposição de que sup X < inf Y leva-nos a uma conclusão contraditória, a
única possibilidade é que seja sup X = inf Y.

Gostaríamos de terminar esta seção apresentando uma aplicação não trivial (e importante)
do conceito de supremo. Para contextualizá-la, observamos que, já no capítulo anterior. em
mais de uma ocasião (cf. parágrafo anterior ao exemplo 41, assim como problema 4.6 página
1

46), valemo-nos da existência de raízes de números reais positivos. O resultado estabelece tal
existência.

TEOREMA 2.9.
Dados um real positivo x e um natural n > 1, existe um único real positivo y tal que y" = .r.

PROVA.
Consideremos o caso em que x > 1, remetendo o leitor ao problema 1.1 e observando que o
caso x = 1 é trivial. Se
X = {a E IR;a > O e a" < x},
então X é não vazio, uma vez que O E X. Também, X é limitado superiormente, pois, para
o > x + 11 o problema 4.6, página 47, juntamente com o item (b) do corolário 2, garante que
Qn > (x + 1r > x + 1 > x; daí, a(/:. X e, assim, X C (-oo, x + 1).

Sendo não vazio e limitado superiormente, X tem supremo, digamos y. Como 1n = 1 < x,
temos que 1 E X e, daí, y > l. Mostraremos que y" = x da seguinte forma:

(i) Se yn < x, obteremos um real positivo z tal que yn < zn < x. Então, a primeira desigualdade
garantirá que y < z e a segunda (com o au.,xiliodo problema 4.6, página 47) que z E X. Com
isto, teremos y < z E X, contradizendo o fato de y ser uma cota superior para X.

(ii) Se yn > x, obteremos um real positivo z tal que x < zn < y". Agora, sendo a E X qualquer,
segue de an < x < zn < yn (novamente pelo problema 4.61 página 47) que a < z < y, de sorte
que zé uma cota superior para X e é menor que y. Mas isso contradiz o fato de que y é a menor
cota superior para X.

Uma vez demonstrados os itens (i) e (ii), não poderemos ter y" < x nem y" > x, de sorte
que a única possibilidade será yn = x.

72
Para a prova de (i), suponha que yn < x. Se z = y + ¼,com k E N, temos, novamente pelo
problema 4.6, página 47, que y" < z". Afumamos que z" < x para k suficientemente grande.
De fato, pela fórmula do binômio (cf. teorema 49), temos

mas, como (;) < 2", 1,< ¾ e y"-J < yn para todo 1 < j < n, segue que

portanto, temos zn < X se yn + n(2ky)" < x, i.e., se k > n(x-y2y~n.


Para a prova de (ii), suponha que y" > x. Se z = y - f, com k E N, então, como no
parágrafo anterior, temos yn > zn. Afirmamos que zn > x para k suficientemente grande.
Realmente, argumentando como acima, temos

z" = ( Y- kl)n = Yn + ~(-l)J


n •(n) 1
j kJyn-J

>y n
- Ln (n.)
-y1
J k1
n-;
>y n
---;n(2y)"
k
j=l

2 2
Portanto ' z" > x se yn - n( y)"
k
> x, i.e., se k > n( y)".
yn-x

Nas notações do teorema anterior, dizemos que y é a raiz n-êsima de x, e denotamos


y = ifx. Para uma outra demonstração do resultado acima, referimos o leitor ao exemplo 62.

Problemas - Seção 2.1


1. 1. * Ainda a respeito de raízes de reais positivos, faça os seguintes itens:

(a) Se O < x < l e n > l é natural, mostre que o número real ~ (cuja existência é
garantida pelo teorema 9, uma vez que ½> 1) é a raiz n-ésima é x.
(b) Para a, b > O e m, n > 1 naturais, mostre que v'ab= y'a v'b, ~ =~ e "'y'a =
~-
(c) Para O< a< b e n > l inteiro, mostre que y'a < v'b.

73
<. \1'111 1 t, .! S1-:q1·1-,:-,;1·1\'-, I·: <'<>:\ 1 l.'\I 11>\l>I·:

(d) Para a > O c m > n inteiros, mostre que y'a > efci,se a > 1, e y'a < efci,se
O<a<l.
(e) Da.dos x < O real e n E N ímpar, seja y = -y'=x. Prove que y" = x (o real y é
também denominado a raiz n-ésima de x) e estenda as propriedades dos itens (b),
(c) e (d) a esse caso.

1.2. * Um conjunto não vazio X e IR é denso em IR se X intersecta todo intervalo, i.e., se


X n J f 0, para todo intervalo I. O objetivo deste problema é mostrar que Q e IR\ Q são
densos em IR. Para tanto, sejam dados números reais a e b, com a < b, e faça os seguintes
itens:

(a) Mostre que basta considerar o caso a> O.


(b) Prove que existe n E N tal que O < ~ < b - a e O < :/!-< b - a.

(c) Sendo a> O e n E N escolhido como em (b), mostre que um dos números ~, ¾,¾,...
e um dos números 2
:/!-,'f!, 'f!,...
3
pertencem ao intervalo (a, b).

1.3. * Um número racional r E [O,1] é dito diádico se existirem k, n E 7l, tais que O < n < 2k
e r = ~- Prove que o conjunto dos racionais diádicos é denso em [O,l].

1.4. * Se Y e IR é não vazio e limitado inferiormente, prove que existe uma maior cota inferior
para Y.

1.5. Sejam X= {x E Q; O< x < 1} e Y = {y E R \ Q; O< y < l}. Prove que

inf X = inf Y = O e sup X = sup Y = 1.

1.6. Se X= {lva - v'bl;a, b EN e a f b}, calcule inf X.

1.7. Sejam

C1 = (O,1)
C2 = (O,1) \ (1/3, 2/3) = (O,1/3] u [2/3, 1)
C3 = ((O,1/3] \ (1/9, 2/9)) u ([2/3, 1) \ (7/9, 8/9)
= (O,1/9] u [2/9, 1/3] u (2/3, 7/9] u (8/9, 1).
Mais geralmente, para cada n E N, obtenha Cn+l a partir de Cn, excluindo-se os terços
ml~diosabertos de cada um dos intervalos que compõem Cn. Se C = Un~l Cn, mostre que
inf G' :::;O e sup C -:-::
1.

74
1.8. * Seja Y C R um conjunto não ,-azio e limitado inferiormente. com m = inf Y. Se n E N~
prove que existe y E Y tal quem< y < m 7 ¾-

1.9. Sejam X. Y C :'..1tconjuntos não ,11Zios.tais que X ê limitado superiormente, Y é limitado


inferiormente e ,up X = inf Y = o. Se o E X U} •. mostre que e.xistem elementos Xn E X
e y,,, E Y tais que Yn - Xn < ¾,para todo n EN.

1.10. ,,,.SeJa X e ....um conjunto não ,uio e limitado superiormente. Para e E lR, seja cX =
{ex, x E X}. ProYe que:

(a) Se e> O, então sup(cX) =e upX.

{b) Se e< O, então cX é limitado irucriormcute e iiú(c_\") = e sup X.

Em seguida, X • limitado inferiormente, t.abel~a propriedades análogas às dos itens


acima, relacionando inf .X ao uprcmo ou no ínfimo de ~\. co1úorme seja e < O ou e > O.

1.11. S<'JamX, Y C conjunto uõo vazio <' X Y = {x + y; x E X e y E Y}.

(a) Se ':\ e Y ...ão limit do HJ>t rionn nh:, pro,·• qu~ X · }' é limitado superiormente e
que up(X + Y) = ,upX up }'.
(b) e \ e ) ..,fiolimitodo infc>riorm~1t , prOh' qu X +Y ~ limitado inferiormente e
Q\W inf( \ - Y) = inf X+ infY.

1.12. • "'l~1,un \, ) e (O, ) oonjunt não , zio limitado superiormente. Se XY =


{.ry; .r e_ \ L' v E Y}, prO\t qu• XY li111itdo !,Upcriormente e que supXY = supX •
~11p) e inf '<) = inf X · inf )'.

1.13. Fncontn"' todn:-.o~ funçõ • /: -+ tai que /(1) = 1 e. para todos x, y E lR, tenhamos

{o) /(.r + y) = /(:r.) + /(y).


(b) /(.r.y) = f(x)f(y).

75
(. \1'111 111 ..! SHl('l-::\<'I.\S F <'<l:\ rt:\l'll>.\lll·:

2.2 Limites de sequências


Da.da uma sequência (an)n~l, estamos interessados em reconhecer se os números reais an
aproximam-se cada vez mais de algum número real l, à medida que n aumenta; por exemplo, se
On = ¾,então é razoável dizer que os números an aproximam-se de O à medida que n aumenta,

haja vista que o resultado da divisão de 1 por n é cada vez menor à medida que n aumenta.
esse sentido, temos a definição central a seguir.

DEFINIÇÃO 2.10.
Dizemos que uma sequência (On)n~l converge para um real l quando, fixado arbitrariamente
um erro f > O para o valor de l, existir um índice noE N tal que lan - li < E, para todo n > n 0 .

Alternativamente, se (an)n>l convergir para l, diremos que a sequência é convergente e que


l é um limite da mesma, fato que denotaremos escrevendo

lim an = l, an ~ l ou an ➔ l.
n-++oo

Por fim, uma sequência que não converge para real algum será dita divergente.
Ainda em relação à definição anterior, é de se esperar que, ao diminuirmos o erro e > O,
tenhamos de aumentar o natural n 0 a fim de que lan - li seja menor que f para n > n 0 ; em
outras palavras, é de se esperar que n 0 dependa de f > O. De qualquer modo, o importante
para assegurar a convergência da sequência (an)n>l para l é que, fixado arbitrariamente um erro
€ > O, sejamos capazes de encontrar n0 EN tal que

No intuito de familiarizar o leitor com esse procedimento, colecionamos, a seguir, vários exemplos
elementares de sequências convergentes e divergentes.

EXEMPLOS 2.11.
(a} Se a,. = ¼,então a,. ~ O: de fato, dado f > O, a fim de que lan - OI < f basta que n > ¼;
assim, fixado noE N tal que no> ¾,temos que

1 1
n > no=>n > - =>- < f =>lan - OI < €.
f n
(b) Se a,.= (-1r, então (an)n~l diverge: realmente, os termos da sequência, sendo alterna-
damenteiguais a 1 e -1, não podem aproximar-se de um mesmo real l. Formalizando esse

76
in ui iY fi. ad l E 1w dn cl<'sigualdad
t d kE

lak - li + la,.. 1 - li lak - li t li - ªk+d


> l(o,- - l) + (1- ªk+i)I
-'- lak - aHil - 2;

por 1o, par tod k E N t mo lak - li > 1 ou lak+l - li > 1, d sorte qu , 1.1111111A1~

O< < l, n~ p d m tornar lan - li< para todo n ufi i nt m nte grand .

( ) ª" = 1+i=.!r.
n ntão ª" ~ 1: i o porque lan - li = ¼ de maneira que la.- li<
par n > !-
(d (an)n>l um
• quAncia con tante com an = e para todo n > 1, então a,. ➔ e.
p rqu p ra todo f > O n E N, temo lan - cl= O < f.

EXE 1PL 2.12.


D do O< lql < 1 ªn = qn nt-ao an n O.
---+

PRO\'.
orno ,!,> 1 pod m r 11
!= 1+ ll:' om ll:' > O. Portan o a fórmula do binômio
i rn
1
lqln = (1 + r > 1 + nll:'
d í
1 7
lan - OI= lql" < 1 + n
1
im qu r m qu lan - OI < b ta impormo qu 1+ n < ou quiv l nt m nt
n>l(l-1).

EXEMPLO 2.13.
A quência (an)n~l, dada por an = Jn + 1 - ./n, conv rg par O.
--~---...:a...a.

im, d d >O omand n 0 E d qu n .al , t r 1ll

1
n > n0 =} Jn + 1 + ✓-ii, > Jno 1 -1 o>

~ -._:_ := ç ~7-
)
daí,
1
n > no ⇒ lan - OI = ✓ Jnn < €.
n+l+

A definição de convergência de uma sequência não deixa claro se o limite de uma sequência
convergente é único. De outra forma, em princípio, poderia ocorrer que uma certa sequência
convergisse para mais de um limite. No entanto, como mostra o resultado a seguir, isto não
ocorre.

PROPOSIÇÃO 2.14.
Se a sequência (<in)n>l convergir, então seu limite é único.

PROVA.
Por contradição, sejam l 1 e l 2 reais distintos e suponha que a sequência converge simullanc-
amente para l1 e l2. Tomando € = ½ll1 - l2 I > O, a definição de Umjte garante a existência de
n1, n2 EN tais que

Daí, pela desigualdade triangular,

o que é um absurdo.

A proposição a seguir lista algumas propriedades básicas de limites de sequências. Para


seu enunciado, dada uma sequência (an)n~l, definimos uma subsequência da mesma como a
restrição de (an)n~1 a um subconjunto infinito N1 = {n 1 < n 2 < n 3 < · · ·} do conjunto de
índices. Como a função de Nem N 1 dada por j 1----+-
n 3 é uma bijeção, toda subsequência de uma
sequência é, ainda, uma sequência. Ademais, podemos denotá-la por (an1cheN·

PROPOSIÇÃO 2.15.
Seja (<in)n~l uma sequência com limite l.

(a) Se l < a (resp. l > a), então existe noE N tal que 0n < a (re.sp. 0n > a) para n > n 0 .
(b) Se a,. > a (resp. 0n < a), para todo n > 1, então l > a (re.sp. l < a) .
(e) Toda 111beequência
(<in.)A:~1de (0n)n~1 a.inda converge para l.
----~-~----~
78
PRO\A
(a) Suponha l < a (o caso l > a pode ser tratado de modo análogo), e tome e = a - l > O. A
definição de limite de sequências garante a existência de um índice n0 tal que n > no=>lan -li <:
e; em particular, para n > n0, temos

ª" < l + e = l + (a - l) = a.
(b) Segue imediatamente de (a), por contraposição.

(c) Seja dado € > O. Como an ~ l, existe nonatural tal que lan - li < e, para n > n 0 . Mas,
como n1 < n2 < n3 < · · ·, existe um índice ni na subsequência tal que n 1 > n 0; portanto,

Mas isso é o mesmo que dizer que an,, ~ l.

Apesar da notação carregada, o item (c) da proposição acima pode ser resumido em palavras
de uma maneira bastante simples: basicamente, ele afirma que, se os termos de uma sequência
aproximam-se de um real l à medida que aumentamos seus índices, então, quando consideramos
somente uma parte (ainda infinita) desses termos, eles continuam se aproximando de l à medida
que aumentamos seus índices.
A proposição anterior po sui a seguinte consequência imediata, a qual fornece uma condição
suficiente para a divergência de uma sequência.

COROLÁRIO 2.16.
Uma sequência que possui duas subsequências convergindo para limites distintos é cli

PRO\\
Realmente, se a sequência inicial convergisse para l, então, pela proposição anterior, todas
as suas subsequências também convergiriam para l.

Até agora, exceto por alguns exemplos bastante simples, não vimos ainda como é p~ível
calcular o limite de uma sequência que sabemos ser convergente. Para remediar essa situação,
precisamos entender como operar com limites de sequências, problema que examinamos no que
segue.

79
Precisaremos, inicialmente, de um resultado auxiliar. Para o enunciado do mesmo, recorde
que toda sequência (an)n~l é, antes de mais nada, uma função de Nem lR. Portanto, de acordo
com a observação que precede o problema 5.2, página 57, a sequência (an)n~l é limitada se
existir Af > O tal que lanl < M, para todo n > 1.

LEMA 2.17.
Toda sequência t:onvergente é limitada.

PROVA.
Seja (an)n~i uma sequência que converge para o limite l. Então, existe n 0 E N tal que
n >no=} lan - li < 1. Portanto, segue da desigualdade triangular que

Por fim, se M = max{l + Ili, la1I,la2I,... , lano-il}, então lanl < AI para todo n EN, de modo
que a sequência em questão é limitada.

PROPOSIÇÃO 2.18.
Sejam (On)n~I e (bn)n~I sequências convergentes de números reais e e é um número real
qualquer.

(a) Se an -+ a, então 00n -+ ca.

(b) Se 0n -+ a e bn -+ b, então an ± bn -+ a ± b.

(e) Se 0n -+ a e bn -+ b, então anbn -+ ab.

(d) Se Cln-+ O e (bn)n~l é limitada, então cinbn -+ O.

Se 0n-+ a e bn-+ b, com b, bn -:f.O para todo n > 1, então t-+ t-

PROVA.
(a) Se e = O, então can = O para todo n E N, e segue do item (d) do exemplo 11 que can -+
O = ca. Suponhamos, pois, que e -:f. O, e seja dado f > O. Então, existe n 0 E N tal que
n > n0 ⇒ lan - ai< ~.
11
Logo,

80
(b) Provemos que an + bn-+ a+ b (provar que an - bn-+ a - b é análogo). Dado€> O, existem
n1, n2 EN tais que
€ €
n > n1 =>lan - ai < e n > n2 => lbn - bl < 2"
2
Portanto, tomando n > ma.x{n1, n 2} e utilizando a desigualdade triangular, obtemos

l(an + bn) - (a+ b)I = l(an - a)+ (bn - b)I


< lan - ai + lbn - bl
€ €
< 2+ 2= €.

(c) Pelo lema anterior, podemos tomar L > O tal que lbnl< L, para todo n EN. Por outro lado,
dado€> O, existem n1, n 2 EN tais que

Então, para n > max{n 1, n 2} e utilizando a desigualdade triangular uma vez mais, obtemos

lanbn- abl = l(an -a)bn + a(bn - b)I < lan - allbnl


+ lallbn- bl
€ € € €
< 2L• L + lal• 2lal+ 1 < 2 + 2 = €.

(d) Seja L > O tal que lbnl< L, para todo n > 1. Dado € > O, tomamos n 0 E N tal que
n >no=> lanl < f. Então, para n > no, temos

de sorte que (anbn)n~l converge para O.

(e) Se mostrarmos que b~ -+ ¼,seguirá do item (c) que

a- -b
1
= -ab.

Para o que falta, comecemos tomando n1 E N tal que lbn- bl< ~ para n > n 1 . Então, para
tais valores de n, temos (novamente com o auxílio da desigualdade triangular)

---
1 1 1 lbn - bl 1
=-·---<-·----
lbn - bl
bn b lbl lbnl - lbl lbl- lbn- bl
1 lbn- bl 2
< Íbl• lbl- lbl/2= lbl2.
lbn - bj.

81
( \1'111 111 _! ~1-:qt'F\'('l\s I" <"O\'ll'\111>\l>F

Agora, dado e > O, tomemos n 2 E N tal que n > n 2 ⇒ lbn - bl < tl~12 . Portanto, para
n > ma.."<
{ri1,112}, temos

EXEMPLO 2.19.
Seja a um real positivo. Se {Cln)n>l é dada porª"= y'ã para todo n ~ 1, então an ~ 1.

Prova.
Se a> 1, então an > 1. Escreva an = l + bn, de modo que bn > O. Como

temos O < bn < ª~ 1 . Portanto, o teorema do confronto (cf. problema 2.5) guarante que bn ~ O,
e o item (b) da Proposição 18 dá an = 1 + bn ~ 1.
Se O <a< 1, seja a~= 0
1
" = i"f,de modo que a~ ~ 1 pela primeira parte. Então, o item
(d) of Proposição 18 nos dá an ~ 1.

EXEM~ 2.20.
A sequência (an)n;?:l,dada por an= v'npara todo n > 1, converge para 1.

Prova.
Como no exemplo anterior, escreva an = 1 + bn para n > 2. Uma vez que bn > O, temos

e, daí,
2
O< b! < n - 1

Assim, novamente pelo teorema do confronto, a desigualdade acima garante que bn ~ O e,


então, que a 11 = 1 + bn ~ 1.

82
Para o que segue, chamemos uma vez mais a atenção do leitor para o fato de que uma
sequência (C1n)n~1é simplesmente uma função f: N ➔ IR,para a qual convencionamos a notação
an = f(n). Como o domínio N da sequência é um subconjunto de IR,podemos adaptar a definição
18 ao caso de sequências, dizendo que (an)n~I é monótona crescente (resp. decrescente, não
decrescente, não crescente) se an < an+l {resp. an > an+I, ª" < an+l, ª" > an+1), para todo
n > 1.
A seguir, enunciamos e demonstramos o resultado mais importante sobre limites de sequên-
1
cias, conhecido na literatura como o teorema de Bolzano-Weierstrass .

TEOREMA 2.21. BOLZANO-\VEIERSTRASS


Toda sequência monótona e limitada é convergente.

PROVA.
Suponhamos que {an)n~I eja uma equência monótona não decrescente e limitada, i.e., que

para algum AI > O (o demais casos pod 'm r tratado de forma análoga). Então, M é uma
cota superior para o conJunto A = { a 1 • a2 • a3 , ... }, de orte que tal conjunto possui supremo,
digamos sup A - l.
Afirmamo que an ➔ l Para tanto, seJa f > O dado; como l - e não é mais cota superior de A,
algum elemento de A é maior que /-f., digamosª"º > l-e. i\las, comoª"º < ano+l < an0+2 < • • •,
concluímos que a 11 > l - e, para n > no. A 1m, para n > no, temos

l - ( < ª" < l < l + e


ou, o que e o mesmo. lan - li < e.

EXEMPLO 2.22.
Seja a um real positivo. Se (an)n~l é dada por Cln= v'ãpara todo n ~ 1, então

PRO\\
As uma que a> 1 (o caso O< a< 1 pode ser tratado como no Exemplo 19). Então, a 1 > a1 >
a3 > • • • > 1, e o teorema de Bolzano-Weierstrass garante a existência de l = limn-.+oo0n > 1.
O item (a) da Proposição 15 dá l > 1, e o item (b) garante que toda subsequência de (On)n>l
1
Apó Bernard Bolzano e Karl Weierstrass, matemâticos alemães dos séculos XV111e XIX.

83
tnmh"m conY<'rg<' pAra /. Porlanlo, ak(k+t) ~ l. Agora, como ak{k+l) = i.*4, segue do item
(d) da Proposição 1 que

Logo,/= l.

Como o leitor pode verificar facilmente, o teorema anterior e a definição de sequência con-
vergente também garantem que, se uma sequência limitada for monótona a partir de um de-
terminado termo, então ela ainda será convergente. Exploramos essa observação no próximo
e.xemplo.

ExEMPLO 2.23.
A sequência (<1n)n~1,dada por <ln = y'n, é convergente e seu limite é igual a 1.
-----

PROVA.
Os termos iniciais da sequência são v'2, '1'3,¼, ... , e é fácil verificar diretamente que
,/2 < '1'3e ~ > ¼ > -Ys.Como 2n > n 2 para n > 4 (por indução, por exemplo), temos
~ > an para n > 4, de sorte que a sequência é limitada; portanto, se mostrarmos que ela
é decrescente a partir de seu terceiro termo sua convergência seguirá do teorema de Bolzano-
\Veierstrass.

~r
Para o que falta, dado n > 2 inteiro, temos

;1/n> ••~n + 1 ç; n•+l > (n + l)" <* n > (1 +


A última desigualdade acima é de verificação imediata para n = 3; para n > 3, basta mostrarmos
que ( 1 + ¼)n < 3. Para tanto, observemos que

(2.1)

e. para k > I,
ri)~= ri! =~-n(n-I)···(n-k+l) 1 1
( k nk k!(n - k)!nk k! nk
< -<--
k! - 2k-l.

Assim,

( ] + ~)
n
n = l + (TI)
.!_+
l 11
(ri)
2_ + ... + (n)
2 n2
~
n nn
l 1 1
<l+l+-+-+···+-
2 22 2n-l
1
= 3- - - < 3.
2" 1

84
Resta mostrarmos que o limite da sequência é l. Para tal fim, observe inicialmente que a
subsequência a2k = fü também converge para l. Por outro lado, 2~
2
= 2❖12 • ffk, com
2
❖12 ~ 1. Agora, segue do problema 2.2 que JVf. ~ Jl. Portanto, aplicando o item {b) da
Proposição 1 , obtemos

EXEMPLO 2.24.
Para k E N e lal> 1, temos limn-++oo o": = O.

SOLUÇÃO.
Inicialmente, ob erve que, pelo exemplo anterior,

Portanto, fixado q E 1Rtal que !1 < q < 1, o item (a) da proposição 15 garante a existência de
n0 E N tal quen > n 0 => n 1: 1~. < q ou, ainda, ,:tn < q". Mas, de acordo com o exemplo 12,
temos que qn ~ O quando n ~ + . Portanto, verificamos facilmente que o mesmo sucede com
nl.
o" •

Por vezes, não precisaremos mostrar que uma dada sequência converge, mas somente garantir
que ela possui uma subsequência convergente. esse sentido, o teorema 26 a seguir, devido
a \,Veierstrass e conhecido na literatura coron () teorema de Weierstrass, dá uma condição
uficiente para a existência de uma tal subsequência. Antes, contudo, precisamos de um resultado
auxiliar importante em si mesmo, conhecido como o lema dos intervalos encaixantes. No
que segue, se I e IRé um intervalo finito, escrevemos III para denotar seu comprimento.

LEMA 2.25.
Sejam dados os intervalos ln = [<ln,bnl,n E N, tais que /1 :> /2 :> /3 :> • • •. Se
O, então existe um único l E R tal que íln~l ln = {l}.
~----------------
85
( • \ l 'I 1 1 1 < 1 .! SE(1l T:\C'I.\S E C'O:\Tl:\I º11>.\IH:

PROVA
Note, inicialmente, que a interseção dos ln é vazia ou unitária: de fato. se existi em reais
a < bem tal interseção, teríamos [a, b) C íln~l ln; em particular, [a. b] C ln e, daí, llnl > b - a,
para todo n EN, o que é uma contradição ao fato de que llnl ➔ O.
Por outro lado, a condição de encaixe / 1 :> 12 :> 13 :> • • • garante que a < a2 < a3 < · · · < bt,
1

e o teorema de Bolzano-Weierstrass garante a existência de l = limn-.+ ªn· firmamo que


l E íln~l ln. Para Verificar tal afirmação, observe que an < l, para todo n E N. Por outro lado.
fixado m E N, temos an < bm, para todo n E N, e o item {b) da proposição 15 garante que
l = limn➔+oo an < bm. Mas, como o natural m foi escolhido arbitrariamente, temo l < b111, parn
todo m EN. Então, temos l E [am, bm] = lm, para todo m EN, conforme desejado.

TEOREMA 2.26. WEIERSTRASS


Toda sequência limitada admite uma subsequência convergente.

PROVA.
Seja lo - [ao, b0 ] um intervalo fechado e limitado contendo todos os Lermos da sequencia
lirrútada {an)n~l- Dentre os intervalos (a 110;tii:i] e [ª ~bu,b escolha um que contenha uma
0
.
0

0
].

infinidade de termos da sequência (a 0 )ra~l• e denote tnl intervalo por / 1 . Proceda do me<,mo
modo com 11, obtendo um intervalo fechado /2 C /1, tal que II21 = ½IJil e 12 contenha uma
infinidade de termos da sequência {a11) 11~ 1. A ·im. construímos indutivamente uma sequência
11 , 12,l3, ... de intervalos fechados e limitados, tais que /1 :> /2 :> h :> • • • e lln+1I - ½llnl, para
todo n > 1. Portanto, pelo lema dos intervalos encai..xantes, existe l E R tal que íl,. ~ 1 h = {/}
Para terminar, escolha n 1 E N tal que an, E / 1. Em seguida, para cada J > 1. após ter
escolhido n 1 E N tal que an, E 11 , torne n1 +1 E N tal que n 1 +1 > n 1 e an,+i E 11+ 1 (isto é
possível, pela definição dos 11). Dessa forma, construímos uma subsequência {an.)k~l de (an)n~I
tal que an,. E h, para todo k EN. Em particular, como an•. l E h, temos

de sorte que a,1,. ➔ l quando k ➔ +oo.

O conceito de sequência convergente tem um apelo geométrico bem forte qual seja. a ideia 1

de que os termos da sequência aproximam-se mais e mais de um certo número real, à medida
que seus índices aumentam. Por outro lado, também é de se esperar que, se os termos de uma

86
sequencia ficarem cada vez mais próximos uns dos outros à medida que seus índices a
então a sequência deva convergir. Esta ob ervação leva à definição a seguir.

DEFINIÇÃO 2.27.
Uma sequência (an)n~l é dita uma sequência de Cauchy se, para todo E> O
71iJEN tal que
m, n > no =>1am - CLn
I < e.
~~----------~----- ---------~-------~--
O teorema a seguir traz o resultado fundamental acerca de sequências de Cauchy.

TEOREMA 2.28.
Uma sequência (an)n~1 é convergente se, e só se, é de Cauchy.

PROVA.
Seja (an)n?l uma sequência convergente, com limite l. Dado e > O,a definição de convergência
garante a existência de n 0 E N tal que
e
n > no =>lan - li < 2.
Portanto, para m, n > n 0 , egue da desigualdade triangular que

e n sequencia (an)n>J é de Cauchy.


Reciprocamente, seja (an)n~J uma equência de Cauchy. Então, existe n 0 E N tal que
larn - anl < 1, para todos m, n > no. Em particular, Iam - ano+il < 1 para todo m > n 0 , de
sorte que a sequência tem todos os seus termos contidos na união

e, assim endo, ê limitada. Portanto, pelo teorema de Weierstrass, a sequência (an)n~l possui
uma sub equência convergente, digamos (ank)k~1, com ank -+ l quando k -+ +oo. Provemos
que, em verdade, limn➔+ ª" = l.
Seja dado e > O. Como ª"k -+ l quando k -+ +oo, existe ko E N tal que

k > ko =>1ª"k - ll <
2.
Por outro lado, como a sequência é de Cauchy, existe n~ E N tal que

m. n > n~ => Iam - anl < r


87
Então, fi._,adok > k0 tal que nk > n~, segue da desigualdade triangular que

n > n~ => lan - li < lan - an1c1 + lan1c- li < ; + ; = €.

Logo, Dn -+ l quando n -+ +oo.

Vejamos um exemplo interessante de aplicação do teorema anterior.

EXEMPLO 2.29.
Seja (<1n)n~l uma sequência de números reais tal que

para todo n E N, onde O < e < 1 é um real fixado. Mostre que tal sequência é convergente e
que, sendo l seu limite, temos

(2 2)

PROVA.
Para a primeira parte, pelo teorema 28 basta mostrarmos que (an)n~l é uma sequência de
Cauchy. Para tanto, iterando a desigualdade do enunciado obtemos, para k E N,

Agora, sejam m e n naturais dados, com m > n. Aplicando sucessivamente a desigualdade


triangular, a desigualdade obtida acima e a fórmula da soma dos termos de uma PG, temos
m-1 m-1

Iam - anl = L(ªk+l - ak) < L lak+l - akl


k=n k=n

< 'f: c•-•1ai- a,I = ( e" /~cem) la2 - a,I


k=n
cn-1
< -- • la2 - a11-
1- c
Como O < e < 1, o exemplo 12, em conjunção com o item (a) da proposição 18, garante
que ~"_; • la2 - a11 -+ O quando n -+ +oo. Portanto, dado € > O, existe n 0 E N tal que
n > n0 ~ e;~:
· la2 - a11 < €. Então, segue dos cálculos acima que, para m > n > n0 ,

(2.3)

88
Logo. a sequência (o.n)n>1
- é de Cauchv. ~

Para o que falta, seja l = limn_..,..00 o.n. Fazendo m -t +oo na primeira desigualdadeem
e utilizando o resultado do item (b) da proposição 15. obtemos (2.2).

Problemas - Seção 2.2


2.1. "' Sejam (an)n~l e (bn)n~l sequências com·ergentes de números reais, com limn-++ooª"=a
bn = b. Generalize o item {b) da proposição 15, mostrando que, se
e limn.......,..oo ª" < bn para
todo n > 1, então a < b.

2.2. * Seja (an)n~1 uma cquéncia de reais po iti\"OScon\"ergindo para a > O. Mostre que
Fn ~ ./ã
2.3. • Dado a E IRtal que lal > L mostre que :~ ➔ Oquando n -t +

2.4. St>jam (an)n I e {bn)n2'.Jsequências de número::. re<rn,e. para cada n E N, seja tn E [O,l]
um real fu..ttdo. Denote por (c.,)n2'.l a !qu">ncindcfmida por

parn todo n EN t>au,b,.-+ I. pro,·~ que C.a-t /.

Em algum, t>xcmplos ao longo do texto. utilizamo por vezes o fato de que, se uma sequência
(nn)n?I atbfaz O < au < bn, onde (bn)n~l é uma sequência que converge para O, então
(a 11)n>t também converge para O. E e re:iultado é um caso particular do teorema do
confronto. que. por ua vez. é o objeto do próximo problema.

2.5. ejrun (an)n~l. {b11 )n~1 e (Cn)n~l sequências tais que ª" < bn < Cn, para todo n E N. Se
ª", cfl ➔ l, para algum l E lR, prO\·e qu ,." -t l.

2.6. Calcule o limite:, abai..xo:

n ✓ri
(a) limn-++ ~-
(b) limn-++ ( Jn 2 + an + b - n), com a. b E lR.
(c) limn-++ y'l + q", onde O < q < 1 é um número real.
(d) limn-++ v'a" + b", com a e b reais positivos tais que a > b.

89
(. \1'11111 > 2 SHJl E:\<'I \SE('():\ 11:\1'11>.\l>L

2. 7. * Este problema estende o conceito de limite de sequências para considerar limites mfimtos
Dizemos que uma sequência (an)n2!:Ide números reais converge para + (resp. - ) se,
dado /1/ > O, existe no E N tal que n > n 0 => an > M (resp. an < -/II). l esse caso,
denotamos limn-++ooan = +oo {resp. limn-++ooan = -oo) ou, ainda, an ➔ + {resp.
an ➔ -oo) quando n ➔ +oo. Em relação a esse conceito, e dadas sequências (an)n2!:le
(bn)n>l de números reais, faça os seguintes itens:

(a) Se ªn ➔ ±oo e (bn)n~l é limitada, então ªn + bn ➔ ±


{b) Se Dn -t ±oo e bn > e> O {resp. bn < e < O) para todo n > 1, então a 11b11 -t ±oo
{resp. anbn ➔ =t=oo).
(c) Se bn ➔ +oo e existe e > O tal que an > cbn (resp. an < -cbn), para todo n > 1,
então an ➔ ±oo.

2.8. Sejam q um número real e (an)n2!:la sequência definida por a,l :,_ qn. Se q > 1, mostre que
an ➔ +oo quando n ➔ +oo. Se q < -1, mostre que (an)n>l não converge para +oo ou
-oo.

2.9. * Dados reais positivos a e q, com q < 1, mo tre que anq2n ➔ O quando n ➔ +oo.

2.10. Seja (an)n2!:luma sequência de números reais tal que, para todos os m, n E N, tenhamos

Prove que a sequência é constante.

2.11. Seja (an)n2!:la sequência definida como segue: a 1 = 1, a2 = ./f+l, a3 = J1 + ./2,


a,= J1 + J1 + J2, a,= /1 + J1 + J1 + J2,.... Mostre que (a.}.~ 1 é convergente
e calcule seu limite.

2.12. Sejam k > 1 um inteiro fixado e t 0, t 1 , ... , tk reais também fixados, tais que t 0 +t 1 +· · ·+tk =
O. Prove que a sequência (an)n2!:l,dada por

converge para O.

2.13. A scquê11cia{an)n>l é tal que Jan+i + 2 < an < 2, para todo n > 1. Mostre que an = 2,
para todo n > 1.

90
2.14. Sejam k um natural fixado e (an)n~l a sequência definida por

com exatamente n raízes quadradas.

(a) Mostre que (an)n~l é convergente.


(b) Mostre que, quando k é ímpar, o limite da sequência é um número irracional.
(e) Encontre todos os valores naturais de k para os quais o limite da sequência seja um
número inteiro.

2.15. Para a> O fixado, seja (an)n~l a sequência definida por a 1 = 1 e, para k > 1 inteiro,

Prove que (a 11 )n~1 converge para .fã. e que ja11 - Jãl < 1
2 " _ 1 la
- li.

2.16. Interprete rigoro amente a igualdndt•

l l+v'S
l + ----=-- - ---
1 2
l + l
l+-

91
, ,, ' ~ I· < l t 1 '\ < 1 \" 1 < < , '\ 1 1'\ t 11, \ 1>1

2.3 O conceito de continuidade


Consideremos, inicialmente, a função f : IR ➔ IR dada por f (x) = {x} (a função parte
fracionária), cujo gráfico está esboçado na Figura 2.1. Após um rápido exame, certamente
sentir-nos-íamos confortáveis em dizer que o gráfico em questão é descontínuo, em alusão à
existência de vários saltos ao longo do mesmo. Note que isso, aparentemente, não ocorre com o
gráfico da função g·: IR ➔ IRdada por g(x) = x2 , o qual, certamente, mereceria ser denominado
contínuo.

-4 -3 -2 -1 O 1 2 3 X

Figura 2.1: gráfico de x ._..{x}.

Surge, então, a questão de obter um critério razoável para identificar a existência ou não
de tais saltos num gráfico, discernindo entre possibilidades como aquelas descritas acima. Para
desenvolvermos alguma intuição sobre como fazê-lo, restrinjamos o domínio da função x r-+ { x}
ao intervalo [¾,J], denotando a nova função assim obtida ainda por /, por simplicidade de
notação; concluímos facilmente que

Im (!) = l rn.
[o,~ U 1) ;

em particular, existem valores de y no intervalo de extremos !(¾) e /(~), por exemplo y = i,


tais que nenhum x E [¾,~] satisfaz f(x) = y.
Por outro lado, é fácil verificar que tal não ocorre com a função g do primeiro parágrafo. Mais
precisamente, fixemos um intervalo [a,b] e IR e denotemos por g([a, b]) a imagem da restrição
de g a [a, b), i.e.,
g([a, b]) = {g(x) E IR;x E [a, b]}.
Então, pondo e = max{ -a, b}, é fácil verificar que
seO<a<b
sea<O<b
sea<b<O

92
Em particular, para todo y no intervalo de extremos g(a) e g(b), existe x E [a,b) (z • ::I:~
conforme o caso). tal que g(x) = y.
Estamos, agora, em posição de generalizar a discussão acima, com a definição a seguir. Pua
tanto, em tudo o que segue, salvo menção em contrário, X denota uma união de intervalos da
reta.

DEFINIÇÃO 2.30.
Dizemos que uma função f : X ➔ R possui a propriedade do valor inte•l"l'IIIAft:
Figura 2.2) se, para todo intervalo (a, b) e X e todo y 0 pertencente ao intervalo de ex1nlllliC
J(a) e J(b), existir xo E (a, b]
-'--~---___;___:_ ______________
tal que y = f (x). ~~~__._.-

G1 y

f(a) 1
1
1
--------~--- 1 ----------- -:- - Y = Yo
1
1

a X

f(b)

Figura 2.2: a propriedade do valor intermediário.

A discu ão acima garante que a função / (x) = { x}, x E lR,não possui a propriedade do valor
intermediário, ao pas o que a função g(x) = x 2 , x E lR,possui tal propriedade. Desse modo, ela
aparentemente nos compele a dizer que, se uma função / : X ➔ lRpossui a propriedade do valor
intermediário, então f é uma função de gráfico contínuo, i.e., sem saltos. Todavia, o leitor pode
verificar facilmente que a função / : (O,2] ➔ ::, dada por

J(x) ={ x, se O < x < l ,


x- ½, se 1 < x < 2
satisfaz a propriedade do valor intermediário, mas seu gráfico apresenta um salto em x = 1.
Assim, f não é o protótipo do que desejamos chamar de função contínua.
A fim de formularmos adequadamente o conceito de função contínua, analisemos a situa-
ção sob outro ponto de vista. Seja / : (a, b) ➔ lR uma função dada, x0 E (a, b) fixado e

93
1 , 11 1, 1 ' SJ·.qt I·,\( 1 \" 1 < ()\ 11\1 11)\J))-'

P0 .r0 .f(.r0 )) E • 1 . ,1run ainda, .z; E (a, b) \ {x 0 } P(x, f(x)) E G!· Para qu o gráfico d
.f m r n - r d n minad contínuo nossa intuição g ométrica diz que para x próximo a Xo
d ~'" m - l r P próximo a Po.

G1 y

Xo b X
1---1

Figura 2.3: ponto de vista correto para função contínua.

1ais precisamente, essa proximidade significa (cf. Figura 2.3) que, arbitrado um erro r > O
para a po ição do ponto P0 (i.e., arbitrado um disco D(P 0 • r), de centro P0 e raio r), dev mo t r
P E D sempre que a abscissa x de P aproximar O com erro suficientemente pequeno, digamo
menor que um certo ó > O (leia-se delta). Em símbolos, arbitrador > O, deve existir ô > O tal
que
lx - xol <ó ⇒ PPo < r. (2.4)
É po~ível mostrar (cf. problema 3.4) que a validade da condição (2.4) equival à seguint
descrição geométrica alternativa (Figura 2.4): arbitrado um erro f > O (lê-se épsilon) para o
valor de f em x 0 , i.e., arbitrada uma faixa horizontal

lR x (J(xo) - f, J(xo) + f)
do plano cartesiano, simétrica em relação ao ponto Po(xo,f(x 0)) (a faixa cinza da Figura 2. ))
dev xistír outro rro ô> O tal que, para x E X sati fazendo lx - xol < ó, o ponto P(x, f(x))
pert nça à faixa em que tão. Isto posto, podemos finalmente enunciar a definição formal d
função continua.

co tfnua m um ponto x 0 E X a seguinte condição for


xis 6 > O tal que

.._,_____.~~-~~~~~~~~--
x e X, lx - xol <ó=> 1/(x) - /(xo)I < f.
-~~----~-~~ (2.5)

94
Xo b X
t--------1

igura 2.4: laborando o pon o d vi ta corr to para função con ínua.

contínua o for m todo x 0 E

E EMPLO 2.32.
'D da função

PR V.
j m um r I d do f : -t funç~ n an igual a e.
r r u J ' ntínu m x 0 E a d finição d on inuidad p d qu dado e.> O
n n r m ó > O al qu p v lidad da d igualdad lx - x 0 1 < ó impliqu
lid d lf(x) - J( o)I < om 1/( ) - !( o)I = lc - cl = O, a d igualdade
lf ( ) - !( o)I < , lid ind p nd nt m n d qualqu r re trição ob 1 - oi- D
u r e rm t man u lqu r r ai p i i o mpr t r mo qu 1 - oi < ó =}
lf(x) - J( o)I < um z qu d igu ldad lf(x) - f(xo)I <e.não t m como r fal a.

fim rmo a on inuidad d funçõ m no impl pr mo laborar um


p u d fini -o qu faz m a gmr.
up qu r m b l on inuidad d f: -t m o E .D acordo com
d fini - mo upor qu um rro > O para /( 0 ) , dado rn guida r capaz d
rr ó> O par o qu orn rdad ira a impli aç- (2.5). rn g r 1, um b
tr g1 n ntido ,

(i) P r ind d um E uj i o um rro do ip 1 - oi < ó tim m o rr l/(2·) - /( o)I


rn t rrn d ó por exc o ob ndo uma d i uald d tip !J(x) - f(xo)I < E( ), ond E
representa uma certa função de ô.

(ii) Em seguida, impomos que tal erro E(ô) não ultrapasse o erro f desejado, descobrindo, então,
os valores apropriados de ô (usualmente, esse segundo passo resume-se a resolver, para ô> O, ª
inequação E(8) < €).

Uma vez executados os dois passos acima, se ô > O satisfizer E(8) < f, teremos claramente
que
xEX e lx - x0 1<ô=> lf(x) - J(xo)I < E(ô) < f,

conforme desejado.

Como uma última observação, o mais das vezes, ao longo da execução dos itens (i) e (ii), a
função E ficará subentendida. Vejamos alguns exemplos relevantes.

EXEMPLO 2.33.
A função modular (cf. exemplo 47) é contínua.

PROVA.
Fixado x 0 E IR, se lx - x0 1< ô, então, pela desigualdade triangular, temos

IJ(x) - f(xo)I = llxl- lxoll< lx - xol< ô.


Portanto, se ô< f, teremos das desigualdades acima que lf(x) - J(xo)I < f para lx - x0 1< ô.

EXEMPLO 2.34.
Sejam a e b números reais dados, sendo a:#=O. Se f: R ➔ Ré dada por f(x) = ax + b, então
/ é contínua.
------~~------------~---------~-_____;
PROVA.
Novamente, fixado x 0 E IR, se lx - xol< 8, temos
lf(x) - f(xo)I = l(ax+ b) - (axo+ b)I = lallx- xol< lalô.
Portanto, se lalô< f (ou, equivalentemente, ô< 1: 1), teremos IJ(x)- f(x 0 )1 < f para lx-xol < 8.

96
EXEMPLO 2.35.
A função raiz quadrada f : [O.+oc) ➔ R. tal que f(x) = ./x para

PROVA.
Sexo= O e O< x < ó, então lf(x) - J(x 0 )1 = ../x< ./J. ~Ias. como ./J <e<=} ó< e2 , para
tais valores de ó> O teremos lf(x) - J(xo)I < e e lx - OI < ó.
Suponha. agora, que x 0 > O. Então. para lx - x 0 1< ó. temos

r= lx - xol 1 ó
f(x) - f(xo)I = lv.L- fiõl = ../x ftõ
x -t- xo
< ~lx -
yio
xol < r:;:-:·
yXo

Agora, observe que }zõ< e <=}ó < ftõ!. Portanto, tomando O < ó < ftõe, temos que
l:i - J.·ol< ó ⇒ IJ(x) - J(.ro)I < e, conforme de,eJado.

A 1:,cguir,estabelecemo a continuid dt• dn funçõ ~ no eco eno. Antes, contudo, precisa-


mo:; de um resultado auxiliai.

LE~IA 2.~lG.
Pnrn todo .r. E lR,temo lscn xi < 1:rl,

PHO\ \
Im,tremo micialmentc qm.. ~ n x < :r, para O < x < i Tal desigualdade é óbvia para x = O;
pn1n O <. .r $ i, marque, no primeiro quadrante do ciclo trigonométrico (cf. Figura 2.5), o ponto
P tnl que f(.1P) = .r.
+-+
e Q é o pé da perpendicular bruxada de P à reta AA', então

enx = PQ < i(AP) = x.

Agora, como :sen(-x) = -senx, segue imediatamente que lsenxl ~ lxl para lxl < J. Por
firn, para !-ri> ~, temo
7r
lsenxl < 1 <
2< lxl.

97
< • \ 1'I 1 1 1 ( > .! S J·:<JI T '\ < •1 \-.. I· < <l '\ 1 I '\ 1 11l \ ll I·:

A'
o Q

B'

Figura 2.5: lsenxi $ lxl.

EXEMPLO 2.37.
As funções_seno e cosseno são contfnuaa.
-------------------------~-•------'

PROVA.
Fixe xo E IR. Se lx - xol < ó, segue das fórmulas de transformação em produto e do lema
anterior que

1 cos x - cos x 0 1 =2 sen (


X Xo)
2
X -Xo
< 2 sen ( X - xo) ~2
2 2
= lx - xol < ó.

Portanto, se ó < t, então teremos Icosx - cosx 0 1< €, sempre que lx - x0 1< 6, conforme
desejado.
Por fim, o argumento para a função seno é totalmente análogo.

Provaremos mais adiante (cf. proposição 47) que, se I é um intervalo e /, g : J --t IR são
contínuas em x 0 E I, então as funções f ± g, f · g : I ➔ IR também são contínuas em x 0.
Por outro lado, na proposição 49 mostraremos que, se g não se anula em /, então a função
lg •• l-+ IR também é contínua em xo. Por ora, assumiremos essas afirmações sem demonstração '
utilizando-as para apresentar mais alguns exemplos de funções contínuas.

98
() ( (l'\("f ))() l>F ( ()'\))'\I ))1\)11

EXEMPLO 2.38.
Dados a E R e k E H. a função x i-+ axk (de lR em lR) pode ser vista como o
função constante e igual a a por k cópias da função identidade, x i--+ x. Portanto, 08 -~
32 e 34, juntamente com a discussão do último parágrafo (estendida ao produto de k fun
contínuas), garantem que a função xi--+ axk é contínua.
Aplicando novamente a discussão do parágrafo que precede este exemplo (dessa ve-zestendida
à soma de um número finito de funções contínuas). concluímos que toda fw1ção polinomial,
i.e., toda função f: R ➔ R do tipo

para certos n E N e a0 . ai, .... an E lR.também é contínua.

EXEMPLO 2.39.
Se / : lR ---+lR é uma função polinomial (cf. exemplo anterior), dizemos que x 0 E lR é
uma raiz de / :-.e/(.r 0) = O. Prova-~c (cf. Capítulo 3 de [141, por exemplo), que o conjunto
das raíz<'-<;ele uma função polinomial ·• finito. Portanto. se f e g são funções polinomiais e
R,, - {.t·1 < .r2 < · · · < .r.k}é o conjunto das rnizes de g, o conjunto

é 11mnunião finito de intervalos nhc1to:, no qual fica bem definida a função


1

-f : IR 1?.9 ➔ R.
g

Umn tal função é dita racional, e o exemplo anterior, juntamente com a discussão do parágrafo
que o precede, garante que funções racionais ~ão contínuas onde estiverem definidas.

E:\.l \IPLO 2.40. 1

A função f : IR➔ R, dada por


2
J(x) = x-i sen x (x + 1) cosx,
-
x8 + 2 + cosx
é continua. De fato. fé o quociente das funções g, h : lR➔ IR, tais que g(x) = x' senz - (r +
1) cosi· e h(x) =x + 2 + cosr. Agora, g eh são contínuas pelos exemplos anteriores,eh~'
~e anula, uma Yez que x ~ O e 2 + cosx > O. ________ --~~---- _._.

Terminamos esta seção examinando a continuidade de uma composição. esse sentido, o

99 A•
t \ 1•1 1 1 1 11 .! S i:q t 1·'\ 1 1 \--. 1 t t , '\ li'\ 1 li> \ 1>I·

·ult ,d ., . Hir ' nh id m e r gra d

2. 1.
C R - uniõ d int rvalo / :X ➔ Y g : Y ➔ R ão funçõ contínu ntão
g oJ: ,,.➔ tamb m ontínu .

R \
4

J 111.oE Yo = f (. 0). D do > O a on inuidad d g g r nt a x1 n ia d >O


tal qu
yE IY- Yol<ô ⇒ lg(y) - g(yo)I < · (2. )
P r ou r lado on inuidad d / garante a xi t n ia d 6' > O tal qu

E , 1 - O1 < {J' =} 1f (X)- ..__,,,,.


f (Xo) <
.._,,,_, 1

y YO

Por anto gu das r l çõ a ima (com y = f(x) m (2.6)) qu

xE lx - oi < 6' =} IJ(x) - f(xo)I <ô ⇒ lg(J( )) - g(J( o))I <

O doí x mplos a guir mo r m u o típico da r gra da cad ia para funçõ on ínu

EXE PLO 2.42.


A funçãof: R ➔ R, dada por J(x) = n (x2 ) é contínua. De fato, e g h : ➔ ão a
2
funções dadas por g(x) = n x h(x) = x então g e h são contínuas f = g o h; portanto f
é contínua, pela regra da cadeia.
------------~-~-------

EXEMPLO 2.43.
A função / : R ➔ R, dada por

J(x) =

4
+ x2 + 2 + JI n xi '

fnua. Realm nt , orno J omp t d fui - r iz quadrada (qu • ontínu ) com


.r2 n 2 +l
a IUDCIOg : R ➔ dad por g(.r) = x•+x 2 +2+
1
n
1
, r gr da cadeia garan
-~-lel'llDOI a continuidad d !/· gor , 9 °
qu i nt d funçõ x H x 2 n (x 2 ) + 1
2 + ✓I n xi, as quai. - ontínu p l x mpl anterior -- r r
___
.,____,_ ____._~---
rv qu <l 11 mim lor n ~ nul vez que 2 +
----~
o.

1
Problemas - Seção 2.3
3.1. Prove que:

(a) A função J: IR\ {O}-+ 1Rdada por f (x) = ~ possui a propriedade do valor interme-
diário e é contínua, mas a função f : 1R-+ 1Rdada por

l sex=rfO
f(x) = x'
{ O, se x =O
não possui tal propriedade.
(b) A função f : R -+ 1Rdada por

x 2 + 1, se x > O
J(x)= { -x, sex<O

não possui a propriedade do valor iutermediârio.

3.2. Sejam/ e IRum intervalo e f: /-+Ruma função contínua. Fixado x 0 E /, seja g: /-+ 1R
a função dada por
g(.r) = { e. e .r. = Xo .
f (x), se x # Xo
t-.lostre que g é contínua se, e só se, e= /(O), i.e., se, e só se, g = f.

3.3. Em cada um dos itens a seguir, e..xplique,com justificativa, se existe um valor de e que
torne conUnua a função / : lR -+ IR dada:

3x - 2, se x < O 3x - 2, se x < O
(a) f(:r) = e, se x = O . (e) J(x) ={ e, se x = O
{
-2, se x > O x2, se x > O

(b) J(x) = { xcosx, se x #O. (d) f(x) ={


~, sex#O
e, se x = O e, se x=O

3.4. * Prove que toda função contínua no sentido da relação (2.4) é contínua no sentido da
definição 31, e vice-versa.

3.5. * Se D= 1R\ {1r/2 + br; k E Z}, use a discussão no parágrafo que precede o exemplo 38
para estabelecer a continuidade da função tangente,

tg : D --+ 1R
X 1----t tg X

101 â-
3.6. Se I e IR é um intervalo e J : J ➔ IR é uma função contínua, explique por que a função
Ili : / ➔ IR também é contínua.
3.7. Justifique a continuidade da função f: [-1, +oo) ➔ IR, dada por J(x) = c°::f:I'-
3.8. Sejam n EN e/: [O,+oo) ➔ IR a função raiz n-ésima, f(x) = fi, para x > O.
(a) Mostre que fé contínua em x 0 = O.
(b) Se x 0 > O ex> O, mostre que

_ lx - xol
IJ(x) - f (xo)I
- ( fi)n-1 + ( fi)n-2 fio+ ... + ( efxõt-1
1
< ( efxõ)n-1 • lx - xol-

(c) Use o resultado de (b) para concluir que fé contínua em x 0 .

3.9. E>..-pliquepor que a função f : IR ➔ lR, dada por f(x)


4
= 3
z ~;~~±1 , é contínua.

3.10. Justifique a continuidade da função f: IR ➔ IR, dada por

f(x) ={ O, se x = O
x sen ~, se x f:. O

3.11. * A função de Dirichlet 2


é a função f : [O,1] ➔ IR tal que

J(x) = { O se x f/.Q
lsexEQ

Prove que fé descontínua em todo x E [O,l].

3.12. Seja f : [O,1] ➔ IR a função dada por

J(x) ={ O, se x f/.Q
¼,se x = r;:com m E Z+, n E N e mdc (m, n) = 1
Prove que f é descontínua nos racionais e contínua nos irracionais 3 do intervalo [O,1].

2 Apói,Gustav Lcjeunc Dirichlet, matemático alemão do século XIX.


3~ J>068Ívclprovar que, dado um intervalo /, não existe função / : / ➔ R que seja contínua em J n Q e
descontínua cm / n (R \ Q). A demonstração, que foge do escopo destas notas, pode ser encontrada no Capítulo
4 de Ili,

102
2. ontinuidad qu ncial
a ção r l ionamo o on i d limi d um qu • n i n rg nt e continui
d uma função. O r ul ado prin • wr qual rã utilizada diverll8
ão caracterizadas pela
d d tr formar m qu·n i onv r nt .
m udo o qu gu . I d no a um in rvalo da r ta.

TE RE 1 2.44.
m funç- f :I ➔ , ontínua . a guint condição for
aE J oda qu·n ia (an)n~l d de I temos

n
lim ª" = a =>n-+lim / (0-n)= f (a).

H V
Im i lm n , u n} u J' nt E/ >O Xl ô > O t l qu

1 E / 1r - oi < => 1/(r) - J(a)I <

n j (an)n~l UI < m r ind p r a. om limn + ª" =


Ho E t I u
11> 11 => la,1 - ai <

nj m - d u ndi - im n u

n> 11 ⇒ la,. - ai < ⇒ IJ(a,.) - f (a)I <


111 111 u iz r u limn J(an) = f (a).
up nh qu J n - , n ínua ma E J. ntão n gação da d finição de
> O l qu para odo ô > O t mo lf (x) - J(a)I > E
x - ai < ô. m p rticular omando n E ô = ¼,verificam

1
-ai< -
n

m d ondi - a 1m u qu·n ia (an)n>l im con trufda con•-•·-


p a, p o qu u·n i (J(an )n~l não onv rg para J(a).

103
' \ 1•1 1 1 1 1 1 ..: S H.J 1 1-:'\ < I \-.. I·. < .l l '\ I I '\ 1 11l \ I l I·.

Ilustremos a importância do resultado anterior em dois exemplos, o primeiro dos quais fornece
outra prova para o resultado do exemplo 19.

EXEMPLO 2.45.
Se a> Oe an = y'a, então an ~ 1.

PROVA.
Façamos a prova no caso em que a > 1, sendo a prova para o caso O < a < l totalmente
análoga. Se a > 1, então an > an+i > 1, para todo n E N. Portanto, a sequência (a,Jn;::1 é
monótona e limitada, logo convergente, pelo teorema de Bolzano-\Veierstrass.
Se l = limn-++ooan, segue do item (a) da proposição 15 que l > l. Por outro lado, a
continuidade da função x H Jx, x > O (cf. exemplo 35) e o resultado do teorema. anterior
garantem que
an ~ l ⇒ -/an~ Ji ==} a2n ~ Ji,
onde utilizamos, na última implicação acima, que Fn = a 2n. Ias, como toda subsequência
de uma sequência convergente converge para o mesmo limite, temos também que a2n ~ l, e a
unicidade do limite de sequências fornece / = Ji, de sorte que l = 1.

EXEMPLO 2.46.
Ache todas as funções contínuas / : R ➔ R, tais que / (x) + f (x2) = O, para todo x E JR.

SOLUÇÃO.
Seja f uma função qualquer satisfazendo as condições do enunciado. Fazendo sucessivamente
x = O ex= 1, obtemos /(O)= J(l) = O.
Agora, para x > O e aplicando a relação do enunciado repetidas vezes. obtemos

1(x) = - 1(vx) = 1(rx) = - 1(~) = ....


Em particular, temos 1/ ( 2vx)I = lf(x)I, para todo n EN.
Sendo an = 2ytx, segue do exemplo anterior que an ~ l. Por outro lado, como a função
1/ 1 também é contínua, o teorema 44 garante que

IJ(an)I ~ 1/(l)I = O.
Mas, como 1/(an)I = 1/(x)I para todo n, também temos que IJ(an)I ~ 1/(x)I. Logo, a unicidade
de limites de sequências implica lf(x)I = O.

104
Por fim, para x < O, temos x 2 > O; portanto, pelo que fizemos acima, /(x 2 ) = O. Ut,.... 1111!111!1!
este fato, juntamente com a relação do enunciado, obtemos f(x) = J(x) + J(x 2) = O.
Logo, a única função satisfazendo as condições do enunciado é a função identicamente n~

O teorema 44 permite provar facilmente as afirmações feitas no parágrafo imediatamente


anterior ao exemplo 38. Comecemos estabelecendo a continuidade da soma e do produto de
duas funções.

PROPOSIÇÃO 2.47.
Se/, g: / ➔ IR são funções contínuas em x 0 E/, então as funções/ ±g, f ·g: / ➔ R também
são contínuas em x 0.

PROVA

Seja (an)n>l uma quenc1a em /, tal que limn-++ ª" = xo. A continuidade das funções f e
g em x 0 garanle, de acordo com o teorema -14,que

Segue do item (b) da propo ição 1 que

= f(xo) ± g(xo) = (J ± g)(xo)-


Portanto, novamente pelo teorema 44, as funções J ± g são contínuas em x 0.
Por fim, o argumento para / • g utiliza o item (c) da proposição 18 e ê totalmente análogo
ao que fizemo acima, de forma que será deixado ao leitor.

Ante de analisar a continuidade do quociente de duas funções, precisamos de um resultado


auxiliar, conhecido como o lema de permanência do sinal para funções contínuas, o qual é
importante em si mesmo.

105
( • \ 1' 1 1 1 1 ( 1 ..! S E(J l •E~< •1 \ :-; 1-: <•o :\ 1 1~ 1•11>\ I>E

LEMA 2.48.
Sejam J e lR um intervalo e f : I ➔ lR uma função contínua. Se x 0 E J é tal que f(xo) > O
(resp. J(x 0) < O), então existe ó > O tal que

x E /, lx - xol <ó ⇒ f(x) > J(;o) (resp. f(x) < _ f(;·o) ).

Em particular,f aiada é positiva (resp. negativa) em J n (x 0 - ó. x 0 + ó).

PROVA.
Façamos a prova no caso em que f (x0) > O,sendo a prova no outro caso totalmente anúlogn
A definição de continuidade garante que, para 1: = f(~o) > O, existe ó > O tal que
J(.ro)
x E/, lx - xol <ó ⇒ lf(:r) - J(.ro)I < 1: = ~-
Por outro lado, a última desigualdade acima implica
J(.ro)
f(.r) - f(.ro) > -~

ou, o que é o mesmo, f (x) > f(~o), para todo .r E / n (.r0 - c5.. r 0 + ó).

Para o que segue, se g : / ➔ R é uma função contínua em :r 0 E J e tal que g(x 0) =/:-O, o
lema de permanência do sinal garante a existencia de c5> O tal que g não se anula no intervalo
J = I n (x 0 - ó, x 0 + ó). Portanto, restringindo g a J. se necessário, podemos supor, sem peida
de generalidade, que g não se anula em J.

PROPOSIÇÃO 2.49.
Sejam f,g : / ➔ R funções contínuas em xo E /. Se g não se anula em I, então a função
; : / ➔ Ré contínua em xo.

PROVA.
Como na demonstração da proposição anterior, se (an)n~I é uma sequência em 1, tal que
an = Xo, então
limn-++oo

Agora, o item (e) da proposição 18 garante que

lim f (an) = lim f(an) = ~n--roo f(an) = f(xo) = f (xo).


n-++oog n-++oog(an) limn-•+oo9(an) g(xo) g

106
Por fim, invocando o teorema 44 uma vez mais, concluímos que Lg é contínua em x 0.

A seguir, definimos uma noção mais f orle de continuidade para funções, a qual mostramos
ser equivalente à noção usual, para funções definidas em intervalos I = [a, b)e IR.

DEFINIÇÃO 2.50.
Uma função J : / ➔ IR é uniformemente contínua se a seguinte condição for sat
para todo f > O, existe ô > O tal que

x, y E /, lx - YI <ô ⇒ IJ(x) - J(y)I < f.


----------------'
Em palavras, a diferença entre a definição de função uniformemente contínua e a definição
de função contínua é que. dado f > O. para uma função uniformemente contínua o ô > O, cuja
existencia é garantida pela definição anterior. 6 depende do f > O escolhido, servindo para todos
.r, y E /; por outro lado, na definição 31 (de função contínua), tal ô depende, em princípio, tanto
do e quanto do r 0 E / fixado.
É imediato, a partir da definição anterior. que toda função uniformemente contínua é contí-
nua: basta fazei y = x 0, onde 1·0 é um elemento de / fixado arbitrariamente. Por outro lado, os
problemas ,1.4 e 1.6 trazem exemplos de funçõ contínuas que não são uniformemente contínuas.
O teorema a seguir garante que, para funçÕC!>definidas em intervalos fechados e limitados,
os conceitos de continuidade e continuidade uniforme coincidem.

TEOR8l\lA 2.51.
Toda função continua f: (a, b] ➔ Ré uniformemente contínua.

PHO\ \.
Por contradição, suponha que f é contínua mas que (2. 7) não seja válida. Então, existe f >O
tal que, para todo ô > O, podemos encontrar x. y E [a, b) satisfazendo

lx - YI < ô e IJ(x) - f(y)I > f.

Em particular, escolhendo ô= ¾,com n E N, concluímos pela existência de elementos Xn, Yn E


[a. b) tais que
(2.8)
Como (xn)n~t é uma sequência em [a, b), o teorema de Weierstrass 26 garante a existência
de uma sub equência (xn"'h~ 1 de {xn)n~l, convergindo para algum Xo E [a, b]. Então, segue da

107
desigualdade triangular que
1 k
IYn1,- Xol < IYnk- Xn1,1+ lxnk - xol < -
nk
+ lxnk - xol -+ o.
Portanto, (Yn1,)k~1 também converge para x0 , e o teorema 44, juntamente com a continuidade
da função modular, assegura que

lim IJ(xn1c)- f(Yn1,)I =


k-++oo
lf(xo) - f (xo)I = O.

Por outro lado, como caso particular de (2.8), temos lf (xn") - f(Yn1,)I > f, desigualdade em
flagrante contradição com o limite acima.

O teorema anterior admite a seguinte consequência importante.

COROLÁRIO 2.52.
Toda função contínua f : (a, b] ➔ R é limitada.
-~----~-----~-~-~---~----------"

PROVA.
Como f é uniformemente contínua, para f = 1 existe 6 > O tal que
x, y E [a, b], lx - YI <ô=} I/(x) - f(y)I < 1. (2.9)

Agora, escolha números reais a = xo < x1 < · · · < Xk = b tais que Xi - Xi-l < ô para
1 < i < k, e seja
1\1 = max{l/(xi)I, ... , I/(xk)I}. (2.10)
Para x E [a, b], existe 1 < j < k tal que x E (x,-1, x 3 ); em particular, lx-xj-il < lx 3 -x 1 _il <
6. Então, segue de (2.9) e (2.10) que

1/(x)I < I/(x) - /(x,)I + IJ(x 3 )1 < 1 + 1/(x3 )1 < 1 + M.


Por fim, como o x E [a, b)do parágrafo anterior foi escolhido arbitrariamente, temos 1/(x)I <
M + 1, para todo x E [a, b), de sorte que a função fé limitada.

A última propriedade de funções contínuas que estudaremos nessa seção é também devida
a Weierstrass, e afirma que toda função contínua J : [a,b) ➔ lR assume valores extremos no
intervalo [a,b).

108
TEORE~IA 2.53. \VEIER.STRASS
Se J : (a, b)➔ IR é urna função contínua, então existem .r"" :r,M E [~, b) tais que

f(xm) = min{f(x); x E [a, b)} e J(xM) = max{/(x); x E (a, b)}.


-----~~-~-~
PROVA.
Provaremos a existência de Xm, sendo a prova da existência de XM totalmente análoga. Como
f é limitada, existe
m = inf{f(x); x E [a, b]}.

Por outro lado, a definição de ínfimo garante (por intermédio do problema 1.8, página 75) a
existência de uma sequência (xn)n~l em (a, b} tal que f(xn) ➔ m. Mas, como toda sequência
limitada possui uma subsequência convergente, podemos tomar uma subsequência (xnk)k~1 de
(xn)n~l, que converge para um real x 0 E [a, b). Então, da continuidade de f obtemos, mediante
o teorema 44, que

Por outro lado, como a sequência (f (xnk))k~l é uma subsequência da sequência (J(xn))n~I
1
(que, por sua vez, converge para m), concluímos, a partir da proposição 15, que (J(xnk))k~l
também converge param. Logo, a unicidade do limite de sequências garante que f (x0 ) = m.

Problemas - Seção 2.4

4.1. Encontre todas as funções contínuas f : IR ➔ IR tais que, para todos x, y E IR, tenha.mos
J(x + y) = J(x) + J(y).

4.2. Seja f : IR ➔ IR uma função sobrejetiva, satisfazendo a seguinte propriedade: para. toda
sequência divergente (an)n~l, a sequênc11:1,
lf (an))n~l também é divergente. Prove que fé
bijetiva e que J- 1 é contínua.

4.3. * Dado um intervalo I e IR,dizemos que uma função f : / ➔ IRé lipschitziana 4 se exi tir
uma constante e > O (denominada a constante de Lipschitz de J) tal que

IJ(x) - f (y)I < clx - YI,


4
Após Rudolf Lipschitz, matemático alemão do século XIX.

109
para todos x, y E /. Por exemplo, vimos no exemplo 37 que

lsenx - senyj < lx - YI e Icosx - cosyl < lx - YI

para todos x, y E lR, de forma que as funções seno e cosseno são lipschitzianas. ambas
com constante de Lipschitz igual a 1. Prove que toda função lipschitziana f : I ~ lR e
uniformemente contínua.

4.4. Para n > 1 inteiro, mostre que a função f : IR. ➔ R. dada por f (x) = x". não é umforme-
mente contínua.

4.5. Sejam I C lR um intervalo e f : I ➔ lR uma função contínua Suponha que c~1stem


f > O e sequências (an)n~l e (bn)n~I em I, tais que lan - bnl ➔ O quando n ➔ + . mm,
IJ(an) - J(bn)I > e, para todo n > 1. ~Iostre que f não é uniformemente contínua.

4.6. Mostre que a função f : (O,+oo) ➔ lR, dada por f(.r) = scn ~- não é uniformcmcnt'
contínua.

4. 7. O propósito deste problema é apresentar uma outrn demonstração do tc01ema 53. Pnrn
tanto, recorde que, dada uma função contrmm J : la, b] ➔ IR.,temo f limitada 1wlo
corolário 52, de sorte que existem

m = inf {f(x); x E [a. b]} e ,\l = sup{J (:r); :.r E [a, bl}.

Faça, agora, os seguintes itens:

(a) Sem ~ Im (J) e g : [a,b] ➔ IRé a função dada por g(x) = f(z;-m 1 para todo x E [a, b],
mostre que existe e> O tal que g(x) < e, para todo x E [a, b].
(b) Ainda sob as hipóteses do item (a), conclua que f(x) > m + ¼,para todo x E [a, b],
e chegue a uma contradição.
(c) Argumente de maneira análoga aos itens (a) e (b) para mostrar que Jvf E Im (J).

4.8. Dados números reais a < b, dê exemplo de uma função contínua f : (a, b) ➔ IR,a qual seja
ilimitada inferior e superiormente.

4.9. Se f : (a, b) ➔ IRé uma função contínua e P(xo, Yo) é um ponto que não pertence ao gráfico
de f, definimos a distância de P a G f, denotada d( P: G f), por

d(P; G1) = inf{ A'P: A' E G1 }.

Se jy 0 -J(x 0 )1 < min{lxo-al, lxo-bl}, prove que existe A E G1 que realiza d(P;G 1), i.e.,
tal que d(P; G1) = AP.

110
4.10. • Seja J: 1R➔ lR a função polinomial dada para x E lR por

com ao. a1, ... , an E lR e an =/-O. Se n é par e an > O, prove que:


( a ) /(z)-ao
z"
> an - la..-11
- lzl -
~
••• - lzl"-1 se r O.
x -J.
(b) f(zJ;:ao > an - 1
! I:;,:; laJ I se !xi > l.
(e) Existe A > O tal que f(x) > aopara lxl > A.
(d) Existe xo E [-A, A] tal que f(x 0) = min{f(x); x E 1R}.

4.11. * O propósito deste problema é provar o teorema do ponto fixo de Banach 5 na reta.
Para tanto, sejam O < e < 1 real e f : IR ➔ 1Ruma função tal que

IJ(x) - J(y)I < clx - YI, \/ x, y E IR. (2.11)

Faça os seguintes item,

(a) E5colha .ro E 1Rarbitrariamente .. defina a ~quência (xn)n~i pondo Xk = f (xk_i),


para todo k E .N. PrO\c qu lx1.-+1- :r1.I < cl.r,..- xk d, para todo k EN.
{b) Conclua, a partir do exemplo 29, qu (:r11 )n~1 é convergente.

{e) Se o. = limn-t+ .r 11 , u~e a continuidade de / (a qual é garantida pelo problema 4.3)


para mostrar que o. é seu único ponto fixo.

4.12. De exemplo de uma fw1ção contínua/: 1R➔ 1R,sem pontos fixos e tal que IJ(x) - f(y)I <
1-r YI, para todos x, y E IR.

5 Após Stefan Banach, matemático polonês do século XX.

111
( \f 111 f 11 ..! SHll'J-::\( 1 \'-, (· ( ()'\ 1 ('\( 11)\()F

2.5 O teorema do valor intermediário


! esta seção, mostramos que funções contínuas definidas num intervalo [a, b] satisfazem a
propriedade do valor intermediário e, em seguida, apresentamos várias aplicações interessantes
desse fato. Começamos analisando o seguinte caso especial, conhecido como o teorema de
Bolzano.

TEOREMA 2.54. BOLZANO


Seja / : [a, b] ➔ R uma função contínua. Se f (a)f (b) < O, então existe e E (a, b) tal que
/(e)= O.

PROVA.
Suponha, sem perda de generalidade, que /(a) < O < J(b), e seja

A= {x E [a, b]; / é negativa no intenalo [a, x]}.

Como a E :A por hipótese, temos A f: 0. Por outro lado. A é limitado (uma vez que A e [a, b])
e, portanto, existe e= sup A. Mostraremos que f (c) = O.
Afirmamos inicialmente que e> a. De fato, como /(a) < O, o lema 4 (de permanência do
sinal) garante a existência de O < ô < b - a tal que f (i:) < O para :r E [a, a+ 6); logo, e > a+ ô.
Agora, suponha que fosse /(e) < O. Então, e< b (uma vez que J(b) > O) e, novam ntc pelo
lema de permanência do sinal, existiria O < ô < b - e tal que / seria negativa em (e - ô, e+ ô).
Mas, como e= sup A, podemos tomar d E (e - ô, e) n A, de sorte que f < O em [a, d]; portanto,
teríamos f < O em [a, d] U (e - ô,e+ ô] = [a, e+ ô), contradizendo o fato de ser e= sup A.
Por fim, suponha que fosse /(e) > O. Então, (uma vez mais pelo lema de permanencia do
sinal) existiria ô> Otal que/ seria positiva em (e-ô, c+EJ)n(a, b];em particular, An(c-ô, e] = 0,
e teríamos sup A < e - ô, o que é contradição.
Logo, a única possibilidade é termos f (e) = O.

~MPLO 2.55.
Se / : R ➔ R é uma função polinomial da forma

•.. , Gn e R, cin I: Oe n ímpar, então a imagem de / é o conjunto de todos os números


MílCIJW, / possui pelo menos uma raiz real.
-------~----~---
112
() llt>I0-\1\ IHI\\I.UH l'\Ill:\IIIH\Hll1 1

PROVA.
Dado d E R, faça g(x) = f(x) - d. Então, g: lR ➔ lR é uma função polinomial satisfazendo
as mesmas hipóteses de f e, para e E R, temos J(c) = d se, e só se, g(c) = O.
O argumento do parãgrafo anterior reduz nosso problema a garantir a existência de e E R
tal que /(e) = O, para o que utilizaremos o teorema de Bolzano. Suponhamos, sem perda de
generalidade, que ª" > O. Então, para x f:- O. repetidas aplicações da desigualdade triangular
fornecem

f(x) On-1 01 ao
=an +--+···+--+-
x" X xn-l X"
ªn-1 01 ao 1
>a
- n
- l --+···+--+-
X xn-l X"

> On -1 1-...-1~ 1-1ao1


On-l
X xn-l X"
lan-d
=a----···----- lad laol
" lxl lxln-l lxl"•
Portanto, se 1.ti > 1,.cnt ão l,r.l< lxl2 < • · · ~ lxl" l!. dni,

o qunl, por sun YCZ, é po:,iti\'o pnrn lxl > :- 1:·.;~la,I.


Em re~umo, s A > mnx { 1. ;!:-L~,-~ la,I}, então '!~) > O para x = ±A. Mas, como n é
impnr, segue que f (-A) < O < J(A) e o teorema de Bolzano garante a existência de e E [-A, A]
tnl que f(c) = O.

l'\o~o próximo resulta.do é um refinamento do teorema de Bolzano, o qual mostra que funções
cont 1mm definidas em inten-alos satisfazem ? .....opriedade do valor intermediário. Por tal razão,
esse rCõultado é conhecido como o teorema do valor intermediário. Doravante, referir-nos-
emos ao mesmo simplesmente como o TVI.

TEOREMA 2.56. TVI


Sejam f, g : [a, b] ➔ R funções contínuas. Se J(a) < g(a) e J(b) > g(b) (ou vice-versa),
então existe e E (a 1 b) tal que J(c) = g(c). Em particular, se um real d pertence ao intervalo de
extremos f (a) e J(b), então existe e E [a, b]tal que J(c) = d.

113
PROVA.
Para a primeira afirmação, note que a função h = f - g é contínua e tal que h(a)h(b)
(J(a)- g(a))(J(b) - g(b)) < O. Portanto, o teorema de Bolzano garante a existência de e E (a. b)
tal que h(c) = O, i.e., tal que f (e) = g(c).
O caso particular em questão é obtido tomando g igual à função constante e igual a d.

Como primeira aplicação do TVI, mostremos que funções contínuas definida em mte1Ynlo::,
têm intervalos por imagens.

COROLÁRIO 2.57.
Se 1 C Ré um intervalo e f: / ➔ Ré uma função contínua. então n imngcm de f tnmbcm
é um intervalo. Se / = (a, b], então exist~m reais e ~ d tais que Im (!) = (e,d].

PROVA.
Suponhamos, inicialmente, que I = (a, b1 PC'lo tcorcmn de \Vc1erslras!:>53, existem .r,,., r,\/ E
[a,b] tais que J atinge seus valores mínimo e mú.\'.imoem .r111 e cm ,l',\J, re p ct1vame11te Sc11clo
f (xm) = e e J(xu) = d, ternos, em pnrticulnr. que Im (J) C (e, e/). Por outro Indo, fixado
y E [e, d], o TVI garante a existência de um real .r pe1tencentc no intervalo d extremob ;1,m e
XM, tal que f(x) = y. Em particular, Im (!)::>(e dj.

Para o caso geral, observe que todo inten-alo / pode ser c~cnto da forma I = Un~l [a11 , b,1 ],
para intervalos [an,bn]tais que [a1, bi] C [a2. b-2]C ... (por e.,xemplo,se I = (a. b), podemos tomar
an =a+! e bn = b - ¾ - os demais casos podem ser considerados de forma análoga) Agora,
sendo [Cn,dn] a imagem do intervalo [an,bn] por J. é imediato verificar que [c ,d e [c ,d e ., 1 1
]

2 2
]

de forma que Un2'. 1 [Cn,dn] é um intervalo J. Por outro lado 1 também é fácil ver que, como
[ = Un2'.1[an,bnl, temos
Im (J) = LJ(en.dn] = J.
n~l

No que segue, discutimos mais algumas aplicações interessantes do TVI.

: [O,1) ➔ [O,1) uma função contínua. Prove que existe um real O < e < 1 tal que
/ tem pelo menos um ponto fixo).
;,...<.:..~~--""---"-~----- --------~--~----~-

114
PRO\A.
Se /{O) = Oou /(1) = 1, nada há a fazer; senão, J(O) > Oe J(l) < 1. Considerandoa fuoçio
g : (O,1] ➔ lR dada por g(x) = x, temos, então, f (a) > g(a) e J(b) < g(b). Portanto, o TVI
garante a existência de O < e < 1 tal que /(e) = g(c) ou, o que é o mesmo, f (c) = e.

EXEMPLO 2.59.
Calcule, para x > O, o número de soluções da equação
1 1
- + 2 =
X X
X + VX+ ifx.

SOLUÇÃO.
Afirmamos que a equação acima admite somente uma solução. Primeiramente, note que,
para x > O e k inteiro positivo, a função x H -:,; é decrescente, ao passo que a função x H .ifi é
crescente. Mas, como uma soma de funções crescentes de mesmo domínio é crescente e uma soma
de funções decrescentes de mesmo domínio é decrescente (prove este fato!), concluímos que as
funções/, g: {O,+oo) ➔ lR, dadas por f(x) =¼+~e g(x) = x+ Jx+ ?txsão, respectivamente,
decrescente e crescente. Como as soluções da equação do enunciado correspondem aos valores
positivos de x tais que f(x) = g(x), o problema 3.17, página 43, garante que tal equação tem.
no máximo, uma solução.
Para mostrarmos que há solução, vamos usar o TVI, observando inicialmente que / e g são
claramente contínuas em (O,+oo). Para tanto, consideremos dois casos separadamente:

• para x > 1, temos ~ < ½ex > y'x > ?'x,de maneira
3
que f(x) < ~ e g(x) > 3~. Em
particular, J(x) < g(x) se ~ < 3-VX,i.e., se x > (i) 4 • Portanto, / (x) < g(x) para x > 1.

• para O < x < 1, temos -}I> ¼ex < ,/x < -Yx,~e maneira que f(x) > ~ e g(x) < 3'7'x. Em
particular, J(x) > g(x) se~ > 3ffe, i.e., se x < (i) 4 . Portanto, f (x) > g(:r) para O< x < (D¾.

Finalmente, a discussão acima garante que podemos tomar reais O < a < l < b tais que
f (a) > g(a) e f (b) < g(b), de maneira que o TVI garante a existência de e E (a, b) tal que
J(c) = g(c).

115
1\1·1111(1..! SHJlT:'-:<'I\"' 1· <'<l\ll'\111)\lll·:

EXEl\f PLO 2.60.


Existe uma função contínua f : lR ➔ 1Rtal que

SOLUÇÃO.
Suponha que exista uma tal J, e defina g : 1R➔ 1Rpor g(x) = J(x + l) - f(:r). Então, g e
contínua (pela regra da cadeia para funções contínuas) e, pela condição do enunciado, trnnsforma
todo número real num irracional. Mas, como todo inten-alo contém número!> racionms (cf
problema 1.2, página 74), a fim de não obtermos uma contradição ao TVI, a única possibiliclnclc
é g ser constante. Assim, existe um número irracional a tal que J(x + 1) - f (x) =o pnin todo
x E lR. Daí,

f (x + 2) - J(x) = f(:r. + 2) - J(:r + 1) + J(.r + 1) - f(:i:) = 2o. (2.12)

também p~ra todo x E 1R.


Agora, afirmamos que existe x 0 E 1Rtnl que J(.r 0) E Q; de foto. tome um real quulqu 't a, ~e
J(a) não for racional, segue de nossas h1pót~c que f (o+ 1) . crá racionnl e basta, enlao, tomai
xo = a ou x 0 = a + 1.
Fixado x 0 como no parágrafo anterior, HS h1pót - . obre f asseguram que

J(xo) E Q => f (.ro + 1) r/:Q => J(:1:0+ 2) E Q.

Logo, f (x0 + 2) - f (x0 ) E Q, o que contradiz (2.12) e termina a demonstração.

Terminamos esta seção utilizando o TVI para estudar a continujdade da inversa de uma
função contínua cujo domínjo ê um intervalo.

1'EoREMA 2.61.
Se I e R é um intervalo e / : / ➔ R é uma função contínua. então / é injetiva se, e só se, J
é crescente ou decrescente. Ademais, nesse caso:

la) A imagem J de f é um intervalo de mesmo tipo (i.e .. aberto. semiaberto ou fechado) que
I.

116
PRO\.\.
Se f não é injetiva, então f claramente não pode ser nem crescente, nem decrescente. Reci-
procamente, se f não é nem crescente, nem decrescente, cnlão existem o < b < e em 1 taí8 qde
/(a) < J(b) > J(c) ou f(a) > J(b) < f(c). Suponha que J(a) < J(b) > J(c) (o outro cuo ~
análogo) e escolha d E 1Rtal que

max{f(a), /(e)} <d< J(b).

O TVI garante a existência de x 0 E (a, b) e x 1 E (b,e) (logo, x0 , x1 E J) tais que f (xo) =de
/(xi) = d, de modo que f(xo) = f(xi). Em particular, f não é injetiva.

(a) Pelo corolário 57, sabemos que J é um intervalo. Como f é injetiva, podemos supor, pelo
item (a), que/ é crescente (o caso em que f é decrescente é análogo). Suponha que I = (a. b),
com a, b E 1R(os demais casos também podem ser tratados de modo análogo).
Se Im (/) = (e, d], tome x 0 E (a, b) tal que f (x 0 ) = d. Então, como f é crescente, para
x E (xo, b) temos J(x) > f(xo) = d, o que contracliz o fato de que f(x 0 ) E Im (/). Analoga-
mente, mostramos que Im (/) -=/[e,d), [e,d], de sorte que Im (/) também é um intervalo aberto.

{b) Analisemos o caso em que I = (a, b) e Im(f) = (e, d), sendo a análise dos demais casos
totalmente análoga. Inicialmente, observamos (cf. problema 4.8, página 47), que 1- 1 é crescente.
Agora, fixado y0 E (e, d), seja x 0 = 1- 1 (y0 ). Dado E> O, queremos ô> O tal que

(2.13)

Para tanto, seja 60 = min{y0 - e, d-y 0 } e suponha, inicialmente, que O< ô < ôo (de sorte que a
condição IY-y 0 1 < ô seja suficiente para garantir que y E (e, d)). Denotando x = J- 1 (y), temos
y = J(x) e podemos reescrever (2.13) da seguinte forma: queremos O< ô< ôo tal que

lf(x) - f(xo)I <ô=> lx - xol < €.

Observe que podemos supor E > O tão pequeno que x 0 ± e E (a, b) (senão, diminua o E >O
dado, de forma que essa condição seja satisfeita). Lembrando que f ê crescente, tome

O< ô< min{ôo,J(xo + E) - J(xo), f(xo) - J(xo - e)}.

Então,

J(x) - f(xo) < ô =>f (x) - f (xo) < f(xo + t:) - f(l·o)
=>J(x) < J(xo + t) =>:r < .ro + t

117
( \1'111 1,, .! SE(2l'E:\<'I.\~ F <"O:\Tl:\l'll).\[)E

e, analogamente,
J(x) - J(xo) > -ó ⇒ x > Xo - €.

Em qualquer caso, a escolha acima para ó garante que


lf (x) - J(xo)I < ó ⇒ -ó< f(x) - J(xo) < ó
==} -€ < X - Xo < €

⇒ lx -xol < E,

conforme desejado.

Como primeira aplicação do teorema acima, damos uma prova alternativa para o problema
3.8, pâgina 102.

EXEMPLO 2.62.
Para n E N, a função potência n-ésima é a função f : IR ➔ IR dada por J(x) .rn. =
Se n é par, denote ainda por f a restrição f : [O+ oo) ➔ IR. Como f é contínua, crescente e
f(k) = k" ·> k, para todo k E N, temos pelo TVI que Im (/) = [O,+oo). Portanto, o teorema
anterior garante a boa definição e a continuidade da função

1- 1 : [O,+oo) ~ [O.+oo)
X ~ ::fi

Analogamente, se n é ímpar, então f é contínua, crescente, lm (/) = IRe


J- 1 : Ilt ~ IR
X ~ ::fi

é continua. Em qualquer um dos casos acima, a função J- 1 é denominada a função raiz


n-êaima.

O próximo exemplo estabelece a continuidade das funções trigonométricas inversas, introdu-


zidas no problema 6.1, página 64.

~PLO 2.63.
A restrição da função seno ao intervalo [-~, ~], que também denotaremos por sen : [-%, ~] ➔
(,,-i, 1),é uma bijeção contínua e crescente. A função arco-seno é sua inversa arcsen : [-1, l] ➔
f-f,f], de sorte que, para x E [-1, l] e y E [-i, iL
arcsen x = y {=} sen y = x.

118
Graças ao teorema 61. a função arco-seno é crescente e contínua.
Analogamente. a inYersa de cos : [O.11]➔ (-1. l] é função ar~cosseno
(O,1r).a qual é contínua (n0\1UDente pelo teorema 61) e decrescente. Veja cmm,ei
x E [-1.1] e y E [O,,.],temos
arccosx = y ç::} cosy = x.
Por fim, a função arco-tangente arctg : R ➔ (-i- i) é a im·ersa da restrição tg : (-j, i
IRda função tangente, de sorte que é contínua e cre-cente. Também. para x E R e y E (-j,l ,
tcm1os
arctgx =y ç=} tgy = x.

Problem ção 2.5


5.1. A fu11çãoconthma f: (- 1) U (l. + )➔ , dnda por

) l

snt isfoz /{O) < ~ < f (2), m não e no domínio de J tal que J(c) = ~-Por que esse
cxelllplo 11iiocontrario o TVI?
r.: •)
,.) ..... 'nlc11l• o 11ú111l)JO dl• oluçõ J1"tu d e d um dns equações abaixo:

(n) 1-1+ l = x4 •
(h) r.o~ • = x 2 .

{e) 'l"'ll .1' = i•

5.3. \lost H' que n full(:ão f : ➔ , dada por J(:r) = :t.3 en x. é tal que seu gráfico intersecta
1odn 1Na nno , ~rt icnJ cm um couj um o infinito de pontos.

5.4. • S1., f : [O 1) ➔ é uma função contínua. tal que J(r) > O para todo racional diádico
r E tO,l]. proYl' q1P J(x) ~ O. para todo x E [O.1).

5.5. • rJ<'Undndo:--um inten-alo I e um ubconjunto não vazio X de I, tendo as seguintes


propnedad~

(i) Para todo x 0 E X. existe J > O tal que I n (xo - ó,xo + ó) e X.


(ii) e (an)n~l é unia equência de pontos de X e l E / é tal que an ➔ L, então l E X.

119
( \l'l 11111 .1 SHJl'l-:\('f\-.; I·. ('<>\ 11\I 11>.\l>I·:

Mostre que X - 1.

5.6. Sejam n > 1 inteiro e a 1 , a 2 , ... , an reais positivos dados. Prove que a equação

possui exatamente uma solução real positiva.

5.7. Sejam x1,x2, ... ,Xn reais escolhidos do intervalo [O,l]. Prove que existe x E [O,l] tal que

5.8. Seja/ : JR➔ JRuma função contínua satisfazendo, para todo x E JR,a relação / (x)f (x +
2) + J(x + 1) = O. Se /(O) > O, mostre que existem infinitos valores reais de x tais que
f(x) = O.

5.9. Seja / : 1R ➔ 1Ruma função contínua tal que f(x)f (J(x)) - 1, para todo x real. Se
/(1000) = 999, calcule / (500).
5.10. Seja/ : JR➔ JRuma função tal que f (x + 1)/(f (x) + 1) + 1 = O, para todo real x. Prove
que f não é contínua.

5.11. Seja/: [O,1] ➔ [O,l] urna função contínua, tal que (/ o J)(x) = x, para todo x E [O,l].

(a) Mostre que f é uma bijeção crescente ou decrescente.


(b) Se/ é crescente, mostre que f(x) = x, para todo x E [O,1).
(c) Mostre que há infinidade de possibilidades para/, se/ é decrescente.

5.12. Seja f : JR➔ JRuma função contínua que assume valores positivos e negativos. Dado k > 2
natural, prove que existem reais distintos a1, a2, ... , ak, em progressão aritmética e tais
que

5.13. As funções contínuas J,g: [O,l] ➔ [O,l] são tais que/ og = gof. Se f for não decrescente,
prove que existe O <a< 1 tal que /(a) = g(a) = a.

120
1 \1·111 11 i \ Ll\1111·.S 1-: ()1-:1<1\·_\l>.\S

osso propósito neste capítulo é colocar em bases sólidas a abordagem heurística do conceito
1
de derivada, delineada na introdução. Ao longo do caminho, mostraremos como utilizar derivadas
para resolver, pelo menos em tese, os problemas da obtenção dos intervalos de monotonicidade e
concavidade de uma função (duas vezes) derivável, bem como do esboço razoavelmente acurado
dos gráficos de tais funções. Por sua vez, a análise de tais problemas suscitará várias aplicações
interessantes do conceito de derivada a problemas de máximos e mínimos.

3.1 Limites de funções


Sejam dados uma função contínua J : (a, b) ➔ IRe x 0 E ( a, b). Recordando nossa discussão
preliminar do conceito de derivada, levada a cabo na introdução, concluímos que, se existir uma
noção razoável de reta tangente ao gráfico de J no ponto A(x 0 , J(x 0 )), e se tal reta não for
vertical, então sua equação deve ter a forma

Y - J(xo) = m(x - xo), (3.1)

onde
1n = lim J(x) - J(xo). (3.2)
x~:ro X - Xo

..A«iui,a expressão do segundo membro acima representa intuitivamente o valor limite dos quo-
cientes f(xl=~~xo) à medida que x tende a (i.e., se aproxima de) x 0 , caso tal "limite' exista em
algum sentido a ser precisado.
A discussão anterior ainda se encontra num patamar relativamente vago, de forma que adi-
amos seu prosseguimento para a próxima seção, a fim definir rigorosamente a noção de limite.
Para tanto, começamos com a definição a seguir.

DBFJJfIÇÃO3.1.
Fixado a e R, uma vizinhança de a é um intervalo da forma (a - r, a+ r), onde r é um real
positivo. Nesse caso, diremos quer é o raio da vizinhança (a-r, a+r) e que todo x E (a-r, a+r)
é uma aproximação de a com erro menor que -~-----------------_J
r.

Sejam I C IR um intervalo, x 0 E J e f : I \ { xo} ➔ IR uma função dada. Gostaríamos de


formular uma definição precisa para a afirmação de que podemos tornar f(x) tão próximo de L
quanto desejado, bastando, para tanto, tomar x E J suficientemente próximo (mas diferente) de
Xo.
Um pouco de reflexão permite concluir que uma formulação razoável desse conceito é obtida
exigindo-se que, para cada vizinhança dada de L, exista uma vizinhança de x 0 que seja aplicada
por / naquela vizinhança.

122
v z qu uma \izinhan d 0 D d Illlil

r,
E ( o - r.xo - r) {xo} <=}O< :e- xol < r.
pdm m 12 a • ( f. Firur .1).

o 3.2.
Je
1 um in rva.lo E l f: f { o} um funç- o dada. Dizem
limi L quar do nd no

lim f(x
:r.
= L. (3.3)

>Od Ir UI >0
(3. )

i u .l: limit un fun -

11\J rr qu n rro > O par L x ir um rro ô > O


m rro m nor do qu ô rr pondam a
m ri am n qu r mo qu para todo
d x0 o pon o do gráfi o d J om ab i
3.1.
ud d n inwdad ) v 1 fri r qu j tifi ar a va.lidad
mpr o d d ima • um Jogo d gato rato: arbitrado
did o L limi . d r apaz d n ontrar um rro ô > O

13
l.1\1111" 1 l li 1;1\ \1 >,__,

para o (o qual m rc l d p nd r· an do dado quan o do própri x 0 ) d modo qu


validad da condiç- O < l:i: - x 0 1 < ó par um l m n o x E I arr a v lidad da n 1ç
1/( ) - LI< .
p o udo d ontinui ad 1i r nt j dquiriu 1 m famili id om
dinâmica d ncon rar ó > O ad quad um > O dado. di v r m
seguir (um pouco a título d r ordação) doi orno impl m n tra 'gia
acima. Em tudo o qu gu por v z omitir mo quai qu r r t r"n i íci
e/ou contradomínio das funçõ nvolvidas on n rando-no õ da
variável independen (x m g ral). mpr qu míni d
função é seu domínio maxim 1 d d finição ( f. ção 1.1), on r domíni • njun o d
números reais.

~➔2(-2x + 7) = 3: s ja dado > O. ar indo d x E uj ito a um rr tipo


O< 1~- 21 < 6 temos

1(-2x + 7) - 31= 1- 2x + 1= 2lr - 21< 28.

Então, escolhendo 6 > O d tal modo qu 2ó < E t m qu

x E 1Re O< lx - 21<ó=> l(-2x + 7) - 31< 2ó <

~119➔:sx2 = 9: seja dado E> O. Par indo d x E uj i o um rro do ipo O< lx- 1< ó

lx2 - 91 = lx - 31I + 31< ólx - 3 + 61


< ó(lx - 31+ 6) < ó(ó + 6).
a desigualdade triangular na penúl ima p
iJif.&IJlos ag m acima. Por ant , e o
vel escolhermos 6 > O de tal forma qu ó(ó + 6) < E, r mo qu

x E 1Re O< lx - 31< ó =>1 2 - 91< ó(ó + ) $ E

Resta mo trar qu a olh d ó > O • po ív 1, para o qu b a


~•-vei-llU ó(ó+ 6) < e. Ao fazê-lo obt mo

O<ó < Yf+9-3.

1
Analisando em retrospecto os dois exemplos acima, concluímos que, se tiverm08 uma funçio
J : I \ {xo} ➔ IR e quisermos provar que limx-+.coJ(:r) - L, então, conforme comentado na
seção 2.3, o segredo para descobrir quais ó > O funcionam para um e > O dado é argumentar da
seguinte forma: partindo de um x E I sujeito a um erro do tipo O < lx-x 0 1< ó, estimamos o erro
1/(x)-LI em termos de ó por excesso, obtendo uma desigualdade do tipo IJ(x)-LI < E(ó), onde
E representa uma certa função de ó (no exemplo do item (a), encontramos E(ó) = 2ó, enquanto
naquele do item (b) encontramos E(ó) = ó(ó + 6)). Em seguida, impomos que tal erro E(ô)
não ultrapasse o erro € desejado, descobrindo, então, os valores apropriados de ,5 ( usualmente,
esse segundo passo resume-se a resolver, para ó > O, a inequação E(ó) < e). Por fim, se ó> O
satisfizer E(ó) < €, teremos claramente que

x E I e O < lx - xol <ó ⇒ lf (x) - LI < E(ó) < €,

conforme desejado.
Observe que a única diferença entre a discussão até aqui e aquela da seção 2.3 reside no
fato de que, diferentemente de lá, aqui nós temos de partir do pressuposto de que lx - x 0 1 > O,
i.e., de que x -=/-x 0 . Por outro lado, conforme explica a proposição a seguir, essa diferença não
transpareceu nos exemplos acima porque as funções x t-+ - 2x + 7 e x t-+ x2 são contínuas (em
todo x 0 E IR).

PROPOSIÇÃO 3.4.
Sejam I e IR um intervalo e/: I ➔ Ruma função dada. Para x 0 E/, temos/ continua em
x 0 se, e só se,
lim / (x)
:r-+:z:o
= /(xo)- (&5)

PROVA.
Suponha, inicialmente, que f seja contínua em xo. Então, a definição 31 garante que, dado
€>O, existe ó> O para o qual

x E I e lx - xol <ó ⇒ IJ(x) - J(xo)I < €.

Em particular, se x E I e O< lx - x 1 < ó, ainda teremos lf(x) - f(xo)I < f, de sorte que (3.4)
J(x) = !( o)-
ê satisfeita. Logo, limx-+:z:o
Reciprocamente, suponha que li x-+xo J(x = J(x 0 ). Então, de acordo com a definição 2,
dado € > O, existe ó > O tal que

x E I e O< 1 - xol < ⇒ 1/(x) - J(i-o)I < f.


um z qu ndi ~ lf ( ) - f( o)I < , rivialm n
nt p d m im m

xEI 1 - xol < =>1/(x) - /(xo)I <


P rtan o, a d finiçã 1 ar n qu f ' n ínu m o-

.s. 3
{±1} ➔ Ré a função dada por/( ) = z ;;~~ 2 , u iliz mo a pr p iç- nt ri r
~•IIUII" lim:r-+lf (x). Para tanto ob rv mo ini iclm n qu l • raiz d num rc l r
llbminador da expr ão qu d fin J(x) om x 3 - 3 2 .., = (x - 1)(.r 2 - __,. - -)
(z - l)(x + 1). Por outr lado m \ {±1} m

(x - l)(x 2 - 2.r - ) x2 - .x - 2
f(x)= (x-l)(x+l) -= .r 1 •

~ que a função g:
2 2 2
\ {-1} IRdad por g(.c) = r ; ~- , e n ínua, pr p i~r o

3
lim J(x) = lim g(x) = g(l) = --
x-+l :r 1 2

m fato impor ant br limi qu limx :ro J(:1:)= L


exist uma vizinhança d o al qu J m inal d L m tod pont
contidos m tal vizinh n a ( x m xo). , gro o modo on
r ultado a eguir o qual onsti ui no an, lo o do 1 m nd mb'm onh id mo
lema de permanência do inal.

1 C Rum int rvalo xo E J f: I\ {.ro} ➔ uma funç~ d da. lim:r .ro f (.r.) = L.
,!: O, então exist 6 > O tal qu

L/2 < f(x) < 3L/-. L>O


z E/ e O< lx - xol <ô=> { _ 3L/ 2 < f (x) < -L/ 2 L<O

ROVA.
uponha L > O (o ). 1 d fini - d limi d do = ½> Ü. XI
6 > O tal qu
L
xEl O< 1• - rol < =>1/( ) - LI < -.

1...
Portanto. para cada um de tais x. temos-½ < J(x)-L < ½ ou, o que é o mesmo,½ < /(z) < 'f•

De posse do lema anterior, colecionamos na proposição a seguir algumas regras operat6rias


que simplificam o cálculo efetiYo de limites. Para o item (c) da mesma, cumpre observarmos o
seguinte: dados I C 1Rintervalo, x 0 E J e g : J \ {x0 } -+ 1Rtal que limx-+zog(x) = M, com
.\1 =/-O, o lema 6 garante a existência de r > O tal que a função g não se anula em J \ {xo},
onde J é o intervalo J = I n (x 0 - r, x 0 + r). Portanto, ao considerarmos a função ¼,tal que
!(x) = 9 /x)' sempre suporemos implicitamente que seu domínio é J \ {x0 }.

PROPOSIÇÃO 3. 7.
Sejam/ C Rum intervalo, x 0 E/ e f.g: I\ {x0 }-+ 1Rduas funções dadas. Se limz➔zo /(z) =
L e lim.c-.cog(x) = Af, então:

(b) limr-+.co(f • g) = L • AI.


(c) lim.c-no (i)(:1')= Íí I caso J\f i=O.

PHO\ \
Em lodo~ o item, a eguir, suponhamos dado f > O.

(a) Façamos a prm·a para/ +g, endo a prova para f-g completamente análoga. Como indicado
anteriormente, tentaremos estimar l(J + g)(x) - (L + M)I por excesso em termos de 1/(x)- LI
e lg(x) - ,'11.Como

l(J + g)(x) - (L + AI)I = (J(x) - L) + (g(x) - M)I


< lf(x) - LI+ jg(x) - MI

(pela de igualdade triangular), a fim de que seja l(J + g)(x) - (L + M)I < f para x E / próximo
a (mas diferente de) x 0, é suficiente que tenhamos lf(x) - LI < ½e lg(x) - MI < ½·Mas, como
½> O e lim.r-+.roJ(x) = L e lim.r-+.rog(x) = iU, a definição de limite garante a existência de reais
po itiYOS81 e 82 tais que
f
x E J e O < lx - xol < 81 ⇒ lf (x) - LI <
2

127
1 1l 1 11 1 ; L 1\ 11I 1 " I 1)f- I{( \ \ I> \"

e

x E J e O < lx - xol < 82 =>lg(x) - MI <
2.
Portanto, sendo 8 = min{8 1 , 82}, temos 8 > O, e a concomitância das condições x E I e O< lx -
.1:ol< 8 acarreta simultaneamente 1/ (x)- LI < ½e lg(x)-Nll < ½·Assim, para O < lx-xol < 8,
temos
€ €
1(/_+ g)(x) - (L + M)I < lf(x) - LI+ lg(x) - MI<
2 2 = €,
+
conforme desejado.

(b) Novamente aqui, estimemos l(Jg)(x)-LMI por excesso, em termos de lf(x)-LI e lg(x)-1\ll.
Inicialmente, segue da desigualdade triangular que

l(Jg)(x) - LMI = IJ(x)(g(x) - A,I)+ (J(x) - L)All


< IJ(x)llg(x) - Ali+ IJ(x) - LIIAJI
< (IJ(x) - LI+ ILl)lg(x) - Ali+ IJ(x) - LIIAifl
= IJ(x) - Lllg(x) - Ali+ ILllg(x) - Ali+ IAlll/ (x) - LI.

Portanto, a fim de que seja l(Jg)(x) - LAll <€para x E J próximo a (mas diferente de) x 0 , é
sl1Ílciente que tenhamos cada uma das parcelas IJ(x)-Lllg(x)-A,ll, ILllg(x)-1"1I e l1"lllf (x)-LI
menor que ! . Para tanto, basta que tenhamos, por exemplo,

1/(x) - LI, lg(x) - MI < i,


€ €
lg(x) - Jvíl < 3(ILI + 1) e IJ(x) - LI < 3(IJ\,f1+ 1).
Em suma, é suficiente que tenhamos

1/(x) - LI < min { i. 3(IM~ + 1)}


e

lg(x) - MI < min { i, 3(IL~+ 1)}.

Mas, se €1 = min { /f, 3(IA;l+l)} e €2 = min { Jf, 3c1LÍ+i)}, então €1,€2 > O, e a definição de
limite garante a existência de erros 81,82 > O trus que lf(x) - LI < t:1, para todo x E J tal que
O < lx - x 0 1< 81 , e lg(x) - A,ll < €2, para todo x E I tal que O < lx - xol < 82. Portanto, se
c5= min{c5i,c52}, então c5> o e a concomitância das condições x E/ e O< lx - xol < c5acarreta,
simultaneamente, 1/(x) - LI < €i e lg(x) - Nll < €2, como necessário.

128
(c) O lema de permanência do sinal garante a existência de 60 > O tal que jg(x)I > ~ JNII&
x E J e O < lx - xol < ôo. Portanto (e conforme o parágrafo anterior à prova), consideramoe~
como a função i :J \ {xo} ➔ lR, onde J = I n (x 0 - 60 , x 0 + 60 ).
Como i = J · } , pelo item (b) basta mostrarmos que limx➔xo = ;,,. Para tanto, como
g(~) DOI

itens anteriores, estimemos I g(~) - por excesso em termos de lg(:r) - MI. Como lg(x)I > ~
)11
para x E J \ {x0}, temos

1 1 lg(x) - A,JI 2
g(x) - M = lg(x)ll1'11 < 1'12 lg(x) - MI;

f
portanto, a fim de que seja 1/x) - 1 1 < 1:, basta termos lg(x) - MI < ~ E.
2

Assim, adicionalmente ao 60 > O acima, basta escolhermos {invocando a definição de limite)


2
um real ô1 > O para o qual lg(x) - MI < A~ t., para todo x E J tal que O< lx - x 0 1 < '51 ; sendo
ô= min{óo, ói} > O, temos, para x E J e O< lx - x 0 1< ó, que lg(x)I < ~ e lg(x) - MI < ,_1;f,
conforme necessário.

Uma fácil indução permite estender as fórmulas dos itens (a) e (b) da proposição anterior a
uma quantidade finita de funções. Especificamente, se I é um intervalo, x 0 E I e /i, ... , fn :
I \ {Xo} ➔ lRsão tais que limz-t:i:oÍJ (x) = LJ para 1 < j < n, então
lim (/i ±
:r-tro
h ± · · · ± f n)(x) = L1 ± L2 ±···±Ln (3.6)

e
(3.7)

Em particular, se k E N e f: J \ {xo} ➔ lRé tal que limx➔xo J(x) = L, então


lim J(xl = Lk_
z-+zo

Doravante, assumiremos as observações acim" "!m maiores comentários.


A proposição a seguir é conhecida na literatura como o teorema do confronto. Conforme
veremos logo em seguida à sua prova, ela é muito útil para o cálculo de limites.

PROPOSIÇÃO 3.8.
Sejam/um intervalo, x 0 E/ ef,g,h: J\{xo} ➔ Rfunçõestaisqueg(z)
de extremidades f(x) e h(x), para todo x E I\ {xo}. Se limx-+zof(x) = Um• •..,
limx-+zo g(x) também existe e é igual a L.

129
( \ 1 1 1 1 1,, : LI\111 F~ 1·: 1)1-:Bl\'.\ll.\-.;

PHO\A.
Dado e> O, queremos encontrar ó> Otal que as condições x E J e O< lx-x 0 1< ó impliquem
lg(.t) - LI < e. Para tanto, se f (x) < g(x) < h(x), então J(x) - L < g(x) - L < h(x) - L e, a
partir daí, é fácil concluir que

lg(x) - LI < max{l/(x) - LI, lh(x) - LI}. (3. )

Se h(x) < g(x) < f(x), concluímos, de modo análogo, a validade de (3. ).
Agora, invocando a definição de limite, sabemos que existem número reais Ó1 , ó2 > O, tais
que
x E J e O < lx - xol < Ó1 =>lf (x) - LI < e
e
x E J e O < lx - xol < Ó2 =>lh(x) - LI < e.
Portanto, se ó= mir1{ó1 , ó2 }, temos ó> O e

x E J e O< lx - xol <ó=> { lf(x) - LI < e


lh(x) - LI< e
•Assim, para x E J tal que O < lx - x0 1< ó, temos
lg(x) - LI < max{l/(x) - LI, lh(x) - LI} < e.

O teorema do confronto torna possível calcularmos o limite trigonométrico fundamental,


o qual se revelará de grande importância na próxima seção.

PROVA.
Como queremos calcular um limite, podemos nos restringir ao intervalo lxl < ~. Suponha,
,_ ~

primeiro, que x > O. Sendo f(AB) = x o comprimento do arco AB (cf. Figura 3.2), temos

sen X = BD < AB < e(AB) = X

130
o

Figura 3.2: o limit trigonomê rico fundamental.

ou ind ; x < 1. Por ou ro lado b m abido ( f. apf ulo 5 d [10), por exemplo) que a
r do tor ir ul r O B • igual a 1r • : = ! , ao p o qu a área do triângulo AOC é igual
2
a½-· por ano amb'm mo

x= (AB) < AC= tgx,

d m n ir < ~.:r ombinando du d igualdad obt mo


nx
CO X < -- < 1. (3.9)

orno funçõ H o x x H ~ :r ão par tai d igual d d ontinuam v rdacl i-


r • p r -~ < < O. Então gue de (3.9) juntam nt om o t r m do onfronto
ntinuid d da função no qu limx o ; x • ' igual 1.

u r f rr m nta mui o ú il no qu on rn o 1 ulo d limit d d p 1 pró ·im pr


p ição. P ra o nunciado da m m , r ord qu um funç- J : J l 'mitada xi tir
f > O l qu
lf(x)I < M 't/ E J.

131
1 \ 1 111 1, 1 ; L 1\ 111 I· -.; I· J) 1-:B 1\ \ I >\ s

PROPOSIÇÃO 3.10.
Sejam / um intervalo, x 0 E / e J,g : / \ {x0} ➔ 1Rfunções tais que J é limitada em J \ { xo}
e limr-u:o g(x) = O. Então, limx-+xo f (x)g(x) = O, mesmo que não exista limx-+:roJ(x).

PROVA.
Dado e > O, queremos encontrar ó > O tal que

x E 1e o< 1x - Xo1 < ó =>IJ(x) g (x) 1 < e.

Para tanto, se 111!> O é tal que lf(x)I < M para todo x E J \ {xo}, então lf(x)g(x)I < l\Ilg(x)I
para todo x E/\ {x0}, e basta encontrarmos ó> O tal que

x E/ e O< lx - xol <ó=> lg(x)I < M"

Para o que falta, é suficiente observarmos que a definição de limite, aplicada a limx-+xo g(x) = O,
garante a existência de ó > O tal que essa última implicação seja verdadeira.

• EXEMPLO 3.11.
Se/: R \ {O} ➔ Ré a função tal que f(x) = sen ¼para x-:/: O, então lf(x)I < 1, para todo
x E R \ {O}. Como limx-+O x = O, segue da proposição anterior que limz-+Ox · sen ¼= O. Observe
que esse resultado está em acordo com (3.5) e com o problema 3.10: página 102.

Sejam dados um intervalo aberto I, um ponto x 0 E / e uma função f : / \ {x0 } ➔ JR. De


modo análogo à discussão até aqui, podemos definir noções de limites laterais para f em x 0 ,
respectivamente à esquerda e à direita, pondo:

(i) limx-+xo- J(x) = L se, para cada e> O dado, existe ó> O tal que

x E 1 e Xo - ó < x < Xo =>1J(x) - L 1 < t. (3.10)

(ii) limx-+;ro+J(x) = L se, para cada e > O dado, existe ó> O tal que

x E I e xo < x < xo +ó=> lf(x) - LI < e. (3.11)

A Figura 3.3 fornece uma interpretação geométrica da noção de limite lateral à direita. rela,
ot>Rerveque, para O < IJ' - .T0 1 < ó, o ponto sobre o gráfico de J com abscissa x só e encontra na
faixa cinza quando .ro < :1:< x0 + ô. Deixamos como exercício para o leitor a composição de uma

132
a Xo b X
H
ó
G,,

Figura 3.3: limite lateral à direita.

figura que forneça a interpretação geométrica correspondente para limites laterais à esquerda.
Ainda cm relação a linut latcra1 é imediato formular e provar, para os mesmos, resultados
análogos àqucl s da propo 1çao 7 (cf. problema 1.4). Ob ervamos também que, para uma
função J : (a, x 0) ➔ IR, a.-, noçõ de limit e limite lateral à e querda de f cm x 0 coincidem.
A~sim, nc~b b c. os, cm g mi •~cre,t>1•mo lim • J(:r) - L, cm vez de limx-+xo-f(x) = L.
Ev1d nlcmcntc, obscrvaçõ análogas n , ,no , t\hclm, pnra limites e limites laterais à direita
cm .r0 , pnia funçõe:, f : (.ro,b) ➔ IR. A • • n~~P1to, v Ja também o problema 1.5.
crminamob ::,tn ção ob~crvnndo que também podemos definir noções úteis de limites
i11fimtm,e lm11te no infinito Com çcmo com o caso de limites infinitos.

DEFINIÇ.~O 3.12.
Dados / intervalo, :.r0 E / e uma função J : / \ {xo} ➔ IR, escrevemos
lim J(x) = +oo
r-+:ro

se, dado JU > O arbitrário. existir ó > O tal que

x E / e O < lx - xol <ó=> J(x) > M.

Dei..xamo ao leitor a tarefa de elaborar definições análogas à acima para os conceitos de


limx-+.ro J(x) = - e limx-+:co± J(x) = ±
A propo ição a eguir estabelece alguns limites infinitos simples. Sua prova não difere em
grau de dificuldade da prO\·a da proposição 7, de modo que a deixamos como exercfcio para o
leitor.

133
, 1 ,,, ; Ll\llfF-.; I•: l)f-:Hl\.\ll\S

PROPOSIÇÃO 3.13.
ejam <indos I intervalo, .r0 E / e funções /, g : I \ {x 0 } ➔ IR.

(a) Se lim J(:c:)= L > Oe lim g(x) = ±oo, então lim J(x)g(x) = ±oo.
~-+:ro r-+.ro .i: ➔xo

(b) Se lim f(x)


z-+:ro
= L < O e x-+1·0
lim g(x) = ±oo, então lim J(x)g(x)
x-+xo
= =fOO.

O protótipo de limite infinito é dado pelo exemplo a seguir, cuja demonstração também
deixamos como exercício para o leitor.

ExEMPLO 3.14.

(a) Se f: (xo, +oo) ➔ IR é dada por J(x) = x~xo, então limx-+xo J(x) = +oo.

(b) Se/: (-oo, Xo) ➔ Ré dada por J(x) = x~xo, então limx-+xo J(x) = -oo.

Voltemo:nos, agora, à consideração de limites no infinito.

~EFINIÇÃO 3.15.
Para/: (a, +oo) ➔ R, escrevemos limx-++oof(x) = L se, dado e > O, existir A > a tal que

x > A=> IJ(x) - LI < e.

Assim como antes, deixamos ao leitor a tarefa de elaborar definições análogas à acima para
limx-.-oo f(x) = L e limx-+±ooJ(x) = ±oo. Por outro lado, observe que as proposições 7, 8 e 13
continuam válidas se trocarmos I\ {x0} por (a,+oo) (resp. (-oo,b)) e x 0 por+ (resp. - );
ademais, as demonstrações nesses casos são essencialmente as mesmas.
Vejamos um exemplo interessante (e útil) de aplicação da proposição 13.

ExafPLO 3.16.
Sejam n E N e / : R ➔ R a função polinomial tal que

com ao,a 1, ... , ª" E R e a,, > O. Prove que:


J(.r) = +oo.
(a) Se n é par, então lim1xl-++oo
(b) Se n é fmpar, então limJ·-+±oo J(.r) =±oo.

134
mo n • p . limr :::: x" n ímpar
..J!.L
:r -1 ~) = an (,- rifiqu

!( X ) =X
n ( ªn-1
tln----···---+- a1 ao) ,
X xn-l x"

pli and .ro.

m d m 1 1mp r n d lillll mfini n infini t • p e id ntifi Ç-0 d


a intola h nz n um fun - m uir.

3.17.
' I t- um iut n l . o E / / : / { } um l qu lim.r-+.roJ(x) = ± ,
dizPm qu , 1 •t . = 0 t. 1d d f. ualogam nt , J:
(a (1 p. / : 1q J ,) = L (r p. lim.r _ f(x) = L),
l /.

1 .
( 1) J l. i 1 \11 l 1.l d fun - 11n int n· l ( -i i). D po
u d, pr 1 13 qu limr-+j tg x =
=- i 1111 1u r t :r. = ±i - íntot
\lt(i1i

(h li m 1 fui - f : \ {- 1 } , dada por f (x) = z':i, obtido


l ltil d fun - d I roJ • u li<léd mv •r ( f. Figura 1.15) com o auxilio
l 1 ult pr b l n ~ .1O p f mo

2
------=2--- (3.12)
X 1 x+ l'
l i m r d f do: prim iro, traçamo
gr nt ri r uma unidad para a urda
grãfi na dir ão vertical pelo fator
ult do no ixo horizontal
X último gr fi o du unidad

13
y

(O 2)
--------------+---------------------
1
1
1
1
1
(-1 O) 1

2x
Figura 3. : gráfi o d X t--+ x+l.

C'A,moWllz➔-1± 2X = =f2 e limx➔-1± x~l = ± gu d propo iç- 13 limx -1± /\ = =i= ,


x =
-1 é assíntota v rtical do gráfi o d f. Por outro lad u f ilm nt d (3.12)
lims-+±ooz2:.i= 2, de ort qu a reta y =2' ín o a horizontal do gráfi d J.

Problemas - Seção 3.1


1.1. jam I um in rvalo /. Dada uma função f : / \ {xo} pr v qu
lim.r--..roJ(x) xi úni o.

1.2. * S jam / um int rv 1 oE l f g: /\ { }


L lim.,, r 0 .9(J') M. J( ) > g( ) p r

1.3. tab 1.ça as g n r, lii~ •

7 p, n limit s IHl

• • •jam / : (a, IJ) 11rn fun ·ii 11-I! .1·0 (a b). u lim.r roJ(:r)
ffÓ ffi•, t:1:tHfrm 8tW 1911a1. s limit , lt l n is lin r .ro t- J(.r) li1 l.r .ro- J(l') d me 1 ,

• <J <·11.no,<'Oll(•h11 q11

J.
J>H<>BI.I \I \"' -.;I 1_ \11 ; ) t .

1.6. Prove a proposição 13 e sua análoga para limites no infinito.

1.7. * Sejam -oo < a < b < +oo e f : (a. b) ➔ R uma função contínua e crescente (resp.
decrescente). Se limr-+a+ f(x) = L e limr_.b- f(x) = 1'1, com -oo < L, A1 < +oo, prove
que Im (!) = (L .. \!).

1.8. Em cada um dos itens abaixo. calcule os limites em questão:


(a) llmx-.0 Z6eD% sen j(.x-1)
1-cosx • (d) lim.x➔l :c-1

(b) limx-+01-ooex cos{


;z}
z2 (e) lim:c_.l :c-1

(c) lim:r-+06e0
leu (2r)
(3:r) (f) lim
r-"O
1-~
z •

1.9. Se /, g : (a, +oo) ➔ R ~ão funções tais que / é limitada e lim.x-++oo g(x) = O, prove que
limr-++ooJ(x)g(x) = O. Em :,c.>guida
1 use este fato para calcular os dois limites abaixo: -

• :s<•nz
( a ) ) llllr-++oo --:;- .

(b) limr-++ r..;::c.

1.10. Sejam / um intcr\"alo, :r0 E f l-./ : J \ {:r,o}-+ umn função tal que lim:i:-+xo f (x) = L. Se
(a 11 ) 11> 1 6 umn ~equênciAem/\ {xo}, tal Qlll' fim 11 ...,.+ a,. = 1·0,prove que limn-++oo/(an) =
L Conclun, a pnrtir dAf. que limr ...o ,en não e.xi te. !
1.11. * Dndn umn. func;Ão/ : (A,+ ) -+ R. dizcmoi:;que a reta y = ax + b é uma assíntota
- (ax + b)) = O. Nesse caso, prove que
oblíqua (do gráfico) de / se limr-. :,(J('.1:)

. J(x) .
a = :r:-++
hm --
.l'
e b = hm (/ (x) - ax).
:c-++oo

Em seguida, elabore o conceito de assíntota oblíqua y = ax + b para uma função /


(- . A) ➔ R. e mostre como calcular a e b nesse caso.

1.12. Em cada um dos itens a seguir, encontre as assíntotas horizontais, verticais e oblíquas (cf.
problema anterior) da função dada:

(a) f: lR\ {O} ➔ IR, tal que J(x) = x + ¼,para todo real x =/=O.
(b) /: lR\ (-a, a) ➔ lR, tal que J(x) = ~Jx2 - a2 , para lxl > a. (Observe que o gráfico
de f é a porção da hipérbole ~ - ~ = 1 situada no semiplano superior do plano
cartesiano. A esse respeito. veja também a discussão sobre hipérboles, na subseção
A.4.3.)

137
( 1111111; l.l\1111"'1· ()11(1\\l>.\:--

(e) J: (O,+ ) ➔ lR, tal que J(x) = x 2 sen ~, para x > O.

1. 13. Calcule lim,-Hoo ( Jx + J x + .,/x- .,/x)-


1.14. Seja J : (O,+oo) ➔ (O,+oo) uma função crescente. Se limx-++oo1)~;/ - 1, prove que
rlffix-++oo/(nx)
/(x) =
1, para to d o n E IM
1,;i.

1.15. Seja (an)n>1 a sequência definida por a 1 - 2v'2 e, para n > l inteiro,

a. = 2" ✓2 - J+J
2 2+ ••• + vÍ2, com n sinais de raiz quadrada. A esse respeito, faça
os seguintes itens:

(a) Se bn = /2 + J2 + · · · + J2 para n > 1, com n - l sinais de raiz quadrada, mostre


que b~ = 2 + bn-1 e, a partir daí, conclua que bn < 2, para todo n > l.
(b) Escrevendo bn = 2 cos 0n, com O < 0n < i, conclua que 0n = ½Bn-1, para todo n > 2.
(c) Mostre que an = 2n+1sen~ = 2n+ 1sen ~- Em seguida, use este fato para mostrar
que limn-++oo ªn = 1T'.

138
3.2 Propriedades básicas de derivadas
Tendo estabelecido firmemente o conceito de limite. retomamos, com a definição a seguirr a
discussão que levou a (3 2).

ÜEFI\'IÇÀO 3.19.
Sejam I C lR um inten-a.Jo aberto e f: J ➔ Ruma função dada. Fixado x 0 E I, •
f é derivável em x 0 se existir o limite

lim J(x) - J(xo)


r-zo X - Xo

Ncss~ caso, tal limite ~erá denominado a derivada de f em x 0, sendo denotado por f'(xo),

Nas 11otaçocs da definição anl >rior, ob~ •n que exigir que o limite acima exista é o mesmo
qu<>exigir que exista o limite
. /(:r:o + h) - /(:r:o)
l1111 J •
h-+O 1
Reahnc11tP1 por um lado, fa7,cndo x 0 + I, = x,
mo h = .i· - .r0 ; além disso, é claro que
t

li -+ O <=>
.r -+ ,r0 . Por outro lAdo, qmrndo < 1l'\ mo /(:r.0 +h ) 1 ... t,lmos bupondo implicitamente

que h é liio pequc110qu ~ :r 0 + h t ml> m p r1t nol" /: mns, como e tamos calculando um limite
<' o dom11110/ de/ é um i11tL•n11lo nb~rlO t 1 upo-..it;-o não impõe restrição alguma à definição.
Em 1c~umo1 !-<'lido J d -irivA\l l t'lll x 0 E J, podt•mo cievcr
'( ) J(x) - f (xo) /(:1·0+ h) - J(x 0)
! :.t'o = 1.llll
:r-+:r-0
-----
x - x0
= ,,1.llll... o -------. h
(3.13)

Do1I\\Hnte, '\ mp1t• que com·-.niente, rcfetir-no::i-emo:sa uma qualquer das frações acima como
um quoci nlc d wton de / cm !ro.
O t''\. 'mplo i, ~egmr trn.z o~ primeiro~ cálculos de denYadas.

EXRMPLO 3.20.
Os it cn~ abaixo c-alculam as deri\'adas de algumas funções simples:

(a) Se f : R -+ Ré uma função constante, então / é derivável em R e /'(x 0) ==O,


x0 E R.

(b) Se n E N e f : R -+ R é a função dada por /(x) -


f'(x 0 ) = nx 0- 1, para todo Xo E R.

(e) Se n E Zé negativo e/: R \{O}-+ Ré a função dada (para x E R \ {O})


então fé derivável em R \ {O} e /'(xo) = nx 0- 1 , para todo Xo E R \ {O}.

139
1 ,1111111: Ll\llll·: .....1-.Ih:Bl\º.\l>.\S

PROVA.
(a) Apliquemos a primeira igualdade em (3.13): sendo f(x) = e para todo x E IR,

J'(xo) = lim f (x) - f (xo) = lim e - e = lim O= O.


x-+xo X - Xo x-+xo X - Xo x-+xo

(b) Apliquemos a segunda igualdade em (3.13): pela fórmula do desenvolvimento binomial,


temos

Então, aplicando sucessivamente (3.6) e (3.5), obtemos

..
(c) Se n = -m, com m > O inteiro, então

f (x) - f (xo) = 1 (_2_


_ ~) =
X - Xo X - Xo xm Xo

Portanto, a proposição 7 e (3.5) fornecem

1 Xm - Xo m) -- lim --· 1 lim ( Xm - Xo


m)
J'(xo) -- lim-- ( -----'-
x-+xoxmx 0 X - Xo x-+xoxmx 0 x-+xo X - Xo
1 . (Xm - xm) o
= -- · l1m
Xõm :.c-+:.co X - Xo •

Agora, aplicando o resultado do item (b) (com m no lugar donde lá), obtemos

! '(Xo) = - - 21-
Xom
• mx m-1
0
-m-1
= -mx 0 n-1
= nx 0 .

140
,

EXEMPLO 3.21. .
As funções seno e cosseno são derh"áYeisem todo x0 E R. com sen'Zo .
...:
,._
..-~
'
\ '~ -
-sen Xo- .. ,

PROVA.
Façamo a prova para o caso da função seno. ::,endoa prO\-a para o caso da função cosseno
totalmentP. anãloga.
A"ci fórmulas de transformação em produto da Trigonometria dão-nos

Como a fu11çãoco •uo {•contínua o prop sição 7. (3.5) "o limite trigonométrico fundamental
(cf. lc•1J1a
9) fornc~m
h
, ,. 1 "i
s n:r-o = .nn~•
h-tO 2

= h-tO
hm ~ li • lim
.1 h....O
, ( ro + -")
2
. nh
-hm-•
h O I, ro = XQ.

A11h. dt:>ptt s.-.t g11ircom o d nvolvimemo da teoria, tt.~amos duas observações <iteis.

(i) Umn Hz q\h~ dl rivndas -o limit , dada uma função /: (a, b) ➔ R, podemos coDlicMrer
a
dc.~rivada lateral A direita m a. i.e., o limite

/~(a) = r /(x) - /(a) 1


..l'l X- a
<'A'iO tal
limite exista. Analogamente para/: (a, b] ➔ R, podemos co •
lateral à esquerda em b, i.e., o limit.e

1:(b) = lim /(x) - f(b)


:r4- X - b

(novamente, caso tal limite exista). Ademais 1 sempre que diaaermoa


[a, b) ➔ R é derivivel, ficará implícito que as derivadas de / em z o =

141
llUmo caso, sempre que não houver perigo de confusão, escreveremos simplesmente
f.(11) (em vez de /~(a) e J~(b)) para denotar tais derivadas. Doravante, utilizaremos
-IN!lll'lçõessem maiores comentários .

.._...._, denotaremos a derivada de uma função f num ponto x 0 utilizando a notação


iva (devida a Leibniz) i(x 0), de sorte que

simbologia justifica-se pelo fato de que, classicamente, a fração do segundo membro


~ma era escrita como ~ lx e, daí, a notação ~(x 0 ) recorda que a derivada é o limite de
• • • Xo
quocientes, 1.e.,
df . 6.f IX
-d· (x0) = hm ~ .
X x-txo uX .r0

A definição de derivada, juntamente com discussão que levou a (3.2). nos permitem apresentar
outra defuµção relevante.

QÃO 3.23.
Sejam / C R um intervalo e f : I ➔ IR uma função derivável em r 0 E /. A reta tangente
00 de f no ponto (xo, f (xo)) ê a reta que passa por tal ponto e tem coeficiente angular
/'(zo)-

Segue da definição acima e de (3.1) que a reta tangente ao gráfico de f em (x 0, JCro)) tem
equaçao
Y - f(xo) = J'(xo)(x - xo). (3.14)
Ilustramos a noção de reta tangente a um gráfico no exemplo a seguir. Para o enunciado do
mesmo, o leitor pode achar conveniente reler o enunciado do problema 5.13, página 59, assim
como a discussão sobre hipérboles, na subseção A.4.3.

,amo da hipérbole equilátera de equação .ry = 1, contido no primeiro quadrante do


+--+
. Se P e Q são pontos sobre os eixos cartesianos, tais que a reta PQ tangencia
igura 3.5), prove que:

do sistema cartesiano em questão, então a área do triângulo POQ indc-


de A sobre 1l.

1,12
PIH>l'IULl>\I>I" 11\"IC \" 1>1· lll·HI\ \I>\" 1
'

o Q X

Figura 3.5: tangente ao gráfico de x H ½-

PROVA.
Evidentemente, 1i coincide com o gráfico da função f: (O,+oo) ➔ JR, tal que J(x) = ¾- Se
Xo > Oe A ê o ponto (xo, f (x0)) = (x0, ; 0),
então a reta tangente a G I em A tem equação
y - ..l..
XO
= J'(x 0)(x - x 0). Pelo exemplo 20, temos f'(x 0) = -~,XO de sorte que a equação da reta
em questão é y - ..l..XO
= -!,-(x
Xo
- x 0) ou, ainda, XõY+ x = 2x0.
SubsLituindo sucessivamente x = O e y = O em tal equação, obtemos P ( O,; 0) e Q(2x 0 , O),
ou vice-versa, e e os itens (a) e (b) seguem imediatamente:

(a) o corolário 53 garante que o ponto médio de PQ tem abscissa e ordenada respectivamente
iguais a -21 • ixo = ..l..
xo
e -21 • 2x 0 = x 0 . Portanto, tal ponto coincide com A.

(b) A(POQ) = ½OP • OQ = ½• ; 0 • 2xo = 2.

Continuando com a análise do conceito de dedvada, mostramos que derivabilidade é uma


propriedade mais forte do que continuidade.

PROPOSIÇÃO 3.25.
Se uma função f : I ➔ R é derivável em x 0 E J, então f é contínua em 1·0 .

143
( 1·111 11,; i.l\llll·~ F l)I.Bl\\l).\S

PROVA.
Inicialmente, note que, para x E I \ {x 0}, temos

f (X) = f (Xo) + ( f (X) - f (Xo)) (X - Xo).


X - Xo

Por outro lado, como estamos assumindo a existência de limx-+xof(xl=!~xo) = f'(:r:o), egue da
proposição 7 que •

lim J(x) = lim (!(xo) + (f(x) - f(xo)) (x - x 0))


x-+xo x-no X - Xo

= J(x 0) + lim
x-+xo
(f(x) - f (xo))
X - Xo
lim (x - xo)
.r-+:ro

= J(xo) + J'(xo) ·O= J(xo).


Logo, a proposição 4 garante a continuidade de / em x 0 .

O exemplo clássico a seguir mostra que eXI&temfunçõe contínuas que não são deriváveis e111
certos pontos de seus domínios 1 .

~ª~·A função/ :R ➔ R dada por f(x) = lxl é contínua em toda a reta mas não é derivável em
X= 0.

PROVA.
Observe que, para x =fO, temos

f(x) - J(O) = El= { 1, se x > O


x-0 x -l,sex<O

Portanto, limx-+O+/(xl=~(o) = 1 e lim:r-+O-f(x;=~(O) = -1. de forma que, pelo problema 1.5,


página 136, não existe limx-+Of(x;=~(O).

Se / e Ré um intervalo e f: I ➔ Ré derivável em todo Xo E J, diremos simplesmente que


/ é derivável em/. Nesse caso, fica bem definida a função derivada f' : J-+ R, que associa a
cada x E / a derivada f'(x) de J em x.
l[nformamos ao leitor que é possível construir exemplos de funções continuas J: R ➔ R, que não são deriváveis
em ponto algum! A prova dessa afirmação foge ao escopo dessas notas. mas pode ser encontrada em 1201.

144
Se I e J são intervalos e/ : J ➔ J é uma bijeção, derivável em todo ponto de 1, ._ ..
proposição anterior que/ é contínua. Ademais, o teorema 61 garante que 1- 1 : J ➔ 1 ~
é contínua. i\osso próximo resultado explica quando 1- 1 é derivável.

TEOREMA 3.27.
Sejam I e J intervalos e / : I ➔ J uma bijeção, derivável em todo ponto de
Yo = f (xo) E J, temos 1- 1 : J ➔ I derivável em y0 se, e só se, /'(x 0 ) :/: O. Ac:I
o caso, temos

PROVA
Para h suficientemente próximo (mas diferente) de O,sejam x = x 0 + h, l = f(x 0 + h)- f(xo)
e y = Yo+ l. Então, y = f (xo) + l = f (x 0 + h), de maneira que

h = (xo + h) - xo = f- 1 (Yo+ l) - 1-1 (yo).


Portanto, as continuidade de/ e 1- 1 garantem que h ➔ O{=} l ➔ O. Veja, também, que

Suponha, agora, que 1- 1 é den,-á,·el em y0 . Fazendo h ➔ O (o que, como vimos, equivale a


fazer / ➔ O), segue do item (b) da propo!-.1ção7 e do câJculos acima que

1 = !i!..'I,
[ ( J(xo + h]- J(xo)) ( 1-'(yo + li- J-'(Yo)) l
1 1
_ . (l(xo + h) - J(xo)) 1. (/- (Yo+ l) - 1- (yo))
- 1im h im l
h-+0 1-+0

= J'(:ro)(/- 1
)'(yo).

Portanto, /'(1·0) =/;O e (/- 1 )'(yo) = /'(~o)•


Reciprocamente, suponha que J'(xo) -:/-O. Como
1- 1 (yo + l) - 1-1 (yo) _ 1
l - (J(xo + h) - J(xo))/ h'
o item (c) da proposição 7, juntamente com o fato de que l ➔ O{=} h ➔ O, fornece

-lim( 1 )
- l-+O (J(x 0 + h) - f (xo))/ h

= J!To
( (J(xo + h/- f(xo))/h) = f'(~o)
145
( \ 1 1 1 1 111 \ LI\IITl·:S E DEIU\º.\D.\S

Logo, 1- 1 é derivável em Yo•

ExEMPLO 3.28.
Se n > 1 é inteiro e/ : [O,+oo) ➔ [O,+oo) é a função raiz n-ésima, f(x) = y'x = x 1fn,
então/ não é derivável em x = O mas é derivãvel em todo x > O, com f'(x) = l • x 1fn- 1 .
------- • ~-n------~

PROVA.
Recorde que/= g- 1 , onde g: [O,+oo)--+ (O,+oo) é a função dada por g(y) = yn. Portanto,
pelo teorema anterior, fé derivável em x = g(y) se, e só se, g'(y) =/=O. Mas, como g'(y) = nyn-l,
temos que / é derivável em x = g(y) se, e só se, y =/=O ou, o que é o mesmo, x =/=O. Nesse caso,
lembrando que x = g(y) = yn <=> y = :fx, segue de (3.15) que

J'(x) __ 1_ _ 1 1 = .!:._.Xl/n-1 _
- g'(y) - nyn-1 n( zyx)n- 1 n


Para uma generalização importante do exemplo anterior, veja o problema 3.4, página 158.
Terminamos esta seção apresentando ao leitor o conceito de derivada de ordem superior.
Para tanto, sejam dados um intervalo I e IR e uma função f : I ➔ IR.
Dizemos que fé duas vezes derivável em x 0 E I se existe uma vizinhança (x 0 - r, x 0 + r)
de x 0 tal que fé derivável em I n (xo - r, xo + r) e a função derivada f' : I n (x 0 - r, x 0 + r) --+lR
é derivável em x 0 E I. Nesse caso, dizemos que (f')'(x 0) é a derivada segunda de J em x 0, e
a denotamos por J"(x0 ). Se J é duas vezes derivável em todo xo E I, dizemos simplesmente que
f é duas vezes derivável em I, e definimos a função derivada segunda de f como a função
J": / ➔ JR,que associa a cada x 0 E/ a derivada segunda f"(x 0 ).
Mais geralmente, seja dado k E N e suponha que já definimos o que se entende por J ser
k vezes derivável em I, bem como o que se entende pela k-ésima derivada J<k>(x 0), de J em
x 0 E /. Denotando a função k-ésima derivada de J por J(k) : I--+ IR, definimos a (k + 1)-ésima
derivada de / em x 0 pondo

caBOtal derivada exista 2 .


2Conformc
chamamos a atenção do leitor à página 37, a notação J(kl ê padrão também para a composição de
uma função / : / ➔ / corlRígo m<•Hum,k vezes. O conLcxto dirimirá. eventuais confusões.

 146
Para k E N, se f : I --t 1Ré k vezes derivável em I e tal que J<k) : I --t IR é contínua em 1,
dizemos que f ê k vezes continuamente derivável em J. Por fim, se fé k vezes derivávelem
I para todo k EN, dizemos que f ê infinitamente derivável em I.

EXE~1PLO 3.29.
Se n E Z \ {O} e f : IR --t lRé a função dada por f (x) = x", segue facilmente do item (b) do
exemplo 20 e do problema 2.1 que fé infinitamente derivável em lR\ {O} (em R, se n > O), com

j(k)(x) = n(n - 1) ... (n - k + l)xn-k

para todo k E N. Em particular, se n > O e k > n, então J(k) é a função identicamente nula.

EXEMPLO 3.30.
As funções sen. cos : R --t IR são infinitamente deriváveis, com

scn(4k) = scn, sen<4k+I) = cos. sen<lk+ 2> = -sen, sen<4k+ 3) = - cos

e
cos<lk} = cos, cos(.ik+i) = -:;cn. cos<·u+2 J = - cos, cos<4k+ 3) = sen,

para lodo/.; EN.--~~~~-~-~

Problemas - Seção 3.2


2.1. Sejam J C IR um intervalo e J :I --t IR uma função derivável em x 0 E J. Fixado e E IR,
plO\'C que:

(a) A função g : I --t IR dada por g(x) = cf(x) também é derivável em x 0 , com g'(x 0 ) =
cf'(::ro).
(b) A função h : I --t IR dada por h~.LJ = J(x) + e também é derivável em x0 , com
h'(.ro) = f'(xo).

O próximo problema estabelece um caso particular simples da regra da cadeia (cf. teorema
3.19).

2.2. * Seja / : (a, b) --t IR uma função derivável em xo E (a, b). Dado e > O (resp. e < O), seja
g : (~. ~) --t lR (resp. g : (~. ~) --t IR) a função dada por g(x) = J(cJ:). Mostre que g é
derivável em~- com g' (~) = cf'(xo).

147
, , 1·1 1 1 1 1, : L1\11n:s 1-: Dr-:u1,·.,n.,s

2.3. io exemplo 63, definimos as funções arcsen : [-1, l]-+ [-~,~]e arccos: [-1, l]-+ [O,rr]
como as inversas das funções sen : [-i, i] ➔ [-1, l] ecos : [O,rr] ➔ [-1, l], respectiva-
mente. Use o teorema 27 para provar que as restrições das funções arcsen e arccos ao
intervalo aberto (-1, 1) são deriváveis, com
1 1
arcsen 'x = ---;::::==::;: e arccos' x = - --;::==::;: ,
Jl - x 2 J1 - x 2
para todo x E (-1, 1).

2.4. Calcule as derivadas das funções abaixo nos pontos em questão (em cada caso, assuma que
a função está definida em seu domínio maximal):

(a) f'(l); f (x) = ~- (e) f'(1r); l (x) = sen (3x).


(b) f'(2); f (x) = 1. ( f) f' ( ~) ; f (x) = 4 cos ( J).
(c) !'(2); f (x) = \!'3x. (g) f'(i); J(x) = arcsen (2x).
(d) f'(l); f(x) = ffx. (h) l'(¾); l(x) = 2arccos(3x).

2.5. Sejam l, g : (a, b) ➔ lR funções deriváveis, cujos gráficos passam por um mesmo ponto A.
• Dizemos que tais gráficos são tangentes em A se as retas tangentes aos mesmos no ponto
A coincidirem. A respeito dessa definição, faça os seguintes itens:

(a) Mostre que, param, n E N, os gráficos de l(x) = xn e g(x) = xm são tangentes em


(O,O), mas só são tangentes em (1, 1) sem= n.
(b) Seja f : (a, b) ➔ (a, b) uma bijeção tal que l e 1- 1 são deriváveis. Se os gráficos de
f e 1- 1 são tangentes em um ponto A(x 0 , y 0 ), calcule os possíveis valores do produto
f'(xo)f'(yo).

2.6. Sejam I e IR um intervalo e l : I ➔ IR uma função contínua em x 0 E /. Se a função


xi-+ xf (x), x E /, é derivável em Xo, prove que l também é derivável em x 0 .

2. 7. * Sejam I um intervalo aberto, Xo E J e l : I ➔ IR uma função derivável em x 0, com


f'(x 0) = L. Dadas sequências (an)n~1 e (bn)n~1 em I, com an < Xo < bn para todo n > 1,
mostre que

2.8. Da.da uma função derivável J : IR ➔ IR, dizemos que um real x 0 é uma raiz de l se
J(xo) = O. O método de Newton 3 para o cálculo de aproximações das raízes de f garante
ªTeremos mais u di1.cr tiObrc o ml!todo de Newton no problema 3.7, página 358.

118
qu, quci (a,. n
d fini • lar , qu f'(an) ':/;O)
n:

( ) p n n n
b
(b) D d a > O xpliqu om o m'todo d rn n ur l d finição da qu ncia
(an)n~l do pr bl m 2.15 p'gin 91 qu 1 on rg p r Jã,.

2.9. D d R > O ja f: (-R R) 4 a função dada por J(x) = JR 2 - x 2 . Prov qu :

( ) gráfi o d J , o mi ír ulo d n ro na orig m do plano cart iano contido no


miplano up rior do m mo om raio R.
(b) o E (-R R) m f'(xo) =- :ro
R2-xi
.

( ) D do o E (-R R) ( Xo J ( o))
a r a ang n ao gráfi o de f obtida de a ordo
om a d finição 23, oincid om a r a qu p a por ' p rp ndicular ao raio
o.
2.10. J m I um in J :I um função d riváv l. Dado a b E
4 uponha
qu Xl um r r ng n o gráfico d f p ando pelo ponto (a b). lo tr qu
r ng n i o gr fi d J m um pont ( o Yo) 1 qu
Yo = f(xo)
{ Yo - b = f' (xo) ( o - a)

m guida r r' -o r t qu p m p lo p nt (1 -1) tang n i ma par bol


d -
quaç o y = :r2
4 u fato para ob r o pon o d tangên i d r r' om par· bol .

r o próximo probl rn , o l itor pod achar ú il r l r d rnon r - do t r m 49 o


rn rial obr ôni n ub ção .4......

2 .11. J 'P a p _.
rábol d fo o F dir triz d. P ' um p nt br d
qu P BP ng n i rn 'P m r qu F E B.

1
1 , 1 1, 1,, : LI\II n:s 1-: DEHl\._\ll.\s

3.3 Regras de derivação


Esta seção é devotada à apresentação das regras de derivação usuais, i.e., fórmulas que
relacionam as derivadas de duas funções deriváveis dadas com as derivadas de certas funções
obtidas a partir das funções iniciais.
Para o item (e) da proposição a seguir, assim como no item (e) da proposição 7, supomo ;
definida em um intervalo apropriado J e I, no qual g =/O.

PROPOSIÇÃO 3.31.
Se /, g : / ➔ R são funções deriváveis em x 0 E J, então:

(a) / ± g é derivável em x 0 , com (/ ± g)'(x 0 ) = J'(xo) ± g'(xo)-


(b) f g é derivável em x 0, com (f g)'(x 0 ) = f'(x 0)g(.r0) + J(xo)g'(.ro)-
(c) Se g(xo) =1-O, então ~ é derivável em x 0 , com

! )' (xo) = f'(:ro)g(.ro) - f(.1·0)9'(.1·0).


(g g(:ro)2

PROVA.

(a) Provemos que f + g é derivável em x 0 , com (J + g)'(x 0 ) = f'(x 0) + g'(x 0 ) (a prova das
afirmações relativas a f - g é completamente análoga). Como
(J + g)(x) - (f + g)(xo) J(x) - J(xo) g(x) - g(xo)
-------- = ---- + ----.
X - Xo X - Xo X - Xo

para x E /\ {x0}, as propriedades operatórias de limites garantem que, se J'(x 0 ) e g'(x 0 ) existem,
então (J + g )' (x 0) também existe e

(J + g)'(xo) = lim (J + g)(x) - (f + g)(xo)


x-+xo X - Xo

= lim (-J(_x_)-_f(_x_o)+ g(x) - g(xo))


x-+xo X - Xo X - Xo

= lim f(x) - f (xo) + lim g(x) - g(x 0 )


:r-+:ro X - Xo x-+zo X - Xo
= J'(xo) + g'(xo)-
(b) Inicialmente, observe que

(f g)(x; =~g)(x 0
) = ( J(x; =!!xo)) g(x) + ( g(x; =:~xo)) f(xo)- (3 16)

150
u g • ontínua m xO da(,

lim g(x) = g(xo)-


:r :ro

P r anto, faz ndo x x 0 m (3.1 ) u ilizand pr pri d d op r tóri d limit , concluí-


m qu
·m (Jg)( ) - (Jg)(xo)
l1 = lim f(x) - !( o) · 1ungx
. ( )
z-t.ro - Xo r ro - Xo r zo

+ J(xo) · lim g(x) - g(xo)


x xo I - Xo

= J'(xo)g(xo) + J(xo)g'( o).


( ) Ini i 1111n , n id r m m qu f • funç ~ on tant igual 1 mo tr mo
q u ( g1), (J.O) = - g'(ro
g(ro)2·
1n g (X) -=/= op X E J e I, t m . par X E J \ { o} qu

1 ( 1 (g(x) - g(x 0))


1 ) 1
x - ro g(~ - g(:ro) = - x - .ro • g(x)g(xo) •
p rln d i u 1 d riv ili d (p r n uin ) ontinuid d d
(J Ili O, 1t ll1 ,

• -- 1
lun (1---- 1) ,
=-g(xo)·--. 1
o .r - ro g( ) g(xo) g(xo)2
p, r i iJ J • llllc , jun e ffi 11 m o r ul ado do i m
(l ):

◄• ·E.1PL 3.' 2.
• ja f: funç - polinomi 1 dada . "

m ao,a1,••·•ª11 E ª" -=/=O. tilizando r p tid • v z o exemplo 20 o i


pr p iça ant rior, n luím qu f d riváv 1 m , com J' : R -+ R tambémJ>Ollilie■
tal qu

para tod

1 1
1 n m n q fun - an fun -
} . tal qu , .r = ~. p r t d r m u d míni .

+kw· k E Z}, ntão funç- t n n tg : \D , d ri á 1


E R \ D. Realm nt , oi m ( ) a pr p iç- nt ri r jun m nt m t'rmul •

1
2 .r.

O x mplo a guir m ra qu fun - r n n ( f. x mpl ) , d ri ;


ua d rivada.

LO 3.34.
função arctg : R ➔ (-i i) , d riv'v l, m

1 1
y- l
+ y-?'
todo 11E R.
---

Inicialm nt ob rv qu g' = 2
X -::j:.
Ü par odo x E ( - ~. ~) . r nt , p l t r lllc

27 a função ar tg • d riv'v l. d m p r y E nd x = ar gy. m y = tg.r.; 1m.

graç a (3.15) ao r ul ad do x mpl b m

1 1 1 1 1
r gy=--=
g' 2x - 1+ g 2x - 1

A mais important d n d d ri aç- ' r r da d i p r d riv d ,


ual ta gro.· ir n d u fun -
como calcula tal d ri .1 di t
isamos d algun pr limin, r ..
um int rval e 1 1l / e u1fü fun - J :J d riv'v 1 m O E J

I .ro - {h E ; .ro h E J}

l --
então J - x 0 é um intervalo aberto contendo O. Definamos a função r : I - xo ➔ R pondo

r(h) = f(xo + h) - J(xo) - J'(xo)h.


Como fé contínua em x 0 , temos

lirn r(h) = lim(f(x 0 + h) - J(xo) - J'(xo)h) =O= r(O),


h-+0 h-+0

de forma que a função r é contínua em O. Também,

lirn r(h) = lim (l(xo + h) - J(xo) _ J'(xo)) = O,


h-+0 h h-+0 h
pela definição da derivada de f em x 0 . (De fato, o último cálculo acima mostra que r é derivável
em O, com r'(O) = O. Entretanto, não utilizaremos esse fato aqui.)
Reciprocamente, temos o seguinte resultado auxiliar.

LEMA 3.35.
Sejam 1 C IR um intervalo aberto, f : 1 ➔ IR uma função dada e x 0 E J. Se existir um
número real L tal que a função r: I - x0 ➔ IRdada por r(h) = f(x 0 + h) - f(x 0 ) - Lh satisfaz
a condição lim11 •0 r~:) = O, então a função J é derivável em x0 , com f'(x 0 ) = L.

PHO\'A.
Como r~) = f(xo+hJ-J(xo) - L, a condição li1111i-+o r~) = O equivale a

.
11m
J(xo + h) - f (xo) _ L
h-+0
h - .

Dados um intervalo aberto I e uma função J : J ➔ IRcontínua em x 0 E J, observe que, para


todo L E IR, a função h H f(x 0 ) + Lh é afim e tal que seu gráfico passa pelo ponto (x0 , f(x 0 )).
Isto posto, é frequentemente útil refrasear a discussão anterior dizendo que J é derivável em x 0
se, e só se, f admite uma melhor aproximação afim numa vizinhança de x 0 , no sentido do lema
35.
Em outras palavras, sendo / : J ➔ IR derivável em xo E J, temos

f (xo + h) = J(xo) + J'(xo)h + r(h), (3.17)

com r : / - x 0 ➔ IR tal que limh-+Ort) = O. Por outro lado, se h H J(x 0 ) + Lh é uma


aproximação afim de f numa vizinhança de Xo tal que, pondo r(h) = f(x 0 + h) - f(J: 0) - Lh,
tenhamos limh-+Or~) = O, então J é derivável em Xo e L = f'(xo).

153
\ 1' 1 1 1 'l ; Ll\1111·:-- 1-: 1)f-:1{(\".\().\S

r'úr Y('IC~, n'Í<.'rir-nos-cmos à igmlldnclc (3.17), válida para h E / - x 0 , como a fórmula de


~ lor d"' ord 111 1 de f mmrn, vizinhança de x 0 . Observe que seu real conteúdo é capturado
pd~ informação de que lim,1 ~or~i) = O quando fé derivável em x 0 .
Para o que segue, é mais útil escrevermos (3.17) como

J(.xo+ h) = f (xo) + J'(xo)h + R(h)h, (3.18)

onde R : / - x 0 -+ IR é tal que

R( h) = { ~, se h i=O .
O, se h = O

Obseve que a continuidade der acarreta a continuidade de R em todo h E / - x0 diferente de


O. e que a condição limh-+O r(;> = O garante que R também é contínua em O.
Reciprocamente, se f puder ser escrita da forma

f(xo + h) = f(xo) + Lh + R(h)h,


.
em I - x 0 , com R contínua em / - x 0 e tal que R(O) = O, então uma modificação simples da
prova do lema 35 garante que fé derivável em x 0 , com f'(x 0 ) = L.
• Podemos finalmente provar o teorema a seguir. Entretanto, tal demonstração pode ser omi-
tida numa primeira leitura, caso o leitor a considere demasiado indigesta.

TEollEMA 3.36. REGRA DA CADEIA


Sejam 1 e J intervalos abertos e g : / -+ J e f : J -+ IR funções dadas. Se g é derivável em
.xoE/ e/ é derivável em g(x 0 ) E J, então f o g : /-+ IR é derivável em x 0 , com

(/ o g)'(xo) = J'(g(xo))g'(xo). (3.19)

PROVA.
Seja Yo = g(xo). De acordo com (3.18), a derivabilidade de f em y 0 nos permite escrever

J(uo +t) = f (Yo)+ J'(uo)t + s(t)t (3.20)

para todo t E .J - /lo, com ,r; : .J - Yo -+ 1RconLínua e lal que s(O) = O. Analogamente, a
derívahílidade de li <>m:1,·0 fornece, para /, E / - .r0 ,

_q(.r:oI h) = g(.to) t g'(.ro)h + r(h)h, (3.21)

com r: / - x 0 ➔ IRco11Uuuue tnJ q11<'r(O) = O.


Agora. para h E J - x 0 . segue de (3.21) que

(! o g)(xo + h) - (! o g)(xo) = f (g(xo + h)) - f(g(:ro))


= f (g(xo) + g'(xo)h + r(h)h) - f (Yo)
= f(Yo + t(h)) - f(yo),
onde t(h) - g'(x 0 )h + r(h)h. Portanto. para h E J - r 0 . egue daí e de (3.20) que

(/ o g)(xo + h) - (J o g)(xo) = f(Yo + t(h)) - f(Yo)


= J' (Yo)t(h) ,- (t( h) )t (h)
= J'(yo)(g'(xo)h + r(h)h) + s(t(h))t(h)
= J'(g(.ro))g'(xo)h + J'(yo)r(h)h + s(t(h))t(h)
= J'(g(.ro))g'(:z·o)h + J'(yo)r(h)h
+ (t(h))(g'(.ro) + r(h))h
= (J o g)(xo) = J'(g(xo))g'(:ro)h + R(h)h,
on<le R(h) = f'(y 0 )1·(h)+ (t(h))(g'(:r.0 ) + r(h)) parn h E/ - .r0 .
Por fim, H discussão do 1wnúlt imo parágrafo nnH• do cnuncindo da regra da cadeia garante
qu<>,pm;\ mo~trnnno~ que/ o 9 t'• deiiváH!l l'lll x 0 l' qm' vnlc (3.19), é suficiente provar que Ré
cont mun cm / .r0 . com R(O) = O. ~ln.-,i so ~guc imediatamente da versão da regra da cadeia
parn fu11çoc-,contínua!'>(cf. propo i<:fio 11), juntamentt• com o fato de quer, se t são contínuas,
<·0111 r(O) .-,(0)= t(O) = O.

F\:l'\IPI.O 3.3i .
. \ ll'!Vª cln cndcia no!:)permite calcular a deri,·ada da função cosseno a partir da derivadada
!- 'no. De foto. cos x = (::..cno g)(:r), onde g : lR -+ lR é a função dada por g(x)
f1111çilo =j - z,
pnrn todo .r E IR. Port auto. temos pela regra da cadeia que

cos' x = (sen o g)'(x) = sen"l(x) • g'(x) = cosg(x) • (-1)

= - cos (; - x) = -sen x.

EXEMPLO 3.38.
Podemos usar a regra da cadeia. juntamente com a regra de derivação de um prod
demonstrar a regra de derivação de um quociente. Para tanto, sejam /, g : / ➔ R
derhih·eis em I. com g(x) =I=O para todo x E J. Se T : lR \ {O} -+ R denota a

155
I.I\1111"1 l>llil\\l>\"

proporcionalidade inversa. i.c., se r(x) = x- 1


, para x # O. então. pelo item (e) do exemplo 20, T

6 derivivel e T'(x) = -.t- 2 , para todo :r E R \ {O}. Por outro lado, temos = /(.r.) • (r o g)(x) ~f;l
paratodo x E /, de sorte que as regras de derivação de um produto e da cadeia fornecem

(:)' (x) = J'(:r) · (r o g)(x) + f(.r.) • (r o g)'(.r.)

= J'(x) • g(~) + J(x) • r'(g(x))g'(.r)

= 1;(~;
+ J(:r)(-g(.t)- 2
)g'(x)
J' (x )g(.r) - J(x )g'( :r:)
- g(x)2

EXEMPLO 3.39.
A regra da cadeia é a ferramenta adequada para o cálculo de cleriYaclasde funções (clcrivávcb)
definidas implicitamente. Especificamente, scJa C um conjunto de pontos do plano cartesiano,
definido por uma equação da forma E(.r., y) - O. Dado (.r0 , y0 ) E C. suponha que, parn um
retângulo R centrado em (x 0 . y0 ), tenhamos que C n 'R r o gráfico de uma função derivável f ·
~' b) ➔ R, com x 0 E (a, b). Então, dizemos que a cquaçao E(.r.. y) = O define implicitamente
y como função derivável de x, numa Yizinhança de (xo, Yo) (cf. Figura 3 6).
Por vezes, contudo, é mais fácil calcular f'('J·) diretamente a partir da equação E(:r:,y) = O,
lembrando que y = f (x) mas sem utilizar a e:\.""pressão para f (que pode ser muito complicada,
ou mesmo impossível de ser obtida expl1citamente). Sendo esse o caso, diremos que f'(.r:) foi
calculada derivando implicitamente a equação E(x, y) = O.
Por exemplo, se E(x, y) = x 2 +y2-R 2 , onde Ré uma constante positiva, então C corresponde
ao círculo de raio R, centrado na origem do plano cartesiano. Se (x 0 . y 0 ) = (O,R) e n é o
ret&ngulo de centro (O,R), base 2R e altura R, então C n n é o gráfico da função denvável
/ : (-R, R) ➔ IR, dada por f(x) = JR - x . Podemos calcular f'(x) diretamente a partir
2 2

dai (com o auxílio da regra da cadeia) ou, então. obserYando que, como x 2 + y2 = R2 , temos
(derivando implicitamente em relação a x)

?
_r + ?-Y-dy -- O.
dx
, obtemos
X :r.
J' (.r.)

namos esta seção com uma aplicação um tanto mais sofisticada da regra da cadeia.

156
y

n
1 1 1 1
I_ - - - - L - .1 - - .,
a Xo b X

Figura 3.6: funções definidas implicitamente.

Prove que a função /: R ➔ R, dada por f(x) = sen(x 2), não é periódica.

PROVA.
Primeiramente, observe que, se f : lR ➔ lRé uma função continuamente derivável e periódica,
então sua derivada /' : lR ➔ JR,além de contínua, também é periódica. Realmente, se f (x) =
J(x+p), para algum real p > O e todo x E lR,então, pela regra da cadeia, temos f'(x) = f'(x+p)
para todo x, de sorte que p também é um período para J'. Segue, daí, que

Im (!') = Im (f(io,p)),

e a continuidade de f' garante, juntamente com o corolário 57, que f' é uma função limitada.
Agora, sendo f a função do enunciado e g: lR ➔ lR dada para x E lR por g(x) = x 2 , temos
f (x) = (sen o g)(x), de sorte que a regra da cadeia fornece

J'(x) = sen 'g(x) • g'(x) = 2x cos(x 2).

Portanto, se f(x) = sen (x 2 ) fosse periódica, a discussão do parágrafo anterior garantiria que f'
seria limitada, o que, obviamente, não é o caso e nos fornece uma contradição.

Problemas - Seção 3.3


3.1. Calcule as derivadas das funções abaixo, assumindo, em cada caso, que a fun~ão e ~un
derivada estão definidas em seu domínios maximais:

157
( • \ l 'I 1 1 1 < 1 : LL\IITES E DEHl\".\D.\S

(a) J(x) = x3 - 2x2 + 7x + 1. (e) J(x) = x 3 sen x.


(b) f(x) = xffe + x4 . (f) J(x) = 7 fi(x 2
+ 4).
(e) J(x) = ~ +~ (g ) f ( X ) =
.r2-3.r
2:cl+l.

(d) f(x) = cosx + tgx. (h) f(x) = 1::~.r·

3.2. Sejam I um intervalo e f : I ---+(O,+oo) uma função derivãvel em x 0 E / Se g I -+


lR é a função dada por g(x) = efl[x), mostre que g é derivável em .r0 , com g'(.r 0 ) =
¾J(xo)¼-1J'(xo).

3.3. Calcule as derivadas das funções abaixo assumindo, em cada ca~o, que a função e ~un
derivada estão definidas estão definidas em seu domínios maxunah,.

(a) j (X)= (COS X)2 . (e) J(.r) = arcsen (1 - .r2 ).


(b) f (x) = cos(x 2). (f) f( ·) _ .r.nrcco'-(:r )
1
.l - x2+1 •

(c) f(x) = J1 - xJl - x. (orr) J( •1·)-- ~


l+.r- 1 '

(d) f (x) = Jl - x cos(x 2 ). (h) J(.r) = cn( en(cosx)).

3.4. Seja r um racional não nulo e f : (O.+oo) -t R a função dada por J(x) = .rr Prove que
fé derivável, com f'(x) = rxr-l para todo :r > O.

Para o próximo problema, recordemos alguns fatos sobre raízes de polinômios. Dadas uma
função polinomial f, de coeficientes reais e grau n > 1, e uma raiz real a de J, o algoritmo
da divisão para polinômios (cf. seção 3.1 de [141) garante a existência de um mteiro
1 < k < n e um polinômio g de grau n - k tais que

(3.22)

com g(a) =fO. Nesse caso, dizemos que k é a multiplicidade de a como raiz de f.

3.5. Seja f : lR---+lR uma função polinomial não constante. ~Iostre que:

(a) Se a é uma raiz de f e f (x) = (x - a)kg(x), com g(a) =fO, então, para x E IR tal que
f(x) =fO, temos
f'(x) k g'(x)
--+--
f(x) X - O: g(x).

158
(b) Se J(x) = a(x - a1)(x - 0:2) ... (x - an), com a, a:1, ... , O:n E R e a -:f O, então, perft
X ":f'U1, ... , U'n, temos

3.6. Se f : IR\ {3} ---+1Ré a função dada por /(x) = 2:!'


3
1
, calcule j< 5>(o).

3.7. Seja/: [O,+oo)---+ 1Ra função dada por

com dez raízes quadradas. Calcule /'(O).

™ most re que are t g (k)( x )


3 . 8 . Para k• E n, = Pk(:r)),,
(1+x 2 onde Pk e, um po 1·
mom10 de grau k - 1 e
A •

coeficiente líder ( - 1)k 1k!.

3.9. Em cada um dos itens a seguir, admita que, em uma vizinhança do ponto (x0, y 0) dado, o
conjunto-solução da equação E(x, y) = O define y como uma função derivável y = f(x) de
x. Calcule f'(x 0 ) derivando E(x, y) = O implicitamente:

(a) x'1 + y 4 = 17, com (xo, Yo) = (1, 2).


(b) fi + Jy = 10, com (xo, Yo) = (16, 36).
(e) cos(xy) + x 2 = O, com (xo, Yo) = (1, 1r).

3.10. Encontre as tangentes comuns às parábolas y = x 2 e y = 2 + (x - 3)2 .

Para o próximo problema, o leitor pode achar útil rever o material da subseção A.4.3.

S
3.11. Sejam E a elipse de equação + ~ = 1 e 1l a hipérbole de equação ~ - ~ = l. Dizemos
que f, e 1í são confocais se possuem os rnesmos focos. A esse respeito, faça os seguintes
itens:

(a) Mostre que a 2 - b2 = a12 + b12•


(b) Mostre que f, e 1í se intersectam em quatro pontos distintos, e que se (x0, y0) é um
deles, então

2(1 1)_1 1 2(1 1)


Xo a2b'2 + al2b2 - b2 + 112 e Yo a21f2 + a12b2 = al21 -
1
a2.

159
1 \ 1·1 1 1 1 ( 1 i LI\IITl•:s E DEBl\._\()_\s

(e) Se Pé um dos pontos de interseção a que se refere o item (b), prove que as tangentes
a E e 1-lem P são perpendiculares.

3.12. Faça os segui11tesitens:

(a) Seja f : R ➔ R a função dada por

2
f(x) ={ x sen ½,se x =/-O
O, se x = O

Prove que fé derivável em toda a reta, mas que f' não é contínua em O.
(b) Dados números reais x 1 < x 2 < ... < Xn, dê exemplo de uma função derivável
g : R ➔ R, tal que g' é descontínua em x 1, x 2, ... , Xn,

3.13. Prove que a função f: (O,+oo) ➔ R, dada para x >Opor f(x) = sen ./x, não é periódica.

3.14. Sejam a1, a2, ... , an reais dados e f: R ➔ R a função dada por f(x) = a 1sen x+a2sen (2x)+
• • • +_ansen (nx). Se lf (x)I < lsen xi para todo x E R, prove que la1 + 2a2 + • • • + nanl < l.

3.15. Dados reais não nulos a 1 , a 2, ... , an, mostre que a função f : R ➔ R, dada para x E R por
.. n

f(x) = I: a cos(jx),
1
j=l

tem período igual a 21r.

3.16. * Prove a versão a seguir da regra de l'Hôpital 4 : sejam f, g : [a, b) ➔ R funções deriváveis
em a e tais que f (a) = g(a) = O. Se g =/-O em (a, b) e g'(a) =/-O, então

lim f (x) = f'(a)


x➔a g(x) g'(a)'

"Após o matemático frnnc~i; do ttl!culo XVTTGuillnumc F. Antoine, 1Iarquês de l'Hôpital Uma versão mais
refinada da regra de l'Ilôpilal 1wrl'l aprc1,c11tmlu nn proposição ,16.

lGO
3.4 O teorema de Rôlle e aplicações
Considere uma função f : (a,b] ➔ IR, a qual é contínua em (a,b], derivável em (a, b) e tal
que /(a)= J(b) = O. Suponha, ademais, que fé positiva em pelo menos um ponto do intervalo
(a, b). Argumentando heuristicamente, translademos o eixo das abscissas paralelamente a si
mesmo, até que ele tangencie o gráfico de f em um ponto (e, J(c)) (cf. Figura 3.21). Então, por
um lado, tal tangente deve ter coeficiente angular igual a f'(c); por outro, sendo paralela ao eixo
das abscissas, seu coeficiente angular deve ser igual a O, de modo que devemos ter f'(c) = O.

a e b
X

Figura 3.7: o teorema de Rôlle.

O primeiro resultado desta seção, conhecido na literatura como o teorema de Rôlle 5, coloca
a discussão do parágrafo anterior em bases rigorosas.

LEMA 3.41. ROLLE

Seja f : (a,b] ➔ IR uma função contínua em (a, b] e derivável em (a, b). Se f (a) = f(b) = O,
então existe e E (a, b) tal que /'(e) = O.

PROVA.
Sejam e, d E [a, b]os pontos em que f atin~e seus valores mínimo e máximo, respectivamente.
Se e, d E {a, b}, então, para todo x E [a, b], temos O = J(c) < f(x) < J(d) = O. Logo, f é
identicamente nula em (a. b], e nada há a fazer.
Suponha, pois, que e E (a, b) (o caso em que d E (a, b) pode ser tratado de modo análogo).
Como J(x) > J(c) para todo x E (a, b], temos que

x E (e, b] ⇒ J(x) - f(c) > O e x E (a,e) ⇒ J(x) - f(c) < O.


x-c x-c
5 Após Michel Rôlle, matemático francês do século XVII.

161
< \ 1•1 1 1 1 1 1 ; L 1\ 111 F s E 1) EH I \ •.\ (). \ "i

Portanto, segue dos problemas 1.2 e 1.5, página 136, que

/'(e) = lim / (x) - J(c) > O e J'(c) = lim J(x) - f (e) < O.
• x-+c+ X - C x-+c- X - C

Logo, f'(c) = O.

Vejamos um exemplo de aplicação do resultado anterior.

EXEMPLO 3.42.
Dados a, b,e E R, mostre que a equação 4ax 3 + 3bx2 + 2cx = a + b + e tem pelo menos uma
raiz entre O e ---
1.

PROVA.
Primeiramente, note que a aplicação do TVI é inconclusiva, pois, se f (x) = 4ax 3 + 3b:c2 +
2cx - (a +·b + e), então f (O) = -(a+ b + e) e f(l) = 3a + 2b + e, de sorte que /(O) e f (1) podem
ter sinais iguais (por exemplo, para a= l e b =e= -~).
Por outro lado, se J(x) = ax 4 + bx3 + cx2 - (a+ b + c)x, então f (O) = J(l) = O. Portanto,
pelo teorema de Rôlle, existe x 0 E (O,1) tal que f'(x 0 ) = O. Mas isso é o mesmo que dizer que
x 0 E (O,1) é uma raiz da equação do enunciado.

Analisemos, agora, a situação geral correspondente àquela do iaício desta seção. Mais pre-
cisamente, consideremos novamente uma função f : [a, b) --► IR, contínua em [a, b] e derivável
em (a, b), mas tal que /(a) e f(b) podem assumir valores quaisquer. A secante ao gráfico de f
passando pelos pontos (a, f (a)) e (b,J(b)) tem coeficiente angular f(bl=~(a). Se a transladarmos
paralelamente a si mesma, até que ela tangencie o gráfico de f em um ponto (e, J(c)) (cf. Figura
3.8), então tal tangente terá coeficiente angular igual a f'(c). l\Ias, como essa tangente também
é paralela à secante inicial, a proposição 59 garante que seu coeficiente angular também deve ser
igual a /(bl=!(a) . Assim,
J'(c) = J(b) - J(a).
b-a
Esse resultado é O conteúdo do teorema do valor médio (abreviado TVM) de Lagrange,
que demonstramos no resultado a seguir.

162
y
J(b) ----------------
.,.
......
.,.
.,. .,.
f(a)
f(c)

a e X
b

Figura 3.8: o teorema do valor médio de Lagrange.

TEOREMA 3.43. LAGRANGE


Se f : [a, b] ➔ 1Ré uma função contínua em [a, b] e derivável em (a, b), então existe e E (a. b)
tal que
f(b) - J(a) = J'(c).
b-a

PROVA.
Seja g : [a, b] -+ 1Ra função dada por

g(x) = f(x)- ( (: :) /(a)+ G=


:) /{b)),

para todo x E [a, b]. (Observe que a função de x dada pela parcela entre parênteses do segundo
membro é, precisamente, a equação da secante ao grâfico de f pelos pontos (a, f(a)) e (b,J(b)).)
Claramente, g é contínua em [a, b] e derivável em (a, b), com g(a) = g(b) = O. Portanto, pelo
teorema de Rólle, existe e E (a, b) tal que g'(c~ -=-O. Mas, um cálculo fácil fornece

g'(X) = J' (T) _ (- f (a) + f (b) ) 1


b-a b-a

de sorte que
0 = g'(c) = J'(c) _ f (b) - f (a).
b-a

163
( \ 1 1 1 1 111 ; Ll\1111•:s J·: Dl-:Hl\'.\l).\S

E .• EMPLO 3.44.
Para :e.y E (- 1, !), mostre que I tg x - t,gYI > lx - YI•

PRO\.\.
Se .r. = y, nada há a fazer. Senão, segue do TVM a existência de e entre x e y, tal que

tg X - tg y = ( tg' C)(X - y) = + (X - y).


cose
?\Ias, como Icos cl < 1, temos
1
1 tgx - tgyl = 2 lx -yl > lx -yl.
cose

No que segue, discutimos uma generalização do teorema do valor médio de Lagrange a duas
funções, conhecida como o teorema do valor médio de Cauchy. ( ote que (3.23) reduz-se
ao TVJ\1 de Lagrange, caso tomemos g(x) = x.)

TmREMA 3.45. CAUCHY


Se/, g : (a,b]➔ R são funções contínuas em [a, b] e deriváveis em (a, b), então existe e E ( a, b)
tal que
(f(b) - f(a))g'(c) = (g(b) - g(a))J'(c). (3.23)

PROVA.
Seja h : [a, b] ➔ IRa função dada, para x E (a,b], por

h(x) = (J(b) - f (a))g(x) - (g(b) - g(a))J(x).


É imediato que h é contínua em [a, b] e derivável em (a, b), de sorte que, pelo TV1I de Lagrange,
existe e E (a, b) tal que
'( ) = h(b) - h(a)
i e b .
' -a
Mas, como h(a) = h(b) = J(b)g(a) - f(a)g(b), Lemosh'(c) = O e basta, agora, observar que

h'(c) = (f(b) - J(a))g'(c) - (g(b) - g(a))J'(c).

JG4
Situações há. no Cálculo. em que temos funções contínuas f. g : [a, b) ➔ R, tais que /(a,)•
g(a) = O, e precisamos avaliar (se existir) o limite

lim J(x).
x-+a g(x).

em geral. referi-mo-nos a uma tal situação como a análise de uma indeterminação da forma g.
~o problema 3.16. página 160. vimos como abordá-la quando f e g são deriváveis em a, com
g'(a) =f O. O TV~t de Cauchy nos permite obter uma versão mais refinada desse resultado,
conhecida na literatura como a regra de l'Hôpital.

PROPOSIÇÃO 3.46. REGRA DE L'HóPITAL PARA ~

Sejam J,g : (a. b) ➔ R funções derivá,·eis. tais que lim.r-+af(x) - lim.r-+a9(x) = O. Se


g. g' =f O em (a, b). então
. f'(x) . J(.r.) L
11m -( -) = L =} luu -(-) = .
:r-+a g' .r, :r-a g .z::

PHOV,\
Estenda J e g continuamente a [a, b), pondo f(a) = g(a) = O. Para x E (a, b), segue do TV~I
de Cauchy que
J(:t) J(.t) - J(n) f'(c(x))
(3.24)
g(.r) = g(.r) - g(a) = g'(c(:r))'
pnrn algum c(.r) E (a .. r.). Então, temos claramente que limx -+a c(x) = a. Daí (e supondo que
hm,-+o ~;~;J
: L), uma pequena modificação da <:>uge~tão dada ao problema 1.10, página 137,
·
gmanle g'(r(.r)) L P ortan t o, (3 .24) d·a-nos que 1·1m.r-+aM
que 1·1111.i-+a /'(c(:r)) g(x) -- L •

EXE\IPLO 3.47.
Podemos utilizar a regra de l'Hôpital para ~ -.ra calcular limx-+O1 -7zr
(observe que x t-+ ,li
não é deri,·ável em :r =O.de forma que não podemos utilizar o resultado do problema 3.16, página
160). Realmente, as funções f(x) = 1 - cosr e g(x) = ./x (para x > O) satisfazem as hipóteaea
da proposição anterior e são tais que

lim J'(x) = lim sen x = lim 2vx sen x = O


z-+0 g' (X) :r-+O 1/ (2 vx) r-+0

(a última igualdade seguindo da proposição 10). Logo, lim.r➔O ~~;~ = O.

165
, \ 1 1 1 1 1, 1 : L1\11·11-:s E DEHl\'.\l>.\S

Problemas - Seção 3.4


4.1. ._ e.iam I C IR um intervalo e f, g : I ➔ IRduas funções deriváveis. Se f' = g' em I, prove
que f - g é constante.

4.2. * Se I é um intervalo e f : I ➔ IR é uma função duas vezes derivável e tal que J" é
constante, pr<;>ve
que ou f é identicamente nula ou é um polinômio de grau menor ou igual
a 2. Mais precisamente, se x 0 E I, mostre que ('
lr'(x)
f(x) = f(xo) + J'(xo)(x - xo) +
,.. 2
° (x - xo)2 .

4.3. Encontre todas as funções f : IR ➔ IR tais que (f(x) - f(y)) 2


< lx - yl3 , para todos os
x,y E IR.

4.4. Se ao, a1, ... , an são números reais tais que~+ T; ·· ·+ nª_;1 = O, mostre que o polinomio
ao+ a1x + • • • + anx" tem pelo menorm~ raiz real.

4.5 . .Moske que a equação x 2 = x sen x + cos x tem exatamente duas raízes reais.

4.6. Refaça o problema 2.7, página Í'48;' dessa vez supondo que f : I ➔ IR é continuamente
derivável em I.

4.7. Em cada um dos itens a seguir, calcule o limite pedido:

(a ) limx-+0 1-cosx .
(C) 1lffix-+0 ✓cosx-~
zl • xl .

. 1-~ . sen 2 {1fx}


(b) 1lfilx-+0 ~- (d) llffix-+-1 (x3+l)l •

4.8. Sejam J,g: {O,i) ➔ IR as funções dadas por f(x) = x 312 sen ~ e g(x) = x 113 - cosx + 1.
• t'Ir, l'lffix-+0
e alCUle, se eXIS /(x)
g(x).

4.9. Se J: (a, b) ➔ 1Ré uma função duas vezes derivável, mostre que, para todo x 0 E (a, b),

. J(xo + h) + J(xo - h) - 2J(xo) _ J"( )


11m
h-+0
h2 - xo .

4.10. Seja J:
[a, b) ➔ lR uma função duas vezes continuamente derivável no intervalo (a,b), com
J(o.) = O e J"(a) # O.

(a) Mostre que existe e E (a, b) tal que f (x) - (x - a)f'(x) # O, para todo x E (a, e).
(b) Calcule lím.i-+a /(A~,~'.)~g},(J)cm função dos valores de JCJ.>(a),
para O< k < 2 (o item
anterior garaute que o denominador da fração tem sentido no intervalo (a, e)).

1G6
.11. Prov a guio - d r gr 1 Hôpi al par ind rminaçõ S em + :
a > O Jg : (a, ➔ . - d ri,á 1 t • qu g g' =f O em (a,+ ).
limx J(x) = limx g(x) = O pro qu

lim f'(x) = L ⇒ lim f ((x))= L.


r g'(x) z 9

.12. J
f (x) = an n + ªn-1Xn-l + ... + a1 + ao
bn n + bn-1 n-l + · · · + b1 + bo

pra od ufi i n m n grand (por x mplo maior do qu od r íz r ai do


p lin mio do d nomin dor).
A a r - o da r gr d l Hôpital d d p lo probl ma ant rior
f(x).

4.13. uma funç- d rh··v l tal qu limi limx + J( ) limx-H xf'(x)


Xl m 1 ul p ív i val r d úl imo limit .

.1 . t I qu lf'(x)I < odo r al ond e um


n nt r n úni oE 1 qu /( o) = O·

.1 . jm/c um in rt r.o E / J I fun -o on ínu d ri áv 1


m / \ {.ro} Xl L = lim.r r J' :r.), pr ,. u f d ri l m x 0 om f'(xo) = L.

u n fun - n, ,. 1 J : ( ) (O + ) 1qu , p ra todo I


y r 1

/(. )- > J(:r y)(J(x) y).

. 7. t d fun - riv v i f : qu /(O) =O IJ'(x)I < 1/(x)I para


.t E .

167
t \ 1 1 1 1 111 : Ll~ll'I r-:sE DEHl\',\f),\S

3.5 pri111eira variação de uma função


\'imo~, nn seção 2.1, que toda função contínua J : [a, b) ➔ IR atinge valores extremos (i.e.,
múxtmo e núnimo) em [o, b]. esta seção, analisaremos se, adicionando hipóteses genéricas
rnzo{n-cb ~obre f (por exemplo, supondo J derivável), existe algum procedimento que nos permita
cnkular efetivamente os pontos extremos correspondentes.
Em tudo o que gegue, I denota um intervalo da reta. Ademais, o interior de I é o intervalo
obtido de J pela exclusão de sua(s) extremidade(s), caso ela(s) exista(m). Assim, se I = [a, b),
(a. b), (a, b]ou (a. b), o interior de I é o intervalo (a, b); analogamente, se I = [a,+ ) ou (a, +oo),
seu interior é o intervalo (a, +oo), e se I = (-oo, b)ou (-oo, b), seu interior é o intervalo (-oo, b).
Nosso propósito nesta seção é resolver o problema descrito no primeiro parágrafo para funções
f : I ➔ 1Rcontínuas em / e deriváveis em seu interior. Para tanto, precisamos, inicialmente, da
definição a seguir.

DEFINIÇÃO 3.48.
Dada uma função f : J ➔ IR, dizemos que x 0 E J é ponto de máximo local (resp.
mínimo lócal) para/ se existe ó > O tal que J(x 0) > f(x) (resp. J(x 0) < f (:r;)), para todo
X E / n (Xo - ô, Xo + ó) .

. ra Figura 3.9, os pontos x 0 , x~ ex~ são pontos de mínimo local para a função f : (a, b) ➔ R
cujo gráfico é esboçado. Observe que, em vizinhanças de cada um desses pontos, os valores da
função são maiores ou iguais que os valores assumidos nos pontos em questão (conforme indicam
os t:rês segmentos pontilhados horizontais). Também, x~ é o único ponto de mínimo global (i.e.,
um ponto onde f assume o menor valor em (a, b)), enquanto x 0 e x~ são pontos de mínimo
locais mas não globais. Por fim, veja que f possui dois pontos de máximo local (você consegue
identíficâ-los?), mas nenhum ponto de máximo global.
Genericamente, um ponto de máximo ou mínimo local para uma função f : I ➔ R é deno-
mina.do um ponto extremo local de f. Se I é um intervalo aberto e f : / ➔ IR é derivável,
mostraremos a seguir que os pontos extremos locais de f são zeros de sua derivada. Entretanto,
urna vez que tais ZPros desmnpcnharão papel preponderante na discussão subsequente, antes de
apresentarmos a prova d<•ssc> rcHtiltnclointroduzimos uma nomenclatura relevante.

Se/ C Ré um intervalo aberto e / : / ➔ IR é uma função derivável, dizemos que :r0 E J é


erftlco de J se f'(xo) = O.

Observe qm• a uoção de>po111ocríLico 11d111i1c


uma intcrprclnção geométrica óbvia: sendo

LG8
a Xo X

Figura 3.9: pontos de mínimo local de f: I ~ IR.

novamente I um intervalo aberto, segue de (3.14) que os pontos críticos de uma função derivável
/ : I ➔ IRsão precisamente aqueles em que a reta tangente ao gráfico de f é horizontal.
Podemos, agora, enunciar e provar o seguinte resultado fundamental, conhecido como o teste
da primeira derivada para pontos extremos locais. Apesar da demonstração ser bastante
similar à do teorema de Rôlle, a repetiremos aqui para a comodidade do leitor.

PROPOSIÇÃO 3.50.
Se I C IR é um intervalo aberto e f : / ➔ IR é uma função derivável, então todo ponto
extremo local de f também é ponto crítico.
-----~-~~---------------'

PROVA.
Analisemos o caso em que x 0 E / é um ponto de mínimo local para /, sendo a prova no outro
caso totalmente análoga.
Tome ó> O tal que (x0 - ó, x 0 + ó) C I (lembre-se de que I é aberto, logo um tal ó sempre
existe) e
O < lx - xol <ó=} f (x) - J(xo) > O.

Então, para x 0 < x < Xo + ó temos f(x;=~~xo) > O, de sorte que

J'(xo) = lim f (x) - f(xo) ~ O.


x➔xo+ X - Xo

Raciocinando de modo análogo com Xo - ó< x < xo, concluímos que

J'(xo) = lim f(x) - f(xo) < O.


x➔xo- X - Xo

169
1 \ 1 , ,, ; LI\IITl·:s E DEBl\'.\l>.\s

Como corolário do t<.'s(cela primeira derivada, lemos o seguinte crilério de pesquisa de pontos
l'X t l'l'lllO '.

COROLÁRIO3.51.
Seja / : / ➔ R uma função contínua cm I e derivável no interior de I. Se / atinge um
valor extremo (máximo ou mínimo) em I, então o ponto extremo correspondente é uma das
extremidades de / ou um ponto crítico de f.

PROVA.
Suponhamos que f atinge um valor mínimo em I (o caso em que f atinge um valor máximo
pode ser tratado de modo análogo). Seja x 0 E / tal que f (x 0 ) = min{/(x); x E I}. Se xo é
uma extremidade de I, nada mais há a fazer. Senão, x 0 pertence ao interior de/; mas, como o
interior de I é um intervalo aberto, o teste da primeira derivada garante que x 0 é ponto crítico
de f.

Conforme o exemplo a seguir deixa claro, o corolário anterior dá informação suficiente para
encontrarmos os valores extremos de funções / : [a,b] ➔ IR,contínuas em [a,b] e deriváveis em
(a. b). contanto que saibamos encontrar as raízes da equação f'(x) = O.

~3.52.
EDMlwtno valor máximo da função f: [O,1] ➔ IR,dada por f(x) = x - x 1.

SOLL'ÇÂO.
Como f é contínua em [O,l], sabemos, pelo teorema 53, que f assume um valor má.ximo em
[O,lj. Pelo corolário anterior, o ponto de máximo correspondente é O, 1 ou um ponto crítico de
f. Agora, como f' (.r) = 1 - 4:r3, temos que 774é o único ponto crítico de f, com f ( ~) = V4.
Por fim, como J(O) = f (1) = O< ~' segue que o valor máximo de J é i·
A proposição a ACguir,conh<'<fllP!lCÍa
cio TVM de Lagrange, nos ensina como obter os intervalos
de monotonícidade <fouma função derivnvcl. Doravante, nos referiremos a ela como o estudo da
primeira variação de urna fuuçao dcrivávclu.
•r-aterminologm 11lu<1,•
w> pa1wl pn•pu11dl'1n11It• dn pn111cirndei ivadn 110rc~ultado em que:;tão, bem como uo
fitode que, claMlc1&11.cutc, ele•,ivncl11dP 111111L
11pri11111lrn c\ciivftvcl c111clC'nommndnsun pm11eira uarzação.
í1111c:no

170
PROPOSIÇÃO 3.53.
Se f : I -> IRé uma função contínua em / e derivável no interior d~ então.

{a) f' > O no interior de I se, e só se, f é não decrescente em /.

{b) Se f' > O no interior de /, então f é crescente em I, mas a recíproca não é válida.

(c) f' < O no interior de / se, e só se, / é não crescente em /.

(d) Se f' < O no interior de J, então f é decrescente em I, mas a recíproca não é válida.

PROVA
Provemos somente os itens (a) e (b). sendo a prova dos itens (c) e (d) inteiramente análoga.

(a) Suponha, primeiro, que fé não decrescente em /. Para um ponto x no interior de/, tome 0

8 > O tal que (x 0


- ô,xo + 8) e I. Se x < x < x + 8, então x E I e f(x) - f (x > O. Portanto,
0 0 0
)

f(.i::)-/C:ro)
r-:ro
> O e1 fazendo x ➔ x O+, obtemo
-

f'(.ro) = lim f (x) - f (xo) > O.


2'-+:r:o+ X - Xo

Reciprocamente, suponha que /' > O no interior de /. Se a, b E I são tais que a < b, a
cont inuidadc de f cm / garante suc1continuidade também em (a, b]. Por outro lado, como (a, b)
está conLido no interior de /, temos f derivável em (a, b), de sorte que o TVM de Lagrange
garante a existência de e E (a, b) tal que

f (b) - f (a) = f'(c) > O.


b-a
Logo, f(b) ~ f(a), e a arbitrariedade da escolha de a< bem I garante que fé não decrescente
cm I.

(b) Suponha que f' > Ono interior de J, e «-,:)1<\lll a< b dois pontos quaisquer de/. Novamente
pelo TV~l de Lagrange, existe e E (a, b) tal que

J(b) - J(a) = J'(c) > O,


b-a
de modo que J(b) > f(a). Logo, fé crescente em /.
Para ver que a recíproca não é verdadeira, considere f: IR ➔ IR dada por f(x) = x3 . Ela é
cre cente em toda a reta, mas J'(O) = O.

171
uir m lrc m u iliz r pr p - 5 p r b rd r pro 1 m nv lv ndo
mínim d fun - fl

é função dada por f(x) = ~:? mo tr qu f ating um valor mínimo


tal valor.

o.
f ting um valor mínimo m [O + ) o or lário 1 gar n qu al o orr m O ou m
um ponto rí i o d f. al uland pnm ir d riv da d f bt m

J'( ) = 2 (X + l) - ( + 3) • 1
2
= _x2__ _
(x + 1)2 (

d r qu f'(x) = O ó x = l (1 mbr - d qu d mo r x > O).


Por ou rolado como x 2 + 2x - 3 = (x + 3)( - 1) on luím qu f' - n ga i a no in rvalo
(0.1) positi a no int rvalo (1 + ). Portanto pr po iç~o 3 gar n qu f , d r n
no intervalo [O 1] e cr en e no in rvalo [1 + ) d modo qu J r alm n a ing um v lor
mínimo o fazendo em x = l. Logo o valor mínim d J ' f (l) = 2.

A seguir aplicamo o tudo da prim ira variação d uma função para d r um d mon raç -o
elementar da desigualdade entre as m •dias aritm 'tica g om - rica.

3.55.
um inteiro n > l e reais positivo a 1 a2 ... an, t m

igualdade

PROVA.
Façamo índuç- br n > 1, L 71 = dm n r d n x mplo J dado
> 2 in iro suponha, p r hip L e d iguald d d nun 1 do
uaisqu r k - 1 r ais p , i iv m i u ld f r m do iguai .
...•-·••• k r is po itiv, a1, ... ,ªk-J a,.., J J: ( ) e fun ; d d p x> p r

J(.i:)- a1 + · · · + a,.. 1 + x - k ij a 1 ... a1.; I • ••

17..,
.\ l'Hl\11.IH \ \ \l<I \<, \< > l>I 1 \I \ 1 t \.<, \< 1 1

Se mostrarmos que f (x) > Opara todo x > O,com igualdade se, e só se, a 1 = · · · = ªk-t = x,
concluiremos em particular que J(ak) ;:> O, com igualdade se, e só se, a 1 = • • · = ªk-1 = ak,
conforme desejado. Para tanto, observe que fé derivável, com

para todo x > O. Portanto, sendo x 0 = k-.ya1 ... ªk-l, temos J' < O em (O,x0 ), f'(xo) = O e
f' > O em (x 0 • +oo), de maneira que, pela proposição 53, fé decrescente em (O,x 0 ] e crescente
em [xo,+oo). Logo, f atinge seu valor mínimo em x = x 0 , sendo f(x) = f(xo) se, e só se, x = xo,
Por firn, uma simples substituição fornece

(3.25)

de modo que f(x 0) > O, pela hipótese de indução. Logo,

f (x) > J(xo) > O


para todo x > O, ocorrendo a igualdade se, e ::,6se, x = x 0 e J(x 0 ) = O. Basta, agora, observar
que, graças a (3.25) e à hipót~e de indução, lemos /(x 0 ) = O se, e só se, a 1 = · · · = ªk-l, de
forma que há igualdade se, e só se, a 1 = • • • = ª1.- 1 = x.

Por vezes, utilizaremos o estudo da primeira variação em conformidade com o corolário a


seguir.

COROLÁRIO 3.56.
Sejnm I um intervalo aberto, f: / ➔ Ruma função duas vezes derivãvel e x 0 E/ um ponto
crítico de f. Se f" > O (resp. J" < O) em/, então xo é o único ponto de mínimo (resp. máximo)
global para f. Em particular, x 0 é o único ponto crítico de f.

PROVA.
Suponha J" > O em / (o caso /" < O em J é totalmente análogo). Como f" = (!')', o item
(b) da proposição 53 (aplicado a f', no lugar de /) garante que f' é crescente em /. Mas, como
f'(x 0 ) = O, segue que f'(x) < O para x < xo e f'(x) > O para x > x 0 . Então, os itens (b) e
(d) da proposição 53 (desta feita aplicada a J) garantem que fé decrescente em I n (-oo, x 0 ] e
crescente em I n [x0 , +oo). Logo, x 0 é o único ponto de mínimo global para/ e, como tal, seu
único ponto crítico.

173
( 1 '' ; Ll\111 FS I·: lh:Ul\'.\l).\S

De po~" do corolário anterior, vejamos uma aplicação do estudo da primeira variação à


Geom ,t ria Euclidiana.

ExBMPLO 3.57.
São dadosuma região angula.r LAOB, tal que AÔB < 90º (cf. Figura 3.10), e um ponto P
-+ -+
em seu interior. Escolha pontos X E O A e Y E O B tais que X, P e Y sejam colineares. Mostre
-+
que existe uma úníca posição de X sobre OA que minimiza o comprimento do segmento XY.

Figura 3.10: segmento XY de comprimento mínimo.

PROVA.
Medindo todos os ângulos envolvidos em radianos, sejam AÔP = a, BÔP = f3 e OXY = 0,
de sorte que a e f3 são conhecidos e 0 é variável (i.e., depende da posição de X). Como a soma dos
.-..
ângulos do triângulo OXY é igual a 1r radianos, temos O < 0 < 1r - a - f3 e OY X = 1r - a - f3- 0.
Aplicando a lei dos senos (cf. seção 7.3 de [10]) aos triângulos XOP e YOP, obtemos

PX sen a PY sen f3
--=--e--=------
0P sen 0 OP sen (a + f3 + 0) '
de forma que
XY = PX + py = OP (sena+ sen/3 ) .
sen0 sen (a+ f3 + 0)
Portanto, sendo f: (O,1r - a - /3) ~ IRa função dada por

f (O)= seno + sen f3


sen 0 sen (a+ f3 + 0)'

bastamostrarmmi que existe um único 0 E (O,1r - a - /3) tal que f atinge seu valor mínimo em
8.

 174
O teste da primeira derivada garante que, se J tiver um ponto de mínimo global, entio tal
ponto ê crítico. Por outro lado, um cáleulo fácil fornece
!'( 0) = _sena
cos 0 _ sen .Bcos(o + .B+ 0).
sen 20 sen 2 (o + .B+ 0)
Daí, pondo 'Y = 7r - o - .B,obtemos facilmente que
lim J'(0) = -oo e lim J'(0) = +oo,
8➔0 8➔~

de sorte que o TVI garante a existência de 00 E (O,-y) tal que !'(0 0 ) = O. Agora,
!"( 0) = sen a{l + cos2 0) + sen,8(1 + cos2 (o + .B+ 0)) > 0
3
sen 0 3
sen (o + .B+
0) '
uma vez que o, {3,0, o+ .B+ 0 E (O,7r). Portanto, o corolário 56 garante que 00 ê seu único ponto
crítico, o qual é um ponto de minimo global.

Terminamos esta seção aplicando a proposição 53 para demonstrar o resultado a seguir,


conhecido na literatura como o teorema de Darboux 7 .

TEOREMA 3.58. DARBOUX

Se I C IRé um intervalo e f : / ➔ R é uma função derivável, então a função f' : / ➔ R,


mnda que não seja contínua, satisfa~ a propriedade do valor intermediário.

PRO\r\.
Sejam a < b em I e d um real pertencente ao intervalo de extremidades /'(a) e f'(b). Que-
remos garantir a existencia de e E (a, b] tal que f'(c) = d. Para tanto, consideremos dois casos
!:>Cparadamente:

(i) f'(a) < O < J'(b) (ou vice-versa): se / não é injetiva em (a, b], então existem o < .Bem
(a, b] tais que /(o) = J(,B). Pelo TV11 de Lagrange, existe e E (o, .B)(logo, e E (a, b)) tal que
f'(c) = [<PJ=!(o) = O. Se f ê injetiva em [a, b], segue do teorema 61 que/ ê monótona em [a, b].
Portanto, pela proposição anterior, temos /' > Oem (a, b] ou J' < Oem [a, b], o que não é o caso.

(ii) /'(a) <d< J'(b) (ou vice-versa): se g: I ➔ IR é a função dada por g(x) = f(x) - dx, então
g é derivável, com g'(x) = J'(x) - d para todo x E /. Portanto, g'(a) < O < g'(b), ou vice-versa,
de forma que, pelo item {i), existe e E (a, b) tal que g'(c) = O. Mas isso é o mesmo que termos
f'(c) = d.

7 Após Jean-Gaston Darboux, matemático francês dos séculos XIX e XX.

175
, ,, : Lt\llTt•:s t•: Dt-:Bl\ •.\l>.\s

Problemas - Seção 3.5


5.1. l roY~ a proposição 24 utilizando os métodos desenvolvidos nesta seção.

5.2. Em cada um dos itens a seguir, encontre, se houver, os valores máximo e mínimo das
funções dadas:

(a) f: [O,+· ) --+IRdada por J(x) = .c;f16 •


(b) J: IR--+IRdada por f (x) = axf+b' onde a e b são reais positivos dados.
(e) f: (O,+ ) --+lR dada por J(x) = x2 + ;, onde a é um real positivo dado.
(d) /: (O,+oo)--+ IR dada por f(x) = xf: 0
, onde a é um real positivo dado.

5.3. Para a, b, e E lR, seja f : IR --+ lR a função dada por f (x) = x 3 + ax 2 + bx + e. Discuta a
primeira variação de / em termos de a, b e e.
2
5.4. ~lostre que sen x < x e cos x > 1 - ; , para todo x > O.
5.5. Em um quadrado ABCD, de diagonais AC e BD, os lados têm comprimento 2. Marque
o ponto médio P do lado AB e, em seguida, um ponto Q E AD e um ponto R E CD tal
que PQR = 90º. Encontre a posição de Q sobre o lado AD, tal que a área do triângulo
PQ R seja a maior possível.

5.6. Sejam f : (a, b) --+ IR uma função derivável e P(x 0 , y0) um ponto que não pertence ao
gráfico de f. Se existe A E G1 que realiza a distância de P ao gráfico de f, i.e., tal que

AP = min { A' P; A' E GJ},


..,_
prove que AP 1-r, onde r denota a reta tangente ao gráfico de J em A. ( esse caso, dizemos
...........
que AP é a reta normal ao gráfico de f em A.)

5.7. Sejam f : (a, b) --+IRe g : (e, d) --+lR funções deriváveis cujos gráficos não se intersectam.
Se existem pontos A(o, /(a,)) e B(f3,f ((3))que realizam a distância entre os gráficos de f
e g, Le., tais que
A l3 = mi11{ A' B 1; A' E G J e B' E G9}, (3.26)
mostre que J'(a,) = !l((-J).

DadaH cforiváveiHJ: (a, b) --+ IRe 9 : (e, d) --+IRcujos gráficos não e intersectam, o
fuuc;õc,)ff
resultadodo prol))t•111n
a11tcrior garn11toque o problc111ade encontrar pontos \ E G 1 e B E

17G
G9 satisfazendo (3.26) equivale ao problema de minimizar a função F: (a, b) x (e, d) ➔ R
dada por
F(x, y) = (x - y) 2 + (J(x) - g(y)) 2 ,

sujeita ao vínculo f'(x) = g'(y). Contudo, as funções J,g: (O,+oo) ➔ R, dadas por
J(x) = ~ e g(x) = O, mostram que nem sempre existe um tal par de pontos A e B. O
próximo problema aplica a discussão acima em um caso positivo.

5.8. Calcule a distância entre os gráficos das funções f, g : 1R ➔ 1R tais que f (x) = x 2 e
g(x) = 1-{x-3) 2 , admitindo (fato geometricamente plausível, se esboçarmos os gráficos em
questão) que existem pontos sobre tais gráficos que realizam e distância entre os mesmos.

5.9. Na Figura 3.11, temos BC = 2 BH. Calcule o maior valor possível da medida do ângulo
BÂC.

H B c
Figura 3.11: maximizando o ângulo BÂC.

5.10. ABCD é um trapézio isósceles de bases AD e BC, com AD> BC, e lados não paralelos
AB e CD. Se BC = a e AB = CD = b, calcule qual deve ser a medida dos ângulos
BÂD = ADC para que ABCD tenha a maior área possível.

5.11. São dados uma região angular LAOB, tal que AÔB < 90° (cf. Figura 3.10), e um ponto P
-+ -+
em seu interior. Escolha pontos X E C ~ e Y E OB tais que X, P e Y sejam colineares.
Mostre que o produto P X • PY é mínimo se, e só se, o triângulo O XY é isósceles, de base
XY.
+--+ +--+
5.12. a Figura 3.12, temos AA', BB' J_r, AA' = a, BB' = b e A' B' = l. Se l < J2(a 2 + b2),
mostre que existe um único ponto P E r tal que AP · BP assume seu valor mínimo.
Mostre, também, que P E A' B'.

5.13. Mostre que C::t:


r+l > c::r,para todos m, n EN.

177
' 1 1 ,, ; LI\IITl-:s F DEBl\.\ll b

A
1
1 B
ai
1
1 :b
A' l B' r

Figura 3.12: minimizando AP · BP.

5.14. Para cada real k > 1, calcule o m nor valor po 111' l" 1/, li i' f 1/ S l 1 ' \IS

tais que (x + J1 + x 2 )(y + J1 12) = k.


5.15. Calcule o menor valor po ív l da . ·pr ]
'2.,:
IJ I'
t r 111 li (ll"ilivis,

.1 i
5.16. Em um triângulo B . n2J
2
,:1 ~ = ...'11 ª-' ... :2• 1J d

5.17. Seja / : lR➔ IR um fun à p linomi,ll 1 11

/ ( r. - /'( /' '( ..


para todo x E IR. Pr v qu /(.r.) -

17
3.6 A segunda variação de uma função
esta seção. continuamos a estudar como o comportamento da derivada de uma função
influencia o formato de seu gráfico. Em tudo o que segue. I e R denota um intervalo.

DEFINIÇÃO 3.59.
Uma função f : I ➔ Ré convexa se, para todos a. b E / e t E [O,l], tivermos

/((1 - t)a + tb)) < (1 - t)f(a) + tf(b).

f(b) ---------------- -
(1 - t)/(a) + tf(b}
/((1 - t)a j~i~

ª (l - t )a + tb b

Figura 3.13: gráfico d uma função convexa.

Qunndo f , t 1 in. de O a 1, um pouco d Geometria Analítica (cf. proposição 52) garante que
o~ ponto~ do forma (1 - t)(a, /(a))+ t(b. f(b)) percorrem o segmento da secante ao gráfico de f,
trnçndn pelo~ pontos (a, /(a)) e (b, f(b)). Portanto, concluímos que f : I ➔ Ré convexa se, e
só M.\ pnrn todo~ a , b em /. a porção do gráfico de f situada entre as retas x = a e x = b não
lsh\ ncinm da r~ta que p~ pelo ponto (a, f(a)) e (b. f(b)).
De outro forma. ~endo

'R+(f) = {(x. y) E R2; x E J e y > f(x)},

temo que f é com-exa e. e só se. 'RT(/) é uma região convexa do plano.


A próxima definição refina a noção de função convexa (observe a diferença na variação de t,
em relação à definição anterior).

179
e 1 1 1 1 1 1 , : L 1\ 11 1 I· '-i 1-: i )f.: H 1\ •.\ 1) \"

DEFINIÇÃO 3.60.
Uma função / : / -+ IR é estritamente convexa se, para todos a, b E / distintos e todo
te (O, 1), tivermos
f ((1 - t)a + tb)) < (1 - t)J(a) + tf (b).

Analogamente à discussão acima, dizemos que uma função f : / -+ R é côncava se - f for


convexa. Portanto, f é côncava se, e só se,

f ((1 - t)a + tb)) > (1 - t)f(a) + tf (b),

para todos a, b E J e t E [O,1). Analogamente, f : / -+ R é estritamente côncava se, e só se,


/((1 - t)a + tb)) > (1 - t)f(a) + tf (b), para todos a, b E / distintos e todo t E (O,1).
As propriedades de funções côncavas (resp. estritamente côncavas) são facilmente obtidas a
partir das propriedades de funções convexas (resp. estritamente convexas), bastando trocar f por
- f, quando necessário. Por isso, no que segue, restringi-mo-nos ao estudo de funções convexas,
deixando ao leitor a tarefa de, sempre que necessário, elaborar os resultados correspondenlcs
para funções côncavas.
A definição 59 garante que toda função estritamente convexa é convexa. Entretanto, nem
toda função convexa é estritamente convexa, conforme atesta o exemplo a seguir.

ExBM_PLO3.11.
Toda função afimé convexa, mas não é estritamente convexa. De fato, se f : IR -+ IR é dada
por /(x) = Ax+B, com A, BE R, é imediato verificar que J((l -t)a + tb) = (1- t)J(a) + tf(b),
para todos a,b E R e t E (O,l].

No corolário 65 mostraremos que, se 1 é um intervalo aberto, então uma função duas vezes
derivável f: J-+ IRé convexa se, e só se, f"(x) > O, para todo x E J. Mostraremos também que
fé estritamente convexa se f"(x) > Opara todo x E J. Assumindo momentaneamente a validade
desses resultados, elencamos, a seguir, alguns exemplos de funções convexas e côncavas (observe
que, anteriormente, ao esboçar os gráficos de tais funções, estávamos assumindo implicitamente
que se tratavam de funções convexas ou côncavas - conforme o caso).

• - da função de proporcionalidade inversa ao conjunto dos reais positivos é uma


,etritamente convexa. De fato, denotando f(x) = ½ para x > O. temos f"(x) =

180
(b) Para n > 1 inteiro, a função f : (O,+oo) ➔ R dada para x > Opor /(z)
convexa, uma vez que J"(x) = n(n - l)xn- 2 > O.

(e) A função seno é estritamente côncava no intervalo (O,1r) uma vez que sen"z •
1

em (O,7r).

(d) A função tangente é estritamente convexa no intervalo (O,~), uma vez que tg'z
tgxsecx > O em (O,~).
-~-----~-~--~~--~-~-----
OBSERVAÇÃO 3.1.
Se f : / ➔ IR é convexa, então, fazendo t = ½ na definição 59, obtemos /(ª! 6) < t<•>;JC•>,
para todos a. b E /. Reciprocamente, se f é contínua e tal que /(ª~b) < f(a>;J(I,),para todos
a, b E /, é possível mostrar (cf. problema 6.8) que f ê convexa. Da mesma forma, também
é possível mostrar (cf. problema 6.9) que uma função contínua / : I ➔ R é estritamente
convexa se f ( a;b) < f(a>;f(b), para todos a, b E J distintos. Essas observações nos permitem
deduzir. de maneira elementar (i.e., com o simples uso de desigualdades elementares), o caráter
(estritamente) convexo ou côncavo de várias das funções com as quais usualmente nos deparamos
no Cálculo. A esse respeito, veja o problema 6.11.

Voltando ao desenvolvimento da teoria, seja/ :J ➔ IR uma função convexa. Então, dados


a < b em I e O < t < l, temos

J((l - t)a + tb) < (1 - t)J(a) + tj(b). (3.27)

Fazendo x = (1 - t)a + tb, temos t = ::: e 1 - t = t=:.


Portanto, a última desigualdade acima
pode cr escrita como
b-x x-a
J(x) < - · J(a) + - · J(b).
b-a b-a
A partir daí, temos que

f (x) - f(a) < _l_ (b - x . f (a)+ x - a . f(b) _ J(a)) = J(b) - f(a)


x-a x-a b-a b-a b-a
e analogamente f(b)-f(.r) > f(b)-/(a). Assim1 sendo J convexa em I temos
• ' b-x - b-a '

f(x) - J(a) J(b) - J(a) J(b) - J(x)


---- < ---- < ----, (3.2 )
x-a b-a b-x
para todos a < x < b em I.
Reciprocamente, se
J(x) - J(a) J(b) - f (x)
----<----, (3.29)
x-a - b-x

181
, , ',,; l,l\1111"1 1)11<1\\11\"

para todos a < .r < b cm /, então (b - x)(J(x) - f(a)) < (x - a)(J(b) - f (x)) ou, ainda,

(b - a)f (x) < (b - x)f(a) + (x - a)J(b).

ubstituindo x = (1 - t)a + tb nessa desigualdade, reobtemos imediatamente (3.27), de forma


que f é convexa.
Observe que os quocientes em (3.28) são os coeficientes angulares das secantes ao gráfico de
f e p~do, respectivamente, pelos pares de pontos (a, J(a)) e (x, f(x)), (a, f (a)) e (b, f (b)),
(x, f(x)) e (b, f(b)). Por outro lado, tais coeficientes angulares são as tangentes dos ângulos
(medidos trigonometricamente) que tais secantes formam com o semieixo positivo das abscissas.
Uma vez que a função tangente é crescente em cada um dos intervalos (O,~) e (i, 1r),sugerimos
80 leitor utilizar os comentários acima para comparar, na Figura 3.13, as inclinações dessas

secantes 80 gráfico de f.
A seguir, utilizamos a discussão anterior para mostrar que uma função convexa, definida em
um intervalo aberto, é contínua.

1D intervalo aberto e f : J ➔ R é convexa, então f _____________


é contínua. ,

PROVA.
Fixe x 0 E J e a< xo < b também em I. Para x E (x 0 , b), a primeira desigualdade em (3.28),
x-xo -< J(b)-J(xo)
com x O no lugar de a , garante que f(x)-J(xo) b-xo '• a segunda desigualdade em (3.28) ' com
x O no lugar de x ex no lugar de b' garante que J(x)-J(xo)
x-xo
> f(xo)-f(a).
- xo-a
Combinando essas duas
desigualdades, obtemos

f(xo) - /(a)) (x - xo) < J(x) - f(xo) < (f(b) - f(xo)) (x - xo).
( xo - a b- x 0

Fazendo x ➔ xo+, segue das desigualdades acima e do teorema do confronto (cf. proposição
8) que
lim J(x)
x-+xo+
= f (xo)-
Argumentando de modo análogo, concluímos que lirn:r-+xo-f(x) = f (x0). Portanto, o problema
1.5, página 136, garante que limr-+roJ(x) = J(To), e segue da proposição 4 que f ê contínua em
Zo·

Conforme comentamos anteriormente, mostraremos a seguir que o caráter (estritamente)


de uma função dua.Rvezes derivável cslá intimamente relacionada ao sinal de sua segunda
eo.DWXO

L82
derivada. Isto será uma consequência imediata do seguinte resultado, no qual se supõe que /
seja somente derivável.

TEOREMA 3.64.
Se I é um intervalo aberto e / : I ➔ IRê derivável, então:

(a) / é convexa em / se, e só se, /' é não decrescente em/.

(b) fé estritamente convexa em / se, e só se, f' é crescente em /.

PROVA.
Suponha, inicialmente, que/ é convexa em/. Dados a< bem I ex E (a, b), segue de {3.28)
que
f (x) - /(a)
___ J(b) - J(a) < ----.
_.;,..< ---- J(b) - f(x)
x-a - b-a b-x
Mas, como / é derivável em J, fazendo x ➔ a+ na primeira desigualdade e x ➔ b- na segunda
desigualdade, obtemos

Logo, f' é não decrescente em I.


Se f é estritamente convexa, então refinamos o argumento acima, tomando e E (a, b) e
x E (a, e), y E (e, b). esse caso, segue de (3.28) e da convexidade estrita de / que
J(x) - /(a) f(c) - f(a) J(b) - /(e) J(b) - J(y)
x-a
< e-a
< b
-e
<----.b-y
Agora, fazendo x ➔ a+ e y ➔ b-, obtemos
J'(a) < f (e) - f (a) < J(b) - f (c) < J'(b).
e-a b-c
Logo, f é crescente em I.
Reciprocamente, suponha que /' é não decrescente em I. Para mostrar que f é convexa,
vimos anteriormente que basta provar que
f(x) - f(a) J(b) - J(x)
----<----,
x-a - b-x
para todos a < x < bem /. Como I é aberto e f é derivável em /, temos f contínua em [a, b]
e derivável em (a, b). Portanto, pelo TVM de Lagrange, existem a E (a, x) e /3 E (x, b) tais
que /(xJ=~(a) = f'(o) e f(bt~(x) = f'(/3). Mas, como a < /3 e estamos supondo que /' é não
decrescente, temos f'(o:) < f'(/3) e, daí,
J (x) - J(a) = J'(o:) < J'(/3) = J(b) - J(.r)
x-a b r

183
1
1 1 1 1 1 1 : I, 1\ 111 1 ..., 1 I ) l·.Ii 1\ \ 1) \...,

conforme desejado.
Se /' é crescente em I, o argumento acima, nesse caso, garante que
J(x) - J(a) = !'(a) < J'(f3) = J(b) - J(x).
x-a b-x
Portanto, temos uma desigualdade estrita em (3.29), para todos a < x < bem /, e f é estrita-
mente convexa.

Se J é um intervalo aberto e f :/ ➔ R é duas vezes derivável, então:

(a) / é convexa em / se, e só se, /" > O em I.

(b) Se /" > O em /, então f é estritamente convexa em /.


-----------------
PROVA.
O item·(a) segue imediatamente do item (a) do teorema anterior, juntamente com o it,em (a)
da proposição 53 (aplicada a /'). Quanto a (b), se f" > O em I, então o item (b) da proposição
53 (novamente aplicada a f') garante que f' é cre cente em 1. Logo, o item (b) do teorema
anterior garante que fé estritamente convexa.

Vejamos, em um exemplo, como aplicar o corolário anterior .

.
: {-\1'2, v'2} ➔ R a função dada por J(x) = 2 ~:i 2 . Encontre os intervalos em que
tiltritámellte
convexa e estrita.mente côncava.

SOLUÇÃO.
Calculando a primeira e a segunda derivadas de f, obtemos J'(x) 2+2:2
(2-x2)2
e J"(x) -
(2 -x 2 ffc
6 +:c2). Portanto,
2
x > Oe 2 - x > O { O< x < J2
ou {::> ou
x < O e 2 - x2 < O x < -../2.
Portanto, / é estritamente convexa em cada um dos intervalos (-oo, -/2) e (O,/2). Analoga-
mente,/ é estritamente côncava em cada um dos intervalos (-../2,0) e (../2,+oo).

 184
O corolário anterior é conhecido como o estudo da segunda variação de uma função
\·ezes derivável. A seguir. obtemos um_aconsequência geométrica importante de tal resultado,
para a qual precisamos. inicialmente. de uma definição.

DEFI~JÇÃO 3.67.
Sejam J um inten-alo aberto e f : J ➔ 1Ruma função contínua. Um ponto Xo
denominado um ponto de inflexão de f se existir ô> O tal que/ é convexa em /n(z 0 -6,
e• côncava cm / n (x x 0 + ô), ou vice-versa.
---------~~---------
.
0

Podemos, agora. enunciar e prO\-aro corolário desejado.

COROLÁRIO 3.68.
Sejam / um intervalo aberto e / : J ➔ R wna função duas vezes derivável. Se zo E / é um
po11tode inflexão de/. então /"(x 0) = O.

f>RO\'A
Suponha que/ é com·e.....-:a 0 + ô), com ó > O escolhido tão
cm {x0 - '5,:c.0 ) • cõnca,,, cm (.r0 , :1·
pequeno qu' (.r0 - ô,.r 0 + 6) C / (o outro ~ pod' , r tratado de forma análoga). Então, pelo
c01oláiio 65, temos /"(x) > O cm (x-0- 6,.r 0) /"(x) < O cm (.r-0. x 0 + ó).
Suponha qm' /"(:r < O, l"> fixt">m oitr 1
0
) a E (:1·
0 - ó, .r
iru1ll'ntl' de sorte que J"(x 0 ) <
0
),

O < J"(a) Como J"(.r 0 ) < ~ < /1'(0) o H~I 'llH\ de Dnrbou.x 5 garante a existência de
lJ E=(a,.r 0
tnl que J"(b) = ~ < O. las, como b E (a,:io) =} b E (xo - ó,x
)

2
deveríamos ter
0
),

J"(b) '> O, o que é uma contradição.


\nnlogmucntc, n suposição dl' qul' J"(:i:o)> O t, mbém 1105 leva a uma contradição, de forma
qm' n umcn po::,sibilidad é t •nno J"{:ro) = O.

,\ ~cguir, demonstramos a de igualdade de Jensen 8 , resultado que, como veremos, confere


i\ tconn de funçõc~ co1wexas (e coucavas) grande importância na discussão de problemas de
mà..'\.1mos e minuno::,

TEOREMA 3.69. JENSEN


Sejam / e R wn inten-alo aberto e f : / ➔ IR uma função convexa.
f 1, ... 'tn E (O, 1), com t1 + ... + tn = 1. então t1X1+ ... + tnXn E / e

J(t1X1 + · · · + tnXn) < t1f (xi)+···+ tnf(xn).


Ademais, no caso em que fé estritamente convexa, a igualdade ocorre se, e só se, z 1 • • •

Apô Joha.n Jeruen, matemãtico e engenheiro d.inamarquê::;dos séculos XIX e XX

185
: , 1"; l.1\1111·"1 ))FHI\\I>.\...,

PRO\'\.
Suponhrunos que f é estritamente convexa (o casos em que f é meramente convexa é total-
mente ru1álogo)e façamos a prova da desigualdade de Jensen por indução sobre n > 1. O caso
n = 2 segue da definição de convexidade estrita de f, uma vez que a condição t 1 +t 2 = 1 equivale
a que t 1 = t e t 2 = 1 - t.
Suponha que, para um certo n > 1 e todos X1, ... , Xn E / e t 1, ... , tn E (O,1), com t1 + · · · +
t'fl = 1, tenhamos tix1 + · · · + tnXn E / e

com igualdade se, e só se, x 1 = • • • = Xn- Consideremos elementos x 1 , ... , Xn, Xn+t E / e
t1, •.. 'tn, tn+l E {O,1) tais que t1 + ... + tn + tn+l = 1. Defina

t1X1 + · · · + tnXn
Y = ------ = S1X1 + · · · + Sn X n,
1 - tn+l

de modo que s1 E (O,1) para 1 < j < n. Portanto, segue da hipótese de indução que y E / e,
daí,
t1X1 + · · · + tnXn + tn+lXn+l= (1 - tn+dY + tn+1Xn+lE /.
Agora, utilizando a convexidade estrita de f, obtemos

J(t1X1 + · · · + tn+1Xn+1) = f((l - tn+dY + tn+1Xn+1)


< (1 - tn+1)f(y) + tn+1f (Xn+1),
com igualdade se, e só se, y = Xn+1. Por outro lado, também por hipótese de indução, temos
que

c.om igualdade se, e só se, x1 = • • • = Xn.


Juntando as duas desigualdades acima, concluímos que

f (t1X1 + · · · + ln+lXn+d < (1 - ln+1)f (y) + tn+1f (Xn+1)


< U1f(xi) + · · · + tnf(xn)) + tn+1Í(Xn+1),

186
ocorrendo a igualdade se, e só se, y = Xn+1 e x 1 = • · • = Xn· Por fim, é imediato verificarque
tais condições equivalem a x 1 = • • • = Xn = Xn+1, conforme desejado.

O mais das vezes, utilizamos a desigualdade de Jensen na forma do corolário a seguir, que
enunciamos para conveniência do leitor.

COROLÁRIO 3. 70. JENSEN


Se I e 1Ré um intervalo aberto e / : / ➔ 1Ré uma função convexa, então, para x 1, ... , x,. E 1,
temos
(3.31)
Ademais, se f é estritamente convexa, então a igualdade ocorre se, e só se, x 1 = · · · = x,..

PROVA.
Faça t1 = t2 = • • • = tn = ¼no teorema anterior.

Deixamos ao leitor a tarefa de enunciar a desigualdade de Jensen (nas formas (3.30) e (3.31))
no caso em que f é côncava (resp. estritamente côncava). A seguir, apresentamos algumas
aplicações intere santes da me ma.

EXEl\fPLO 3. 71.
Dados n > 1 reais positivos a 1 , a2 , ... , ª", prove que

com igualdade se, e só se, a1 = a2 = • • • = On-


---------~-~~~-----
PROVA.
Vimos, no exemplo 62, que a função de proporcionalidade inversa f : (O,+oo) ➔ JR, dada
por f(x) = ¼,é estritamente convexa. Portanto, segue de (3.31) que

____ n ___
ª1 + ª2 + • • • + an
=f (ª1 + a2 +n·.. + ªn)
< /(ai)+ f(a2) + • • • + f(an)
n
l/a1 + l/a2 + • • • + 1/an
- n

187
1 ,, : Ll\1111'-I· l)J,'1{1\\1>\"

ocorrendo a igunldade se, e só se, a 1 = a 2 = · · · = ªn· Agora, basta notar que a desigualdade

n < l/a1 + l/a2 + · · · + l/an


a1 + a2 + · · · + an - n
é, precisamente, a que desejávamos provar.

Nosso próximo exemplo utiliza a desigualdade de Jensen para resolver um interessante pro-
blema de geometria .

..._.__,l' um semicírculo de raio R e diâmetro A0 A 1 . Para cada inteiro n > 2, mostre que existe
um 6nico n-égono AoA1A2 ... An-l satisfazendo as seguintes condições:

(a) A2, ... , Ân-1 E r.


(b) A Aiea.de AoA1A2 ... An-1 é a maior possível.

SOLUÇÃO.
Considere a Figura 3.14 como representativa da situação do problema. Sejam O o centro
der e .ÂiÔAi+I = ai, para 1 < i < n - l (com a convenção de que An = A0), de forma que
a: 1 + a: 2 + · · · + a:n-l = 7T'. Aplicando a fórmula do seno para a área de um triângulo (cf. seção
7.3 de [10)), obtemos (novamente convencionando An = A0)
..
A(AoA1 ... An-1) =~
L...,A(AiOAi+1) = ~l
L..., R 2 sen AiOAi+1
2
---

i=l i=l
n-1

= 21 R 2"'
L...,sen ª'i·
i=l

Agora, uma vez que a função seno é estritamente côncava no intervalo [O,7r),segue de (3.31)
que
n-1 ( 1 n-1 )
'°'sena, < (n - 1)sen "' a, = (n - l) sen _7T'_,
L..J n-1L..J n-l
í=l i=l

ocorrendo igualdade RC, e só se, o 1 = • • • = o,l-l


a = : .
11 1
Logo, há um único polígono de área
máxíma 1atiBfazendo 88 condições do enunciado.

188
Ao o
Figura 3.14: polígono de ãrea máxima in crito em um midrculo.

Problem - Seção 3.6


6.1. j m I, J e in ervalo g: I ➔ J e f: J ➔ funçõ tritam nte conv xas. fé
cr ent , prov qu / o g: I R também , tritamente conv xa.

6.2. jam I, J e int rvalo /: I J uma bijeção contínua. e/ é stritamente conv xa,
prov qu 1- 1 : J I ou tritaro nte conv a ou tritaro nt côncava em J.

6.3. Prov que a ma d um núm ro finito d funçõ tritam nt convexas (r p. trita-


m nt côncav: ) d m mo domínio ambém , uma função tritament conv xa (r p.
trit m nt côncav ).

6 .. ' 1) a funç- o dada por / (x) = J:-:r. Encontr o int rvalo em qu / é


on x ou cônca a.

6.5. P ra a, b, E ja f : ➔ função dada por f (x) = x3 + ax 2 + bx + e. ncontr o


ponto d inflexão d f, b m como~ intervalo no quai / é convexa ou côn ava.

6.6. ja / : (O,+ ) ➔ a função dada por f(x) = x2 n i, mo tr qu f po ui infinito


pon o de infl ao.

6. 7. J : (O + ) ➔ é uma função duas d rivável, não d r n onv xa prov


qu função g: (O,+ ) ➔ , dada por g(x) = xf(x), também conv xa.

6. . j f :I ➔ uma contínua tal qu f(:r~ 11) < J(:r);f(y), para tod x, y E I (r p ti-
m nt J(:r~y) > f(:r);f(y), para todo x, y E I). Prove qu / é conv xa (r p ti m nt
côncava).

6.9. ja f : I ➔ uma contínua e tal que J(ª~b) < f(a>;t(b) (r p ti m n f(ª+b)


f(a>;J(b)) para todo a, b E I dis intos. Prove qu f , conv x (r p tiv: m n n \! ).

19
1 , '" : l,J\1111 ..._1 1)J·.HI\ \1, \"

6.10. f : (O,+ ) --; IR é uma função contínua, crescente (resp. não decrescente) e convexa
(rc p. estritamente convexa), prove que a função g: (O,+oo)--; IR,dada por g(x) = xf(x),
também é estritamente convexa.

6.11. Use os resultados dos problemas 6.8 e 6.9 para estabelecer os caracteres (estritamente)
convexo ou côncavo de cada uma das funções do exemplo 62.

6.12. Sejam I um intervalo aberto e f : J --; IR uma função convexa e duas vezes derivável.
Mostre que, para todo x 0 E J, o gráfico de f está situado acima da reta tangente ao
mesmo em x 0 . Mais precisamente, mostre que, para todos x, x 0 E /, temos

J (x) > J(xo) + J'(xo)(x - xo).

6.13. Dados k > l inteiro e a 1 , a 2 , ... , an reais positivos, prove que

ocorrendo a igualdade se, e só se, a1 = a2 = · • • = an.

6.14. Sejam r um círculo de centro O e raio R, n > 2 um inteiro fixado e A 1A 2 ... An um


n-ágono convexo inscrito em r.

(a) Se O não pertence ao interior de A 1A 2 ... An, use um argumento geométrico para
mostrar que há n-ágonos convexos inscritos em r e com área maior do que aquela
de A1A2 ... An.
(b) Use a desigualdade de Jensen para mostrar que, dentre todos os polígonos convexos
de n lados inscritos em r, os regulares são os únicos de área máxima.

6.15. Sejam dados um círculo r, de centro O e raio r, e um inteiro n > 2. Dentre todos
os n-ágonos convexos A 1A 2 ... An circunscritos a r, prove que os regulares são os de
perímetro mínimo.

6.16. Seja. ABC um triângulo acutângulo de lados AB = e, AC= b e BC= a. Se Ré o raio


do círculo circunscrito a ABC, prove que

com igualdade se, e só Rc, ABC é equilátero.

190
6.17. Sejam n > l inteiro e x 1 : x 2 : .... Xn reais positivos com soma igual a 1. Prove que

t x, • >
Jl - x- -
✓ n > /Xi+ •••+ Jx.
n- l - Jn - l '
~· l

com igualdade em uma qualquer das desigualdades acima se. e só se, X1 = · · · = Xn = t .

191
1 l11: l,J\l(fl'-,J [)fl(J\\J)\'-,

3. 7 Construindo gráficos
cjam dados um intervalo I e uma função contínua f : / ➔ IR, duas vezes derivável no
int crior de /. A teoria desenvolvida até aqui nos fornece, a princípio, uma quantidade suficiente
de informações para construir um esboço razoavelmente acurado do gráfico de f. Senão, vejamos:

(i) A utilização dos resultados do problema 5.10, página 59, pode reduzir o problema do esboço
do gráfico de / a um intervalo J e I.

(ii) O teste da primeira derivada garante que os pontos extremos de f são ou extremidades de
/ ou as soluções da equação J'(x) = O.

(iii) O estudo da primeira variação de / garante que os intervalos em que f é crescente (resp.
decrescente) são os intervalos-solução da inequação f'(x) > O (resp. f'(x) < O).

(iv) Os pontos de inflexão de / encontram-se dentre as soluções da equação J"(x) = O.


(v) O estudo da segunda variação de f garante que os intervalos em que f é estritamente
convexa (resp. estritamente côncava) são os intervalos-solução da inequação J"(x) > O
(resp. f"(x) < O).

(vi) As retas tangentes ao gráfico de f em seus pontos de inflexão ajudam a esboçar melhor o
gráfico numa vizinhança de cada um desses pontos.

(vii) A definição 17 e o problema 1.11, página 137, ensinam como procurar, se existirem, as
assíntotas ao gráfico de f.

(viíí) O cálculo dos limites limx-+±oof(x), se fizer sentido, ajuda a entender o comportamento
do gráfico de J para valores grandes de lxl.

(i.x) A marcação de alguns pontos sobre o gráfico (para além dos eventuais pontos extremos e
pontos de inflexão) pode auxiliar em muito a construção do esboço do gráfico de f.

à guisa de ilustração, a seguir uUlizamos o programa delineado acima para esboçar os gráficos
de algurnaBfunções.
----=-----
' :: .
'\~

• •1
1
'
,··
' ' ; k E Z}, esboce o gráfico da função secante, sec : D ➔ IR.

 192
(i) Inicialment que· u.fici n boçar o gráfi o d m (-i i)U(i, 3;), poi , m
co • p riódica de período 21r t mo tamb'm p riódi , d p ríodo 21r. Por outro lado, como
co (x + 1r)= - co para odo E , t mo (x + 1r) = - x, para todo x E D; portanto,
o probl m 5.10 pâgina 59, garant que o gráfico d no interv lo (i 3;) obtido a partir
do grãfi o d no int rvalo ( - i ; ) por m io de uma r flexão em torno do ixo d ab ci as
guida d uma translação de 1r unidad parai lament a tal eixo.

Doravan e, restringimo análi do gráfico d ao intervalo ( -i ~).


(ii) (iii) orno

' = (co -l x)' = -(co xt 2(- nx) = nx = tgx x, (3.32)


co 2 x
t mo qu n = O <=> x = O. (D fato tinha d r im uma vez qu
0< <l > 1 o orr ndo a igu ld cox=l1. ,e Ó , X= 0.
Port =O x r mo lo ai - logo pon o crí i o .) Por outro lado, é cr nt
<=} / >0<=> n X > o<=> E (o ;) • lo o éd r nt em (-i O).

(iv) ( ) Prim ir mplo 33 (3.32) forn m


li
= tg' X+ t X IX= X

Logo, li =/.o, n- h ponto de infl xão. Por ou ro lado, como c tritaro nt coo x
11x >O o qu por ua vez ocorr 6 co x > O, con luímo qu
6
tri m n on xa m ( -i ~).
( ii) • p riódi a, u grâfi o não contêm íntot horizont i ou oblíqu . Por outro
1 do im di to qu lim.r-+i- x = limx-+-i + x =+ , d modo qu r t X= ±21f -

ín ot rti ai do gráfico d

( iii) orno p riódica não on tant , não xi t limx ± f (x).

(ix) 1 ui ndo co x para x = O,±;';, ±i, ±~, ±J, ± 1;;, marcamo


ponto (0,1) (±i,73) (±i,vli) (±J 2).

193
1 ,,, ; l.l\111('-,f l)ff!l\\11\"

Rc~ta r:-unir ns informações acima e esboçar o gráfico de sec. Fazemos isso na Figura 3.15,
destacando a porção do gráfico em (-i, ~) U (;, 3;) (i.e., a porção que se repete).

y
1 1
1 1
1 1
1 1
1 1
1 1
1 1
1 1
1 1
1 1 1 1
____ J _____
y = l --t---- 1
l _____
1
L_
1
1 1 1
1 1 1
_Jr 1
2,
o .?r 1
2 1
'K
1
3,r 1
2 1
X

y = -1 --+----- ----7----
1 1
- - - -,-
1
-
1 1 1
1 1 1
1 1 1
1 1 1
·'
1
1
1
1
1
1 1 1
1 1 1
1 1 1
1 1 1

Figura 3 .15: grâfico da função secante em ( -; , ; ) U (;, 3;).

fuçio polinomial / : R ➔ R, dada por / (x) = 2x 3 + 4x 2 + 2x - l.

SOLUÇÃO.

(ii) e (ííí) Como f'(x) = 6x 2 + Bx + 2, os pontos críticos de f são x = -1 ex = ~- Daí,


f'(x) <O# x E (-1, -½), e segue que J é decrescente{=} f'(x) <O{=> x E (-1, -½)- Logo, fé
crescente # x E (-oo, -1) U ( -½, +oo). Segue que x = -1 ex = -½ de f são, respectivamente,
pontos de máximo e mínimo local.

(iv) e (v) Como /"(x) = 12:r + 8, temos que Jé est,rit,amente convexa em um intervalo J se,
e IÓ 1e, /"(x) > O em /. Logo, f é cst,rit,amcnt,e convexa em (-j, +oo) e, analogamente, f é

194
estritamente côncava em (-oo, -}). Por fim, o ponto x = -} é o único ponto de inflexão do
gráfico de J.

(vi) Veja que !(-}) = -~~, e que a reta tangente ao grâfico de f em tem coeficiente
x = -}
angular f'(-i) = -l Sendo (O,e) o ponto em que tal reta tangente intersecta o eixo vertical,
2 d e
temos c-(-31/27) _
0 _, _ 213 ) - -
_ 43
3 , e 1orma que e - - 27 .

(vii) Como f está definida em toda a reta, seu grâfico não possui assíntotas verticais. Por outro
lado, como limx-+±oo/~) = limx-+±oo(2x 2
+ 4x + 2 - ¾) = +oo, segue do problema 1.11, pâgina
137, que o gráfico de f também não possui assíntotas oblíquas.

(viii) Segue do exemplo 16 que limx-+±oof (x) = ±oo.

(ix) Como /(O) = -1, segue de (viii) e do TVI que / tem uma raiz positiva a. Calculando
f(x) para x = -~, -1, -½, O, ¼,½, ½ e 1, marcamos sobre o gráfico de/ os pontos(-!, -i),
-J
{-1, -1), (-½, -~~), (O,-1), (¼, 2), (½, 2\), (½,¾)e (1, 7). (Recorde que, no item (vi), já
obtivéramos o ponto (-i, -~~) sobre o gráfico.) Em particular, concluímos que a E (¼,½).

Por fim, como no exemplo anterior, reunimos as informações obtidas acima para esboçar o
grâfico de J. Fazemos isso na Figura 3.16.

EXEMPLO 3.75.
Esboce o gráfico da função/: R \ {-1} ➔ R, tal que f(x) = x2 + x!i.

SOLUÇÃO.

(ii) e (iii) Como J'(x) = 2x - (x~ir2 = 2:r


3
,!~ 3 2
1);.-.tl, temos J'(x) =O<=} 2x + 4x + 2x - 1 = O.
Portanto, o único ponto crítico de / é a única raiz positiva a da função do exemplo anterior,
de modo que a E (¼,½). Daí, J'(x) > O <=}2x3 + 4x 2 + 2x - 1 > O <=}x > a, e segue que f é
crescente se, e só se, x E (a, +oo). Veja que x = a é ponto de mínimo local para/.

(iv) e (v) Como J"(x) = 2 + (x.;l)3, temos que/ é estritamente convexa em um intervalo I e, e
só se, f"(x) > O em I ou, ainda, se, e só se, x~I > -1 em/. Portanto,/ é estritamente convexa
em cada um dos intervalos (-oo, -2) e (-1, +oo), e estritamente côncava em (-2, -1). O único

195
( ,1·1111,1: LI\1111•:s I·. DFHl\\ll\,

ª
4
1
1
1
1
1
1 .!
X

43
-=,--27
-4

Figura 3.16: gráfico da função polinomial f(x) = 2x 3 + 4x 2 + 2x - 1.

ponto de inflexão de J é x = - 2.

(vi) Veja que J(-2) = 3, e que a reta tangente ao gráfico de f em x = -2 tem coeficiente
angular f'(-2) = -5. Sendo (O,e) o ponto em que tal reta tangente intersecta o eixo vertical,
temos 0 ~,!2) = -5, de forma que e= -7.

(vii) Como f só não está definida para x = -l, a reta de abscissa -1 é a única candidata
a assíntota vertical de seu gráfico. Esse é de fato o caso, uma vez que limx-+-l- f (x) = -oo
e limz-+-1+f (x) = +oo. Por outro lado, como lim:r-+±oo/~z) = limx-+±oo( x + :c(z~l)) = ±oo,
segue do problema 1.11, página 137, que o gráfico de f não possui assíntotas oblíquas.

(viii) Segue do exemplo 16 que limx-+±ooJ(x) = +oo.

(ix) Calculando J(x) para x = -~, -!, -?, -¾, -½, -¾, O, ¼,½,1 e 2, marcamos sobre o grá-
fico de J os pontos(-~,~~),(-!,-~),(-¾,-~~),(-¾, i!),
(-½, ?), (-¼,~~),(O, -1), (¼, 2), -J
3
(½, !), (1, J) e (2, \ ). (Recorde que, no item (vi), já obtivéramos o ponto (-2, 3) sobre o gráfico.)

Mais uma vez, reunimos as informações obtidas acima para esboçar o gráfico de f, na Figura
3.17. Para uma melhor visualização qualitativa do mesmo, utilizamos uma escala 2: 1 nos eixos

196
horizontal e vertical, respectivamente.

y
1
1
1
1
1
1
1
1
1
1
- - .i1 -
1
1
1
1

-2 \ ,1 Q X
1
1
1
1
1
1
1
1
1
1
1

Figura 3.17: gráfico da função J(x) = x 2 + xii.

Problemas - Seção 3. 7
7.1. Esboce o gráfico da função cossecante, csc: IR\ {k7r; k E Z} ➔ IR, tal que cscx = IM!~:r·
7.2. Em cada um dos itens a seguir, esboce o grãfico da função dada (definida em seu domí-
nio maximal), calculando ou estimando, quando for o caso, pontos críticos, intervalos de
monotonicidade, pontos de inflexão, retas tangentes nos pontos de inflexão, intervalos de
convexidade ou concavidade, assíntotas e comportamento no infinito:

(a) f (x) = x 4 + 2x2 - 2x + 1. (e) J(x) = ~ + :,;2~ 1 •


(b) f (x) = x + ~- (f) J(x) = :,;2~ 1 •
(c) f(x) = :r2~ 1 • (g) f (X) = :r2-;:r+2 •
(d) f (x) = :,;S~1 • (h) f(x) = (x:i+1~::rL1) •

197
1 ,,, ; l,l\lllf>,Ll)FHl\\l>\"

7.3 . .. .,<'.IHf : (a, b) ➔ IR uma função duas vezes derivável em x 0 E (a, b), com J"(xo) :/- O.
Como fé, cm particular, derivável em x 0 , a reta r, tangente ao gráfico de f em A(xo, J(xo)),
núo é vertical. Definimos o círculo osculador ao gráfico de f no ponto A como o círculo
r 1 t migcntc a r em A e que melhor aproxima o formato do gráfico de f numa vizinhança
de .t 0 , no seguinte sentido: tomando e E (a, x 0 ) e d E (x0 , b) tais que um dos arcos de r
situado entre as retas .t = e e x = d é o gráfico de uma função g : (e, d) ➔ IR, temos
g(.to) = J(.r 0)', g'(x 0) = f'(x 0) e g"(x 0) = f"(x 0). A esse respeito, faça os seguintes itens:

(a) Seja O(a, /3) o centro de r. Se f'(x 0) = O, mostre que a = x 0; se f'(x 0) :/- O, mostre
que fJ-f(xo) = __ 1_
o-xo f'(xo) •

(b) 1fostre que a equação de r é dada por (x - a) 2 + (y - /3)2 = R 2 , onde R =


J(xo - a) 2 + (J(x 0 ) - /3)2 é o raio de r.
(c) Para x E (e, d), substitua y = g(x) em (b) e derive a relação assim obtida para chegar
a g'(x O) = /3-g(xo)
xo-o = J'(x O) e g"(x O) = 1+J'(xo)2.
/3- f(xo)
2 2
(d) .eonclua que /3= f( x0) + l+f'(xo)
f"(xo) e a= x0 - J'(x0) (l+f'(xo)
f"(xo) ) • A partir
• d a1,
, mostre
que O está situado no semiplano acima (resp. abaixo) der se fé estritamente convexa
(resp. estritamente côncava) numa vizinhança de x 0 .
(e) Mostre que R = (l+f'(xo)2)3/2
ir(xo)Í .
(Em particular, observe que o círculo osculador ao grá-
fico de f em x 0 é, realmente, unicamente determinado pelas condições do enunciado.)
(f) Mostre que o vetor geométrico ti = (1, f'(x 0 )), de origem A, é paralelo à reta r.
Mostre também que, aplicando ao vetor ti uma rotação anti-horária de centro A e
--+
ângulo 90º, obtemos um vetor da forma >.AO,com >.E lR*.

O raio R der é o raio de curvatura do gráfico de f em x 0 . A curvatura do gráfico de


f em x 0 é o número real
f7 f7 J"(xo)
k - - - -----'---
- R - (l + f'(xo)2) 312 '
onde <1= J1IE {±1} é o sinal de>.. (Observe que lkl será tanto mais próximo de O quanto
maíor for R, o que condiz com nossa intuição geométrica.)

(g) Se R > Oe f: (-R, R) ➔ lR é a função dada por J(x) = J R2 - x 2 (de forma que o
gráfico de / 6 um semicírculo de raio R), mostre que o raio de curvatura do gráfico
de J l>constante e igual a R.

Como ílWJtração, a Figura 3.18 esboça o círculo osculRclorao gráfico de f(x) = ~. :r > O,
no ponto A.

198
y

Figura 3.18: círculo osculador ao gráfico de x ~ ½em A.

7.4. O objetivo deste problema é esboçar o gráfico da função f : 1R➔ 1Rdada por

f(x) = { O, se x = O
x sen ½, se x O
=/=,

Para tanto, faça os seguintes itens:

(a) Mostre que basta considerar o caso x > O.


(b) 11ostre que o conjunto dos pontos críticos de / em (O,+oo) é o conjunto dos x tais
que x =!,onde
y
y > O satisfaz a equação seny-ycosy = O.
(c) Mostre que o conjunto das soluções y > O da equação do item (b) é uma sequência
(Yn)n~1,tal que n1r < Yn < n1r + ~ e a sequência (n1r + ~ - Yn)n~I decresce para O
quando n ➔ +oo.
(d) Se Xn = 111n para n > 1, mostre que X2k-1 é ponto de mínimo local e x 2k é ponto de
máximo local para /, para todo k > 1.
(e) l/(x1)I > l/(x2)I > lf (x3)I > •• • ➔ o.
(f) fé crescente em (x1,+oo), com limx-++oof(x) = 1.
(g) 11ostre que f é estritamente convexa em ( 2!1r, (2k~l}1r) e estritamente côncava em
( (2k! l}1r , 2 t1r), para cada inteiro k > 1.

(h) Esboce o grãfico de f.


7.5. Acesse a página www.wolframalpha.com e utilize o WolframAlpha para esboçar os gráficos
dos exemplos discutidos nesta seção, bem como aqueles dos problemas 2 e 4. Por exemplo,
para esboçar os gráficos de x H ½+ :r2~ 1 e x H x sen ½,digite respectivamente

199
1
1
111
', Li\1I1I--1 1)1I:I\\I)\'-

plot x/2 + 1/(x-2+1);

plot x sin(l/x).

• ~-\ ~ 200
• r "'\

·-..:
3.8 Algumas aplicações à Física
Esta seção utiliza as ferramentas de· que dispomos até agora para discutir algumas aplic~
simples do Cálculo à Física.
Em tudo o que segue, adotamos o sistema de unidades MKS (também conhecido como
sistema internacional- abreviado SI), no qual as unidades padrão de distância, massa e tempo
são, respectivamente, o metro (m), o quilograma (kg) e o segundo (s). Comecemos recordando
a segunda lei de !ewton e as equações dos movimentos retilíneos uniforme e uniformemente
variado.

EXEMPLO 3. 76.
Considere um obJeto pontual (i.e.. um objeto cujas dimensões e formato sejam irrelevantes
cm relação ao fenomeno físico sob consideração) de massa m, movendo-se ao longo de uma
1cta r. Adote um sistema de coordenadas em r, de origem O e coordenada x, de forma que r
scJa identificada com uma reta real. Denote por x(t) a posição objeto pontual no instante t, e
suponha que a função t ~ .c(t) é duas \'ezcs derivável.
Fixado um instante t 0 > O, a velocidade v(t 0 ) do objeto no instante t0 é definida como a
tara temporal de vari.ação de sua posição. calculada no instante t 0 , i.e.,
. x(t) - :r(to) ,
1•(t0) = t-+to
luu ----
f - f0
= x (to)-
Da mesma forma, a aceleração a(t 0 ) do objeto no instante t0 é definida como a taxa temporal
de variação de sua velocidade. calculada no instante t 0 , i.e.,
v(t) - u(to) = v '( t ) = x "( t ) .
a(f 0) = lim ---- 0 0
t-+to f - to

Portanto, a~ funções velocidade e aceleração do objeto pontual são dadas por v(t) = x'(t) e
a(t) = v'(t) = :r"(t), para todo t > O. 'o que concerne unidades, a discussão acima deixa claro
que vclocidadel:>são medidas em metros por segundo (m/s) e acelerações em metros por segundo
ao quadrado (m/s 2 ).
Suponha. agora. que o objeto se move sob a ação de uma força F, de direção paralela ar,
cujos sentido e magnitude só dependem do valor de x. Então, temos F = F(x), para alguma
função xi--+ F(x), a qual supomos contínua. A segunda lei de Newton garante que F imprime
ao objeto uma aceleração a(t), de mesma direção e sentido que F em cada instante, tal que
F(x(t)) = m • a(t).
Então, uma vez que a(t) = x"(t). obtemos a equação diferencial

x"(t) = .!_F(x(t)).
m

201
1 1 1 1,, : Ll\111 F~ t·: Dt-:HI\ \I> \-.;

A unidade de força no sistema MKS é o Newton (abreviado N), de forma que, pela segunda lei
de Newton, lN = 1 ~t.
No caso particular em que F é constante em sentido e magnitude, a segunda lei de Newton
(3.33) garante que a aceleração a(t) do objeto também é constante. Denotando simplesmente
J;)Ora tal constante, concluímos que x" é constante e igual a a. Se o objeto parte da posição
inicial xo no instante t = O, com velocidade inicial v0 , segue do problema 4.2, página 166, que
• 1 1
x(t) = x(O) + x'(O)t + 2x"(0)t 2 = Xo + vot + 2at 2 . (3.35)

Logo,
v(t) = x'(t) = vo + at. (3.36)
Di7.emosque as fórmulas acima descrevem as equações do movimento retilíneo e uni-
forme, se a= O, e uniformemente variado, se a f. O.
--------'---------
0

Ainda em relação ao exemplo anterior, grosso m odo, denominamos equação diferencial


ordinâria·(que abreviamos por EDO) a uma equação envolvendo uma função-incógnita de uma
variável, que se supõe derivâvel um certo número de vezes, e uma ou mais de suas derivadas.
Assim, uma vez que (3.34) envolve uma função-íncógnita x = x(t), de uma única variável t,
temos que (3.34) é um exemplo de EDO.
Utilizamos o adjetivo ordinária (i.e., comum) para contrapor as EDOs às EDPs. Aqui (e
também grosso modo), uma EDP, ou equação diferencial parcial é uma equação envolvendo
uma função-incógnita de mais de uma variável, que se supõe derivável um certo número de vezes
em relação a cada urna dessas variáveis, juntamente com uma ou mais de tais derivadas. Para
um exemplo, seja f = f (x, y) urna função de duas variáveis reais x e y, e escreva ~(x, y) e
~(x, y) para denotar, respectivamente, as derivadas das funções x i---+ f(x, y) e y i---+ f (x, y).
Então,

é uma EDP.
Nestas notas, abordaremos vários aspectos elementares da teoria de EDOs, mas não teremos
espaço para discutir EDPs. Ao leitor interessado, sugerimos a referência (19].

Voltando à descrição de movimentos, o próximo exemplo utiliza o teste da primeira derivada


e o estudo da primeira variação de funções para analisar algumas questões relativas a lança-
mentos obUquos.

202
EXEMPLO 3.77.
Em um terreno plano. temos um canhão apontado com ângulo de elevação a em
horizontal. o qual atira um projétil que sai do mesmo com velocidade escalar v0 . Despr
força de resisténcia do ar durante o movimento subsequente do projétil, concluímos que a única
força que atua sobre ele é sua força peso F = mg. orientada verticalmente para baixo, onde m
é a massa do projétil e g"' 9. 8 m s2 é o ,-alor da aceleração da gravidade nas proximidades da
superfície da Terra.

y
1F= mg

....
........
--------- --
Vo,.......... ----....
....
v 0 coso
l'oS<ll O X

Figura 3.19: lançamento obliquo.


Para analisar o movinH'nto do projétil, utiliznmo:>o princípio da independência de Ga-
lileu, qut> nfinnn, 11 •ss' caso, que o movimento oblíqtto do projétil pode ser visto como a super-
posição ele dois movinwnt os ~t>pnrndo: um horizont ai, com velocidade escalar inicial v0 coso e
ncclcrnçiio nuln (pois n força J>l' o não tem componente horizontal), e outro vertical, com velo-
cidnclPc~cnlnr iniciAl u0 ~en o t• aceleração -g (cm relação ao sistema de eixos cartesianos fixado
na Figura 3.19).

Figura 3.20: Galileu Galilei, um dos maiores cientistas de todos os tem-


pos. Além de ter sido o inventor do telescópio, Galileu foi o primeiro
a compreender. dentre outras coisas, que, desprezando eventuais forças
de resistência do ar. corpos com massas iguais caem de alturas iguais
em intervalos de tempo iguais. Um relato muito interessante da vida de
,___~~~~_G_al_il_e_u _P.odeser lido em (15]_.------~---~-~~-~~-----'

203
1

1.1 1111" 1 1lii,I\ \1 1 ,-

, l3-.35) garante que o movimento horizontal é retilíneo e uniforme, com equaçã


do instante inicial t 0 = O e do ponto de lançamento x 0 = O dada por

x (t) = (v0 cos a) t. (3.37)

assegura que a equação horária do movimento vertical a partir do


to~ O e do ponto de lançamento y 0 = O é

(3.38)

i(t) = O se, e só se, t = O ou t = 2~sen


9
a, concluímos que o projétil toca novamente
o instante 2~sena. Tal instante define o tempo de voo do projétil.
outro lado, o teste da primeira derivada garante que o projétil atinge sua altura má..-xima
te tM, tal que y'(tM) = O. Resolvendo a equação y'(t) = O, obtemos tM = ~sena, ele
forma que tM é igual à metade do tempo de voo. Assim, a altura máxima atingida pelo projétil
é
2
(v 0 sen a)
y (tM ) = ----
2
g ( vosena )
- - --- = -sen 2 a.
v5
g 2 g 2g
O alcance horizontal A do projétil é o valor de :r(t) quando t é igual ao tempo de voo, de
forma que
• vo Vo
A= (v0 cosa) · 2-sena
g
= -sen2a,
g
onde, na última igualdade, utilizamos a fórmula para o seno do arco duplo (cf. corolãrio 72).
Por fim, a trajetória descrita pelo projétil é uma parábola, fato que pode ser deduzido
escrevendo tem função de x em (3.37) e, em seguida, substituindo o resultado em (3.38):

- ~ (
2
y(x) = (v0 sena:) x x )
vo cosa 2 v0 cos a
(3.39)
9 2
= (tga:)x - 2 (v cosa )2 x •
0
iu-.-------~- ---------~-----~

Antes de continuar com a Física, precisamos discutir alguns fatos elementares sobre EDOs,
o que fazemos a partir de agora.
Para o que segue, observe que, se f(x) = senx e g(x) = cosx, então J"(x) + f(x) =
i'(x) + g'(x) = O, para todo x E IR. Res11mimos isso dizendo que senx e cosx são soluções da
EDO y'' + y = O. Aqui, y = y(x) é a função-incógnita y : IR ➔ IR, a qual supomos duas vezes
clerivé.vel. Mostraremos, no teorema 79, que essas são, essencialmente, as únicas soluções dessa
• diferencial. Antes, contudo, precisamos de um resultado auxiliar.

204
LEMA 3.78.
Se /, g : IR ➔ IR são funções deriváveis e tais que f'(x) = g(x) e g'(x) • -'
x E IR, então f(x) = /(O) cosx + g(O)senxe g(x) = g(O)cosx - /(O)senz, para

PROVA.
Sendo h(x) = f(x)senx + g(x)cosx e l(x) = J(x)cosx - g(x)senx, temos h(O) = g(O),
l(O) = /(O) e

h' (X) = /' (X)sen X + f (X)cos X + g' (X)cos X - g (X)sen X


= (J'(x) - g(x)) senx + (J(x) + g'(x)) cosx = O.

Analogamente, l'(x) = O para todo x E IR, de forma que, pelo problema 4.1, página 166, h e
l são constantes. Então, temos

f(x) sen x + g(x) cosx = g(O)


{ f (X) cos X - g (X) sen X = f (o)

As igualdades acima formam um sistema linear de equações em f (x) e g(x). Podemos resolvê-
lo sem dificuldades, obtendo f(x) = /(O) cosx + g(O)senx e g(x) = g(O)cosx - /{O) senx.

TEOREMA 3. 79.
Se/: IR ➔ IR é uma função duas vezes derivável e tal que f"(x) + f(x) = Opara todo z e R,
então
f (x) = J(O)cos x + /'(O)sen x,

para todo x E IR.


~~------------------------- - -- ......

PROVA.
Sejam g, h : IR ➔ IR as funções dadas por g(x) J(x) - J(O)cosx e h(x) = g'(x) -
J'(x) + /{O) sen x, para todo x E IR. Como

h'(x) = J"(x) + /(O) cosx = - f(x) + J(O)cosx = -g(x),

segue do lema anterior que g(x) = g(O)cosx + h(O)sen x = f'(O) en x. Portanto, f(x) -
/(0)cosx = f'(0)senx, conforme desejado.

205
' 1 1 '1 1 1 1 11 1: 1 '1

um tr m n ma1 g r 1 d

➔ um funç- du v z ' d ri áv l tal qu /"( ) >..J(x) = O para

/(x) = /(O) co ( ✓-\x) + !]) n ( ✓-\r) (3.40)

PR \
J g : ~ a fun - d d por g( ) = /(-J-;_). P 1 r gr d ad i , g , du V Z

d ri , 1 al qu g'( ) = ~ •f'(Jx)

g"( ) = ½.!"( / ✓-\) = -!( /✓-\) = -g( )

para todo ;e E . Por anto p lo or ma n rior mo

g(x) = g(O) o + g'(O) n

amb'm para odo x E . B a agor ob ; ar qu g(O) = J(O) g'(O) = 1Jl_) f(x) = g(v'>:x).

O corolário ant rior no p rmi apr n ar no x mpl gmr um b l plic -o do


Cálculo a d nç d um i t ma mas a-mola unidim n i nal pli u r mona
época do próprio wton.

•lea'ifflM>&uma mola com coo tant lâ ti a k > O (te 1 n tant r pr nt . m última


ida da r ist n ·ia da m la u • - ), pr a
dad a uma pnr d •. o ,
11treii111·
•™·:a, hoj I id m la
DrllmeilltOx qu não induza d forma br
t ur dor (i. ., qu
o , p is, F - -kI p, r
r r . t ur l r.
.\U,1 \I \"' \l'I I< \<<li"' \ Fl"'I< \

Figura 3.21: o sistema massa-mola unidimensional.

Agora, prendemos um bloco de massa m à outra extremidade da mola (que supomos ter
massa desprezível em relação a m). de forma tal que o sistema massa-mola resultante repouse
horizontalmente numa superfície plana. a qual suporemos tão polida que nos permita negli-
genciar, na análise de eventuais movimentos, quaisquer efeitos resultantes de atrito estático ou
cinético. Em seguida. elongamos lentamente a mola de um comprimento x 0 (puxando ou empur-
rando o bloco de massa m, conforme o caso) para, por fim, imprimir ao bloco uma velocidade
instantânea v0 e deixar o sistema massa-mola o~cilar li\Temente.
Suponha que tal movimento oscilatório se dê ao longo de uma reta, e denote por t o tempo
transcorrido a partir do instante inicial t0 = O. Também, seja x(t) a elongação da mola no
instante /, e suponha que a função ti-+ .r.(t) é duas vezes derivável.
o exemplo 76, \'imos que a velocidade v(t) e a aceleração a(t) do bloco no instante t são
dadas re~pcctivamente por v(t) = ::r'(t) e a(t) = x"(t). Portanto, aplicando a lei de Hooke a
(3.34), obtemos m:r"(t) = -k.t(f) ou. ainda.

k
x"(t) + -x(t) = O,
m
com .r(O) = .ro e .r'(O) = v(O) = vo.
Fazendo w /F-i
concluímos, com o auxílio do corolário 80, que

Vo
x(t) = x 0 costwt + - sen (wt). (3.41)
vJ

A expressão acima, juntamente com o resultado do problema 6.4, página 65, garante que a
função t i-+ x( t) é periódica, de período 2;. Portanto, o movimento do sistema massa-mola é
periódico, de período (i.e., o menor intervalo de tempo necessário para urna oscilação com-
pleta) T = 2;. Por sua vez, a frequência do sistema massa-mola (i.e., o número de oscilações
completas por unidade de tempo) é w = 2-;.No sistema ~1KS, frequências são medidas em
Hertz9(abreviado Hz). onde 1 Hz = 1 ciclo/s.
------------~-~----------
207
55, juntam nt m o it m (e) d pr bl ma .3 p gina 64, garante qu
il õ , do i t ma m ·a-m la m qu -o (i.. , a longa ã máxima d

2
o

w

d pr m quiv rn ia n r o
pnn ipto d Fi nn at l i d r fl x~ d m' ri .

um meio mat rial , dito homog neo u propri d d na variam


para outro. Um do po tulad da Óti a G om' rica • qu um raio d luz ao
em um meio homogêneo t m v lo id d alar on u ro po ul do • o
• de Fermat 10 que afirma qu um raio d luz ao propagar- m um m io homog·n o,
a outro, escolh a trajetória d m nor t mpo d p r ur o po ív l. não
UUIR,8,C1.
os ao longo do percurso do raio d luz ntr doi pon , o ív 1 mo tra
não o faremos aqui, poi a d monstração fog opo d not ) qu u movim nto
dois pontos em questão é retilíneo uniform .
~-·dere,

agora, a situação da Figura 3.22 m qu A r pr na um font puntiforme de
Bum anteparo puntiforme, e a reta rum p lho p ditam nt polido odo itu d
meio homogêneo. A partir de A a m raio d luz m tod
Dll!!811ll0 dir - ;
raio de luz diretamente de A para B m movim nto r tilín o uniform
incide em B após ser refletido no sp lho r em um ponto P (ind d p d p
to retilíneo e uniforme).

A' p B'
Figura 3.22'. o printfr ia d •rn1't.

luz, d fi11irnoHi-; •u , ngul d in id n m P m an ulo


xão 111 ngul LB B'.

1, Jln t1., IM, u 11J1•1111w do i; •(•111 >


, de J•1•1lllltf, Jlllll 1•111 1 icO frllllC' 1110 II

o
1

.\1.LI \I , .... \l'I ll \(, l li-... \ F1-...11 \

Aqui, nosso objetivo é deduzir, a partir do princípio de Fermat, n lei da reftexio da.
Geométrica, que afirma que os ângulos de mcidê11c1.oe refle:z:aoem P I/Jm medidas íguaía.
subproduto de nossa análise, mostraremos também que o percurso de A a B, tocando a reta f'
no ponto P tal que AP A' = BP B', é, efetivamente, aquele de tempo rnfnimo (dentre todoe OI
percursos que partem de A, tocam a reta r e chegam em B).
Para tanto (cf. Figura 3.22), sejam a e b, respectivamente, as distáncias dos pontos A e B à
reta r, APA' = o e BPB' = /3,com 0:,/3E (0,1r) (observe que, se tivéssemos desenhado o ponto
P à esquerda do ponto A', teríamos o > %; analogamente, se tivéssemos desenhado o ponto P
à direita do ponto B', teríamos /3> i)-
Uma vez que a velocidade da luz em um meio homogêneo é constante e o movimento entre
d01s pontos (sem interrupções) é retilíneo e uniforme, o problema de minimizar o tempo de
percurso de A a B equivale àquele de minimizar o comprimento AP + P B.
Seja e - A' B'. Se P = A', segue do teorema de Pitágoras que AP + P B = a+ Jb2 + c2 : se
P = B'. temos analogamente que AP + P B = b + J a2 + c2 . Em geral, calculando os senos de
o e ;3 nos triângulos retângulos AP A' e BP B', obtemos
- - a b
AP+PB=--+--.
seno sen/3
Analogamente, supondo P =I=A', B', obtemos, a partir de A' P + B' P = e, a igualdade
a b
- + - =e (3.42)
tgo tg/3
(mesmo nos casos em que o> ~ ou /3> ~).
Agora, observe que /3 é uma função contínua de o. Por outro lado, a partir de {3.42) e
considerando separadamente os casos em que P está à esquerda de A', entre A' e B' ou à direita
de B', concluímos que, nesses casos, í3é uma função derivável de o. Por exemplo, no caso em
que P está entre A' e B', temos /3= arctg ( b/( e - t;o)).
Então, resta minimizar
a b
d(o) = seno + sen f3' (3 •43 )
com f3 definido por (3.42).
Pelo teste da primeira derivada, sabemos que os pontos de mínimo de d, se existirem, são
pontos críticos. Derivando (3.43) e lembrando que /3= /3(0:),obtemos
d' (0 ) = _ a cos o _ b cos /3 . /3'(0 ) 1
sen2 a sen 2 {3
de forma que d'(o) = O se, e só se,
2
/3'(0 ) = _ a . cos o sen j3
b cos ,Bscn 2 o

209
l.l\1111"1 l>IHl\\ll\~

P :/:B' ~ /3-::j:.~ =>cos/3-::j:.O). Por outro lado, derivando (3.42) em relação a o,

2
/3'(o) = -~ . sen /3.
b sen 2a
do as duas expressões acima envolvendo /3'(o), concluímos que
, cosa
d (a) =O<=> - = 1 ç:> a= /3.
cos 13

mostrar que a posição de P em que a = /3realmente minimiza AP + P B, dentre


as posições de P sobre r, tais que P -::j:.A', B'. Para tanto, calculando d" (a) (para um a
qualquer), e lembrando que o:,/3E (O,1r),obtemos

d"(a) = ª
sen 3 a
(l+cos 2
a)+ \
sen 13
(1+cos 2
/3) > O.

Portanto, o corolário 56 garante que o único ponto crítico de d ê o úruco ponto de mínimo global,
dentre todas as posições de P sobre r, com P -::j:.A', B'.
Veja agora que, na posição em que a= /3.temos, a partir de (3.42) e (3.43),

a+b a+b
d(a) = -- e -- = e.
sena tga
Segue daf e do problema 4, página 482, que

a+b = -- e = cl✓
d(o:) = -- + tg 2 a
sena cosa
2
= e 1+ + b)
(~ = Jc2 +(a+ b)2 .

J c2 + (a + b)2 < a + Jb + c2,b + J a


2 2
+ c2
valores de AP + P B quando P = A' ou B'), concluímos finalmente que a posição
f3 é a de comprimento mínimo, dentre todas as posições de P ao longo der.
--~

210
Problemas - Seção 3.8
8.1. Prove a equação de Torricellf se t i--+ x(t) é a equação horária de um movímento
retilíneo uniformemente variado de velocidade li--+ v(t) e aceleração a, então

para todos os instantes t 1, t2.

8.2. Na situação descrita no exemplo 77, suponha que o canhão está situado no pé de uma
colina que tem o formato aproximado de uma rampa retilínea, inclinada em relação ao
solo de um ângulo agudo 0. Qual deve ser o ângulo o de elevação do canhão em relação
à horizontal, a fim de que os projéteis por ele arremessados atinjam a maior distância
possível colina acima?

8.3. Nas notações do exemplo 81, mostre que a amplitude A das oscilações de um siste~a
massa-mola de frequência w é tal que
v(t) 2
A= x(t)2 + -2-,
w
para todo t > O.

8.4. A lei de Snell-Descartes da refração afirma que um raio de luz, ao atravessar uma
superfície plana que separa dois meios homogêneos (cf. Figura 3.23) o faz de maneira tal
que
sen o 1 sen 02
-----
V1 V2
onde o 1 e o 2 são os ângulos do raio incidente e do raio refratado com a normal n e v1 e v 2
são as velocidades da luz nos meios de origem e de destino, respectivamente. Deduza a lei

~n
º'

Figura 3.23: a lei de Snell-Descartes.

de Snell-Descartes a partir do princípio de Fermat.

211
AI TEGRAL DE RIE-
MA
( \ 1 1 1 1 1 1 l 1 .\ 1:\ 1 u ;1( \ (, 1>1·:H li·:\ 1.\ :\ \;

&tt> capítulo formaliza a primeira das ideias descritas heuristicamente na introdução, qual
:-ejn. o cálculo <lnúrcn ::;obo gráfico de urna função não negativa f : [a, b] ➔ R; mais precisamente,
concluímo::; que uma tal área pode ser calculada quando, além de não negativa, J for uma
função mtcgrável. Dentre outros subprodutos da discussão do conceito de integral, estudamos
A~ propriedades de duas das mais ubíquas funções da Matemática, as funções logaritmo natural

e c.xponencial.

4.1 O conceito de integral


Esta seção introduz o conceito de integral de uma função limitada f : (a, b] ➔ IR e estabelece
a integrabtltdade de dois tipos importantes de tais funções, quais sejam, as funções contínuas e
as funções monótonas. Para tanto precisamos, inicialmente, fixar algumas notações.
Dado um intervalo [a,b], uma partição de (a,b] é a escolha de um subconjunto finito

P ={a= xo < X1 < x2 < · · · < Xk = b} (4.1)

de (a, b]. Fixadas uma tal partição e uma função limitada (mas não necessariamente não negativa)
f : [a.b] ➔ R, denotaremos sistematicamente

e
Mi = sup{f(x); x 1 -1 < x < x;} := sup f.
[x,-1,x,)
Observe que os números mi e Mi estão bem definidos, graças à limitação da função J.
Os análogos das aproximações inferior e superior para a área sob o gráfico de f, discuti-
das na introdução, são a soma inferior e a soma superior de J em relação a P, definidas
respectivamente por
k k

s(f; P) = L m (x
1 1
- Xj-1) e S(f; P) = L !vl (x1 1
- XJ-d-

Fixada a partição P, certamente temos mí < A/1 para todo j, de forma que s(f; P) < S(J; P).
Portanto, continuando o paralelo com a discussão da introdução, gostaríamos de declarar a
função / como ínteg,-ávelse

sup{s(J; P); Pé partição de [a, b]} = inf {S(J; P); Pê partição de [a, b]}.

Contudo, há CjU(! se>f cr cuidado, pois a desigualdade s(J; P) < S(J; P) (a qual só vale para uma
me,ma partição P) wlo nccc.'lsar,,amenle implica uma rcla,ão entre o supremo e o ínfimo acima.

21'1
Isto se dá porque não há vínculo algum entre a partição genérica P que declara o conjunto do
primeiro membro e aquela que declara o conjunto do segundo membro; a escolha de uma mes~
letra para denotá-las foi uma questão de mera conveniência.
Assim, para prosseguir em direção a uma definição coerente de integral, temos que comparar
s(J; P) e S(J; Q), para duas partições quaisquer P e Q de [a,b]. Fazemos isso no lema a seguir,
cuja demonstração pode ser omitida numa primeira leitura.

LEMA 4.1.
Seja/ : [a,b] ~ R uma função limitada. Dadas partições P e Q de [a, b], tais que P Q (I
temos
s(J; P) < s(f; Q) e S(J; Q) < S(J; P).
--------------'
PROVA.
Seja P como em {4.1). Considere inicialmente o caso em que Q = P U {x'}, com x' f:.
xo, x1, ... , xk, e tome o único índice t E {l, 2 ..... k} tal que x,_ 1 < x' < xi. Então,

t 1

S(J; Q) = L Af,(x,
J=l
- x,_i) + ( sup f)(x' - x,_ 1)
[r,-1,z'I

k
(4.2)
+ ( sup J)(x, - x') +
1r' .r,)
L lvl (x
J=t+l
1 1 - x1 _i).

Agora, como [x,_1 , x'], [x',x,] e [xi-1,xi], temos

sup J, sup J < sup J = !11i,


(r, 1,r') (r' ,r,) (:r,-1,:r,)

de forma que

( sup J)(x' - Xi-i) + ( sup J)(x, - x') < M1(x' - x,_i) + M,(x, - x')
[r, -1,:r') (:r',:r,)

Portanto, segue de (4.2) que


,-1 k

S(J; Q) < L A1 (x 1 1 - x,-1) + !11i(xt- x,-1) + L M (x 1 1 - x1-1)


J=l J=i+l
k
= L 1\1,(x, - x1_i) = S(f; P).
j=l

215
' 1 1 'l 1 .\ I'\ 11-.(;f<.\I. f>f·: Hll·.\f \\\

Cem 1<l<'re1 agora, unrn parLição qualquer Q contendo P. Como Q ê um conjunto finito,
po<l'mos pa,' 'ar de P a Q em um número finito de passos, ajuntando a P um ponto de Q \ P
por vez. A~"im fnzendo, obtemos partições P = P1 e P2 e • • • e Pt = Q, tais que, para
2 < i < 11 obtemos P, adicionando exatamente um ponto a Pi-l· Então, segue da primeira parte
da demonstração que

S(J; P) = S(J; Pi) > S(J; P2) > · · · > S(J; Pt) = S(J; Q).
Por fim, a demonstração da desigualdade envolvendo as sornas inferiores é análoga, e será
deixada ao leitor (cf. problema 1.1).

Como consequência imediata do lema anterior, dadas uma função limitada f : [a,b] ➔ IR e
partições quaisquer P e Q de [a,b], temos

s(f; P) < s(f; P u Q) < S(f; P u Q) < S(f; Q). (4.3)

Conforme veremos a partir de agora, essas desigualdades é que nos permitirão desenvolver ade-
quadamente a teoria de integração.
Seja / : [a, b] ➔ IR uma função limitada. Fixada uma partição Q de [a, b), as desigualdades
(4.3) garantem que S(f; Q) é uma cota superior para o conjunto {s(f; P); Pé partição de [a,b]}
das somas inferiores de f. Portanto, o axioma' 3 garante que tal conjunto possui um supremo, o
qual será denotado por supp s(f; P):

sups(J; P)
p
= sup{s(J; P); Pé partição de [a,b]}.
Agora, como o supremo de um conjunto limitado superiormente é a menor de suas cotas
superiores(uma das quais é S(J; Q)), concluímos que

sup s(f; P) < S(J; Q).


p

Mas, como a partição Q foi escolhida arbitrariamente, essa última desigualdade é válida para
toda partição Q de [a,b]. De outra forma, o número supps(f;P) é uma cota inferior para
{S(/; Q); Q é partição de [a, b]}, o conjunto das somas superioresde f. Portanto, segue do pro-
blema 1.4, página 74, que tal conjunto possui um ínfimo, o qual será denotado por infQ S(J; Q):

ínf S(J;Q)
Q
= inf{S(J;Q); Q é partição de [a,b]}.

Por fim, como o ínfimo de um conjunto limitado inferiormente é a maior de suas cotas inferiores
(uma daHquaiR é RUJ)p s(f; P)), couclufmos que

(4.4)

21G
Estamos finalmente em condições de apre entar a definição central deste capítulo.

DEFINIÇÃO 4.2. '


Uma função limitada/ : [a, b] ➔ lR é integrável se

sup s(f; P)
p
= inf
p
S(J; P).

Conforme antecipamos no primeiro parágrafo desta seção, funções contínuas e funções mo-
nótonas com domínio [a,b] são exemplos de funções integráveis. Contudo, com o material de
que dispomos até aqui, é mais conveniente apresentar, inicialmente, o exemplo clássico de uma
função não integrável.

EXEMPLO 4.3.
Recorde (cf. problema 3.11, página 102) que a função de Dirichlet é a função/: [O,1] ➔ R
tal que
f(x) = { O se x ~Q .
lsexEQ
Fixada uma partição P = {O = x 0 < x 1 < x 2 < · · · < Xk = 1} do intervalo [O,1], segue do
problema 1.21 página 74, que todos os intervalos [x,_1 , x 1 ] contêm números racionais e irracionais.
Portanto, para 1 < j < k, temos m 1 = O e A,/1 = 1, de forma que
k
s(f; P) = L m (x 1 1 - Xj-d =O
J=l

e
k k

S(f; P) = L Afj(x 1
- x 1
-1) = L(x 1
- Xj-i) = 1.
Então,
sup
p
s(J; P) =O<,-. 1 = inf
p
S(f; P),

e f não é integrável.
-------~~-~-------------~~---
A fim de apresentar de maneira mais transparente os exemplos prometidos de funções in-
tegráveis, é interessante, inicialmente, reelaborar a definição de função integrável conforme o
resultado a seguir, o qual também encontrará utilidade na próxima seção. Tal re~ultado é co-
nhecido como o critério de integrabilidade de Cauchy, e sua demonstração tmubém pode
ser omitida numa primeira leitura.

217
1 ' '1 111 1 .\ 1\ 11·.Li( \1. Ili·. Bll·.\I \:\:\

TEOREMA 4.4. CAUCHY


Uma função limitada f : [a, b] ➔ lR é integrável se, e só se, a seguinte condição for satisfeita:
para todo f > O dado, existe uma partição PEde [a, b] tal que

S(J;PE)- s(J;Pc) < €. (4.5)


----------~~~

PROVA.
Suponha, inicialmente, que fé integrável, de sorte que supp s(f; P) = infp S(f; P). Denote
tal valor comum por I, e seja dado € > O. Como I = sup p s(f; P) é a menor cota superior
para o conjunto da somas inferiores de f e I - ½ < I, existe uma partição P 1 de [a,b] tal que
I - ½< s(f; P1). Analogamente, como I = inf p S(f; P) é a maior cota inferior para o conjunto
da somas superiores de f e I + ½> I, existe uma partição P2 de [a,b] tal que S(f; P2) < I + ½-
Pondo Pc = P 1 U P2, segue de (4.3) que

S(f; P,J - s(f; Pc) < S(f; P2) - s(f; Pi)< (1+ ~) - (1- ~)= €.

Recipro~amente, suponha satisfeita a condição do enunciado. Então, dado € > O, tomando


Pc como lá e levando (4.4) em consideração, obtemos

O < inpfS(J; P) - sup s(J; P) < S(J; Pc) - s(f; Pc) < €.
p •

Mas, uma vez que€> O foi escolhido arbitrariamente, segue que infp S(J; P)-supp s(J; P) = O,
conforme desejado.

Podemos finalmente estabelecer a integrabilidade de funções monótonas e funções contínuas.

~ iuoçio monótona / : [a,b]➔ R é integrável.


-----~--------~~~_.,

PROVA.
Suponha que f ê não decrescente, sendo o caso em que f é não crescente totalmente aná-
logo. Observe que f ê claramente limitada, com imagem contida no intervalo [J(a), J(b)]. Para
estabelecer a integrabilidade de f ê suficiente, pelo critério de Cauchy e dado € > O, encontrar
uma partição P = { a = xo < xi < · · · < Xk = b} de (a, b] tal que a condição (4.5) seja satisfeita
para P,. = P.
Para uma partição qualquer P como acima, segue do fato de f ser não decrescente que

218
()(()'\ll))()J)) )'\)11,)!\I 1 1

e. analogamente, Af 3 = f(x 3 ). Portanto,


·k k

S(J; P) - s(J; P) =L Al3 (x 3 - x 3 _i) - L m (x; - 3 X 1 -1)


3=1 J=l
k
= L(f (x 3
) - J(x _i))(x
1 3
- Xi-d·
J=l

Sendo ó = ma.x{x1 - x 3 -1; 1 < j < k}, segue da última igualdade acima e da fórmula (A.27)
para somas telescópicas que
k
S(J; P) - s(J; P) < L(f(x;) - f(xi_i))ó = ó(J(b) - J(a)).
j=l

Portanto, basta escolhermos a partição P de tal forma que ó(f (b) - J(a)) < E; isso ocorre se,
por exemplo, o< /(b)-Í(a)+l, i.e., se

x1 - Xj-l < J(b) _;(a)+ 1 , \;/ 1 < j < k.


Como é sempre possfvel escolher uma partição P satisfazendo tal condição, é sempre possível
obter uma partição PE de (a,b) satisfazendo {4.5).

TEOREMA 4.6.
Toda função contínua / : (a, b) ➔ R é integrável.

PROVA.
O corolãrio 52 garante que uma tal f é limitada. Portanto, novamente pelo critério de
Cauchy e dado f > O, para estabelecer a integra.bilidade de fé suficiente encontrar uma partição
P ={a= x 0 < x 1 < · · · < Xk = b} de (a,b) tal que (4.5) seja satisfeita para Pi= P.
Como na demonstração do exemplo anterior, temos
k k
S(J; P) - s(J; P) = L M1 (xj - x3 -1) - L mj(x 1 - x1 -1)
j=l j=l
k
(4.6)
= L(M 1
- m )(x 1 3
- xj_ 1
).

j=l

Lá, também tínhamos m 3 = M3 -1, uma vez que f era não decrescente. Para uma funçiio
contínua mas não necessariamente não decrescente, esse não é, em geral, o ca o. Contudo,
podemos utilizar a continuidade de f para controlar o fator Ali - m;, em vez do fator J'J - .r:1-t ·

219
'. \1·111111 1 .\ I'\ 11-.CH \1. Ili-: HIE\I.\:\:\

Como f tem por domínio um intervalo fechado e limitado (a,b], o teorema 51 garante que f
é uniformemente contínua. Logo, consoante a definição 50, dado É > O, existe ô> O tal que

x, y E [a, b], lx - YI <ô=> lf(x) - f(y)I < f.'.

Portanto, começando com uma partição P tal que x 1 - Xj-l < ô para 1 < J < k, concluímos que

· x1 -1 < x, y < x1 =>lx - YI <ô=> lf(x) - J(y)I < l.

Mas, uma vez que MJ = sup{f(x); Xi-I < x < xi} e m 1 = inf{f(y); x1 -1 < y < x1 }, segue
facilmente da desigualdade acima que

(<!.7)

Por fim, sendo Puma partição tal que x 1 - x 1 _ 1 < 8 para 1 < J < k, segue de (4.6), { 1.7) e
novamente da fórmula (A.27) que
k
S(f; P) - s(f; P) = I).\/ 1 - m 1 )(i·1 - x 1 _i)
J-1
k
< L l(.1· 1 - .1·1 _i) = l(b - a).
J=l

Logo, para que S(J; P) - s(f; P) < t, basta começarmos com E'= 2 (b~a).

Se f : [a, b] -+ 1Ré uma função integrável, denotamos a integral de f sobre o intervalo [a, b]
'

t
\
escrevendo
J(x)dx. (4.8)

Vale observar que a notação acima, devida a Leibniz, tem sua razão de ser: o símbolo J lembra
um S estilizado, e está presente para recordar que o cálculo do valor da integral envolve um
processo de aproximação por somas; os números a e b recordam o domínio da função f que está
sendo integrada; dx recorda que as somas que aproximam o valor da integral envolvem diferenças
Xi - x,_ 1 de elementos de uma partição P de [a, b], diferenças estas que, classicamente, eram

denotadas por óx,. Note, ainda, que tanto faz denotarmos a integral de f sobre o intervalo [a, b]
como em (4.8) ou escrevendo
[ J(t)dt.

De fato, uma tal mudança de notação equivale a mudarmos o nome da variável que denota os
elementos do domínio [a, b] da função f, o que certamente não afeta o valor de sua integral.

220
Em que pese a densidade do material desta seção, infelizmente ainda não dispomos de um
procedimento geral para calcular, para-funções f : (a,b] ➔ 1Respecíficas, o valor de J,;J(.r)rl.r.
Remediaremos essa situação na seção 4.4. Contudo, veja os problemas 1.2, 1.4, 1.5 e 1.10.

Problemas - Seção 4.1


1. 1. * Complete a prova do lema 1 mostrando que, se f : [a, b] ➔ 1Ré limitada e P e Q são
partições de [a, b] tais que P e Q, então s(J; P) < s(J; Q).

1.2. * Se J: [a, b) ➔ 1Ré uma função constante, digamos f (x) = e para todo x E [a, b), mostre
que
l f(x)dx = c(b - a).

1.3. Sejam J : [a, b) ➔


1Ruma função monótona e Pk = {a = x 0 < x 1 < · · · < Xk = b} uma
partição equiespaçada de [a, b), i.e., tal que X; - X;-i = b·t, para 1 < j < k. Mostre que

1.4. * Para uma função não negativa J : [a, b) ➔ IR, verifique que as aproximações
por falta e
por excesso (1) e (2) para a área da região 'R, são casos particulares de somas inferiores e
superiores de f. Em seguida, use esse fato, (6) e o resultado do problema anterior para
mostrar que, dados n E N e b > O, temos
b bn+l

1 0
xndx= --.
n+l

1.5. Para calcular J: ../xdx, faça os dois itens a seg~

(a) Mostre que, para reais u > v > O, tem-se


2 1 1
3(uvu - vvv) - 3(u - v)( y'u- Jv) < 2( vu+ vv)(u - v)

e
1 2
2( vu+ vv)(u - v) < 3(u,/ü - v\l'v).

(b) Se Pk = { a = x 0 < x 1 < · · · < Xk = b} é uma partição equiespaçada do intervulo


[a, b], use o resultado do item (a) para mostrar que

2 r. ~) (b - a)( v'b- Jã) ( r::: p ) ./b-k ,/ã.


(bvb- ava - k < s vx; k +
3 3 2

221
\ 1 1 , , ,1 1:11 1 ,

e
./b-Ja 2 r,-
s( Jx;Pk) + k < (bv b - ava).
2 3
(e) Conclua que J:./x dx = ~(b./b- aJa).

1.6. Admita que f01 ✓:+i dx = 2( ./2- 1). (Calcularemos esta integral no item (d) do problema
4.1, página 259.) Se an = 7n(Ji~n + ✓21+n + · · · + ¼n)para n E N, mostre que a
sequência (an)n>l converge para 2( v'2- 1).

1.7. Mostraremos no exemplo 40 que f 1


0 1
;:r2 dx - ~- Use esse fato para mostrar que, se
<ln= n (n2~ 12 + n2~ 22 + · · · + n2!n2), então a sequência (an)n~l converge para ~.

1.8. Sejam f : (a, b] -+ lR uma função limitada e a > O um real dado. Se, para cada k E N,
existe uma partição Pk de [a,b] tal que S(f; Pk) - s(f; Pk) < i, mostre que f ê integrável
e que

J.J(x)dx =
ª
b lim s(f; Pk) = lim S(f; Pk)-
k-++oo k-++oo

Para o próximo problema, o leitor pode achar conveniente reler o enunciado do problema
4.3, página 109.

1.9. Sejam f : [a,b] -+ lR urna função lipschitziana, com constante de Lipschitz e, e Pk = {a=
x0 < x 1 < · · · < Xk = b} uma partição equiespaçada do intervalo [a, b].
2
(a) Mostre que S(f; Pk) - s(f; Pk) < c(b~a.> .

(b) Para 1 < j < k, escolha um real t.kJ E [xj-1, xj] e defina a soma E(J; Pk; t,k) por
E(J; Pk; t.k) = Í:~=l f (t.ki)(x 3 - X1-1). Prove que

J.
ª
b J(x)dx = lim E(J; Pk;t,k)-
k-++oo

1.10. * Faça os dois itens a seguir:

(a) Dados a, h E IR, com h # 2l1r para todo l E Z, prove que

k sen (a+ (k-l)h) sen (k+1)h


L sen (a + j h) =
3-0
2

sen 2
h
2

(b) Mostre que fab sen x dx =cosa - cos b.

222
lllilll:11\I\'- "-.!1\1111 ,

1.11. Seja f: (O,1] ➔ IR a função dada por

f(x) = { O, se x = O ou x ~ Q
~, se x = '; com m, n E N e mdc (m, n) =1
Prove que f é integrável.

223
, 1 \ 1 1 11 1 , , 1 .\ [\: 1 H ; 1<\ 1. ll l·. H IF \ 1 \ :\ :\

4.2 O Teorema de Riemann e algumas observações


1 esta seção, nós enunciamos um importante teorema de Riemann sobre a caracterização de
funções integráveis a Riemann, bem como tecemos algumas observações importantes sobre os
limites da teoria de integração desenvolvida na seção anterior. Em particular, colocamos em
bases sólidas o conceito de área da região sob o gráfico de uma função integrável e não negativa,
o qual foi discutido informalmente n@-:introdução.

I. O mais das vezes, a apresentação da integral nos cursos de Cálculo faz uso do conceito
de somas de Riemann 1 . Mais precisamente, dada uma função limitada J : [a,b] ➔ IR, define-
se f como integrâvel (a Riemann) se existir um número real J (a integral de Riemann
de J) satisfazendo a seguinte condição: dado t: > O, existe ô > O tal que, para toda partição
P ={a= Xo < x1 < · · · < Xk = b} de [a,b] e toda escolha de pontos ç1 E [x1 -1,x 1 ], 1 < j < k,
tenhamos
k
max{lx 1 - Xj-1 I; 1 < j < k} < ô=> L J(çi)(x 1 - x 1 _i) - I < t:. (4.9)
J=l

Nas notações acima, dizemos que max{lxi - xi-d; 1 < J < k} é a norma da partição P e
que E~=l J(çi)(xi - xi-d é a soma de Riemann de f com respeito a P e ao pontilhamento
ç = (çi)i$i$k de P. Denotando
k

IPI = max{lx 1 - XJ-1 I; 1 < j < k} e E(J; P; ç) = L J(ç )(x 1 1 - x 1 _ 1 ),


J=l

resumimos (4.9) dizendo que I é o limite das somas de Riemann E(J; P; ç), quando IPI ➔ O e
para todo pontilhamento ç de P. Ademais, nesse caso, escrevemos simplesmente

I = lim E(f; P; ç).


IPl-+O
O resultado a seguir, cuja demonstração omitimos, mostra que a definição de integral dada
acima coincide com aquela adotada na seção anterior. Ao leitor interessado, sugerimos consultar
[1), (20)ou (27).

,,.,.JAA 4. 7. RIEMANN
Uma função limitada/ : [a, b] ➔ R é integrável (no sentido da seção anterior) se, e só se,
ate o limite limlPl➔O E(/; P; {) de suas somas de Riemann, para todo pontilhamento ç de P.
-ai•~ neaaecaso, tem-se

1Após
1
a
b

o matemático alemão do século XIX B. füemann.


f(x)dx = lim E(/; P; ç).
IPla-+O ~----------------

224
Apesar do fato de a introdução da integral por meio de somas de Riemann ter sido a maneira
pela qual a teoria se desenvolveu historicamente, a abordagem por meio de somas superiores"
e inferiores torna tecnicamente bem mais simples a discussão do material da próxima seção.
Entretanto, conforme veremos a partir de agora, a caracterização da integral dada pelo teórema
de Riemann nos permite alicerçar a discussão heurística da introdução, acerca do cálculo de
áreas.
Dada uma função limitada e não negativa f : [a, b] --t IR, denote (como na introdução) por
n a região sob o gráfico de f (e situada acima do eixo das abscissas e entre as retas x = a e
X= b):

Seja Pk = { a = Xo < x1 < · · · < Xk = b} uma partição equiespaçada de [a, b), i.e., tal que
x 1 - x 1-1 = b-,_a,
para 1 < j < k.
Se f é conUnua, então, pelo teorema 53, para 1 < j < k existem Çkj, ç1iE [x3 _ 1 , xj] tal que

Portanto, as aproximações por falta e por excesso A(J; k) e A(f; k) para a área de 1?.,{definidas
por (1) e (2), página 3)) coincidem, respectivamente, com as somas de Riemann E{/; Pk; çk) e
E(f; Pk; (D (observe que, aqui, Çk e ç1denotam pontilhamentos do intervalo [a, b), em vez dos
elementos de um pontilhamento). Mas, como IPk1 = ¼,segue do teorema de Riemann que

rb J(x)dx = lim E(J; Pk; Çk) = lim A(J; k)


}a k-++oo k-++oo

e, analogamente,
fb J(x)dx = lim A(J; k).
Ía k-++oo

Assim, J:J(x)dx é o único valor razoável para a área de n.


Por outro lado, se f é meramente integrável (mas ainda não negativa), então, a menos da
troca dos valores mínimo e máximo de f no intervalo [xj-l, xj] por seus ínfimo e supremo em
tal intervalo, A(J; k) e A(J; k) resumem-se às somas inferior e superior s(f; Pk) e S(J; Pt),
respectivamente.
A discussão acima torna a definição a seguir bastante natural.

225
( \llilli, 1 .\ l'\IILH\I Ili BII\I\'\'\

DEFI IÇÃO 4.8.


J : [a, b] ➔ IRé uma função integrável e não negativa definimo a área d r gião 'R ob o
gráfico de f por ·

~--------------
A(R) = t f (x)dx.
-----------~~-----

a próxima s ção, a definição acima fornecerá argum nto h urí ico p r pi u 1 ilid d
algumas das propriedades operatória da integral d Riemann.

II. Provamo no teorema 6 qu toda função contínua f : [a b] ➔ int r ,. l. m m,


vimo no exemplo 3 que a função d Dirichl t (a qual d on ínu m od p nt d
[O,1), pelo problema 3.11, página 102) n -o ' in grá 1. Por ou ro l o, pro m
ção (cf. propo ição 18) qu uma funÇo f : [a b] ➔ lm nt continua (i. m Ulll
número finito de ponto d d ontinuid d O E [a b], to q limi
limx➔xo± J(x)) ainda é integráv l. m qu 1 f rn id 1 1 f m Ao
definida no problema 1.11 págin 223 m p u1r um núm r infini
de pontos de de continuidade.
A <li u ão do parágr fo an rior - íntim ml -
grabilidade de uma função limitad f : (a. b tamanho o D1
de de continuidad . E d f to o c o, um f rmul - pr r n r int -
grabilidad e o tamanho de D f u it n io conJimto d m d1da nu.la, xpli i do
seguir.

4.9.
Dw.emos que X e Ré um conjunto de medida (de Lebesgue) nula , dado f > O,
existem intervalos abertos 11, /2, /3 ... tais que

e LJI, L 111<
j~l
e
j~l
1 f,

,1denota o comprimento de l;.

Um teorema d Leb gue2 garante que uma função limitada f : [a b] ➔ é in egrável a


Riemann , e só o conjunto D J de us ponto de d continuidade em medida nula. E e
r ult::.do dá uma explicação unificada para a integrabilidade ou não integrabilidade d odo o
tipos d funçõ di cutido no penúltimo parágrafo ant da definição 9: como o conjun o do
2 Apól H. Le

226
pontos de descontinuidade de funções contínuas ou seccionalmente contínuas são finitos (logo,
certamente de medida nula), tais função são integráveis. Por outro lado, é possível provar que
[O.l] não tem medida nula, mas Q tem medida nula; isso explica o fato de a função de Dirich1et
não ser integrável (uma vez que o conjunto de seus pontos de descontinuidade é todo o intervalo
[O,l]), bem como o fato de a função do problema 1.11, página 223, ser integrável (posto que o
conjunto de seus pontos de descontinuidade é Q n [O,l]).
É possível provar que o conjunto ndos números irracionais contidos em um intervalo qualquer
não tem medida nula. Portanto, uma função f : [a, b] ➔ 1R.cujo conjunto dos pontos de
descontinuidade seja [a, b]n Il não é integrável. Mas, uma função com um tal conjunto de pontos
de descontinuidade pode existir? Pode-se provar que não, mas a demonstração também foge ao
escopo dessas notas.
Para uma discussão detalhada dos fatos acima, sugerimos novamente ao leitor consultar uma
das referências (l], [20], (26) ou (27).

III. Posteriormente a Riemann, no início do século XX, uma versão mais flexível do conceito
de integral foi introduzida pelo matemático francês H. Lebesgue. A fim de apreciar a diferença
entre tais conceitos, precisamos da definição a seguir.

DEFINIÇÃO 4.10.
A função caracterlstica de um conjunto A e R é a função X,4 [a, b) ➔ R, tal que
XA(x) = 1, se x E A, e O, se x </.A.

Consideremos, agora, uma função f : [a, b] ➔ [O,L].


o contexto da integral de Riemann, começamos escolhendo uma partição P = { a = x 0 <
x 1 < · · · < Xk = b} do domínio [a, b] de f. Em seguida, definimos as funções-escada f P,-, f P,+:
[a, b] ➔ JR.,pondo
k k

ÍP,- =L mjX[x,-1,x1l e ÍP,+ = L MjX[X3-l,X3]1


J=l J=l

onde m 1 = inf1x,_1 ,:r1 J f e M1 = SUP(x,_ 1 ,x1 J f. ~:~.--próximaseção, mostraremos (cf. exemplo 19)
que tais funções são integráveis a Riemann, com

J. J.
b k b k
JP, (x)dx = ~ m1 (x1 - x,-1) e ÍP,+(x)dx = ~ AI,(x1 - x,-1).

Portanto, de acordo com a discussão da seção anterior, / é integrável a Riemann se

sup J.b ÍP,-(x)dx = irJfJ.b ÍP,+(x)dx;


P a a

227
< \1'111 1, 1 1 .-\ IYI u;JL\I. IH·: HIE\I.\\'\

ademais, nesse caso, a integral de f coincide com o valor comum acima.


Já no âmbito da integral de Lebesgue, começamos com uma partição Q = {O= Yo < Y1 <
· · · < y, = L} do contradomínio [O,L] de f. Então, para 1 < j < k - l, consideramos a imagem
inversa A, do intervalo [Y,-1,y1 ) por f, definida por

assim como a imagem inversa Ak do intervalo [Yk-1,yk], tal que

Se os conjuntos Ai não forem muito complicados (num sentido que não nos interessa aqui),
mostramos que é possível associar a cada Aj um número real não negativo m(A 1), denominado
a medida de Lebesgue de A,, de uma maneira tal que, se A 1 for um intervalo, então m(AJ)
coincide com o comprimento de A,; nesse caso, dizemos ainda que A 1 ê um conjunto mensu-
rável (a Lebesgue). Em seguida, supondo que cada A 1 ê mensurável, consideramos a função
simples
k

ÍQ = L
j=l
Y1-1XA)

e definimos sua integral de Lebesgue, denotàda Íia,b) ÍQ, por

(4.10)

Por fim, se para toda partição Q de [O,L] todos os conjuntos A 1 definidos como acima forem
mensuráveis, dizemos que f ê uma função mensurável; nesse caso, definimos a integral de
Lebesgue de f, denotada Íia,b) f, por

f f = sup {JJQ; Q ê partição de [O,LJ}. (4.11)


Í[a,b) Q

Apesar de termos considerado somente o caso de funções li.mitadas e não negativas na discus-
são acima, é possível considerar a noção de integral de Lebesgue no contexto de funções limitadas
e mensuráveis f: [a,b] ➔ lR quaisquer. (De fato, podemos considerar também o caso de funções
não limitadas, mas isso não vem ao caso aqui.)
É possível mostrar que a integral de Lebesgue é mais abrangente do que a integral de Riemann,
de duas maneiras: por um lado, toda função f : [a,b] ➔ lR (li.mitada e) integrável a Riemann
é também integrável a Lebesgue, e os valores das integrais de Riemann e de Lebesgue de f

228
coin id m· por o -
int • L b a fun - iri
- 5.4 ( f. o gu t mbém
dispõ d teo m d con erg-nc ·a uj d mo raçõ h no ont 'to da int gral
d Ri mann. r Y 1am impr cindi,· • para o tudo d d m • profundas d
cqu-nc d funç - . m últim an • • raz p 1 quai
am gu m ad u da do qu al d muit propó ito
u apli"°""'v~ tr am nto mod rno da
dit r n i •
r p udam dir t u nt m gr 1 d
raz o qu . por um 1 do m qu
,
p int r 1 d Lb gu
111 ral d Rim 111

( 'Ili ( 1.11) d
p r d1 mui itu -

Pnt d r 11 u 1-7]p um in r du - b nt
1 IP111 •11t 1 1 int 1 1 1

r l 11 .2
'....1. 11 m ( ) i p bl u .l u fol X n(1r )d =l 1f
f l )l 1 n)n;?:l n rg limn + an = 1.

t1 U d iri hl t / : ( 1) ( f. x mpl 3) m gr v l no ntido


l} f =
..... , L) um n . m tr m no or m 5 qu J int gráv 1
tr u qu J:J(x)dx = Íia.b)J.
~ J: [ . ) [ . ) wn bij - n on ínu Mo tr qu

f.ªf(x)dx + J.'r 1
(x)dx > ab

p t r n n tiv a b.

1/.9:[ .l)" (O.l]du funç- ndofn- nt . Prov qu

[ (f o g)(x)dx < [ f(x)dx + [ g( )dx.

22
( \ l 'I 1 1 1 ( l 1 :\ '\:TECI{.\(, I>E H IE\I.\\:\:

4.3 Operações com funções integráveis


Esta ::,eçãoé devotada ao estabelecimento de algumas propriedades operatórias úteis para
a integral de Riemann. Apesar de que tais propriedades serão utilizadas extensivamente no
restante do livro, suas demonstrações podem ser omitidas numa primeira leitura, sem perda
apreciável de continuidade.
Ao longo de toda esta seção, dadas funções limitadas J,g : (a,b] ➔ IR e uma partição
P ={a= xo < X1 < · · · < Xk = b} de (a, b],denotaremos os ínfimos de f e g no intervalo (x1 -1, x 3 ]
respectivamente por m 3(J) e m 3(g); analogamente, M1(J) e A,J3(g) denotarão os supremos de f
e g no intervalo [x3 _ 1 , x 3 ], também respectivamente.
Se J,g : [a,b] ➔ IR são funções integráveis e não negativas, com f < g, então as regiões
'R-1e 'R9 , situadas respectivamente sob os gráficos de f e g, são tais que 'R1 e 'R9 ; esperamos,
pois, que A('R1) < A('R 9). Esse é de fato o caso, e é uma propriedade da integral de Riemann
conhecida como sua monotonicidade.

PROPOSIÇÃO 4.11.
Se/, g.: [a, b]➔ R são funções integrá.veis e tais que f < g, então 1:J(x)dx < 1:g(x)dx.

PROVA.
Sendo P = { a = Xo < x1 < · · · < xk = b} uma partição de [a, b], segue imediatamente de
f < g que M;(J) < M 3 (g), para 1 < j < k. Portanto,
k k
S(J; P) = L M;(J)(x; - x3 -1) < L J\ll (g)(x; -
3 Xj- 1) = S(g; P).

Mas, como 1:f (x)dx < S(J; P), concluímos que

t f (x)dx < S(g; P),

para toda partição P de (a, b]. Assim, 1:J(x)dx é uma cota inferior para o conjunto das somas
superiores S(g; P), de forma que

b f (x)dx < inf S(g; P) = J.b g(x)dx.


/. a p a

 230
Consideremos, agora, uma função integrável e não negativa f: [a, b] ➔ lRe um real p08ítívoe.
O problema 5.10, página 59, garante que a região 'Rc/ sob o gráfico da função cf pode ser obtida
da região 'RJ sob o gráfico de / por meio de uma dilatação vertical por um fator e; esperamos,
pois, que A('Rcf) = cA('R1 ). Por outro lado, se g : [a,b] ➔ lR é outra função integrável e não
negativa, então, para cada x 0 E [a,b), o segmento da reta x = x 0 contido em 'R1+u (a região
sob o grãfico de / + g) tem comprimento igual à soma dos comprimentos dos segmentos de tal
reta contidos em n 1 e 'R9 ; isto sugere que 'R1+9 pode ser obtida colando 'R9 acima de n 1 e que,
portanto, devamos ter A('R 1+9 ) = A('R 1) + A('R 9 ).
A seguir, veremos que as duas propriedades da integral sugeridas pelos argumentos heurísticos
do parágrafo anterior são realmente vâlidas. Doravante, nos referiremos a tais propriedades
respectivamente como a linearidade e a aditividade da integral de Riemann.

PROPOSIÇÃO 4.12.
Se f, g : (a, b) ➔ IRsão funções integráveis e e E IR,então:

(a) e/: [a,b] ➔ IRé integrável, com J: cf(x)dx = e J: f(x)dx.


(b) f + g: (a,b] ➔ IRé integrável, com J:(J(x) + g(x))dx = J: f(x)dx + J: g(x)dx.
----~

PROVA.
(a) Corno na prova da proposição anterior, seja P ={a= x 0 < x 1 < · · · < Xk = b} uma partição
de (a, b].
Se e > O, é imediato verificar (cf. problema 1.10, página 75) que

e, analogamente, !v11(cf) = cM1 (!). Assim,


k k
s(cf; P) = L m (cf)(x
1 1 - Xj-1) = e L m (f)(x 1 1 - x 1 _i) = cs(J; P)
3=1 j=l

e, analogamente, S(cf; P) = cS(J; P). A integrabilidade de f e as propriedades de supremos e


ínfimos (novamente conforme o problema 1.10, página 75) dão-nos

sups(cf; P) = sup(cs(f; P)) = e sups(f; P) = clb J(x)dx


p p p a

e
inf S(cf; P) = inf(cS(/; P)) = e inf s(f; P) = clb f(x)d:r.
p p p a

231
< \1111 1,, 1 .\ 1:\11-:c;B\I. Ili·: Hll-:\1\'-.\

, 'e< O, a demonstração das duas relações acima é inteiramente análoga, bastando observar,
sue'· ·ivmnente, que mi(cf) = e 1'vl
1 (f), Mi(cf) = cmi(f), s(cf; P) = e S(f; P) e S(cf; P) =
e ·(/; P), de forma que

sups(c/; P)
p
= sup(cS(/;
p
P)) = e inf
p
S(J; P) = cf.b
a
f(x)dx
.
e, analogamente, infp S(cf; P) = e J:f(x)dx.
Em qualquer caso, temos infp S(c/; P) = supp s(cf; P) = e J:f(x)dx. Logo, cf é integrável,
com

J.bcf(x)dx =
a
inf S(cf; P)
p
= cf.b
a
J(x)dx.

(b) Sejam P e Q partições de [a,b] tais que P U Q = {a = x 0 < x1 < · · · < Xk = b}. Segue
prontamente do problema 1.11, página 75, que

sup (/ + g) < sup f + sup g


(x1 -i,x 1) (x1 -i,x 11 [xJ-1,:i: 3)

ou, ainda (em relação à partição P U Q) Mi(!+ g) < Mj(f) + Ali(9), Portanto,
k
S(f + g; P U Q) = E Mi(!+ g)(xj - x 1 -1)

= S(J; P u Q) + S(g; P u Q)
< S(J; P) + S(g; Q).
Mas, como infR S(J + g; R) < S(J + g; P U Q), segue das desigualdades acima que

i~f S(f + g; R) < S(J; P) + S(g; Q),

para todas as partições P, Q e R do intervalo [a, b].


Na última desigualdade acima, tomando o ínfimo sobre todas as partições p e Q de [a, b] e
utilizando novamente o resultado do problema 1.11, página 75, obtemos

inf S(f + g; R) < inf {S(J; P) + S(g; Q); P, Q partições de [a, b]}
R
~ inf S(f; P) + inf S(g; Q)
P Q (4.12)
= J.ºJ(.r)d:r+ jb g(~r)dx.
h h

232
Argumentando de forma análoga ao que fizemos até aqui, obtemos sucessivamente m1 (J+g) ~
m1 (f)+mí(g), s(f +g: PUQ) > s(f; P).+s(g; Q) e supR s(J+g; R) > s(f; P)+s(g; Q). Portanto,
utilizando uma vez mais o resultado do problema 1.11, página 75, obtemos

sup s(J + g; R) > sup s(f; P) + sup s(g; Q)


R p Q
(4.13)
= J.'J(x)dx + J.'g(x)dx.
Por fim, como supRs(f + g:R) < infRS(J + g;R), segue de (4.12) e (4.13) que

irJ! S(f + g; R) = sup s(f + g; R) = 1b f(x)dx + 1b g(x)dx.


R a a

Assim, J + g é integrável, com J:(f(x) + g(x))d.x = J: J(x)dx + J: g(x)dx.

Um corolârio óbvio da propo ição anterior é que. e /, g : [a.b}➔ IR são funções integráveis,
então J - g : [a, b1 ➔ IR também é integrável, com

J.'u(x) - g(.r.))d.t= l f(.r)dx - l g(x)dx. (4.14)

Realmente, o item (a) garante a intcgrabilidadc da função - /, ao passo que o item (b) garante
aquela de f - g-= f + (-g). A relação (4.14). egue, agora, das fórmulas dos itens (a) e (b) da
proposição anterior:

J.'u(x) - g(x))dx =l f(x)dx + l(-g(x))dx

=l f(x)dx + (-1) l g(x)dx.

Se f : [a, b}➔ IR é uma função contínua, a regra da cadeia para funções contínuas garante
que Ili = l · Io f: [a, b}➔ IRtambém o é; em particular, l/1também é integrável. No que segue,
e tabelecemo a integrabilidade de lf I supondo que / é simplesmente integrável. Obtemos, ainda,
uma de igualdade muito útil entre as integrai~ de / e de 1/1, conhecida como a desigualdade
triangular para integrais.

PROPOSIÇÃO 4.13.
Se J : [a, b}➔ R, é uma função integrável, então a função 1/1: [a, b] ➔ R tambéuié
e vale a desigualdade abaixo:
[ J(x)dx < [ lf(x)ldx.

233
< \1·111111 I .\ 1'--111,1:\I 1>1 Hll\l\'\\

PROVA.
Sejam f +, f- : [a, b) ➔ [O,+oo) as funções dadas por
f +(x) = ma.x{f(x), O} e J_(x) = - min{J(x), O}.

É imediato verificar que J = J + - f- e Ili= J + + J_. Portanto, se f + for integrável, então,


pela discussão que precede o enunciado desta proposição, f- = J+ - f também será integrável;
mas, sendo esse o ·caso, o item (b) da proposição anterior garante que o mesmo sucederá com
Ili= f+ + J_.
A partir daí, para demonstrar a desigualdade (4.15), observe inicialmente que a monotonici-
dade da integral, juntamente com o fato de que f- > O, garante que J:J_(x)dx > O. Por sua
vez, aplicando sucessivamente (4.14), essa desigualdade e a fórmula do item (b) da proposição
anterior, obtemos

[ f(x)dx = [ f+(x)dx- [ J_(x)dx

< 1• f+(x)dx+ 1• J_(x)dx


(4.16)
= 1•(f+(x)dx+f-(x))dx

= t IJ(x)ldx

(Para uma outra prova de (4.15), sugerimos ao leitor o problema 3.7.) Analogamente, mostramos
que
- [ J(x)dx = [<-J)(x)dx < [ 1(-J)(x)ldx = [ IJ(x)ldx,

de sorte que
- [ lf(x)ldx < [ J(x)dx < [ u<x)ldx,
o que por sua vez equivale a (4.15).
Resta demonstrar a integrabilidade de f+• Para tanto, seja P ={a= x0 < x 1 < ... < xk =
b} uma partição de [a, b). Se J(x) > O para algum x E [x1 -1, x1), então M1 (J+) = M1 (J); por
outro lado, como f + > J, temos m 1 (J+) > m 1 (J), de sorte que

Sef(xj < O para todo x E [xj-1, x1], então f + = O em [xi-l, x 1] e, daí,

234
Em qualquer caso, temos 1'13(f +) - m 3(J+) < li/ 1(/) - m 3(J), de forma que
k
S(f+; P) - s(J +; P) = L(1'1 1 (f+) - m 1 (f+))(x 3 - x 1 -1)
;=l
k
< L(i\tl 3(J) - m3(J))(x 3 - x3_ 1)
J=l

= S(J; P) - s(f; P).

Agora, como fé integrável, o critério de Cauchy garante que, dado t > O, podemos tomar a
partição P de tal forma que S(J; P) - s(J; P) < t. Mas, sendo esse o caso, segue dos cálculos
acima que S(J +; P)-s(J +; P) < t. Portanto, novamente pelo critério de Cauchy, f + é integrável.

EXEMPLO 4.14.
Dada uma função f: [a, b] ➔ R, pode muito bem ocorrer que 1/1seja integrável sem que/
o seja. O exemplo clássico é fornecido pela função/: (O,l] ➔ R, tal que

f(x) ={ -1, se x ~Q .
1, se X E Q

Um argumento análogo ao do exemplo 3 garante que/ não é integrável. Por outro lado, 1/1é a
função constante e igual a 1, logo, integrável.

A primeira e a última igualdades em (4.16) deixam claro o porquê da validade da desigualdade


triangular para integrais: a primeira igualdade garante que J:f(x)dx pode ser calculada como
a diferença entre as âreas de regiões 'R+ e n_ do plano cartesiano, onde 'R+ está situada abaixo
do gráfico de f e acima do eixo das abscissas, enquanto n_ está situada acima do gráfico de /
e abaixo do eixo das abscissas; por outro lado, a última igualdade garante que J: lf(x)ldx é a
soma das âreas de 'R+ e n_.
osso próximo resultado garante que a integral também é aditiva em relação aos domínios
[a, e] e [e, b] de duas funções integráveis. Novamente, se raciocinarmos heuristicamente com
uma função não negativa / : [a, b] ➔ R, tal que suas restrições aos intervalos [a, e] e (e,b]
sejam integráveis, concluímos que esse resultado é bastante plausível: sendo n, 'Rl(a,c) e 'Rllb,c)
as regiões respectivamente sob os gráficos de / e de suas restrições aos intervalos [a, e] e [e, b],
temos n = nl[a,c) u nl(b,c]) com 'Rl[a,c] e nl[b,c) sem pontos interiores comuns; portanto, é de se
esperar que tenhamos A('Tl) = A('Tll[a,c]) + A('Tl1[b,c])-

235
<. \l'l 11111 I .-\ )\ 1 F<;f<.\L rn-: HIE\I.\\\

PROPOSIÇÃO 4.15.
Scjnm dados uma função f : [a,b] -t IR e um real e E (a, b). Se as restrições de f aos
intervalos [a, e) e [e, b) são funções integráveis, então f também é integrável, com

t J(.r)dT ~[ J(x)dx + l J(x)dx.


--~~---~~-~

PROVA.
Denote por f1(a,c) e f1[c,b)as restrições de f aos intervalos [a, e) e [e,b), respectivamente. Dado
€ > O, as integrabilidades de !l[a,c) e Íl(c,b) garantem, por intermédio do critério de Cauchy, a

existência de partições P de [a, e) e Q de [e, b) tais que


€ €
S(fl[a,c);P) - s(f1[a,c];P) <
2 e S(fl[c,b);Q) - s(fl[c,b);Q) <
2.
Se R = P U Q, então Ré uma partição de [a, b] e é imediato que

S(J; R) = S(f11a,c); P) + S(f11c,bJ; P) + s(fl[c,bJ;Q).


Q) e s(f; R) = s(f11a,cJ;
Portanto, •

S(f; R) - s(f; R) = S(fl[a,c);P) - s(fl[a.c);P) + S(fl[c,b);Q) - s(fl[c,b);Q) < €,

de forma que, invocando uma vez mais o critério de Cauchy, concluímos pela integrabilidade de

f.
Para o que falta, seja ó. = J:J(x)dx - J:f(x)dx - fcb J(x)dx. as notações da discussão
acima, temos

t,. < S(J; R) - [ J(x)dx - l J(x)dx

= S(111.,,1;P) + S(111,,,1;
Q)- [ f(x)dx - l J(x)dx

< S(fl[a,c]; P) + S(fl[c,b];Q) - s(fl[a,c];P) - s(fl[c,b);Q)


= S(/l[a,c]; P) - s(f11a,c];
P) + S(fl[c,b);Q) - s(fl[c,b];Q) < e;
analogamente,

t,. > s(J; R) - [ f(x)dx - l J(x)dx

> s(fl[a,c]; P) + s(f1(c,b); Q) - S(/l[a,c]; P) - S(fl[c,b]; Q)


1 = -(S(fl[a,c]; P) - s(f/[a,c]; Q) - s(fl[c,b];Q)) >
P)) - (S(f1[c,b]; -€.

Logo, 16.I< t:. Por fim, como t: > O foi escolhido arbitrariamente, concluímos que 6. = O.

236
• ão ant ri permi irá ar m mpl d funções
in gráv i on udo. an d faz· lo. prtÃ.i.:><1ll.lO
do ruint r ult do uxiliar.

LE 1A .16.
f. g: [a. b) ➔ - funçõ ais que f • int grá\ l f = g em (a, b) então g é integral
J:f(x)d;1, = J:g(x)dx.
pq \' .
ini i lm n o e o m qu f = g m (a b). r m t ndo o l itor ao probl ma
3 p r g ral.
P - {a = Xo < 1 < ··· < x = b} • um p tiç- d {a, b) in id"n ia d f g m
[:.r.ox -1J f rn

1 (/: P) - (g· P)I = 1 ( inf f - inf g) (x1 - x1 _i)


:r,-1 ,1 (:r,-1 z

= ( (:rinf
-• 1
J- inf
(:r -a.:r
g)(.rk - ·k 1)

< IJ u - (b)I (b - , _i);


m nt
( f· )- (g· P)I < IJ(b) - g(b)l(b - XJ:_i).

lll t ric ll lcr u

(q P) - (g· P) - ( (g· P) - (/' P)) + ( (/, P) - (f; P))


( (f • P) - (g; P))
( (J;P)- (J-P))
1 (J· P) - (g. P)I
~ -lf(b) - g(b)l(b - X _i) (S(J- P) - (J- P)).

d > O in bilid d d f garan , p lo ri 'rio d au h a xi t"ncia d uma


(J- P) < ½ (r finando P n e ário) b - Xk-1 < (IJ(b)-~(b)l+i).
m um t d ulo cima qu

(g, P) - . (g; P) < 2lf(b) - g(b)l(b - Xk i) + (S(J· P) - (J- P))



< 2 lf (b) - g(b)I • 4{lf (b) - g(b)I + 1) + 2 < €.
L , inv do um v z mai o ri 'rio d Cauch oncluímo qu g • int gráv 1.

237
< \1'1111(1 1 .\ J'..:ILCH\l.1>1-. Htl-:\1.\.'\\

Para a igualdade das integrais de J e de g, note que (novamente pelos cálculos acima)

l J(x)dx < S(J; P) = (S(J; P) - S(g; P)) + S(g; P)

< IS(J; P) - S(g; P)I + S(g; P)


< IJ(b) - g(b)l(b - Xk-1) + S(g; P).
Portanto,

t f(x)dx - l g(x)dx < IJ(b) - g(b)l(b- Xk-J) + S(g; P) - l g(x)dx.

Escolhendo a partição P de forma tal que b- xk-l < 2(1f(b)-Eg(b}l+l} e S(g; P) - J:g(x)dx <½
(esta última escolha sendo possível graças ao critério de Cauchy), obtemos

[ f(x)dx - [ g(x)dx < IJ(b) - g(b)l(b - x,_i) + S(g; P) -


f
l f
g(x)dx

< IJ(b) - g(b)I. 2(1/(b) - g(b)I + 1) + 2 < f.


Mas, como f > O foi escolhido arbitrariamente, segue daí que

[ J(x)dx ~[ g(x)dx < O.

Por fim, trocando os papéis de / e g no raciocínio acima (o que é perfeitamente lícito, uma
vez que já estabelecemos a integrabilidade de g), obtemos a desigualdade contrária à última
desigualdade acima, o que demonstra a igualdade das integrais.

Precisamos, agora, da definição a seguir.

PBmUÇÃO4.17.
Umá função / : [a, b] ➔ R é seccionalmente continua se existem a = x 0 < x 1 < • • • <
a:,•b tail,,que / 6 contínuano intervalo(x;- 1 , x;}, para 1 < j < k, e existem os limites laterais
1.._, t.+ /(s), Hm11 •b-- /(x) e limz-+z,± /(x}, para 1 < j < k.

A Figura 4.1 esboça uma função seccionalmente contínua f : (a,b] ➔ IR,a qual é descontínua
em exatamente três pontos.

238
y

a b X

Figura 4.1: uma função seccional.mente contínua / : [a, b] ➔ IR.

PROPOSIÇÃO 4.18.
Se f . [a, b] ➔ IR é uma função seccionalmente contínua, então / é integrável, com

(' k rr,
Ía f(x)dx = L, Ír f(x)dx. (4.17)
o J•I r,-1

PHO\ \
Se Íi: [.r1 -1ix,) ➔ Ré tal que J, = J em (x,-1,x,) e Jj(x 1 _i) = limx--+x,1 +f(x), J1 (x1 ) =
lim~-+,,- /(.t), então J, e contínua, logo intcgrAvel. Agora, o resultado do lema 16 garante que
n rest11ção de / ao inter\"alo [:r,-1, x,l também é integrável, com fx'
:r,-1
f(x)dx = J:rj-1
rx, J1 (x)dx.
Po1 fim, repetidas aplicações da propo ição 15 garantem que J ê integrável em [a, b], e que vale
(4.17).

Para o exemplo a ~eguir, o leitor pode acr~..útil rever o conceito de função característica de
um conjunto, à definição 10.

EXEMPLO 4.19.
Sejam a = x0 < x 1 < • • • < x,. = b uma partição do intervalo [a, b], e J : (a, b) ➔ R •
definida por
1c
/(x) = L CjX[:r,_ 1 ,z,],
J=l

239
\ 1 1 '
1
\ 1 '1 1: 1 1 1
\

R, para 1 < j < k. Uma vez que fé claramente seccionalmente contínua, a proposição
,prante sua integrabilidade, com
b k rx 1

J.J(x)dx L lx
a
=
J=l x 1 -1
f(x)dx.

outro lado,o resultado do problema 1.2, página 221, fornece

Por fim, combinando as duas igualdades acima, obtemos

O próximo resultado, o qual estabelece a recíproca da propo 1ção 15, será de crucial impor
tância para o cálculo efeti'~o de integrais, na próxima s ção.

PROPOSIÇÃO 4.20.
Se f : (a, b) ➔ R é uma função integrável e a < e < b, então as restrições de f aos intervalos
(a, e) e [e, b)(as quais serão denotadas simpl~mente por f) também são integrávei5, com

J.'f(x)dx= /.e f(x)dx+ 1• J(x)dx. (4.18)

PROVA.

É suficiente estabelecermos a integrabilidade das restrições de f aos intervalos [a,e) e (e.b).


Realmente, uma vez feito isso, a igualdade do enunciado seguirá da proposição 15.
Para o que falta, dado f > O, tome, pelo critério de Cauchy, uma partição R do intervalo
(t&,b) tal que S(f; R) - s(J; R) < f. Se R' = R U {e}, segue do lema 1 que S(J; R') - s(f; R') <
(/; ti) - s(f; R) < f; portanto, podemos supor de início que e E R.
J>.eaotepor a restrição de J ao intenralo (a, e). Se R ={a= Xo < x 1 < • • • < x, =e<
/l[a,cl
• • b e P ={a= x 0 < x1 < · · · < x, = e}, então Pé uma partição de [a, e). tal que

240
l
S(fl[a.cJ; P) - s(fl[a,ci:P) = L/AI 1 (f) - m 1 (f))(xj - X 1-1)
;=1
k
< L(Al 1 (f) - m3 (J))(x 1 - x1 _ 1 )
J=l

= S(f; R) - s(f; R) < €.

Assim, novamente pelo critério de Cauchy, Íl[a,c) é integrável.


Analogamente, Íl[c,b] é integrável.

O último resultado desta seção mostra que o produto de duas funções integráveis é também
integrável. Entretanto, conforme mostra o problema 3.10, o valor da integral do produto das
funções não guarda uma relação simples com os valores das integrais dos fatores. (Veja, contudo,
o problema 3.12.)

PROPOSIÇÃO 4.21.
Se/, g: [a, b] -+ R são funções integráveis, então f g: [a, b]-+ R também é integrável.

PRO\'A.
Suponha que já mostramos que o quadrado de uma função integrável é integrável. Então,
(J +g) 1 e g2 são integráveis, de forma que, como f g = ½((f+g)
2

,
2

f -g repetidas aplicações
2

-
2 2

),

da proposição 12, juntamente com a observação que a sucede, garantem a integrabilidade de f g.


Para o que falta, mostremos que / 2 é integrável. Para tal fim, observe primeiro que, como
f é limitada, existe uma constante real e tal yu /+e > O em [a, b]. Veja e como uma função
constante em [a, b). Como 12 = (I + c)2 - 2cf - c2,se (f + c)2 for integrável, o mesmo sucederá
com J2 (novamente por repetidas aplicações da proposição 12 e da observação a ela posterior).
Resta, pois, mostrarmos que, se f é integrável e não negativa, então 1 também é integrável. 2

Para tanto, note inicialmente que 12 é certamente limitada. Agora, dada uma partição P =
{a= x 0 < x 1 < · · · < xk = b} de [a, b], segue do problema 1.12, página 75, que

sup f 2 = ( sup /) 2 = A1J


lxi-1,xil l:rJ-1,:rJ)

241
( \1111 111 1 .\ hllC:H\I. )))·: HIF\I\\:\:

e, analoganPntc, inf1.r,_1 ,xil J2 = m;. Port.anto,


k
2
S(f 2
; P) - s(/ ; P) = L}M} - m;)(xi - x 1 -1)
J=l
k
= :~:::)M
+ )(M 1 m1 1 - m 1)(xi - x 1-1)

k
< 2 sup f · I)M 1 - m1)(x1 - x1_i)
(a,b) J=l

= 2sup/ • (S(J;P)- s(f;P)).


(a,b]

Invocando o critério de Cauchy uma vez mais, dado f > O e graças à. integrabilidade de f,
podemos supor que S(f; P) - s(J; P) < 2(sup /b /)+l. Logo, os cálculos acima garantem que
1 1

2 2
S(/ ; P) - s(/ ; P) < 2(sup /) • 2( € /) < f.
(a,b) SUP[a,b) +1
Portanto, também pelo critério de Cauchy, / 2 é integrável.

Problemás - Seção 4.3


3.1. Dê exemplo de uma função derivável f: [a, b] ➔ JR,tal que f' não é limitada (e, portanto,
não é integrável).

3.2. Seja f : [a, b] ➔ 1Ruma função contínua, com um número finito de zeros no intervalo [a, b],
digamos x 1 < x 2 < · · · < xk. Se

Rj = {(x, y) 2
E 1R ; a < x < b e O < y < f (x)}
e

prove que

J.
a
b f(x)dx = A(Rj) - A(R 1).

Para o próximoproblema, dadas funções contínuas f, g : [a, b] ➔ JR,tais que g(x) < J(x)
para todo x E (a, b],seja

n,(/ -= {(.r, y) E IR2 ; Q <X< bC g(x) < y < f(x)}

242
a região do plano situada entre os gráficos de f e g. Definimos a área ele Riu por

A(R1,) = J.'u<x) - g(x))dx.

3.3. Prove o princípio de Cavalieri: parai= 1, 2, sejam fi, 9i : [a,b] ➔ IR funções contínuas,
tais que g,(x) < !t(x) para todo x E [a,b]. Se, para todo x E [a,b], o segmento que une
os pontos (x, /1 (x)) e (x, g 1 (x)) tem comprimento igual ao segmento que une os pontos
(x, h(x)) e (x, g2(x)), então A(R1i9 J = A(R1292 ).

3.4. Seja L·J: 1R➔ 1Ra função parte inteira (cf. problema 1.9, página 23). Para n EN, calcule
lonlxJdx.

3.5. Seja{-}: IR ➔ IR a função parte fracionária (cf. problema 1.10, página 23). Para n EN,
faça os seguintes itens:

(a) Mostre que 1:+1{x}dx 1


= l 0 {x}dx.
(b) Calcule lon{ x}dx.

3.6. * Sejam J,g: [a,b] ➔ IR funções contínuas, tais que J(x) < g(x) para todo x E [a,b]. Se
1:J(x)dx = 1:g(x)dx, prove que J = g.
3.7. Seja f : [a, b] ➔ IR uma função integrável. Admitindo que 1/1também
é integrável, use o
fato de que -IJ(x)I < J(x) < lf(x)I, para todo x E [a,b), para deduzir (4.15).

3.8. * Complete a demonstração do lema 16, analisando o caso em que f = g em (a, b).

3.9. Dados O < a < b e n E N, calcule 1:xndx. Em seguida, calcule 1:f(x)dx, onde f(x) =
E7=o a1x 1 , com ao, a1, ... , an E IR e an =f O.

3.10. Dê exemplo de funções contínuas e não negativas /, g; [a,b] ➔ IR tais que 1:J(x)dx > O,
1:g(x)dx > O mas 1:J(x)g(x)dx = O.

3.11. * Seja f : [a, b] ➔ IR uma função integrí...":. Para O < € < b;a, mostre que

b-< 1b
lim
<➔O J.
+t
J(x)dx =
a
f (x)dx.

3.12. * Prove a desigualdade de Cauchy para integrais: se /, g : [a, b] ➔ R são funções


integráveis, então

J. b
J(x)g(x)dx <
(
J.b 2 )
f(x) dx
1/2 (
J. b
g(x) 2dl·
) 1/2

243
f e g são contínuas mostre qu a igualdad ocorre se ó E
tal que J(x) = Àg(x), para todo x E [a b].

3.13. * S J : [a, bl ~ :R é uma função contínua e convexa 1 prove que

J(a): J(b) < b 1 a t J(x)dx.

244
4.4 O teorema fundamental do Cálculo
O propósito principal desta seção é tomar rigoroso o argumento da introdução que levou a
(11). página 7. Ao longo da mesma, salvo menção E!Àrplícita em contrário, denotamos por / um
intervalo da reta e por/: / ➔ Ruma função integrável em todo intervalo [a,b] e/.
Para e E / e [a,b] e I. definimos

[ f(x)dx = O e [ f(x)dx = - [ f(x)dx.

A partir daí, utilizando a segunda parte da proposição 20. é imediato verificar que

t f(x)dx = [ f(x)dx + l f(x)dx, (4.19)

para todos a, b, e E / {1.e.,não necec..ariamente tru que a < b e e E (a, b)).


Isto posto, temos a seguinte defimção. central para tudo o que segue.

ÜEFINlÇÃO 4.22.
SeJam I C IR um intervalo e / : / ➔ uma funçüo integrável em cada intervalo [a, b) e /.
[·1xado e E /. a integral indefinida dt."'/ baseada cm e é a função F : I ➔ R definida por

F(x) = 1:r f(t)dt.

Se f, F: / ➔ IR ão como na definição anterior, segue prontamente de (4.19) que

F(b) - F(a) = l f(t)dt - [ f(t)dt = [ f(t)dt, (4.20)

para todo a, b E /.
Agora, podemos enunciar e provar o result"-4') principal desta seção, o qual se constitui em um
do~ mah, importantes resultados básicos do Cálculo e é conhecido na literatura como o teorema
fundaiuental do Cálculo. (Por vezes. abreviado TFC).

TEOREMA 4.23. TFC


Sejam / e JRum intervalo, f : I ➔ R uma função integrável em todo intervaló J
F : / ➔ 1Ra integral indefinida de / baseada em e E /. Se / é contínua em Xo E l
derivável em x 0 , com F'(xo) = J(xo)-

245
<. \1'111 1< > I :\ (\; ru;H.\L DE HtE:\I.\\\

PROV\.
Fixe .t E J\ {x0}. Aplicando sucessivamente (4.20), o resultado do problema 1.2, página 221,
a aditividade da integral e a desigualdade triangular para integrais, obtemos

F(x) - F(x 0) _ J(xo) 1 !.X


X -Xo
- x - Xo xo J(t)dt - J(xo)

1 !.X
-o (J(t) - J(x 0 ))dt
lx - xol ~

< lx ~ xol { lf(t) - f(xo)ldt •

(Na última igualdade acima, o módulo fora da integral deve-se ao fato de que, se x < x 0 , então
fx:lf(t)
- f(xo)ldt = - J:ºIJ(t) - f(xo)ldt.)
A continuidade de f em x 0 garante que, para E > O dado, existe ô > O tal que

t E J, lt - xol <ô ⇒ IJ(t) - J(xo)I < E.

Portanto, .para O < lx - x0 1 < ô, temos lt - x0 1 < ô para todo t pertencente ao intervalo de
extremidades x 0 ex, de modo que IJ(t)- f(xo)I < E. Segue, pois, da monotonicidade da integral
e novamente do resultado do problema 1.2, página 221, que

J.
x lf(t) - J(xo)ldt < J.:r Edt = Ejx - xol•
XQ Xo

Por fim, os cálculos acima garantem que, para x E J tal que O< lx - x 0 1 < ô, temos

F(x) - F(x 0 ) 1
- f (xo) < IX I • Ejx - xol = E.
X - Xo - Xo

Logo, Fé derivável em x0 , com F'(xo) = J(xo).

O TFC é a peça que faltava para podermos calcular efetivamente e de maneira simples as
integrais de várias funções usuais. Antes de fazê-lo, contudo, precisamos de urna definição.

l)JWINlÇÃO4. 24.
Sejasn J C Rum intervalo e/ : /--+Ruma função integrável em todo intervalo [a,b] e I.
IYa para/ em/ é uma função derivável F: /--+ R, tal que F' = f em I.

Em termos da definição anterior, o TFC garante que toda integral indefinida de uma função
contínua f: l--+ Ré uma primitiva de f em I. De fato, sendo F : J--+ R a integral indefinida

246
de F baseada em e E I, segue do TFC e da continuidade de J em I que F' (x) = J(x), para todo
x E/. De outra forma, temos que:

f contínua =}
d 1xe J(t)dt
dx = J(x). (4.21)

O próximo resultado garante que, mesmo para uma função meramente integrável, não há
outras primitivas possíveis para f. De outra forma, ele garante que, para/ meramente integrável,
não há outras soluções possíveis para (4.24).

TEOREMA 4.25.
Sejam I e 1Rum intervalo e f : I ➔ 1Ruma função integrável em todo intervalo [a, b) C J.
Se F : I ➔ 1Ré urna primitiva de f, então, fixado x 0 E I, temos

F(x) = F(x 0 ) + fx J(t)dt, (4.22}


Íxo

para todo x E/.

PROVA.
Suponha x > x 0 ( o caso x = x 0 é trivial e o caso x < x 0 pode ser tratado de modo análogo,
levando-se em conta que fx: J(t)dt = - J:ºJ(t)dt).
Seja P = {x 0 < x 1 < · · · < xk = x} urna partição de (x0 , x]. Como F é derivável em I, F é
contínua em cada intervalo [a, b] e I. Portanto, pelo TVM de Lagrange, existe E,3 E (x3 _ 1 , x 1 )
tal que
F(x 1 ) - F(x 3 _i) = F'(E,1 )(x 1 - x3 -1) = J(E,i)(xi - Xj-1),
para 1 < j < k. Segue, pois, que
k
F(x) - F(xo) = I:)F(x 3) - F(x;-1))
J=l
k
= L f(E,1 )(x 1 - x 1 -1)
i=l
= E(J; P; E,).
Recorde agora que, de acordo com o teorema de Riemann 7, temos

fx J(t)dt = lim E(f; P; E,),


Íxo IPl-+0
para todo pontilhamento ç de P. Portanto, em face aos cálculos do parágrafo anterior, não há
outra alternativa que não termos a igualdade do enunciado.

247
( \ 1•1 1 1 1 1 i 1 :\ 1\ 1 Ee; H. \ L 1>E H 11-:
:\ 1.\ \' \'

l' umn função f : 1 -+ IR tem primitiva, então, graças ao teorema anterior, é costume
J
dcnot.u· umn primitiva genérica de J escrevendo J(t)dt. Bem entendido, o leitor deve ficar
atento pnrn não confundir as notações f J(t)dt e J:J(l)dt: enquanto a primeira notação refere-
se a uma. função de J em IR cuja derivada coincide com f em /, a segunda denota um número
1'!!.al. Observe, ainda, que o teorema anterior garante que, se J
J(t)dt é uma primitiva de f em
J, então as primitivas de f em I são as funções da forma

J f(t)dt + C,

onde C é uma constante real.


De posse do teorema anterior, o corolário a seguir garante que, se f : [a,b] -+ IR é uma
função integrável, então, para calcular efetivamente a integral J:J(t)dt, é suficiente encontrar
uma primitiva de f. Doravante, dada uma função contínua F : [a,b] -+ IR, denotamos

F(x)i::: = F(b) - F(a).

COROLÁRIO ·4.26.
Se / : [a, b) ➔ R é uma •função integrável e F : [a, b] -+ IR é uma primitiva de f, então

1
b • lx-b
/ (t )dt = F (X) :r=a. (4.23)
0

PROVA.
Faça x 0 = a ex= bem (4.22).

Conforme antecipamos, o corolário anterior fornece uma estratégia geral para o cálculo do
valor de J:J(t)dt:
basta conseguirmos visualizar o integrando f como a derivada de uma função
F, aplicando (4.23) em seguida, para obter J:J(t)dt = F(b) - F(a). Por isso, é costume
escrevermos (4.23) como
b l:r-b

J.ª F' (t)dl = F(x) :r=a.

Vejamos alguns exemplos.

➔ 1 tal que f(x) = E.i=Oa1 xi


para todo x E IR, onde a 0 , a 1 , ... , ª" são números
O, Como F(x) = E;=o i~l x 1 +1 é claramente uma primitiva de f, temos,

 2,18
para reais a < b, que
.
rb f(t)dt = F(x)1.r=b=
}0 x:a
t
i=O
~(b1+l
J +1
- aj+l).

EXEMPLO 4.28.
Calcule J01r sen x dx, f 011' sen2 x dx e f 011' cos2 x dx.

SOLUÇÃO.
Como a função - cos é uma primitiva da função sen, segue do corolârio anterior que

1O
1r
sen x dx = - cos x
1x-7r
x=O
= - cos 1r + cos O = 2.

Quanto à segunda integral, como sen2 x = ½(1 - cos 2x) e fx sen 2x = 2 cos 2x, temos, novamente
pelo corolârio anterior, que

71'
sen2 x dx = 1 /.71' (1- cos 2x )dx = 1 ( x - 1 sen 2x ) 1X=71'
x=O= 1í
.
/. 0 2 0 2 2 2
Por fim, como sen2 x + cos2 x = 1, temos

/.71'cos2 xdx = J.1r(1 - sen2 x)dx = J.1r 1 dx - 1.11' sen2 xdx

= x1x=1r-111' sen2 x dx = 1r - 1r = 1r.


x=O o 2 2

Uma outra maneira de refrasear (4.21), a qual é de grande importância para a teoria de
EDOs, é colecionada na proposição a seguir.

PROPOSIÇÃO 4.29.
Sejam / e 1Rum intervalo e f : / --+ 1Ruma função contínua.
problema de valor inicial
y' = J(x)
{ y(xo) = Yo

tem como única solução a função F : J --+1Rdada por

249
\ I'\ 11 (,1: \I Ili 1:11 '1 \'

RO\'.
m luç~ d ( .2 ) ' um fun - d riv 1 F : I ➔ ai qu F'( ) = f(x) para
tod x E / F(. 0 ) = y0 . orno f , on ínu ( .21) g ran qu a funç~o F d finid m
(4.2 ) uma oluç~o d (4.24). r u r 1 do F 1 F2 : I ➔ ( .2 ), n o
F{ = f = F~• lo o o pr bl m .1 p gin 16 g r n qu F 1 - F2 , on an mo
(F1 - F2)(.ro) = Yo- Yo= O n luím u F1 - F2 = O, i .. F1 = F2.

proposição anterior nos permi e n ontrar a quaç ~ o horári da po ição d um obj to


pon ual em movimento retilíneo uniform m nt variado d man ir mai dir ta do qu
utiliAda no exemplo 76. Para tanto uponha qu o obj t p rt d p i ~o ini ial . 0 n in • ant
t = O com velocidade inicial v0 . jam, ainda a o v lor d ua e 1 r ç~o t H (t) t H v(t),
respectivamente, as equações horárias d ua po iç- v lo id orno t H (t) r olv o

problema de valor inicial { y' = ª gu d ( .25) qu


• y{O) = vo

1
t , -t
v(t) = Vo+ ad •= vo + a = vo + at.
O • =0

Agora, como t ....+x(t) resolve o problema d valor ini ial { y' = (t) apli ando ( .25) nova-
y(O) = xo
mente, obtemos

O mais das vez , a ar fa d n ontr r um pnm1 1 uma função in


par áv 1 d d
(quando possível) n m mpr im i ta. r1 n d ál ulo, on id ráv 1 quan i-
de mpo é devotada d nv lvim n d p ili 1 ul d in gr i ,
conh idas g n ri m n d mt gração. prop lÇ lf

to do TF , traz a qu z ma1 ú il d d fó mul d

5
Oll<>Hl\l\ll'\l>\\ll'\I\IIH>('\1<11<> 1 1

PROPOSIÇÃO 4.31.
Se f, g : [a, b] ➔ R são funções deriva.veise com derivadas integráveis, então

[ J'(x)g(x)dx = f(x)g(x{:: - t f(x)g'(x)dx.


-~-------------'
(4.26)

PROVA.
Primeiramente, observe que as integrais em ambos os membros de (4.26) estão bem defini-
das. Realmente, as funções f e g, sendo deriváveis, são contínuas; portanto, como f' e g' são
integráveis, a proposição 21 garante que f' g e f g' também são integráveis.
Agora, como (/ g )' = f' g + f g', segue do corolário 26 que

[u'(x)g(x) + f(x)g'(x))dx = l(fg)'(x)dx = f(x)g(x)[:.

Por fim, para obter (4.26), basta aplicar a aditividade da integral ao primeiro membro da última
igualdade acima.

EXEMPLO 4.32.
Calcule as integrais f 0,r x sen x dx e f 0,r x2 cos x dx.

SOLUÇÃO.
Para a primeira integral, fazendo f(x) = -cosx e g(x) = x em (4.26), obtemos

,r 1,rx(-cos'x)dx 1x=1r 1,r


1 O
xsenxdx =
O
= -xcosx
x=O
- (-cosx)dx
O

= 1r +
1
11"

o
cos x dx :::;:1r + sen x
lx-,r
x=O
= 1r.

Para a segunda integral, fazendo f (x) = sen x e g(x) = x 2 em (4.26), obtemos


f1r x 2 cos x dx = 11r x 2 sen 'x dx = x2 sen x Ix=1r -11r 2x sen x dx
lo o x=O o
= -2111" xsenxdx = -21r,

onde utilizamos o resultado da primeira parte na última igualdade acima.

251
Observe que, em termos de primitivas, a fórmula de integração por partes garante que

j J'(x)g(x)dx = J(x)g(x) - j f(x)g'(x)dx.


Em palavras, uma primitiva para f'g pode ser obtida subtraindo, de f g, uma primitiva de f g'.
O caso particular a seguir da fórmula de integração por partes é, por vezes, bastante útil.

COROLÁRIO4.33.
Se f: [a, b] ➔ Ré uma função derivável e com derivada integrável, então

!.
a
b
f (X)dx = XJ (X) 1a
b
-
!.b
a XJ' (X)d:r. ( 1 27)

PROVA.

Trocando f por g em (4.26), obtemos


'

1 a
b
J(x)g'(x)dx = J(x)g(x)j
b
0
-
1b
ª J'(x)g(x)dx.

Basta, agora, fazer g(x) = x para todo x E [a, b].

A seguir, vejamos um exemplo numérico de aplicação da fórmula de integração por partes, o


qual encontrará útilidade na seção 5.2.

34.
e ln= f0w/2(cosx)"dx, mostre que:

• (n .L- l)ln-2, para todo inteiro n > 2.


1 1
• , para todo m > o, e I 2k-1 = <22k(kk!)
• k _
• t e1ro
2
d • • k> 1
2 para to o mte1ro • _ .
k)',

252
PRO\".\.
(a) Para n > 2, segue da fórmula de integração por partes que

1
1r/2 J.1r/2
ln = O (cosxr- 1 cosxdx = O (cosx)11-1sen'xdx

1 7r /2 /.7r/2
= (cosxr- senx
l O - O (n- l)(cosxr- 2
(-senx)senxdx
1r/2
= (n - 1)
J. 0
/2
7r
(cosxt- 2
(1 - cos2 x)dx

/.7r /2
= (n - 1) (cosxt- 2
dx - (n - 1) cos" xdx
/. 0 0
= (n - l)In-2 - (n - l)In.
Portanto, nln = (n - l)In-2·

{b) É imediato que 10 = f 01r/2 dx = xi::: = ~- Por hipótese de indução, suponha que I2m =
·~,para
(2<~:),~2 algum inteiro m > O. Fazendo n = 2m+2 na recorrência do item (a) e utilizando
a hipótese de indução, obtemos
_ 2m + 1 . _ 2m + 1 {2m)! 1r
12m+2 - 2m + 2
12
m - 2m + 2. (2mm!)2 2
(2m + 2){2m + 1) (2m)! 1r
(2(m + 1))2 (2mm!)2 2
(2m + 2)! 1r
-
{2m+1(m+ 1)!)2 2 •
Portanto, a primeira parte de (e) é vâlida para todo k > O.
Por fim, a prova da validade da segunda parte é totalmente análoga, e será deixada como
exercício para o leitor.

Voltando ao desenvolvimento da teoria, apres ntamos, no próximo resultado, o teorema de


mudança de variâveis. Conforme veremos em seguida, ele se constitui em outra técnica de
integração bastante útil.

TEOREMA 4.35.
Seja / : [a, b] ➔ R uma função contínua. Se g : [e, d] ➔ (a, b) é derivê.vel,
integrável, então
g(d) /.d
J(t)dt = J(g(s))g'(s)ds.
/. g(c) e

253
< \1 1: 1 1,, 1 .\ 1\ 11-:c;H \1. l>I·: H11·:\1 \\\

PHO\'\.
Pnra calcular a integral do primeiro membro, comecemos tomando uma primitiva F : [a, b] -+
R para J, a qual existe, pelo TFC. Então, graças ao corolário 26, obtemos

g(d) jg(d) 1x=g(d)

J.
g(c)
f(t)dt =
g(c)
F'(t)dt = F(x)
x=g(c)

= F(g(d)) - F(g(c)) = (F o g){s)i:::.


Agora, como F e g são deriváveis, a regra da cadeia garante que a função F o g : [e,d] -+ IR
também é derivável, com

(F o g)'(s) = F'(g(s))g'(s) = f (g(s))g'(s) = (J o g)(s)g'(s).


Por outro lado, como f e g são contínuas, temos f o g contínua, logo, integrável; mas, como g' é
integrável, segue da proposição 21 que (J o g)g' integrável. Portanto, aplicando mais uma vez o
corolârio 26, podemos escrever

(F o :)(s)
I
s-d
s=c = lde (F o g)'(s)ds = lde f(g(s))g'(s)ds.

OBSBRVAÇÕES 4.36.
Ainda em relação ao teorema de mudança de variáveis, temos:

(i) A derivada g' de uma função derivável g : [a, b] -+ 1Rpode não ser integrável. Para um
exemplo, veja o problema 3.1, página 242.

(li) Para uma variante do teorema anterior, aplicável ao caso em que a função f ê seccional-
~ c.outfnua,veja o problema 4.15.

O corolário a seguir traz um caso particular bastante simples, mas muito útil, da fórmula de
integração por substituição.

uma função contínua e À E JR•,então

ds = ~ f>.b J(t) dt.


f.
b j(Ãs)
a ___ AJ>.a ------------~
(4.29)

254
() 1 I· 1 li i 1 \ 1 \ 1 1 '\ 11 \ \ 11 '\ 1 \ 1 1><1 ( ' \ 1< 1 1 1 1 < , 1, ,

PROVA.
Para >. > O é suficiente, no teorema 35, tomar g [a, b] ➔ [>.r,,,
>.b]dada por g(s) - À-!1.
Realmente, com tal escolha, temos

>.b J.g(b) J.b


J(t) dt = J(t) dt = J(g(s))g'(s) ds = J.b J(>.s)>.ds.
1 >.a g(a) a a

Para >. < O, tome g : [a, b] ➔ [>.b,>.a) dada por g(s) = >.s. Então, argumentando como no
caso anterior e lembrando que Jbª = - J: e J:bª = - f>.~,obtemos

>.b 1>.a J.g(a)


f (t) dt = - f (t) dt = - J(t) dt
1 >.a >.b g(b)

= - [ f(g(s))g'(s)ds = l J(>.s)>.ds.

Por vezes, nos referiremos a (4.28) como a fórmula de integração por substituição,
nomenclatura que explicamos heuristicamente a seguir.
Para calcular a integral J:J(t)dt com o auxílio do TFC, para alguma função contínua
f : [a, b) ➔ lR, precisamos obter uma primitiva para F. Nesse sentido, após examinarmos
com cuidado a fórmula que define f(t), podemos eventualmente notar que ela resultará bem
mais simples se operarmos uma substituição de variável, trocando t por uma expressão g(s),
dependente de uma nova variável s. Mas, como f está definida no intervalo [a, b],para que uma
tal troca tenha sentido é necessário que g(s) pertença ao intervalo [a, b]quando s varia em algum
intervalo (e,d]; para identificar um tal intervalo [e,d), a melhor maneira é, via de regra, resolver
as inequações a < g(s) < b, descobrindo, assim, quais devem ser e e d para que elas tenham
sentido. Agora, derivando fonnalmente a igualdade t = g(s), obtemos dt = g'(s) ds, de forma
que a integral original deve ser corrigida com o fator g' (s), antes de ser transformada numa nova
integral.
Em resumo, executando as duas passagens descritas nas igualdades a seguir, obtemos a
fórmula de integração por substituição em cada caso específico com o qual nos defrontemos:

Por fim, observamos que, para que a última integral acima tenha sentido, a função 8 ~
f(g(s))g'(s) deve ser integrável, de forma que (como vimos na demonstração do teorema) é
natural supormos que g é derivável em [e,d), com derivada integrável nesse intm vnlo.

255
1 1i 111 1 ') 1 :\ I\' I 1-:(;I{,\[. IH: BIE\I \\'\:

A ' guir, ilustramos como a discussão informal dos dois parágrafos anteriores simplifica o
emprego do fórmula de integração por substituição.

E EMPLO 4.38.
Calcule J:f
tsen (t2) dt.

SOLUÇÃO.
Operando a substituição t = Js, obtemos

2,;; 2 /,? 1 l /,?


../ii t sen (t ) dt = ? Js(sen s) VSds = ? sen s ds,
/. 2 2
onde o fator de correção 2 Jsfoi obtido derivando formalmente a igualdade t = Js, para obter
dt = 2Jsds.
Para corrigir os extremos ..fii e 2..,/ii da integral, resolvemos as inequações ..fii < vs< 2..,/ii,
obtendo 1T < s < 41T. Portanto,

2../ii 1 1411' l s=4'11'


t sen (t 2) dt = - sen s ds = -( - cos s)
/.../ii 2 11' 2 8='11' I
= -1,

onde utilizamos o TFC na última igualdade.•

Ainda em relação ao exemplo anterior, observe que, à luz do teorema 35, todas as passagens
que executamos são lícitas. De fato, o que estamos realmente fazendo é usando a função g .
[7r,47r) ➔ [..fii,2..fii} tal que g(s) - vs,
e tal função é derivável e tem derivada g'(s) = 2 ~
integrâvel no intervalo [7r,47r].

SOLUÇÃO.
No exemplo 34, calculamos ln = f0'11'12(cosxtdx. Mostremos que f0'11'12(sen x)ndx = ln tam-
bém. Para tanto, recordando que cosx = sen (~ - x), e fazendo a substituição x = ~ - y (de
sorte que dx = ,-dy), obtemos

1(/2 /,? /,?


1. 0
(senxtdx =
1
(scn (7r/2 -y))'1(-l)dy = -
7
(cosytdy.

256
Os eÃ'iremos corretos da última integral podem ser obLidos facilmente, notando que O <
i - y ~ se, e só se, O y
< conLüdo, há que se Ler cuidado, uma vez que x =O=> y = J e
< < ~;

x = i =>y = O. Portanto, o correto é escrevermos

J. 1r /2
(senx)"dx = -
!.º (cosy)»dy =
J.1r
/2
(cosy)"dy = Ín.
O 1r/2 O

De um ponto de vista mais formal, observe que a substituição empregada no exemplo anterior
reduziu-se à utilização da função g: [O,i] ➔ [O,~] tal que g(x) = ~-x. Como g(O) = !, g(í) = O
e g'(x) = -x, temos
/2 !,º
J.
1r J.g(O)
(senx)"dx = (senx)"dx = (seng(x)tg'(x)dx
o g(i) i
= -1 o

2
(sen (1r/2 - x))"dx = J.~
2
(cosx)"dx.

Um dos mais importantes usos da fórmula de integração por substituição é relacionado a


integrações por substituição trigonométrica. Para entender do que se trata, suponha, por
exemplo, que tenhamos urna função contínua f : [-1, l] ➔ IR e queiramos calcular f~1 f(x)dx.
Então, lembrando que a restrição da função seno ao intervalo [- ~, ~] ainda é derivável, com
derivada integrável, e que sen (-~) = -1 e sen ~ = 1, segue de (4.28) (com g(t) = sen t) que
1
/_ f(x)dx = 1sen½ J(x) dx = 1! J(sen t) cos t dt.
-1 sen( - i) - J

Dependendo da expressão algébrica que define J, a última integral acima pode ser mais simples
de calcular do que a integral inicial.
A discussão acima encerra um sem-número de variações, as quais não podem ser listadas
uma a uma. Vejamos, contudo, um exemplo que mostra, em um caso específico relevante, como
utilizar uma substituição trigonométrica adequada para calcular uma integral.

EXEMPLO 4.40.
Calcule f01 1).r2 dx.-------
PROVA.
Se recordarmos (cf. exemplo 34) que!, arctgx = 1.;:i:2, podemos utilizar o TFC para obter
1 1 1::t=l 7r
dx = arctgx = arctg 1 = .
/. o 1+X 2
.t:=O 4

257
\ 1 1 1 ' '1: \ 1 111 1:11 ' 1\

Alternativamente, fazendo a substituição trigonométrica x = tg t e observando que O < x ~


1 * O < t < ¾,obtemos, a partir da fórmula de integração por substituição e do exemplo 33,
que
1

1 1
1+ x2
dx = 1¾ 1 +
O
1
tg 2t
• tg'tdt = 1¾ -\-
O sec t
·sec 2 tdt

= 1o
¾1 dt = t It= ¾ =
t=O
1r .
4

Sejam dados os intervalos I e J e as funções g : J ➔ J e f : J ➔ JR, sendo f contínua


e g continuamente derivável (cf. seção 3.2). É imediato verificar que (4.28) fornece a seguinte
igualdade entre primitivas:

(/ J(t)dt) (g(x)) = j J(g(x))g'(x)dx + C, (4.30)

onde C é wna constante real. Um caso específico relevante é isolado no exemplo a seguir.

dados um intervalo I, uma função continuamente derivável f : I ➔ (O,+oo) e um


• nulor. Como •
1
!1(xr • r f(xr- J'(x),

t é uma primitiva para a função x ....+ f(xy- 1


f'(x). De outro modo, temos

(4.31)

pode ser lembrada facilmente, a partir de (4.30) (com f no lugar de g e x M xr-I

• por um lado, o TFC dâ-nos

/(z) tr 1/(-z) 1
tr- 1 dt =-;: +e= -:;.f(xr + C;
/

de \'ari6.vel t = f (x) fornece

~~~;.c;ufmos'esta seção observando que, para além dos problemas reunidos a seguir, veremos
exemplos relevantes de integrações por substituição trigonométrica na próxima seção,
DOIproblem88 d88 seções 4.6 e 4.7.

258
PHOl\11 \I \'-- '"'' ( \() 1 1 1

Problemas - Seção 4.4


4.1. Em cada um dos itens abaixo, calcule a integral em questão:

(a) J;1r(1 - cos t) 2 dt. (d) J; J;+i dx.


(b) f 02 Jl -
1r cos t dt. 12
(e) J; Jt-:r;1 dx.
(c) f01 x sen( 1rx)dx. (f) f1../21.:x,dx.

4.2. Em cada um dos itens abaixo, calcule as primitivas pedidas, explicitando os intervalos em
que as mesmas têm sentido:

(a) J(sen x) 3 cos x dx. (d) Jx 5 Jl + x 3dx.


(b) Jx 2senxdx. (e) Jx3 Jl - x 2dx.
(c) Jx 2 Jl + x 3dx. (f) f cos VXdx.
4.3. Param, n E N, mostre que

1o
2
"'
sen(mx) sen(nx)dx = 1o
2
"'cos(mx) cos(nx)dx = { O1 se m =,fn
1r, sem= n

4.4. Em cada um dos itens a seguir, resolva o problema de valor inicial dado, no maior intervalo
I e 1Rpossível.

y' =
(a) { y(l)
vx
=2
(b) { y' = sec x
2

Y(l) = 1

4.5. Seja/: [-a, a) ➔ 1Ruma função contínua. Mostre que:

= { 2 J0ª J(x)dx, se f ê par


J_ª J(x)dx
-a O, se f é ímpar.

e f_2! cos(1rj)
1
l sen(1rr)d dX.
4 ,6 , Cal cule J -1 1+:rl X l+:r
2

4. 7. Use a fórmula de integração por substituição para reobter a equação de Torricelli (cf.
problema 8.1, página 211).

4.8. * Sejam g, h: [e, d) ➔ 1Rfunções deriváveis e/ : (a,b] ➔ 1Ruma função contínua. Mostre
que
d lh(:r)
- f (t)dt = f (h(x))h'(x) - f (g(x))g'(x).
dx g(x)

259
< \l'I 11 1 e, I :\ IYI EC:H.\I. DE H11-::\I.\\:\:

4.9. Seja J : IR.➔ IR.uma função contínua e periódica, de período p > O. Prove que J:+P J( t)dt =
JtJ(t)dt, para todo a E IR..
4.10. Seja f : [a, b1➔ IR.a restrição de uma função afim ao intervalo [a, b). Se f é não negativa,
a região Ri é um triângulo ou um trapézio. Mostre que, em um qualquer de tais casos, o
valor obtido para a área de R 1 ao utilizarmos o TFC coincide com o valor obtido mediante
o emprego das fórmulas usuais da Geometria Euclidiana.

4.11. Seja r um círculo de centro O e raio R, e escolha um sistema cartesiano de coordenadas


tal que 0(0, O). Se f: [-R, R) ➔ IR.é a função dada por J(x) = JR2 - x2 , então a região
R1 sob o gráfico de fé a porção do disco delimitado porre situada no semiplano superior
2
do plano cartesiano. A partir daí, conclua que A(r) = 1r R .

4.12. Generalize a fórmula para a área de um círculo, provando que a área de uma elipse de eixo
maior 2a e eixo menor 2b é igual a 1rab.

4.13. Seja f : (a, b) ➔ IR.uma função contínua, não negativa e crescente (resp. decrescenle).
Prove que toda integral indefinida de f (cf. definição 22) é estritamente convexa (rcsp.
estritamente côncava,).

4.14. Se f : [a, b) ➔ [a, b) é uma bijeção cr scente e derivável, calcule os possíveis valores de
J:(J(x) + 1-1 (x))dx.
4.15. Prove a seguinte versão do teorema 35: se f : [a, b) ➔ R é seccionalmente contínua e
g : [e, d) ➔ [a, b) é crescente (resp. decrescente) e derivável, com derivada integrável, então

g(d) 1d

J.
g(c)
J(t)dt =
e
f(g(s))g'(s)ds.

4.16. Prove o teorema do valor médio para integrais: dada uma função contínua f: [a, b) ➔
R, existe e E [a, b) tal que
1 J.b
b _ a ª J(x)dx = J(c).

4.17. O objetivo deste problema é utilizar o material desenvolvido nesta seção para estabelecer
a irracionalidade de 1r. Para tanto, faça os seguintes itens:
2
(a) Mostre que é suficiente estabelecer a irracionalidade de 1r .

(b) Fixado n EN, seja f(x) = ~1xn(1 - x)". Prove que:


i. A função J é da forma J(x) = ~'E!:nakxk, com ak E Z para n < k < 2n.

260
u. Se O < x < 1, então O< /(x) < -\.
n.
lll. Para todo k > 1, as k-ésimas derivadas /Ck)(Q)e JCk)(l) de / nos pontos O e 1
são números inteiros.
(e) Suponha que 1T
2
= ?, com a, b EN. Se

mostre que

d~ (g'(x) sen (üx) - üg(x) cos(1rx))= 1r2 a"f(x) sen (1rx).

(d) Conclua, a partir do item anterior, que

O< ,ra" J.'/(r) eu (,rx)dx = -g(O) - g(l) E Z.

(e) Por fim, obtenha uma contradição mo trnndo que, se n for escolhido inicialmente de
forma tal que 1ra"< n!l então

O< ,rn" J.'f(x) cu (,rx)dx < 1.

261
( \ 1 '1 1 1 1 1 1 1 .\ "' 1 1 ( • I{ \ 1. )) I·. H I F \ 1.\ \ \

4.5 Algumas aplicações à Geometria


Recordemos que nossa motivação inicial para o estudo do conceito de integral foi geométrica.
Em última análise, para uma função não negativa e integrável / : [a, b) ---+R, definimos a área
da região n sob seu gráfico como a integral de f, de a a b:

A('/?.)= [ J(x)dx

Nesta seção, colecionamos mais algumas aplicações do conceito de integral à Geometria. Iais
precisamente, abordamos os três problemas a seguir:

(I) como definir e calcular o comprimento do gráfico de uma função f : [a, b) ---+R;

(II) no caso em que fé positiva, como definir e calcular o volume do s6lido de revolução gerado
pela rotação da região sob o gráfico de J em torno do eixo das abscissas;

(III) ainda no caso em que fé positiva, como definir e calcular a ârea da superfície de revoluçao
gerada pela rotação do gráfico de f em torno do eixo das abscissas.

Comecemos analisando o problema (I), para o que consideramos uma função J : [a, b) ---+R,
contínua em [a, b] e continuamente deriváv~l em (a, b). Fixe (cf. Figura 4.2) um intervalo
[e, d) e (a, b), uma partição P = {e= x 0 < x 1 < · · · < xk = d} de [e, d) e denote ó.x 1 = x 1 -x 1 -1,
para 1 < j < k.

...

e= x 0

Figura 4.2: aproximando o comprimento de um grafico.

Para IPI suficientemente pequena, é razoável supor que o segmento que une os pontos
(x1_ 1 , J(x 1_i)) e (x1, J(x1)) seja uma boa aproximação para a porção do gráfico de J situada
entre as retas x = x 1 _ 1 e x = x;. Portanto, é também razoável supor que o comprimento e1 de
tal segmento constitua uma aproximação razoável para o comprimento da porção do gráfico de
/ situada entre tais retas.

262
A fórmula (A.36) para a distância entre dois pontos fornece

f.3 = J(xj _:_x3 _i) 2 + (f(xj) - J(xj_ 1 )) 2

1 + (I(xi) - J(x1-d)2 6.x,.


X3 - X3-l

Agora, pelo TV11 de Lagrange, existe ç1 E (x3 _ 1 , x 3 ) tal que f(xi)-f(xi-i>


x,-x,-1
= J'(ç3·), de modo que

Adicionando as aproximações acima para os comprimentos dos k segmentos assim obtidos,


concluímos que
k

L J1 + f'(ç 3 )2(x1 - x 1 _ 1) (4.32)


J=l

é uma aproximação razoável para o que gostaríamos de definir como o comprimento do gráõco
da restrição de f ao intervalo [e.d] Ademais, esperamos também que tal aproximação torne-se
cada vez melhor, à medida que IPI-4 O.
Por outro lado, como (4.32) coincide com a oma de Riemann

E( J1 + (!')2; P; ç).

segue do teorema de Riemann 7 que

à medida que IPI-4 O.


A di cu ão acima permite concluir que fcd J1 + f'(x)2dx é a única definição razoável para
o comprimento do gráfico da restrição de J ao intervalo [e,d]. Observando que a união dos
gráfico de tai re trições (quando [e, d] varia sobre todos os subintervalos fechados e limitados
do intervalo (a, b)) coincide, exceto pelos pontos (a, f(a)) e (b, /(b)), com o gráfico de /, temos
a definição a seguir.

DEFINIÇÃO 4.42.
Seja f : [a, b] ➔ R uma função contínua em [a,b) e continuamente derivivel
existe o limite 1>-(

diremos que o gráfico de/ tem comprimento


= lim
t:➔O 1
a+t
Jt + /'(x) 2dx,
t.
-------------

263
Ainda cm relação à definição anterior, observe que, se f for derivável em [a, b) e J1 + f'(x) 2
for integrável em [a,b), então o problema 3.11, página 243, garante que

t
e= ✓1 + f'(x) 2
dx. (4.34)

4.43.
a r um círculo de centro O e raio R, e escolha um sistema cartesiano de coordenadas
0(0,0). Se f: [-R,R) ➔ Ré a função dada por f(x) ,_ JR 2 - x 2 , então o gráfico
1 o aemicfrculo de r situado no semiplano superior do plano cartesiano. Observe que f é
ua em (-R, R) e continuamente derivável em (-R. R). Portanto, de acordo com ('1.33), o
fCbmp:rimentoder é igual ao dobro do limite

se tal limite existir.


Para o que falta, observe inicialmente que

✓n f'(x) ✓1 + ( JR2-x-
2
2
R
= )
x2 JR2 - x2

Assim, perfazendoa substituição trigonométrica x = R cos t e observando que -R ..J. € <x ~


R- E<=> arccos(-1_+ í) < t < arccos(l - -k), obtemos
R-t ,--- f_R-t R
J1 + f'(x) 2
dx = ---;:::==dx
f_-R+t -R+( JR 2 - x2
arccos(l-"h) R
=
larccos(-1+-h>
--;::::;=:==:::::;::;::;;:=.=

arccos( 1- fi)
JR 2
-
2
R cos t 2
• (- R sen t) dt

=-
larccos( -1+-h)
Rdt

= R ( arccos ( -1 + ~) - arccos ( 1 - ~)) .


a função arccos: [-1, 1) ➔ [O,rr) é contínua, obtemos

: ✓1+ /'(x) 2dx = !i~ R ( arccos (-1+;) - arccos ( 1 - ;) )

= R (arccos(-1) - arccos(l)) = Rrr.


, o comprimento de r é igual a 2rr.

264
Em que pese o exemplo anterior, a inLegralem (4.33) é, cm geral, difícil (ou mesmo imposa(vel
- cf. problema 7.5, página 299) de calc~lar exaLamente. Vejamos um exemplo ilustrativo.

EXEMPLO 4.44.
Dado b > O, calculemos o comprimento e da porção da parábola y = x 2, situada entreOI
pontos (O,O) e (b, b2 ). Com f(x) = x 2 , segue de (4.34) que e=J:JI + 4x 2dx.
A fim de calcular essa integral, façamos a substituição trigonométrica x = ½tg t. Como
1 + tg 2 t = sec2 t, Ít tg t = sec2 t e O < x < b # O < t < arctg (2b), segue da fórmulade
integração por substituição que
arctg (2b) ,--- arctg (2b)
e= 10
J1 + tg t · sec2 tdt =
2
1 0
sec3 tdt.

Contudo, o cálculo da última integral acima terá de esperar até a seção 4. 7 (cf. problema 7.3,
página 298, e problema 7.4, página 298), pois que precisamos, antes, conhecer as propriedades
básicas da função logaritmo natural.

Voltemo-nos, agora, aos problemas (II) e (III) elencados no início desta seção. Para a análise
dos mesmos, seguiremos essencialmente as discussões das seções 9.1 e 10.2 de [8).
Sejam a< b números reais dados e f : [a, b)-+ IRuma função contínua em [a, b)e positiva em
(a, b). Fixe, em um plano do espaço, um sistema cartesiano com eixo das abscissas e, e seja 9
o gráfico de f em tal sistema. A superfície de revolução de eixo e e geratriz 9 (cf. Figura
4.3) é o conjunto S(e; 9) dos pontos do espaço, obtidos pela rotação de 9 em torno de e, de tal
forma que, para x E [a, b), o ponto (x, J(x)) E 9 descreve o círculo de raio f(x), centrado no
ponto de e com abscissa x e contido no plano que passa por x e é perpendicular a e.
O sólido de revolução definido por S(e; 9) (cf. Figura 4.4) é a região limitada do espaço,
delimitada pela união de S( e; 9) com os discos delimitados pelos círculos gerados pelas rotações
dos pontos (a, J(a)) e (b,J(b)) em torno de e. Doravante, sempre que não houver perigo de
confusão, denotemos simplesmente por S o sólido de revolução definido por S(e; 9).
Para u < v em [a, b] e r > O, seja C(u, v; r) o cilindro sólido de raio r, tendo por bases os
discos de raios J(u) e J(v), centrados respect: .:.mente nos pontos (u, O)e (v, O)e perpendiculares
a e (cf. Figura 4.5). Considere uma partição P = {a = Xo < X1 < · · · < Xk = b} de [a, b] e
denote ~x 3 := x3 - x1 _1, para 1 < j < k. Se
m

1
= min{J(x); x E [x3 -1, x

1
]} e M = max{f(x);
3
x E [x3 -1, x

3
]},

temos (novamente cf. Figura 4.5)


k k
LJC(x 3 -1, x 1 ; m 1 ) e Se LJC(x
1 -1, x 1 ; 1'11 ).
J=l 3=1

265
( ' 1 1 1 1 1 ' 1 .\ h 1 1-.<; H \ 1. 1>1-:H ti-:\ 1.\ :\ :\

g
' 1\
, 1\ /\
, 1 1 I 1
' 1 1 I 11
' 1 1 I 1
1
l I 1 I 1
1
' 1 1 1 1
1
' 1 1 1
1
,af ,
1 1 1 • 1
61 1 e
1 I 1 1 1 1
1 I 1 1 1 I
1 1 ' 1 1 I
1 1 ' 1''
1 ' / li I
\ 1/ \/

\ 1'

Figura 4.3: superfície de revolução S(e, Ç)

Figura 4.4: o sólido de revolução delimitado por S(e, Ç)

Portanto, se pudermos definir o volume de S, devemos ter


k k
L V(C(x 1 -1, xj; mj)) < \/(S) < L V(C(x 1_ 1: x1 ; M 1 )),
J=l

onde V(-) denota o volume do sólido declarado entre parênteses. las. como o volume de um
cilindro de raio da base R e altura h ê igual a 1rR 2 h, segue das desigualdades acima que
k k
L 1rm;ó.xj < V(S) < L 1r1\J;ó.x 1.
j=l 1=1

Agora, observe que o primeiro e o terceiro membros das desigualdades acima são, respecti-
vamente, iguais às somas inferior e superior, em relação à partição P. da função 1rf 2 . Portanto.

266
s

Figura 4.5: aproximações por falta e por excessopara V(S).

fazendo P variar sobre todas as partições possíveis de (a, b},obtemos

ou, ainda (uma vez que 1rf 2 é contínua logo, integrável),

,r J.'f(x) 2
dx < V(S) < ,r J.'f(x) 2
dx.

Portanto, concluímos que 1rJ:f (x) 2 dx é a única definição razoável para o volume de S, o
que nos leva à definição a seguir.

DEFINIÇÃO 4.45.
Se f : [a, b] ➔ IR.uma função contínua em [a, b] e positiva em (a, b), então

V= ,r 1.•f(x) 2
dx, (4.35)

é o volume do sólido de revolução gerado pe~otação do gráfico de J em tomo elo


abscissas.---~----------------~----~-~......&..::.cr.i

EXEMPLO 4.46.
Seja/: [-R, R] ➔ R a função dada por J(x) = v'R2- x2, de sorte que o a6liclo
gerado pela rotação do gráfico de / em tomo do eixo das abscissas é uma bola ,
Como

267
1 , 1
1 1,, 1 .\ 1\ I I-:CH.\I. l>I·. HII-:\1 \.\\

concluímosque o volume de uma bola de raio R é igual a j1rR 3 .

Voltemo-nos, agora, ao problema do cálculo da área da superfície de revolução S(e; 9). Para
tanto, dados números reais e, d E (a, b), com e< d, seja (cf. Figura 4.6) S[c,d] a porção de S(e; 9)
ituada entre os planos perpendiculares ao eixo e nos pontos de abscissas e e d.
Para definir a área de S1c,d), considere a partição P ={e= x 0 < x1 < • • • < Xk = d} de [e,d]
e denote 6.x 1 = x 1 - Xj-l, para 1 < j < k. Para IPIsuficientemente pequena, é razoável supor
que o tronco de cone de revolução de raios f (x 1 _i) e f (x 1 ) e altura 6.x 1 seja uma aproximação
bastante satisfatória para a porção de S1c,d] situada entre os planos que passam por x 1 -1 e x 1 e são
perpendiculares ao eixo de rotação e (cf. Figura 4.6). Portanto, é também razoável supor que a
área A1 da superfície lateral de um tal tronco de cone constitua uma aproximação razoável para
a área da porção de S1c,d] situada entre os planos em questão. Nesse ponto, a fim de continuar

9
'1 \
1 1\ I\
1 I 1
1 I 1 /' 1li 1
1 I 1 1 1 1
1 I 1 1 1 1
' 1 1 1 1 1
1 1 , , 1 e
e= ~o , '
1
d= Xk
1 I 1 1 1 1
1 1 ' 1 1 1
1 I 1 1 1 1
1 I 1 1 1/
1 1 , \ li
\ 1 1 \/
\ 1'

Figura 4.6: aproximando a área de uma superfície de revolução.

argumentação,precisamos de uma breve digressão sobre troncos de cones de revolução e


ll0888.
o cAlculo das áreas de suas superfícies laterais.
Seja dado um cone de revolução de vértice V, altura VO = h e geratriz g, cuja base é o
+---+
círculo r, de centro O e raio R, contido no plano perpendicular a VO (cf. Figura 4.7). Sejam
n > 3 um número natural e A 1A 2 ... An um polígono regular inscrito em r. A pirâmide de base
AiÂ2 ... Ane vértice V é regular e tem altura h. Sendo 1\11 o ponto médio de A1 A1+1 (com a
convenção de que An+I = Ai), é imediato que V ft./1= V 1112= • • • = V !lln. Denotando por an
tal comprimento, é imediato que a área lateral da pirâmide (i.e., a sorna das áreas de suas faces

268
V

Figura 4.7: pirâmide regular de n lados inscrita em um cone.

laterais) é igual a

Ln A(V A1Ai+1) = Ln 21 -,----,--


AA 1 1 +1 •
--
V l\tf 1 =2
1 n -,--~-
L AiAi+l • ª"
1
= 2Pnan,
t=l i=l i=l

onde Pn é o perímetro de A1 A2 ... An. Agora, à medida que n ➔ +oo, a união das faces laterais
da pirâmide forma uma aproximação cada vez melhor para o cone de revolução em questão.
Portanto, a área lateral da pirâmide constitui uma aproximação cada vez melhor para o que
gostaríamos de definir como a área lateral A do cone revolução de raio R e altura h. Mas, como
Pn ➔ 21rR e ª" ➔ g quando n ➔ +oo, definimos tal área lateral por
1
A= · 21rRg = 1rRg.
2 (4.36)
Ainda nas notações da figura 4.7, trace, à distância d< h do plano da base do cone e entre
tal plano e V, um segundo plano, que intersecta o cone segundo um círculo r', de raio R'. A
porção do cone de revolução situada entre os planos der e f' é o tronco de cone de revolução
de bases r e r' (ou raios R e R') e altura d. O lema a seguir utiliza (4.36) para calcular sua área
lateral.

LEMA 4.47. .,
A área lateral A de um tronco de cone de revolução de raios R e Ir e altura d é u,.....-.

PROVA.
Segue de (4.36) que A = 1r(Rg- R' g'), onde g e g' são, respectivamente, as geratrizes do cone
t-+
dado e daquele de base f'. A seção de tais cones por um plano que contém V O é mostrada na

269
figura 4.8. Nela, XY e X'Y' são diâmetros dos círculos r e r', respectivamente, de sorte que a

X o y

+--+
Figura 4.8: seccionando os cones por um plano que contém VO.

semelhança entre os triângulos V XY e V X'Y' fornece a relação 2~ = ~R?. Então, h = R ~<~, ,


h - d= R1:~,e, aplicando o teorema de Pitágoras aos triângulos XOV e X'O'V, obtemos

2 2 2
g =-IR2+·h2= R2+--- R d -- Rd J(R--- R') +1
V (R - R')2 = R - R' • d

e, analogamente, gt = R'd
R-R' ·1✓ + (R-R'
-d-
)2. Logo,
'

2 2 2

A = 1r( Rg - R' g')


R
= 1r ( R _ R' (R') )
- R _ R' d 1 +
(R
~ R')

= ir(R+ Fl!)d✓l + ( R ~ R')'

Voltando à discussão sobre como definir a área da superfície de revolução S(e; Q), recorde
que denotamos por A1 a área lateral do tronco de cone de revolução de raios J(x 1 _i) e f(x 1 ) e
altura Ãx; = x; - x;_ 1 . Portanto, segue do lema anterior que

Suponha, doravante, que f é continuamente derivável em (a, b). Então, pelo T\11'1 de
Lagrange, existe e,·E (x1·-1, x 1·) tal que J(x,)-J(x
XJ-XJ-1
1 -d = f'(E,1). Por outro lado, levando em

270
conta que f varia muito pouco no intervalo (x3 _ 1 , xj] para !PIsuficientemente pequena, temoe
J(x 1 -1), J(x 1 ) ""J(ç 1) e, daí,

A3 ::'. 21rf(ç1 )J1 + f'(f.j)2Dlx 3 = 21rf(f,1)J1 + f'(f,/f(x1 - X 1-1).

Adicionando as aproximações acima para as áreas dos k troncos de cone assim obtidos,
concluímos que
k
L21rf(f, 1)J1 + f'(ç 1)2 (x1 - x 1_1 ) (4.37)
j=l

é uma aproximação razoável para o que gostaríamos de definir como a área de S[c,dJ· Ademais,
também esperamos que tal aproximação torne-se cada vez melhor, à medida que IPI➔ O.
Por outro lado, como (4.37) coincide com a sorna de Riemann

invocando novamente o teorema de Riemann 7, concluímos que


k
L21rf(f.
J=l
1
,----
)J1 + f'(f, 1 )2Dlx1 ➔ 21r 1e
d
f(x)Jl + f'(x) 2 dx

à medida que IPI➔ O.


Graças à discussão acima, concluímos que 21rfcd f(x)Jl + f'(x) 2 dx é a única definição ra-
zoável para a área A de S[c,d). Mas, como

u
(c,d)C(a,b)
S[c,d) = S(e; Ç) \ (C1LJrb),

onde r ª e rb denotam os círculos gerados pelas rotações dos pontos (a, J(a)) e (b,J(b)) em torno
de e, temos a definição a seguir.

DEFINIÇÃO 4.48.
Seja J: (a, b] ➔ lR uma função contínua em [a, b], a qual é positiva e continuamen•
em (a, b). Se existe o limite
b-i
A= lim 21r
f ➔O J.a+E
f (x)Jl + f'(x) 2 dx,

diremos que a superfície de revolução gerada pela rotação do grAficode/ em.


abscissas tem área A.

271
Em rela,ão à definição anterior, observe que, se f for derivável em [a, b] e J1 + f'(x) 2 for
integrável em [a, bj, então o problema 3.11, página 243, garante que

A= 2,r J.'f(x) ✓1 + f'(x)'dx. (4.39)

ExEMPLO 4.49.
Na discussão anterior, seja / : [- R, R] -+ 1Ra função dada por f (x) = J R 2 - x 2, de sorte
que S(e;íi) é uma esfera E, de raio R. Como f'(x) = ✓R'i~x 2 para !xi < R, temos que J é
oontinuamente derivável em ( - R, R). Como

✓H'-x'• ✓.l+ ( )2dx=R


l
-R+E
R-<
~x
✓R - X 2
1R-E
-R+E
dx=2R(R-<),

temos que

.
Portanto, ·oonforme esperado, concluímos que a área de uma esfera de raio R é igual a 41rR 2 .

Problemàs - Seção 4.5


5.1. Sejam dados, no plano, uma reta r e um círculo r, de raio 1, tal que r e r são tangentes
entre si. A cicloide gerada por r (cf. Figura 4.9) é a curva descrita por um ponto P de
r, à medida que r rola sobre r, sem deslizar. A esse respeito, faça os seguintes itens:

, -----
I
I r
1
\
\

' ...
o t T 1T 21r X

Figura 4.9: a cicloide gerada por r.

(a) Fixe uma posição de r e denote por O seu ponto de tangência com a reta r. Em
1
seguida, escolha um sistema cartesiano xOy, tendo r como eixo das abscissas e tal
quer esteja sempre situada no semiplano superior em relação ar. t-.Iostre que, após
r rolar para a direita (i.c., no sentido positivo do eixo das abscissas) por t radianos

272
-

I '1 il li: 1 1 • 1 \" "'1 1 \ 1 • 1 -,

a partir de O, o ponto O ocupará uma posição P(x(t), y(l)), com x(t) =t - sente
y( t) = 1 - cos t.
(b) Observe, agora, que a função t f--t x(t), de [O,21r]em si mesmo, é uma bijeção derivável,
com inversa x f--t t(x) derivável em (O,21r). Portanto, se f: [O,21r]➔ IRé a composta
f(x) = y(t(x)), o gráfico de f ê a porção da cicloide contida na faixa do plano
cartesiano definida pelas desigualdades O < x < 21r. Tal porção é denominada um
passo da cicloide. Mostre que a área A da região situada abaixo do passo da cicloide
(e, como sempre, acima do eixo das abscissas) é igual a 31r.
(c) Use (4.33) para calcular o comprimento de um passo da cicloide.

5.2. Um segmento esférico é o sólido obtido pela interseção de uma bola com a região deli-
mitada por dois planos paralelos. Os raios dos discos de interseção dos planos com a bola
são os raios do segmento esférico, ao passo que a distância entre os planos é sua altura.
Mostre que o volume V de um segmento esférico de raios r 1 e r 2 e altura h é dado por

5.3. Generalize a definição 45 para o sólido de revolução gerado pela rotação, em torno do
eixo das abscissas, da região situada entre os gráficos de duas funções /, g : [a, b] ➔ IR,
satisfazendo as seguintes condições: / e g são contínuas em [a, b] e tais que O < g < f
em (a, b). Mais precisamente, mostre que a única definição razoável para o volume de tal
sólido é
2
,r 1.•(f(x) - g(x) 2 )dx.

5.4. Sejam dados, em um plano, uma retare um círculo r, de centro O e raio R, estando O
situado à distância d > R der. Se S é o sólido gerado pela rotação de r em torno der,
calcule o volume de S em termos de R e d (o sólido S é denominado o toro de revolução
de raios R e d).

Para os problemas 5.5 a 5.10, precisamos de uma pequena digressão física. Dada uma
região simples 'R.,no plano, queremos definir o que se entende pelo bancentro de R. Para
tanto, imaginemos 'R.,como uma chapa fina de metal, de massa me homogénea (i.c., com
densidade superficial de massa constante). Então, o baricentro ou centro de gravidade
G de 'R.,é o ponto de aplicação do vetor m onde g( g
ê o vetor aceleração da gravidade)
para fins de momento de n. De outro modo, isso significa que, particionando 'R em wn

273
1 11,1 1,, 1 .-\ l'\1u;1u1, 1>1-:HIE\I\\::\

número finito de pequenas chapas n 1 , de massas m 1 e baricentros (conhecidos) G1 , devemos


ter
(4.40)

o vetor nulo.

5.5. Sejam f : [a, p) ➔ R uma função contínua em [a, b) e positiva em (a, b), e n a região sob
seu gráfico. Mostre que o ponto G(xc, Yc) tal que

Xc
_ 1:bxf(x)dx
-
_ ½1:f(x) 2 dx
e YG - b
la f (x)dx la f(x)dx
é a única definição razoável para o baricentro de n.
5.6. Dado no plano um triângulo ABC, suponha, sem perda de generalidade, que Ê, ê < 90º.
Considere um sistema cartesiano de coordenadas de origem B, tal que C(a, O), com a> O,
e A(b, h), com O < b < a e h > O. Em um tal sistema, mostre que a região triangular
delin:tltada por ABC coincide com a região n sob o gráfico da função f : [O,a) ➔ R, dada
por
hx seO<x<b
J(x) = { ~;-a) -b <- < .
, b-a , se _ X _ a.
Em seguida, mostre que o baricentro dê ABC, conforme usualmente definido nos livros de
Geometria Euclidiana (cf. parágrafo que precede o exemplo 54), coincide com o baricentro
da região 'R, calculado com o auxílio das fórmulas do problema anterior.

5. 7. Generalize o problema 5.5 ao caso da região n situada entre os gráficos de funções J,g :
[a, b] ➔ R, contínuas em [a, b] e tais que g < f em (a, b).

5.8. Prove o teorema de Pappus: nas notações do problema anterior, se g > O, e denota
o eixo das abscissas e d a distância do baricentro de n a e, então o volume do sólido de
é é
revolução obtido pela rotação de n em torno de e 21rAd, onde A a ârea de n.

5.9. Utilize o teorema de Pappus, juntamente com o resultado do exemplo 46, para calcular a
posição do baricentro de um scmidisco de raio R.

5.10. Utilize o teorema de Pappus para calcular novamente o volume do toro de revolução de
raios R e d, com d > R.

Adaptandoa discussão que precede o problema 5.5, podemos definir o que se entende pelo
de uma fu11ção.Mais precisamente, sendo f : [a, b] ➔ R uma função
baríe,eritT'{J do g1·áfir:o

 274
contínua e Ç o gráfico de f, definimos o baricentro ou centro de gravidade G de Odo
seguinte modo: imaginamos Ç como um fino fio de metal, de massa me homog~neo(i.e.,
com densidade linear de massa constante). Em segujda, definimos G exatamente como
g
feito anteriormente, i.e., como o ponto de aplicação do vetor m para fins de momento de
Ç. Como antes, isso significa que, dividindo Ç em um número finito de pequenas porções
91 de fio, de massas m 1 e baricentros (supostos conhecidos) G1, devemos ter a validade de
(4.40). A esse respeito, faça os problemas 5.11 a 5.15.

5.11. Seja J : [a, b] ➔ 1Ruma função contínua em [a, b], a qual é positiva e continuamente
derivável em (a, b). Mostre que a única definição razoável para o baricentro de Ç é o ponto
G(xc, Yc) tal que
limE-+O J:;EExJl + f'(x)2dx
XG = b
limE-+0 fa+cf J1 + f'(x) 2dx
e
limc-+O J:;:J(x)Jl + f'(x)2dx
Yc= b E ,
~-+O fa+< ✓l + f'(x) 2dx
contanto que tais limites existam.

5.12. Generalize o problema anterior ao caso em que Ç é a união dos gráficos das funções f, g:
[a, b] ➔ JR,contínuas em [a, b], continuamente deriváveis em (a, b) e tais que g < f em
(a, b).

5.13. Prove o teorema de Pappus: nas notações do problema anterior, se g > O em (a, b), e
denota o eixo das abscissas e d a distância do baricentro de Ç a e, então a área da superfície
de revolução obtida pela rotação da união dos gráficos de f e g em torno de e é 21rfd, onde
eê a soma dos comprimentos dos gráficos de f e g.
5.14. Utilize a versão do teorema de Pappus dada pelo problema anterior, juntamente com o
resultado do exemplo 49, para calcular a posição do baricentro de um semicírculo de raio
R.

5.15. Utilize a versão do teorema de Pappus dada pelo problema 5.13 para calcular a área do
toro de revolução de raios R e d, com d > R.

275
1 , 1, 1 1,, 1 .\ l'\11-:<;1<\f. llF HIF\I\'\'\

4.6 Logaritmos e exponenciais


Nesta seção, introduzimos e discutimos as propriedades de duas das mais importantes funções
da 11atemática, a função logaritmo natural e a função exponencial. a seção 4.9, o leitor terá a
oportunidade de apreciar o papel central dessas funções na Física.

DUINIQÃO 4.50 ..
A função logaritmo natural, log: (O,+oo) ➔ JR,é a função definida por
x 1
logx =
1
1
-dt.
t

Segue prontamente da definição acima que log 1 = O, log x > O se x > 1 e log x < O se
O < x < 1. De outra forma, sendo f : (O,+oo) ➔ IRa função de proporcionalidade inversa, i.e.,
a função tal que f(x) = ~ para todo x > O, então, em termos de área da região do primeiro
quadrante situada sob o gráfico de f (cf. Figura 4.10), temos

A(f111,x]), se x > 1
logx ={ O, se x = 1
-,1.(fl(x,1]), se X< 1

o 1 X t

Figura 4.10: definição de logx para x > 1.

276
Pelo TFC, temos que log é uma função derivável, com
/ 1
log x = -.
X

Daí, log é infinitamente deriYáYele, como log' > O, o estudo da primeira variação de funções
garante que log é uma função crescente. Também, como
,, 1
log x = --x2 < O l

segue do corolário 65 que log é estritamente côncava.


A proposição a seguir e seus corolários trazem mais propriedades importantes da função
logaritmo natural.

PROPOSIÇÃO 4.51.
Para x. y > O, temos
log(xy) = logx + logy. {4.41)

PROVA
Para a primeira parte, a ad1t1vidadc dn integral garante que

=1 12'"JI 1xy-dt.
7
1 1 1 1
log xy = rt1
-dt -dt + -dt = log x +
1) f lf r f X t
Agora, se g : (1, y) ➔ [x, xy] é a função dadn por g(t) = xt, o teorema de mudança de variâveis
35 fornece

1 ZIJ 1
-dt =
t
1g(y) 1
-dt =
1JI
-
1
· g'(s)ds =
1" -
1
· xds
r

=
1" g( 1)

1
1
i

- • ds = log y.
s
l 9 (S) l XS

COROLÁRIO 4.52.
Para x > O e r E Q, temos logxr = r logx.

PRO\'A.
Aplicando (4.41) y = x, obtemos logx 2 = 2logx. Agora, se logxk = klogx para um certo
k EN. então, fazendo y = xk em (4.41), obtemos
log xk+l = log(x · xk) = log x + log xk
= log x + k log x = (k + 1) log x.

277
( • 1 1' 1
1 1 1 1 :\ I '\ 1 I·.( ,l<.\I. 1>J·: H 11·.:\I.\\\

Portanto, cgue por indução que log x" = n log x, para todo n E N.
Agorn, aplicando essa igualdade com x 1/n no lugar de x, obtemos

log X = log(x 11nr = n log x 1ln

ou, o que é o mesmo, log x 1/n = ¼log x. De posse dos casos particulares acima e sendo r = ';:,
com m, n EN, temos

1
log xr = log xm/n = log(x 1ln)m = m log x 1ln = m. - log X= r log X.
n
Para estender essa igualdade aos racionais negativos, faça primeiro y = x- 1 em (4.41) para
obter
O= log 1 = log(x · x- 1) = logx + logx- 1

ou, ainda, log x- 1 = - log x. Em seguida, se r é um racional negativo e s = -r > O, temos

logxr = log(x 5)- 1 = -logx" = -(s · logx) = rlogx.

PROVA.
Fixe a> 1. Como log a> O, segue do corolário anterior que

logan = nloga ~ +oo e loga-n = -nloga ~ -

Por outro lado, como O < ¼< 1, o exemplo 12 fornece, sucessivamente,


1
lim a-n = lim (1/at = O e lim a"= lim - - = +oo.
n-++oo n-++oo n-t+oo n-++oo a-n

Por fim, os limites acima e o fat,o de que log é crescente garantem que limx-+O+log x =-
e límz-++oolog x 1= +oo.
Para o que falta, segue do problema 1.7, página 137, que Im (!) = IR.

278
l.1H,\lill'\1h 1 1 \\'11\\ \1 1\1'-

Aplicando a desigualdade de Jensen à função logariLmo uA..Lurnl,poclcmoHdar outr& prova


da desigualdade entre as médias ariL11)éiicae gcoméLrica (cf. exemplo 55). Para apresentá-
la, precisamos da seguinte generalização de (4.41), a qual pode ser estabelecida facilmente por
indução: para x 1, X2,... , Xn > O, temos

log(x1x2 ... Xn) = log Xi + log X2+ · · · + log Xn, (4.42)

EXEMPLO 4. 54.
Sejam dados um inteiro n > 2 e reais positivos x 1, x 2, ... , Xn. Como log : (O,+oo) ➔ Ré
estritamente côncava, a versão do corolário 70 para funções estritamente côncavas dá--noe

logx1 + logx2 + · · · + logxn < 1og (X1 + X2+ · · · + Xn) .


------------ --------
n n

Aplicando sucessivamente (4.42) e o corolário 52 (com x = x 1x 2 ... Xn er = ¾) ao primeiro


membro, obtemos
(4.43)

com igualdade se, e só se, x 1 = x2 = • • • = Xn. Mas, como log é uma função crescente, segue daí
que

ocorrendo a igualdade se, e só se, X1= x2 = • • • = Xn.


----------------~
A seguir, utilizamos a sobrejetividade da função logaritmo natural para introduzir um dos
dois3 mais famosos números da Matemática, o número e.

DEFINIÇÃO 4.55.
O número e é o único real positivo tal que log e = 1.
~-----------------_J

Na seção 5.1, mostraremos que e é um número irracional 4 tal que

e rv 2, 71828, (4.44)

com cinco casas decimais corretas. Por ora, o resultado a seguir dá uma intepreta.,ão mais
tangível para o número e.
3 Como o leitor deve suspeitar, o outro é o número 1T.
4 Para conhecimento do leitor, informamos que os números e e 1Tsão, de fato, transcendentes, i.t'., e e 1r não
são raízes de polinômio algum de coeficientes inteiros. Uma prova desse fato fogt.•no t-':icopo dt'.,tt\., notas, wa.1
pode ser encontrada em l22J.

279
( \ 1 '1 1 1 1 1 1 1 .\ ('.; 1 )-( ; I{ \ 1. 1) t: I{ 1 (· \ 1 .\:\ \:

TEOREMA 4.56.
e= limn-++oo (1+ ¼f-

PROVA.
Inicialmente, observe (cf. Figura 4.11, onde esboçamos a porção do gráfico de t H ¾ situada
de t = 1 a t = 1 + ! ) que

1 n
n+l

1 1 + ln

Figura 4.11: estimando log (1 + ¾)

1 < 11+¼ dt = t 1t=1+.!." =


log ( 1 + -1) • 11+¼ -dt ( 1 + -1) - 1 = -1
n 1 t 1 t=l n n

e, analogamente,

1 > 11+¼ -- 1 dt = -- n
log ( 1 + -1) = 11+¼ -dt · -1 = --.1
n 1 t 1 1+~ 1 n+l n n+l
Portanto,
-- 1 < log ( 1 + -1) < -,1 (4.45)
n+ 1 n n
de modo que n~l < n log (1 + ¼) < 1. O teorema do confronto (cf. problema 2.5, página 89)
garante, então, que limn-++oon log (1 + ¼)= 1 ou, ainda, que

lim log
n-++oo
(1+ n .!.)n = 1.

Mas, como log é crescente e contínua, segue daí que limn-++oo( 1 + ¾)" existe, e

log lim
n-++oo
(1+ n .!.)n = lim log
n-++oo
(1+ n .!.)n = 1.

Logo,limn➔+oo (1 + ¼f= e.

280
y
,, y =X
,,,,
,, log
,,,,
,,
1 --~----- ,,,, 1
,, 1
,,
fJ e X

Figura 4.12: gráfico de log : (O,+oo) -+ IR.

Reunindo as informações que obtivemos atê aqui sobre a função logaritmo natural, esboçamos
seu gráfico na Figura 4.12.
O esboço do gráfico da função logaritmo natural sugere que ela cresce muito lentamente. O
teorema a seguir quantifica essa suspeita.

TEOREMA 4.57.
P ara n > 1 m • t emos 1·
• t erro, log:z:
o/%= Ü.
1m:z:-++oo

PROVA.
Inicialmente, observe que t > t 1- 2~ para t > 1. Portanto, lt < -:-1-r
t -rn
1
para t > 1, de forma
que, para x > 1,

log x =
l l
x-dt
1
t
< 1:z:
1
-- 1
t 1-2n 1 dt = lx
1
1
trn- 1dt
1
trn 1t=x
= -- = 2n( 2y'x - 1) < 2n 2y'x.
l/2n t=l

Assim, para x > 1, temos


log x y'x
2
2n
0<--<2n--=--+0
y'x y'x 2y1x
quando x -+ +oo. Segue, pois, do teorema do confronto {cf. problema 2.5, página 89) que
w-+ o quando
1
X -+ +oo.

281
1
1 , 1 1 1 1 1 .\ 1" 1 1-,c; 1<\ 1. 1 li·. H 11·.\ 1 \""

Umn ,·cz que a função logaritmo natural log (O,+oo) ➔ IR é uma bijeção contínua e
crescente, podemos considerar sua inversa

exp: 1R➔ (O,+oo), (4.46)

denominada a função exponencial. Observe que exp também é uma bijeção contínua e cres-
cente. Assim, para x, y E JR,sendo x > O, temos

logx = y ~ x = exp(y);
em particular, segue de log 1 = O e log e= 1 que exp(0) = 1 e exp(l) = e.
Por outro lado, como log' x = ¾ =f O, o teorema 27 garante que exp é uma função derivável;
ademais sendo x > O e y = log x, temos

1
exp'(y) = -1,-
log x
= - -
1/x
= x = exp(y),

exp' = exp;
.
em particular, exp é infinitamente derivável. Também, como exp" = exp > O, segue do corolário
65 que exp é uma função estritamente conveta .

A proposição a seguir traduz, para a função exponencial, as propriedades da função logaritmo
natural expressas na proposição 51 e no corolário 52.

t) = exp(x). exp(y).

PROVA.
(a) Sendo a= exp(x) e b = exp(y), temos a,b > O e Ioga= x, logb = y. Portanto, segue da
proposição 51 que
x + y =Ioga+ logb = log(ab),

de sorte que exp(x + y) = ab. Por fim, os cálculos acima fornecem

cxp(x) · cxp(y) = ab = exp(x + y).

282
(b) Como log e exp são inversas uma da outra e log e = 1, segue da proposição 51 que

logexp(r) =r = loger.

Daí, a injetividade de log fornece exp(r) = er.

Graças ao item (b) da proposição anterior, doravante denotaremos

exp(x) = ex,
frisando que o segundo membro acima coincide com o sentido usual de e:r:quando x E Q. Por
outro lado, nessa nova notação, o item (a) da proposição anterior pode ser escrito como

para todos x, y E lR..


Mais geralmente, a função exponencial nos permite definir rigorosamente o que se entende
pela potência a:r, para a real positivo e x E IR. De fato, nas notações do parágrafo anterior,
pomos
a-z: = C:rloga, (4.47)
de sorte que

e
log ar = log(e:rlogO ) = x log a, (4.48)
ptU·ntodos x, y E IR.
Por outro lado, ser E Q, digamos r = ';, com m E Zen EN, segue do corolário 52 que

em outras palavras, para x = r E Q, a definição (4.47) concorda com o sentido que temos
atribuído a ar até agora. Em particular, temos a 0 = 1 e a 1 = a.
Se a = 1, segue de (4.47) que a:r = 1 para todo x E IR. Por outro lado, fixado um real positivo
a =/:-1, a função exponencial de base a é a função la : lR.➔ (O,+oo) tal que J0 (x) = ax.
A relação (4.47), juntamente com as propriedades da função exponencial e a regra da cadeia,
garante que / 0 é uma bijeção derivãvel, com derivada

J~(x) = exp:r:loga-Ioga= ax Ioga.

283
< ,111111 \ 1 .\ I\ r1-:c1<.\L 1>1-:H11-::\I.\\\

Em particular, como loga >O<=>a> l, segue que

!a é crescente <=>f~ > O<=>a > l;

por conseguinte, / 0 é decrescente se, e só se, O < a < l.


A inversa da função la é a função log0 : (O,+oo) ---t lR, denominada função logaritmo na
base a, cujas propriedades estudamos no problema 6.1.
Uma variante importante da discussão acima é a seguinte: dados um intervalo I e funções
J: I -4 (01 +oo) e g: I ---t lR,definimos J(x)g(x) por

f(x)g(x) = eg(x)log/(.i:)_

Observe que a exigência de que f(x) seja positivo dá sentido a tal definição. Também, o segundo
membro da igualdade acima, juntamente com a regra da cadeia, garante que a função x E J 1-t
f(x)g(x) é derivável se f ego forem. A esse respeito, veja também o problema 6.15.
PodeID:os,agora, enunciar e provar urna extensão útil do teorema 56.

~-
e=~➔+oo (1+ ~r-
PROVA.
Um argumento análogo ao utilizado na prova do teorema 56 garante que, para x > O, temos

-- 1 < log ( 1 + -1) < -.1 (4.49)


x+ 1 X X

Agora, segue de (4.48) que log (1+ir= x log(l + ¼)-Portanto, multiplicando as desigualdades
acima por x > O, obtemos
_x_ < log (1 + .!.):r
< 1,
x+l x
e o teorema do confronto (cf. problema 2.5, página 89) garante que

lirn log
:r-t+oo
(1+ !):r=
X
1.

Por fim, como a função exponencial é contínua, podemos escrever

284
A seguir, utilizamos o teorema 57 para mostrar que a função exponencial cresce mais rapi-
damente que qualquer polinômio, à medida que x ➔ +oo.

TEOREMA 4.60.
Se p é um polinômio, então limz-++oo~ = O.

PROVA.
Seja p(x) = amxm + ªm-1Xm-l o. Como
+ · · · + a1x + ao, comam=/:-

basta mostrar que limx-.+ :: = O, para todo n E N. Para tanto, fazendo y ez, temos
x = log y e, daí,
xn = (logy)n = (logy)n
ez y efy
Por fim, como y ➔ +oo quando x ➔ +oo, segue do teorema 57 que

lim ::_n
x-++oo eX
= lim
y-++oo
(1ogy
efy
)n= ( limy-++oo
1ogy
efy
)n= O.

Problemas - Seção 4.6


6.1. * Para O< a=/:-1, recorde que a função log0 (O,+oo) ➔ IR(a função logaritmo na base a)
:

foi definida como a inversa da função la : IR ➔ (O,+oo), tal que !a(x) = az, para x E R.
Prove que log0 é uma bijeção infinitamente derivável, com
logx , 1
logo x = -1oga
- e loga x = x loga , (4.50)

285
1 Ili l(l 1 .-\ 1\11-:c:U.\L Ili·: HII-:\1.\\\

para todos O < a, x =/-1. Ademais, dados reais positivos a, b, e, x, y, com a, b, e=/- 1, prove
também que:

(a) Iog = Ioge.


(b) logª é crescente, se a > 1, e decrescente, se O < a < 1.
(c) Ioga(xy) = logª x + Iogay.
(d) Iogae = Iogbe • logª b.
(e) logª b • logbe· logca= l.

6.2. Prove que o número log10 2 é irracional.

6.3. Explique, com justificativa, qual dos números e1f ou 1reé o maior.

6.4. Encontre, com justificativa, todos os a, b E N tais que a =/-b e ab = bª.

6.5. * Mostre que f Iogxdx = xlogx - x+ e.


6.6. * Móstre que Iog(l + x) < x, para todo x > O.

6. 7. Mostre que ~~t~ dt = O, para todo x > O.


fi~x


6.8. Use o corolário 52, juntamente com o resultado do problema 6.8, página 189, para dar uma
outra prova do fato de que log é estritamente côncava.

6.9. * Prove que a função f : (0,1r) ➔ IR, dada para x E (0,1r) por J(x) - logsenx, é
estritamente côncava.

6.10. * Prove a desigualdade de Young 5 : sejam p e q reais positivos tais que!+


p
lq = 1. Dados
a, b > O, temos ab < ~ + b;, ocorrendo a igualdade se, e só se, aP = bq.

6.11. Dados reais positivos a, b e e, prove que aªbbcc > (ª+~+cr+b+c,com igualdade se, e SÓ se,
a= b = e.

6.12. Calcule Je-zdx.


6.13. Encontre todos os valores negativos de a para os quais a função f : IR ➔ IR, dada por
f(x) = ~ + ax 3 , tem um único ponto de inflexão.

6.14. Sejam o;/: O um número real e f: (O,+oo) ➔ IR a função dada por f(x) = xº. Mostre
que f'(x) = ox 0 - 1 , para todo x > O.
6 Ap68 Wílliam H. Young, matemático inglês dos séculos XIX e XX.

286
Dados um intervalo I e funções deriváveis / : I ➔ (O,+oo), g : I ➔ R, calcule a derivada
da função h : I ➔ IR dada por h(;t) = f (x )g(x).

6.16. Dados a, b > O, seja f : (O,l] ➔ IR a função dada por J(x) = xª(l - x) 6 . Calcule o valor
máximo de/.

O conjunto dos racionais não é fechado em relação à potenciação; por exemplo, se a =


b = ½,então a6 = 72,
que é um número irracional. Se a e b forem irracionais positivos, é
sempre verdade que a 6 é irracional? Justifique sua resposta.

Prove que a equação x 2 = 2x tem exatamente três raízes reais, uma das quais é irracional.

Se a é um real positivo, prove que limx➔+oo (1+ !t = eª.

Encontre todos os valores reais de a para os quais limx➔+oo (:~:f= e.


6.21. Seja f : IR ➔ IR uma função derivável, tal que /{O) = O e f'(x) > f(x), para todo x E IR.
Prove que f(x) > O, para todo x > O.

6.22. * As funções seno hiperbólico e cosseno hiperbólico 6 são as funções senh, cosh : IR➔
IR, definidas por
eX - e-X eX + e-X
senhx = 2
e coshx =
2
Faça os seguintes itens:

(a) Mostre que cosh2 x - senh 2 x = 1, senh 'x = cosh x e cosh' x = senh x.
{b) Mostre que cosh é uma função par e estritamente convexa, com imagem (1, +oo), tal
que coshx = 1 <=>x = O.
(c) Mostre que senh é uma função ímpar e estritamente crescente, com imagem lR. ~fos-
tre, também, que senh é estritawi:>nt convexa em (O,+oo) e estritamente côncava
em (-oo, O), e que x = O é seu único ponto de inflexão.
(d) Esboce os gráficos das funções seno hiperbólico e cosseno hiperbólico em um mesmo
sistema cartesiano de coordenadas; esboce ainda, nesse mesmo sistema, os gráficos
das funções x H ½e:r(x > O) ex H ½e-:r, x H -½e-::c (x < O).
6A razão para tal nomenclatura é explicada pelos itens {a) e (b): graças a eles, a aplicação t H (cosh t, senht)
uma parametnzação para o ramo da hipérbole equilâtera x 2 - y 2 = 1 situado no primeiro e quarto quadrantes
o plano cartesiano.

287
1 \l'I' 1 l 1) 1 .-\ [:\; 11-:(;({.\(, Ili·: HIF\I.\:'\'\

(e) O gráfico da função cosseno hiperbólico é uma curva conhecida como catenãria 7 .
Para x 0 E lR, mostre que a curvatura da catenária em x 0 ( cf. problema 7.3, página
198) é k(xo) = cos~2xo.
(f) Para x 0 f:. O, calcule o comprimento da porção da catenária situada entre os pontos
de abscissas O e x 0 .

O próximo problema explica o porquê dos nomes seno e cosseno hiperbólicos. Para colocâ-
lo em contexto, recorde (cf. Figura 4.13, à esquerda) que a área do setor circular de raio
1, contido no primeiro quadrante do plano cartesiano, centrado na origem do mesmo e
delimitado pelos raios que unem a origem aos pontos (1, O) e (cos t, sen t) é igual a ½-

_ (cos t, sen t)

Figura 4.13: seno e cosseno circulares e hiperbólicos.

6.23. Para todo t E lR, observe que o ponto (cosh t, senh t) pertence ao ramo direito da hipérbole
x 2 -y 2
reta y = (tanht)x, onde tanht = :S~~-
= 1e à Fixado t > O, seja A a área da
porção limitada do plano cartesiano, delimitada pelo eixo das abscissas, pelo ramo direito
de x 2 - y 2 = 1 e pela reta y = (tanh t)x (cf. Figura 4.13, à direita). Mostre que A= ½-

6.24. Dado À > O, mostre que limx➔O+ x~ log x = O.


6.25. • Se f : (1,+oo) ➔ lR é a função definida por

J(x) = log(x + 1/2) - (x + 1) log(x + 1) + x logx + 1,

prove que/ é positiva em (1, +oo).

7A Duperficie 4e revolução gerada pela. rotação da catenária em torno do eixo das abscissas é um exemplo de
wna •uperf(cie m(nima, nesse c&BOo catenoide. Esse tipo de superfície tem muitas propriedades interessantes
dentre u ql.UiÍ8 algumu aplicações à const.ruçiio civil. Para mais sobre superfícies em geral, e superfícies mínimas
em particular, 1ugerir110Hao leitor o excelente livro 1181.

288
obr d
.
lil
-
gr çao
- d.is u
um pou m obr • ru d in !IT - m um duplo prop ito: por
nd m d ixar 1 o o import p p 1d p nh d p 1 fun õ log xp m
i ui - : por outro. o d.is u irm o m todo d d ompo ição
or um conjun o ma compl to d ni d in gração
ulo d in prim.i i\ d
n o nal ão. di limi d t
á - b n • pl qu n qu p n r ·pr

.• mpl m fun - log xp


m 1 r fl ·i ilid d rmul ( ·- ) ( ·- ).

t i!iz rn du \ J) primi i, d J n(log )dx:

J )d - )- / (l
1
)•-d.r

- )-J·(- n ( l g x)) • 1
dx)

ll l x)- )-J 'n (l x)dx.

u ld illl d

l
J )d = -· n(l :I) - (l )) +e.

11 llll m Ic um fun - f: I ➔ , d riv v 1


( m i n, i mt 1 r r d
.!!_f(r} }J'( ).
dx
1 i . u - _l qu fun - X /(x}/'{ ) in gr v l d forma qu

J f(r} J'(x)dx = f(r)


(4.51)

ul b n • • 1• 1m nt qu
lill
d r,;
d.:rv x = 2 rz.
1 Ent-ao,

-------- 1- -dxli =_ 1- ___


r •
d
-'1r,dx
....,__
____
=2 ../r ir=2
~ -1
= 2{e~ - e).
~~--~............
--
1 \ 1 '1 1 1 1' 1 1 .\ "' 1 I· e; ({ \1. () F H lt-: \ 1.\ :'\ :'\

Ainda em relação a (4.51), chamamos a atenção do leitor para o fato de que, em princípio,
podemos emprega.r tal fórmula ao cálculo de primitivas da forma Jef(x>g(x)dx. r esse caso,
supondo que f'(x) # O em I, empregamos (4.51) seguida por uma integração por partes:

• J
ef(x)g(x)dx = f ef(x)J'(x) • g(x) dx
f'(x)
= ef(x) . g(x) -
f'(x)
J ef(x) . .!!:_( g(x))
dx f'(x)
dx

Com sorte, a última integral pode ser mais simples de calcular do que a primeira.
Continuando nossas discussões, definamos a função L: JR.•➔ R pondo, para x # O,

L(x) = log lxl.

(A Figura 4.14 mostra o gráfico de L.) Se x > O, temos L(x) = log x, de forma que L'(x) = ~; se

,.
Figura 4.14: gráfico de L : R* ➔ R.

x < O, temos L(x) = log(-x), e a regra da cadeia fornece L'(x) = !x · (-1) = ~- Em qualquer
caso, temos L'(x) = ;, de sorte que L é uma primitiva para a função de proporcionalidade
inversa x i-+ l,% definida para x # O. Assim,

L' (x) = -l
X
⇒ J- l = L( x)
X
+ e.

Agora, seja I um intervalo e f :I ➔ R \ {O} uma função derivável, com derivada integrável
em todo intervalo [a, b] e /. Como f é em particular contínua, o TVI garante que J > O em I

290
ou f < O em I. Portanto, pela regra da cadeia, a função Lo f = log 1/1é tal que
(Lo f)'(x) = L'(J(x))J'(x) = j[;;. (4.52)

7 7.
Por outro lado, escrevendo = /' · e utilizando as hipóteses sobre /, segue da proposição
21 que 7 é integrável em todo intervalo [a,b] e I. Portanto, Lo f = log 1/1é uma primitiva
para 7em I, de modo que

j jf;i= log 1/(x)I + C. (4.53)

Para x -1Í + k7r, com k E Z, temos

tgx = senx
--
COSX
= cos' x
---.
COSI

o k E Z, faça I = (-~ + krr, ~ + kír) e tome f(x) = cosx em I. Segue de (4.53) que, em

j tgxdx = - log Icos.r:I +e= log I secxl + e.

Voltemo-nos, agora, ao e.'>tudoda técnica de integração por decomposição em frações parciais,


a qual é bastante útil para a mtegração de quocientes de funções polinomiais. Tal técnica é
baseada em alg1ms fato algébricos que. para a conveniencia do leitor, elencamos a seguir. Para
umn discussão pormenonzada dos mesmos. recomendamos a referência [141.
Dada uma função polinomial g(.r) = anx" + an-1x"- 1+ · · · + a1x + ao, com ao, a 1, ... , ª" E IR
e a,. -1O. dizemos que n é o grau de g e denotamos n = 8g. (Observe que âg = O se g for
uma função con lante e não nula; por outro lado, não definimos o grau de g se g for a função
identicamente nula.)
O teorema fundamental da Álgebra (cf. seção 3.3 de [14]) garante que a função g pode
ser fatorada, de maneira única. a menos de uma reordenação dos fatores, da forma
k l
g(x) = an II (x - Xj)mJ • II ((x - Zj)(x - Zj))nJ, (4.54)
J=l J=l

m x 1, ... , xk E IRreais distintos, z1, ... , Zt E C\IR complexos distintos e m1, ... , mk, n 1, ... , n 1 E
+· E crevendo ::1 = a1 + ibj, com a1 , b1 E IR, temos z1 = a1 - ib1 e, daí, (x - z1 )(x - zi) =
2
z - 2a1 x +(a;+ b;). Portanto, a partir de (4.54), obtemos para g a fatoração alternativa
k l
g(x) =ª"II(x - x 1 )mJ • II (x 2 + a x + /3)"J,
1 1 (4.55)
J=l j=l

291
( \1'1\1 \11 1 :\ l:\(H;f{.\f. f)(-' ({ff-:\1.\:\:\

ond<.'o 1 ,/31 E lR são tnis que :r2 + a,1 x + (31 não tem raízes reais.
e f,g: lR ➔ lR são funções polinomiais dadas e (como acima) x 1 , ... . xk são as raízes reais
d g, dizemos que; : lR\ {x1, ... , xk} ➔ IRé uma função racional. Se8J > 89, o algoritmo da
dwtsão para polinômios garante a existência de funções polinomiais únicas q e r, (efetivamente
calculáveis e) tais que J(x) = g(x)q(x) + r(x) para todo x E lR, com r = O ou O < 8r < 8g.
- e caso, temos
J(x) _ ( ) r(x) (4.56)
g(x) - q x + 9(x)'
para todo x =j:.x1, ... , Xk-
Elabora.ndo um pouco mais a discussão anterior, pode-se provar o seguinte teorema de de-
composição em frações parciais para funções racionais.

TEOREMA 4.64.
Sejam r, g : R ➔ 1Rfunções polinomiais não nulas dadas, com O < 8r < 89

(a.) Se g está fatora.da como em {4.55), então, para x =j:.x 1, ... ,xk, temos

onde r 1, ... , rk, s 1, ... , s, são funçõ~ polinomiais tais que r 1 = O ou O < 8r 1 < m 1 para

1 < j < k, e Sj = O ou O < asj < 2nj para 1 < j < l.

. r 1 (x) Lmi C t
(b) Para 1 < J < k ex=! Xj, temos (x-x )m] = (x-x 1 )t, com Cj 1 , ... , Cjm
] E IR.
J t=l J

. Sj(X)
( e) Para 1 < J < l, temos (X 2 + Q·X + (3·)ni
J J
e;., E R. ,.

A discussão que fizemos até aqui nos permite explicar, de maneira relativamente simples, a
filosofia por trás da técnica de integração por decomposição em frações parciais: dadas funções
polinomiais não nulas J,g : lR ➔ IR,suponha que somos capazes de obter efetivamente as funções
q, r, r 1, ... , rk, s 1, ... , s1 de (4.56) e do item (a) do teorema anterior. Então, temos

J f (:r)
g(~r)dx
f
=. q(:r)d1·+
j r(x)(x) dx
9

L j ----r (.r)
k
Lj
1

J r(.c)d
--
q(;,:)
'
1' =
• .
r=-1
(.r -
1
.1· )mJ
J
d'.l' +

J= L
s1 (x)
-----------
(x2 + a ·X+ (3 )ni
J
dX

l

292
\I \I" ....,,>111<1-1 l.t "\11 • \'.-- 1>1. I"\ 11 t ,H \<,' \< > 1 ,

um v z qu o álculo d Jq(x)dx ' im di (p o qu q J lin mini), n•d111,irn. o


âl ulo d pnrru 1 d fr o ál ulo çl rimi i

--~---d.
j

umindo a or qu om paz d ob r f iv m n o r
(b) ( ) do eor m an rior mo

( .57)

Q
1(x)
1X f3·)n, X=
d ~1
L...,, (x2
d1tX+ 1t d
+ Q 1X+ {31)t X. ( .5 )
1 t=l

primi ivas do gund m mbro d ( .57) ão f i d e l ular:

1 dx-{ l~t(x-x3)l-t+c t>l


/ (x-x 1)t - lo lx-x 1l+C t=l

Quan o ( . ) 1t = ~(2 + 1) +( 1t - d,~a,) ob mo inicialm n e

D du primiti do gundo m mbro a im a prim ir pod r al ulad f ilm n an -


li ndo doi o p rad m n :

(i) t = 1, gu d (4.53) do fa o d qu x 2 + o 1 x + /31 > O par todo x E qu

2
J x
X2 + 0 1X
1 /3 dx
+ 1
= lbg(x 2 + 1x + /31) +

2
(ii) t > 1 ntão f z ndo r = 1 - t J(x) = + 1 + /31 m ( . 1) bt m

J (x 2
2x + a-1
+ 0 1X + f31i
dx = _1_(
1- t
2+ 1
x + /3 )1-t +
1

ort o r duzim o álculo d ( .5 ) qu l d

(
! X
2
+ 1 +f3)Ldx,

2
( \l'l 11 1 e 1 1 :\ (\; ru;tL\I. f)E HIE:\I.\\"\"

com t > l e a, /3E R tais que x2 + ax + f3 não tem raízes reais. Para tanto, note primeiramente
2
que o 2 - 4/3 < O, de sorte que, pondo k = f3 - : , temos k > O. Completando quadrados e
utilizando a substituição y = Tk (x + ~), obtemos sucessivamente

/ 2
1
dx =
(x + ax + f3)L
J l
((x + o/2) 2 + k) 1
dx

1 / 1
= kL 2 t dx
( ( *(x + o/2)) + 1) (4.59)

=
1 /
kL
-/Te
(y2 + l)L dy.
Resta, pois, calcularmos as primitivas

J (y2: l)t dy,


onde t E N. Se t = 1. o resultado é dado pelo exemplo 40:
1
dy = arctg y + C,
/ y2 + l
onde C é uma constante r~al. Se t > 1, então, integrando por parte , obtemos

/
1 d - y f
d (( 2
(y2 + l)t-1 Y - (y2 + l)t-1 - • Y. dy Y + l
)1-t)d
Y
y
= (y2 + l)t-1 + 2(t - 1)
J y2
(y2 + l)tdy

= (y2 :1i•-• + 2(t- 1); cy2 +\i•-1 - (y2 ~ l)') dy.

Resolvendo a igualdade acima para f (y2 ~1) 1 dy, obtemos finalmente

1 d 1 y ( 2t - 3) / 1 , (4.60)
/ (y2 + l)t y = 2(t - 1) • (y2 + l)t-1 + 2t - 2 (y2 + l)t-1 dy.

A ideia por trás da fórmula acima é que, se pudermos calcular f (y2 +;)t-i dy, então podemos
J
calcular c112~i)t dy. Como, de inicio, fomos capazes de calcular Y2~ 1 dy, segue que podemosJ
calcular J(
112~ 1>,dy, para todo t E N.

Resumimos a discussão anterior com o resultado a seguir. Para o enunciado do mesmo, dize-
mos que / : / ➔ R é uma função elementar se / pode ser obtida a partir de funções racionais,
trigonométricas, logarítmicas e exponenciais, pela aplicação de um número finito de operações
usuais sobre funções (quais sejam, adição, subtração, multiplicação, divisão, radiciação, compo-
sição e inversão).

294
TEORE~lA 4.65.
Se J.g : 1R ➔ 1Rsão funções polinomiais não nulas, então a primitiva f ~~=~clx pode _..
calculada explicitamente em termos de funções elementares. Ademais, a expressão que a define
em termos de tais funções vale em 1R\ {x 1, ... , xk}, onde x 1, ... , x1csão as raízes reais de g.

Seja dada uma função racional não nula L.


g
A fim de pôr em prática a discussão que antecede
o teorema anterior, temos de ser capazes de:

{i) escrever f como em (4.56);


{ii) fatorar g como em (4.55);

(iii) decompor ~ em frações parciais, como descrito no teorema 64.

A execução do item (i) é sempre viável, uma vez que, como deve ser familiar ao leitor, sempre
podemos dividir polinômios de coeficientes reais, obtendo como resultado um quociente e um
resto polinomiais satisfazendo as condições desejadas {i.e., sendo r o resto, temos r = O ou
O< ar< ag).
A execução de (ii) é imponderável; de outra forma, não existe um algoritmo simples para
fatorar g como em {4.55). Portanto, a obtenção de uma tal fatoração para g deve ser analisada
caso a caso.
Por fim, se conseguirmos cumprir o item {ii), a execução de (iii) é sempre possível, sendo
conhecida como o método dos coeficientes a determinar. Para entender como ele funciona,
suponha dadas funções polinomiais r e g, tais quer= O ou O < ar < ag. Então, o teorema 64
garante que

r(x)
g(x)
(4.61)

para certas constantes reais c1 t, d1 t e e1 t. Como já executamos (ii), estamos supondo que, na
última expressão acima, conhecemos os coeficientes der e x 1 , ... , Xk, a1, /31, ... , a,, /31. Portanto,
para encontrar os reais c1 t, d1 t e e1 t, temos simplesmente que: (a) reduzir a expressão após a
segunda igualdade em (4.61) a um mesmo denominador; (b) igualar os coeficientes dos monômio
de mesmo grau do numerador obtido e der; (c) resolver o sistema linear de equações resultante
(e cujas incógnitas são exatamente os coeficientes c1 t, d1 t e e1 t). (Observe que tal si tema ~cmpre
admite soluções, exatamente porque o teorema 64 garante a existência dos reais CJt, dJt e C;t·)

295
1 , 1• 1 1 1 1 1 , 1 .\ I'\ 1 1·<: B \ 1. 1>I·: H 11·.\ 1.\ .\ .\

A execução dos procedimentos acima é mais bem compreendida em um exemplo específico.


i ele, observe que já assumimos cumprida a etapa (ii).

ExEMPLO 4.66.
Obtenha as primitivas da função racional

R(x) = 10x4 + 42x3 + 85x2 + 84x + 16.


(x + 1)2(x2 + 2x + 3)2
~-------~~~---- ------------

SOLUÇÃO.
De acordo com (4.61), temos k = l = 1 e devemos começar procurando reais C1t, d11 e elt tais
que
R( )
x
'°' 2
'°' d1tx + eu
= 8 (.x+ l)t + 8 (x2 + 2x + 3)t.
C1t
2

Escrevendo simplesmente Ct, dt e et no lugar de C1t, d1t e e1t, segue da igualdade acima que
devemos ter
4
1:0x + 42x3 + 85x2 + 84x + 16 = c1 (x + l)(x 2 + 2x + 3)2 + c2 (x 2 + 2x + 3)2
+ (dix + ei)(x + 1)2 (x2 + 2x + 3)
+ (d2x + e2)(x + 1)2.

Comparando os coeficientes dos monômios d~ mesmo grau em ambos os membros da igualdade
acima, obtemos claramente um sistema linear de cinco equações e cinco incógnitas, cuja solução
(faça os cálculos para obtê-la!) é c 1 = O, c2 = 3, d1 = O, e1 = 7, d2 = 2, e2 = -5. Portanto,
3 7 2x - 5
R (x) = -- 2 + ---+----.
2 2
(4 62)
(x+1) x +2x+3 (x +2x+3) 2 •

Para o que falta, observe inicialmente que


1 1
( X + l )2dx =-X + l + e. ,. (4.63)
/
Por outro lado, como

2
x' + 2x + 3 = (x + 1) + 2 = 2 ( ( x-;/ )' + 1),

aplicando a substituição y = xj/ obtemos

1 d 11 l2 d 11 ./2 d
,/ x 2 + 2x + 3 x =2 ( xÃl) + 1 x =2 y2 + 1 y
(4.6-1)
= ~arctgy +C = ~ arctg (x;/)+e.
296
Por fim,

f (x !\:!3)2dx =f
2 Cx' !\!! 3)2 - (x' + ;x + 3)2) dx
1
= - x 2 + 2x + 3 -
J 7
(x 2 + 2x + 3)2 dx
(4.65)

e, como acima (veja (4.59) e (4.59)), obtemos

J 1 d _li
(x 2 + 2x + 3)2 x - 4
.,/2, d
(y 2 + 1)2 y

-- J2 ( y
4 2(y2 +
!J
1) + 2
1 d)
y2 + 1 y
(4.66)

x+l 1 (x+l) C
= 4(x2 + 2x + 3) + 4/2 arctg J2 + •

Calculando primitivas em (4.62) e substituindo (4.63), (4.64), (4.65) e (4.66) em seguida,


obtemos
3 7
J R(x)dx = -- - +- -arctg (-x_+_l)- __2 l __
x + 1 ./2 ./2 x + 2x + 3
7(X + 1) 7
- 4(x2 + 2x + 3) - 4./2 arctg
(X+
J2 +
C 1)
2
19x +42x+47 35 rd (x+l) C
= - 4(x + 1)(x 2 + 2x + 3) + 4J2 ª g J2 + •

Gostaríamos de terminar esta seção com um comentãrio muito importante sobre as limitações
das técnicas de integração que estudamos ao longo deste capítulo.
Apesar de todos os esforços que engendramos para calcular exatamente integrais e primitivas
de funções contínuas, observamos que há funções contínuas bastante simples cujas primitivas não
podem ser construídas explicitamente a partir de funções elementares. Um exemplo relevante
dado pela função de Jacobi 8
j J 1 + k sen t dt,
2 (4.67)

onde k é um real positivo.


Ia literatura do Cálculo, é usual dizermos que (4.67) é uma função elíptica ou, ainda, uma
função altamente transcendente. A explicação de tal nomenclatura é devida ao fato de que
8 Após Carl Gustav Jacob Jacobi, matemâtico alemão do século XIX.

297
( \1'111 11 l 1 :\ l'\ 1 f,,(;(L\I. l>F Hll-:.\1.\:\:\

to.i' funções aparecem naturalmente no cálculo do comprimento de elipses. De fato, conforme o


leitor verá no problema 7.5, o comprimento de uma elipse de eixo menor 2b e distância focal 2c
é dado pela integral elíptica

2b 11r 1+ (~)2
sen t dt. 2 (4.68)

Problemas - Seção 4. 7
7.1. Em cada um dos itens a seguir, calcule as primitivas pedidas, explicitando os intervalos
em que as mesmas têm sentido:

(a) f cos(logx)dx. (e) J 1.;_ezdx.


2
3x +2 d
(b) J :z:3+2:z:+l X. (f) f x2~1:+ 3 dx.
(c) J 1_:x2dx. (g) J (x-l)(x+I)
x 2 +2x+3 d
2 X.

(d) J ekx sen x dx. (h) J (:z:2+:z:+1)2


3x +2x+l d
2
x.

7.3. O propósito deste problema é calcular é\5primitivas da função ti--+ sec3 t, onde sec denota a
restrição da função secante a um intervalo da forma ( -i + br, i + k1r), com k E Z. Para
tanto, faça os seguintes itens:

(a) Mostre que sec' t = tg t • sec t e que ! log Isec t - tg ti = - sec t.


(b) Conclua que J sectdt = -log Isect - tgtl + C.
(c) Escreva sec 2 t =I+ tg 2t e integre Jsec3 t dt por partes para obter

/ sec3 t dt = - log Isec t - tg ti + sec t · tg t - j sec t dt3

e, daí,
/ sec3 t dt = ~ (sec t • tg t - log Isec t - tg ti) + C.

7.4. Use o resultado do problema anterior para completar o exemplo 44. Mais precisamente,
mostre que o comprimento do arco da parábola y = x 2, situado entre os pontos (O,O) e
(b, Ir), com b > O, é igual a

1
bJl + 4b2
2log( ✓1 + 4b
- 2 - 2b).

298
7.5. Seja E uma elipse de eixo maior AA' e eixo menor BB', com AA' = 2a e BB' - 2b.
Fixado o sistema cartesiano tal que A'(a, O) e B'(O, b), é bem sabido que E tem equação
~ +· ~ = 1. A esse respeito, faça os seguintes itens:

(a) Se f: [-a,a] ➔ 1Ré a função tal que J(x) = ~Ja2 - x2, mostre que o gráfico de f
é a porção de E situada no semiplano superior em relação ao eixo das abscissas. Em
seguida, conclua que o comprimento de E é dado por limE-+O 2 J~~~E 1 + a2c";:.x2) dx. J
(b) Faça e = J a2 - b2 e x = a cos t, com O < t < 1r. Em seguida, use a fórmula de
integração por substituição para mostrar que o comprimento de E é dado por (4.68).

Para os problemas (7.6) e (7.8), denote

R(x ) = """
L...O$i,j$m
a·•J xiy 3
,y """ b-iJ'
(4.69)
L...O$iJ$n 13X Y

com ai 3 , b.j E 1Re não todos nulos.

7.6. Se J(t) = R(cos t, sen t), dizemos que R é uma função racional de cos t e sen t. Para
calcular
j J(t)dt = j R(cos t, sen t)dt,

faça os seguintes itens:

l-s 2
(a ) Se s = t g 2t , most re que cos t -_ 1+s2 e sen t _- 2s
1+s2·

{b) t-.Iostre que a substituição t = 2 arctg s (o que equivale as= tg ½)garante que

J(
2

J J(t)dt = R
1- s
l+s 2
,
2s )
l+s 2
2
l+s 2
ds.

(Observe que s ~ R ( ~~:;, 1!:2) 1_;8 2 é uma função racional ordinária de s, de forma que,
em princípio, podemos obter suas prim: ~: BSvia decomposição em frações parciais.)

7. 7. Use o procedimento descrito no problema anterior para obter sucessivamente

J sec t dt = j-cost dt = j
1
-
2
1 - s2
ds
1- s 1 - tg .!:.
= log -- + C = log ; + C.
1+s 1 + tg 2

Em seguida, mostre que o resultado coincide com aquele do item (b) do problema 7.3.

299
( \l'I 11 1 ( 1 1 .\ h I 1-.CB \(. Ili·: Hti-:\1 \:\:\

7.8. Se R(x, y) é como em (4.69), adapte o procedimento descrito no problema 7.6 para reduzir
o problema do cálculo de
j R(t, Ja 2 - t2 )dt

(a > O uma constante real) ao cálculo de


2

J R (a(l - s
1 + s2
)
'
2as ) 2a d
1 + s2 (1 + s 2 ) 2 s.
2
Em seguida, use a primeira parte para calcular J :/~3:r dx.

7.9. Acesse a página www.wolframalpha.com e utilize o WolframAlpha para caJculnr a~ mtc-


grais (definidas ou indefinidas) dos problemas 1, 3, 7 e . Por exemplo, pnra calcular
J cos(logx)dx, f0 x 5 e-x dx e J ✓~0/ dx, digite respectivamente
1 2

indefinite integral of cos(log x);

definite integral of x-se-{-x} from O to 1;

indefinite integral
. of \sqrt{4-x-2}/x-3.

300
4.8 Integração imprópria
Jesta seção, estendemos a integral de Riemann ao caso de funções f : 1 ➔ R, não necess&r
riamente limitadas e definidas em um intervalo arbitrário J.
Se 1 C 1Ré um intervalo, então J é de uma das formas [A, B], (A, B), [A, B), (A, B], (A, +oo)
[A, +oo), (-oo, A), (-oo, A] ou (-oo, +oo) = JR,com A, BE JR. A fim de não precisarmos con-
siderar uma definição de integral para cada um de tais casos, adotamos as seguintes convenções:

(i) referimo-nos aos números reais A, B ou aos símbolos -oo e +oo como as extremidades de
I (muito embora -oo e+ sejam simplesmente símbolos formais, e não números reais),
as quais serão genericamente denotadas por o e /3;

(ii) impomos que -oo < A, B < + , para todos A, BE lR;

Assim, se I tem extremidades a e {3, com a < f3,então

(a,/3) e I e I \ (a.{3) e {o,/3}.

Por exemplo, se / tem extremidades a = A e f3 = + , temos (A, +oo) e I e I \ (A, +oo) e


{A,+ }, de forma que/= (A,+ ) ou (A,+ )
Fixado um intervalo I. supomos dada uma função f : / ➔ lR, tal que f é integrável em todo
intervalo [a,b] e I. (Em particular. se/ - [a,b],então supomos que fé integrável em J.) Isso
implica que / é limitada em cada inten·alo (a, b] C I, muito embora possa ocorrer de f não ser
limitada em /. Observe, contudo, que se f ê contínua em I, então, ainda que f seja ilimitada
em /, o teorema de \Veierstrass 53 garante que ela é limitada em todo intervalo [a, bJe I.

Sejam I e IR um intervalo de extremidades o, /3, com o < /3, e / : I ➔ R WD$ fuil~


tegrável cm todo inten·alo [a, bJ e I. Dizemos que f é integrável em / se, para~
E (a. /3). existirem os limites
11
!~ l:r: J(t) dt
f:r::r:o e lim 1
11➔/3 :r:o
f (t) dt.

do esse o caso, definimos a integral imprópria de f em / denotada. J: f(t) dt,


1

1.
o
/3f (t) dt = lim 1:r:ºf (t) dt +
:r:-+o :r:
lim
11➔/3
1 11

:ro
/ (t) dt,

• mos também que a integral imprópria J: f(t) dt converge.

301
<. \ l 'I 1 1 1 t > I :\ h:TE<:B.\L DE ílIE:\l.\~X

Note que a definição anterior independe do x 0 E I escolhido. Mais precisamente, suponha


que :ro é tal que existem os limites (4.70), e seja x~ outro ponto de I. Então,

xóJ(t) dt = 1xo J(t) dt + 1:z:óJ(t) dt,


1X X XO

de forma que também existe o limite limx-+oJ;óJ(t) dt, com

xó 1xo 1:z:ó
lim J(t) dt = lim J(t) dt + J(t) dt.
X-+Q 1 X x-+o X ~

J;óJ(t) dt, com


Analogamente, também existe o limite limy-+.B

= 1:z:º+ 1
11
lim
y-+{3 1 y
x'o
J(t) dt
x'o
f (t) dt lim
11-+/J xo
f (t) dt.

Portanto, uma vez que J


XO
XO
J(t) dt + f ,º J(t) dt = O, temos
X
Xo

Iim
x-+o 1x

f (t~ dt + lim
y➔/3
1 Y
:i:ó
f (t) dt = lim
.r_.o
1.ro
-.r
J(t) dt + lim
y-+/3
1!1
xo
J(t) dt.

Por outro lado, no caso em que I =


B), afirmamos que a definição anterior coincide com


[A,

a definição usual de integral. Para verificar isso, seja f : [A,B) ~ IR (limitada e) integrável, e
fixe x 0 E (A, B). Corno f ê, por hipótese, integrável em todo intervalo fechado e limitado, Lemos
f integrável em [A, e, para x E
xo] (A, xo),

Sendo M uma cota superior para lfl em [A,B], segue da desigualdade triangular para integrais
e da proposição 11 que
L:z:f(t) dt < Lx lf (t)ldt < J\1/(x- A),

de forma que, pelo teorema do confronto, limx-+A J; J(t) dt = O. Logo,

xo
lim
x➔ A
J(t) dt = 1xoA J(t) dt - lim
1x
f (t) dt = 1:z:o f (t) dt.
1x x-+A A A

Como f também é integrável em [xo, B], um argumento análogo ao acima permite concluir que

lim
y-+B lxo
fYJ(t) dt = fª J(t)
lxo
dt - lim
y➔B
1ª f (t) dt =
y
1 8

xo
J(t) dt.

302
Por fim, invocando a proposição 20, obtemos

lim
z-+A J.
~
zo
J(t) dt + lim
y-+B
J.Y
J(t)• dt = 1zo
A
J(t) dt + J.B
J(t) dt = 1B
J(t) dt.
A
• zo xo

Graças às discussões acima, podemos restringir a análise do conceito de integral imprópria


aos casos em que I = (A, B], [A, B), (-oo, B] ou (A,+ ). o restante dessa seção, discutimos
os casos em que I = [A,B) ou (A,+oo), deixando ao leitor a tarefa de formular os resultados
análogos para os casos em que I = (A, B] ou (-oo, B].
Vejamos alguns exemplos.

EXEMPLO 4.68.
A integral imprópria J/ 00
e-t dt converge. :t,.Iaisprecisamente, temos J/ 00e-t dt = l.

PROVA.
Para x > O, segue do TFC que

:r -tdt ,,-:r
1O
e = -e -t
t=O
= 1- e -x .

Agora, segue do teorema 60 que limx-+-+ e-r = O e, daí, J/ 00


e-t dt = l.

A mtcgral J/ 00
tº dt converge se, e só se, a: < -1.
-------~-~-~~~----------'
PRO\.\
Para x > 1 e a-=/- -1, observe que
:r 1 1t=:r 1
/,1
tº dt = --t 0
+1 = --(xº+l - 1). (4.71)
O:+ 1 lt=l Q + 1

Agora, como x 0 +1 = e(o+l) logx, temos (novamente pelo teorema 60) que

. o+l O, se a: + 1 < O
hm x
z-++oo
={ +oo, se a:+ 1 > O
.

Portanto, existe o limite (finito) do segundo membro de (4.71) quando x ➔ +oo se, e só se,
a+ 1 < O, i.e., se, e só e, a:< -1.

303
1 \ 1 1 1 1 11 1 1 .-\ J\: 1 1-:c; H , 1, r>r-: B rr:\ 1 , :\ :\

Por fim, se o = -1, noLe que


T/ 1
= logtl t =x = logx-+
J: dt
l=l
+oo

quando x-+ +oo.

Ainda em relação ao exemplo anterior, como



lim - = x-++oo
lim x 0
+1 = O
x-++oo x- 1

J 00
se, e só se, o < -1, costumamos dizer que 1+ tº dt converge se, e só se, xº decai para zero mais
rapidamente do que x- 1 , à medida que x -+ +oo. A esse respeito, veja também o problema 8.3,
1
que trata o caso 0 tºdt. f
ExEMPLO 4. 70.
Seja / : (a, b) -+ R uma função contínua em [a, b] e continuamente derivável em (a,b)
É imediato verificar que o limite que define o comprimento e do gráfico de f (cf. definição
42), res~e-se à integral imprópria J: JI + f'(x)2 dx. Assim, e existe se, e só se, tal integral
imprópria converge, em cujo caso temos e = J: J1 + f'(x) 2 dx. Também, se f é positiva em
(a, b), então o limite que define a área A da superfície de revolução gerada pela rotação do

gráfico de / em torno do eixo das abscissas (cf. definição 48) resume-se à integral imprópna
21rJ:f(x)JI + f'(x) 2 dx. Então, a área A pode ser calculada se, e só se, tal integral imprópria
• b
converge, e, nesse caso, temos A= 21rfa f(x) JI + f'(x)2 dx.

O teorema a seguir é o resultado fundamental acerca da convergência de integrais impróprias,


sendo o análogo, para integrais impróprias, do teorema 28. Para o enunciado do mesmo, bem
como para a observação que o sucede, o leitor pode achar conveniente reler o enunciado do
problema 2.7, página 90.

TB0REMA 4. 71.
Seja/: (A, +oo)-+ Ruma função integrável em todo intervalo [a, b] e [A, +oo). Então, as
ammações a seguir são equivalentes:

(a) /1 00
f(t) dt converge.

P•• todo t > O,existe M > A tal que Xi, X2 > !vi => lfx~2 J(t) dtl < E.
L e R tal que, para toda sequência (xn)n~l em [A, +oo), com limn-++ooXn = +oo,
=~--~-----,
J;"f(t) dt = L.
Jlm..-.+oo

304
l'\11.<;H\f,·\11 l\ll'B<>l'lil\ 1

PRO\'.
ja F: [A + ) -+ a função da por F(x) = fz:f(t) dt.

(a)=} (b): uponha qu J; f(t) dt con erg com J+ J(t) dt = L. Então por d finição, t mo
limx + F(x) = L d modo que dado > O I > O tal qu

x > I ⇒ IF(x) - LI < 2.
Port nto para x 1 x 2 > 1 gu da d igualdad triangular qu

[' f(t) dt = IF(x,) - F(xi)I

< IF(x2) - LI+ IL - F(xi)I


<2+2=
(b) =} ( ): uponha qu ondiç- do i m (b) ja (xn)n~I uma equ n ia tal
qu Xn > p r t d > 1 limn + Xn = + D do > O tom 1 orno no nun iado
no E I qu >no=} n > !. n - . p r m n > no t mo Xm, n > M , daí,

P rl n (F(xn))n~l um qu·n i d u h' l g on rg n , p lo t r m 2 .


L = hmn-++ F( n) rm qu hmn-++ F(x~) = L par toda qu·n ia ( ~)n~l tal
qu . ~1 > > 1 lii ~ = + . f o, di us ão no parâgrafo an rior
( 111 (J·~)n~I n ( n)n~l xi ·n i d L' = limn-++ F(x~). M fo
L' =/-L n - u·n i ( ~)n~l d p k ~ 1 por x;k-l = X2k-1 x~k = ;k ri al qu

P r n omo L =/-L' n - xi iria limn-++ F(x~) o qu , uma ontradi ão ao argum no do


n rior ( om (x~)n~l no lugar d (xn)n~1).

( ) =} ( ): uponh mo qu xi ta L E tal qu para toda qu·ncia (xn)n~l m [A + m


limn-++ Xn = + nhamo limn-++ F( Xn) = L. limx-++ F( X) não xi t ou n -
L n ~o xi m > O para odo B > um r al B > B t 1qu IF(xs) - LI > . nC ,
tomando B u ivam n igu l + 1 + 2 A + 3, . . . ob ríamo r i
qu Xn > + n IF( n) - LI > para odo n > l. Logo limn-++ Xn =+ m (F( ,1)) 11
>1
n -o on rgiri para L o qu ' um ab urdo.

30
ÜBSERVAÇÃO 4.1.
É imediato adaptar o resultado anterior ao caso de uma função f : [A, B) ----+IR,integrável em
todo intervalo [A, b] tal que A < b < B. Mais precisamente, e conforme o leitor pode verificar
facilmente, as seguintes afirmações são equivalentes:

(a) J: f (t) dt converge.


{b) Para todo f ; O dado, existe ô> O tal que B - ô < x 1 , x 2 < B =} IJ;J(t) dtl <
2
L

{e) Existe L E 1Rtal que, para toda sequência (xn)n~l no intervalo [A, B), com limn-++ .c,1==
B, temos limn-++ooJ;nJ(t) dt = L.
~----
A seguir, estabelecemos uma importante consequencia do teorema anterior, conhecida na
literatura como o teste da comparação para integrais impróprias.

PROPOSIÇÃO 4.72.
Sejamf,g: [A,+oo)----+1Rfunções tais que IJ(i·)I < g(.1:),para todo .e E [A,+oo). Se f e
g são integráveis em todo.intervalo [a,b] e [A,+oo) e J...7g(t)dt converge, então J...~f(t)dt
00 00

converge e

PROVA.
Inicialmente, observe que g(x) > O para todo x > A, de forma que J: g(t) dt > O. Então,
00

dado f > O, a convergência de


00
J;
g(t) dt garante, mediante o teorema 71, a existência de
M > A tal que
X2

X1, X2 > JU=?


1
X}
g(t) dt < f.

Agora, a desigualdade triangular para integrais, juntamente com a proposição 11, garante
que, também para x 1 , x2 > /'vl, temos

2 2
fx J(t) dt < fx IJ(t)ldt < {X'J g(t) dt < f.
Íxi Íx1 Íx1

Então, novamente pelo teorema 71, concluímos pela convergência das integrais J: 00
J(t) dt e
J:00
1/(t)I dt.

306
Por fim, segue do que fizemos acima e das propriedades de limites de funções (cf. seção 3.1)
que

L-rOO J(t) dt = lim {:e J(t) dt = lim {z J(t) dt


z-+-oo }A :e➔+ }A
< lim rr lf(t)ldt < lim rz g(t) dt
:c➔~oo}A z_.+oo}
A

= L-oo g(t) dt.

ÜBSERVAÇÀO 4.2.
Como na observação anterior, a adaptação da proposição 72 ao caso em que I = [A,B) é
iuwdiata. Mais precisamente, se as funções J,g : (A.B) ➔ 1Rsão tais que 1/(x)I < g(x), para
todo :r E [A,B), e se J e g !:>ãointcgrá\'cb em todo intervalo [A, b], com A < b < B, então a
convcrgência de J::g(t) dt acarreta aquela de J: /(t) dt: ademais, nesse caso vale a desigualdade
1

{11 {º
l~ J(t)dJ < }A g(t)clt.

A seguir, colecionamos do1~ex mplo~ 1 1 ,m1t de aplicação da proposição anterior.

Exg~tPLO 4. 73.
Utiliz<.'mosa propo~ição anterior para estabelecer a convergência da integral de Dirichlet
sen t
1
+
-dt.
o t
Parn tanto, obsen·e inicialmente que, pelo limite trigonométrico fundamental (cf. lema 9), a
função/ H -f1 estende-se continuamente a O. Portanto, é suficiente estabelecer a convergência
de f1+< • ;" dt.
Parn o que falta, para x > 1. segue da fórmaj, de integração por partes que

1
:csent
-dt =--
COS f 1t=z
-
1z COS
-dt2
t
l f

= cosl
t t=l
cosx
- --
X
-
1z 1

1
t
cost
--dt.
t2

Como lcos xi < 1 para todo x, concluímos dos cãlculos acima que J1+ -:1'
00
dt converp

se, Jtx.(it dt converge. Portanto, basta estabelecermos a convergência dessa (ai •


imprópria.
-~-----------~~------------~___._,.__

307
1 1 1 1 1 1 1 ' 1 \ 1'\ 1 1 ( , I: \ 1 1) 1 1~ 1(· \ 1 \ '\ '\

1
Por fim, é suficiente aplicar a proposição anterior, observando que 1~:r 1 < zi e que (cf.
exemplo 69) J1+ fI dt converge.
00

Para uma outra demonstração da convergência da integral de Dinchlet, ,·eja o problema 1 1 .


página 340. Por outro lado, é possível mostrar (cf. apêndice ao capítulo 6 de 1191.por exemplo)
que
+oosen t
1
1r
-dt
t
= -.
2
0
Por sua vez, esse câlculo é de importância fundamental para a teoria de série~ d< Fo111 i<'t' (cf.
problema 4.6, página 366). A Figura 4.15 esboça o gráfico da funçiio x H ;:r: .

• 1.0

• 0.5

-80 o
-0.5

Figura 4.15: gráfico de xi-+ c;:r.

Segue da proposição 72 que, se J : [A, +oo) ➔ R é integrável em todo intervalo 1o, b] C


[A,+oo) e J; lf(t)I dt converge (obserYe que, pela proposição 13, lfl também
00
é integrável em
todo intervalo [a, b) e [A.+oo)). então J-:00J(t) dt converge e

[ J(t) dt < [~ IJ(t)I dt. {4.72)

Em um tal caso, dizemos que J: J(t) dt é absolutamente


00
convergente. ·em toda inte-
gral imprópria convergente é absolutamente convergente; de fato. o problema 1.19, págma 3-l0,
garante que fo+oo11e~
t dt não é absolutamente convergente.

O próximo exemplo utiliza a proposição 72 para introduzir e discutir algumas propriedade;


da função Gama.

308
1 , 11 <,li\< \« 1 l\ll'Ht ll'lil \

EXEMPLO 4 74.
A função Gama é a função r: (O:+oc) ➔ (O.+oc), definida por

r(x) = !.+ ,-•1.r-l dt..

Corno
I' ,-•t'- 1
dt. = J.',-•e- dt.+ 1~,-•1r-l
1
00

dt,

para ver que f(x) está bem definida é suficiente mostrar que as duas integrais impróprias do
segundo membro acima convergem.
Para o que falta. observe inicialmente que. como x > O. o problema 8.3 garante a convergência
de Jd 1:r:-l dt. Portanto. como c-tF-
1
< rz:-1 para O < t < 1, a observação 2 (modificada de
maneira ób,·ia, a fim de er aplicada ao intervalo (O.1)) garante a convergência de f0 e-ttr-l dt.
1

Por outro lado, fixado x > O. afirmamo. que e-tt,z- 1 < Ce t 2 • para t > 1, onde C é uma
constante positiva que só depende de x. De fato. sendo n um natural maior que x + 1, segue do
tcor<>maGOque
t"
lim
t-+
.J/"'
(:;" -
= O.
Portanto, existe C' > O tal que c~j
2 < C, para todo t > 1. Ma.-,,sendo assim, ternos

t -ti:r:-1 < ,-tfn < cc-t/2, (4.73)


pnrn todo t > 1, confm mt• dc::;ejado.
Por fim, veja que uma variame óbvia do Pxemplo G gc1rante a convergência da integral
imprópria Jt < -t/ 2 dt. Portanto, a dl~">igualdadc(4.73) permite aplicar a proposição 72 (com
A = 1, J(t) = e-' /7 - 1 e g(t) = Cc-t1 2 ) para concluir que J1+ e trr 1 dt converge.
A funçào Gama tem muitas aplicações importante.:; cm Matemática, notadamente à Anfise,
à Fisic,, :\lntcnu\tica e à Teoria do~ Números. Em última análise, isso se deve ao fato de que ela
estl'llcll', pnrn o~ reais positiYos. a noção de fatorial de um número natural. Para verificar ~a
última afirmação, comecemos notando que. pelo exemplo G8,
r+oo
f(l) = L e-tdt = 1.
Jo
Por outro lado. para x > O, a fórmula de integração por partes fornece

r(x)

309
' \1·111 111 1 .\ f\:IJ·:<;JL\I. 1>1·:HIJ-:\1.\:'\:\

Agora, de Reore.locom o teorema 60, temos lims-++oo~e- 8 sx = O. Portanto, segue dos cálculos
adma que

Então,
r(x + 1) = Xr(x)
para todo x > O, e segue por indução, a partir de r (1) = 1, que r (n) = (n - 1)!, para todo
nE N.

Problemas - Seção 4.8


8.1. Examine a convergência das integrais abaixo:

" 1
(a) .J!½ sec t dt. (c) Ío lo~t dt.
00
{b) J0+ e-t sen t dt .. (d) +oo i dt
f-oo l+t 2 •

8.2. * Sejam J : (A, +oo) ➔ 1Ruma


função contínua e g : (B, +oo) ➔ [A, +oo) uma fun-

ção derivável, com derivada integrável em cada intervalo (a, b) e [B, +oo) e tal que
00
limx-++oog(x) = +oo. Prove que f9~; J(t) dt converge se, e só se J; J(g(s))g'(s) ds
converge. Ademais, sendo esse o caso, prove também que

+oo J,+oo
J.
g(B)
J(t) dt =
B
J(g(s))g'(s) ds.

8.3. * Mostre que f0 tº dt converge se, e só se, a > -1. Em seguida, para a < O, conclua que
1

f01 t° dt converge se, e só se, xº cresce para +oo mais lentamente do que x- 1 , à medida
que X ➔ 0+.
00
8.4. É possível mostrar que J/ e_ 8:2 ds = Yf-.(Veja, por exemplo, o capítulo 8 de (27].) Use
esse fato para calcular r( ½)= .J,rr.

8.5. Mostre que, para x > O, a integral J/00e-t ,x-l log t dt ê absolutamente convergente.

O propóHito do próximo problema é generalizar o resultado do problema 3.12, página 2-13,


CHtahelccendo urna df'.l1tf1ualdade1,nlegralconhecida como a desigualdade de Holder.

310
8.6. Sejam I e R um intervalo de extremidades a e /3,f, g : I ➔ (O,+oo) funções intcgrávcís
(no sentido impróprio) e p, q > o.tais que !p + !q = 1.

(a) Se J: f (t)Pdt = J! g(t)q dt = l, mostre que J: J(t)g(l) dl < l.


(b) Mostre a desigualdade de Hõlder 9:

1 f3
J(t)g(t) dt <
(
1 /3
J(t)P dt
)1/p (
1
f3
g(t)q dt
)1/q

8. 7. O propósito deste problema é mostrar que a função r


é contínua e que a função log r :
(O,+oo) ➔ Ré convexa em (O,+oo). Para tanto, faça os itens a seguir:

(a) Mostre que log ré convexa se, e só se,

(4.74)

para todos x, y > O e todos p, q > O tais que ~ + ¼= 1.


(b) Aplique o resultado do problema anterior para mostrar que (4.74) é verdadeira, para
todos x, y, p, q como no item (b).
(c) Conclua que a função ré contínua.

9 Após Otto Holder, matemático alemão do século XIX.

311
.\ 1\11<,li\l. Ili 1{11 \I\\\

4.9 Mais aplicações à Física


&"ta última !;CÇàodeste capítulo retoma as aplicações do Cálculo à Física, iniciadas na seção
3. . Começamos examinando o processo físico de decaimento radioativo.

EMPLO 4.75.
O decaimento_ radioativo é um procPsso pelo qual um núcleo de um átomo instável perde
energia, emitindo radiação ionizante (i.e., ondas eletromagnéticas de certos níveis de frequência).
'Ia.isradiações incluem, por exemplo, partículas alfa, partículas beta e raios gama. Um decai-
mento radioativo pode ocorrer como resultado de uma colisão de um feixe de prótons de alta
energia (i.e., movendo-se a velocidades muito próximas da velocidade da luz) com um núcleo de
um átomo pesado. Isso é o que ocorre em todo acelerador de partículas, como, por exemplo, no
LHC - Large Hadron Collider , localizado em Genebra, Suíça). Contudo, há substancias, dilas
radioativas que emitem radiação ionizante espontaneamente, como, por exemplo, o Urânio 239,
que decai para Plutônio 239 com a emissão de partículas beta (por isso, dizemos que se trata de
um decaimento beta).
No início do século XX, E. Rutherford 10 verificou experimentalmente a validade da lei física
hoje conhecida como a lei de decaimento radioativo de Rutherford: a taxa temporal instan-
tânea segundo a qual uma certa substância radzoativa decai é diretamente proporczonal à massa

de substância presente. Ademais, a constanJ,e de proporcionalidade só depende da substancrn
em questão. Tal constante de proporcionalidade é denominada a constante radioativa da
substância.
Suponha que urna certa substância radioativa tem constante radioativa k > O, e denote por
M(t) sua massa no instante t, medido a partir de t 0 = O. A taxa temporal média de decaimento
da substância entre os instantes t 1 = t e t 2 = t + íit > t 1 , é definida por
fvl(t2) - A,f (ti)

Por definição,a taxa temporal instantânea de decaimento da substância, no instante t = t 1 , é o


limite dos quocientes acima, quando o intervalo de tempo decorrido ó.t tende a O. Ora, isso é
8Ílllplesmentea derivada M'(t 1), que, então, supõe-se existir. A lei de decaimento radioativo de
Rutherfordgarante, pois, que
l\I'(t) = -kl\í(t),
-_,doo sinal - devido a.o fato de que l\I'(t) < O, uma vez que a massa da substancia diminui
O"pa8881' do tempo.

10ErDHt Rutlwrford, físico n<.'07<•1a11cl~s dos i.éculos XIX e ~X, ganhador do prêmio. obel de Qwmica de 190 ,
por sua J>CHClllÍHBttt,lm• f1•11ô111t•11osrndioat ivos.

312
Supondo que a massa da substância radioativa presente no instante inicialto•
a M0, queremos calcular M(t) em funçffe,o
de t.

O problema matemático resultante da situação física descrita no exemplo anterior é um caso


particular do problema de resolver a EDO y' + ky = O para uma função-incógnita derivável
y = y(x), definida em um intervalo contendo O e satisfazendo a condição inicial y(O) = y0 , onde
Yo é uma constante real dada. Como na proposição 29, dizemos que

y' + ky = O
{ y(O) = Yo

é um problema de valor inicial associado à EDO y' + ky = O. Por sua vez, esse problema é
um caso particular de uma situação ainda mais geral, resolvida pelo teorema a seguir.

TEOREMA 4.76.
Sejam I um intervalo aberto, x 0 E /, Yo E R e / : / ➔ R uma função contfnua dada. O
problema de valor inicial
y' + f(x)y = O
{ y(xo) = Yo
tem como única solução a função y : / ➔ R tal que

(4.75)

Heuristicamente, para resolver a equação y' + J(x)y = O, supomos que a solução y é positiva
em uma vizinhança de x0 , de forma que

y'(x) = - f(x).
y(x)

Em seguida, integramos ambos os membros da igualdade acima entre x 0 e x (o que tem sentido,
uma vez que estamos supondo f contínua), para obter, com o auxílio do TFC,

:r 1x y'(s) 1:r
logy
l
xo
=
:ro
-ds
y( S)
= -
:ro
f(s)ds.

Como y(xo) = Yo, segue que


log y(x) = logy(x) - logy(xo) = -1:r J(s)ds;
Yo :ro

por fim, exponenciando, chegamos a (4.75).

313
< \1·11111) 1 .\ I\ 11-:c;JL\I. l>I·: HIE:\l.\.\.\

Observe que, uo final das contas, se y0 > O, então a solução dada por (4.75) é realmente
positiva. e está definida em todo o intervalo I. Contudo, como chegamos à solução nos apoiando
cm hipóteses que não sabíamos serem satisfeitas, a discussão acima não constitui uma prova.
Apesar disso, vereinos a seguir que ela dá uma pista muito boa sobre como provar o teorema.

PROVA DO TEOREMA 76.


Suponha que y _:I ➔ IRresolve o problema de valor inicial do enunciado, e seja Y : I ➔ IRa
função dada por

Então, Y é derivável e, pela regra da cadeia e pelo TFC, é tal que

Y'(x) = eI:of(s)cu f (x)y(x) + ef:of(s)cLsy'(x)


= eI:af(s)cu(J(x)y(x) +y'(x)) = O.

Portanto, o resultado do problema 4.1, página 166, garante que Y é constante e, como Y(x 0 ) =
y(xo) = Yo,concluímos que Y(x) = y0 , para todo x E I. A partir daí, é imediato que y é dada
como em (4.75).
Reciprocamente, sendo, y dada como em (4.75), é imediato verificar que y' + J(x)y = O e
y(x 0 ) = y 0 . Portanto, tal função realmente resolve o problema de valor inicial do enunciado.

Podemos, agora, voltar ao exemplo anterior, observando que, naquele caso, a função f é
constante e igual a k. Assim, de acordo com (4.75), a solução do problema de valor inicial

1\1'+ kAI = O
{ l\tl(O) = lHo

é a função
(4.76)
Voltando à discussão sobre equações diferenciais ordinárias, situações físicas há (conforme
veremos no próximo exemplo) em que se faz necessário considerar problemas de valor inicial do
tipo
y' + J(x)y = g(x) ,
{ y(xo) = Yo (4.77)

r
onde g : I ➔ ~ são funções contínuas dadas e Xo E I' YoE IR.
Uma tal equação pode ser facilmente integrada (i.e., resolvida) com o auxílio do teorema
anterior e de um método heurístico conhecido como variação dos parâmetros. Tal método

314
\1 \I" \l'l.11 \1, 1 >I..., \ I· 1"1' \ 1

consiste no seguinte: inicialmente, multiplicamos ambos os membros de (4.77) por uma função
auxiliar derivável e não nula h : J ➔. IR (à qual nos referiremos como o fator integrante),
obtendo
h(x)y'(x) + f(x)h(x)y(x) = g(x)h(x);
em seguida, procuramos h de tal forma que o primeiro membro da igualdade acima seja a
derivada da função hy.
Se isto for possível, deveremos ter

(hy)'(x) = h(x)y'(x) + f(x)h(x)y(x) = g(x)h(x); (4.78)

daí, o TFC dar-nos-á


hyr= = 1:r (hy)'(s)ds = 1:r g(s)h(s)ds,
:to xo :ro
de forma que
h(x)y(x) - h(x0 )y(x0 ) = E g(s)h(s)ds

ou, ainda,
y(x) = htx) ( h(xo)y(xo)+ 1: g(s)h(s)ds) . (4.79)

Portanto, para fazer o método de variação dos parâmetros funcionar e terminar de resolver
(4.77), resta mostrarmos que é possível escolher uma função derivável e não nula h : / ➔ IR tal
que (cf. (4.78))

h(x)y'(x) + f(x)h(x)y(x) = (hy)'(x) = h'(x)y(x) + h(x)y'(x).


A saída natural é impor que h satisfaça, em I, a igualdade h'(x) = f(x)h(x), i.e., que h resolva
a EDO y' - f(x)y = O. Conforme vimos no teorema anterior, podemos tomar

(4.80)

a qual é, de fato, uma função derivável e não nula, definida em /.


Por fim, um cálculo (enfadonho mas) direto permite verificar que, com h: / ➔ IR dada como
em (4.80), a função y: J ➔ IRdada como em (4.79) realmente resolve o problema de valor inicial
(4.77). (Evidentemente, ainda temos que fazer essa checagem, pois encontramos y supondo que
ela existisse, o que não é um procedimento logicamente correto.)
Vejamos um exemplo onde é necessário considerar o caso mais geral de (4.77). Uma vez que
a discussão das leis físicas concernentes ao mesmo é consideravelmente mais complicada do que
nos exemplos anteriores, caso ache necessário o leitor pode iniciar a leitura a partir do parl\grafo
que antecede a equação (4.83).

315
<. \l'l 11 111 I A l~TEGH,\L DE HIE~IA~~

EXEMPLO 4 77.
Grosso modo, um circuito elétrico é um dispositivo formado pela ligação de vários elemen-
tos elétricos, tais como resistores, indutores, capacitares, fontes de tensão, fontes de corrente e
interruptores, conectatos entre si de modo a formarem pelo menos um caminho fechado para o
füLxode corrente elétrica.
Por exemplo, um circuito elétrico simples, alimentado por pilhas, baterias ou tomadas, sempre
apresenta uma fonte de energia elétrica, um aparelho elétrico, fios ou placas de ligação e um
interruptor para ligar e desligar o aparelho; estando ligado, o circuito elétrico estará fechado e
uma corrente elétrica o atravessará; por sua vez, essa corrente pode produzir efeitos os mais
variados, como luz, movimentos, aquecimentos, sons etc.
Em um circuito elétrico, a tensão elétrica U entre as extremidades de um elemento é a
diferença de potencial elétrico entre as mesmas, i.e., a diferença de energia elétrica potencial,
por unidade de carga elétrica, entre os dois pontos em questão. No SI, a unidade de energia é
o Joule (abreviado J) e a unidade de carga é o Coulomb (abreviado C). Portanto, a unidade de
medida para tensão elétrica é J/C, também conhecida por volt (abreviado V), em homenagem
a Alessandro Volta, físico italiano dos séculos XVIII e XIX. Assim, dizer que a tensão existente
entre as extremidades de um elemento de um circuito é de 1V é o mesmo que dizer que 1C de
carga que atravessa esse elemento transmite lJ de energia.
Em um circuito elétrico, a corrente elétrica que atravessa um de seus elementos (dito,
então, um condutor) é o fluxo de elétrons observado no mesmo. A intensidade i(t) dessa
corrente, calculada no instante t, é a taxa de variação temporal inst~tânea· da quantidade ó.q
de carga elétrica que atravessa o circuito entre as extremidades do condutor em questão:
. ó.q dq
i(t) = hm - = -.
~t-+o ó.t dt
Assim, medindo cargas em Coulombs e intervalos de tempo em segundos, concluímos que a
unidade de medida de corrente é C/s, também conhecida como Ampere (abreviado A), em
homenagem a Adrien-Marie Ampere, físico francês dos séculos XVIII e XIX.
Um resistor é um elemento elétrico condutor utilizado em circuitos com uma de duas fun-
ções básicas: transformar energia elétrica em energia térmica ou limitar a corrente elétrica que o
atravessa (oferecendo, para tanto, resistência à passagem de carga). Resistores satisfazem a pri-
meira lei de Ohmll: uma tcusão U(t), aplicada às extremidades de um resistor, é diretamente
proporcional à intcm~idade i(t) da corrente que o atravessa. Denotando por R tal constante de
proporcionalidade, temos
(4 81)

11 Ap618C1•,1rgC.,i111011Oli111,fri;ic:o•de11rnodo s6culo XIX.

316
\I \I"- \l'LW \<:<>!·." .\ FhW.\ ,
1

e dizemos que R é a resistência elétrica do resistor. !\1edindo U em volts e i em Am~;


temos R medida em V/ A. unidade conhecida como Ohm (abreviamos f2). Assim, um reailt9r
com uma resistência elétrica de lf2 causará uma queda de tensão de 1V a cada IA de corrente
que o atravessar
A lei de Ampere do eletromagnetismo garante que um campo magnético é gerado sempre
que uma corrente elétrica atraYessa um condutor. Em circuitos elétricos, um indutor é um
condutor, construído geralmente como uma bobina de material condutor (como, por exemplo,
um fio de cobre) envolvendo um núcleo de material ferromagnético, que se vale da lei de Ampêre
para armazenar energia potencial elétrica no campo magnético gerado ao fazermos uma corrente
elétrica atravessar as várias ~piras da bobina. (Em Física, esse fenômeno de armazenamento de
energia potencial elétrica é conhecido como indução eletromagnética, razão do nome indutor.)
É possível mostrar que a tensão instantânea entre os terminais de um indutor é diretamente
proporcional à taxa de variação instantânea da corrente que o atravessa. Em símbolos,
di
U(t) = L dt, {4.82)

SPndo n constante de proporcionalidadl• L denominada a indutância do indutor. Assim, vemos


que L e medida cm V • ...,;A, unidade conlwcidn como Henry 12 e abreviada por H.
Em um circuito, um nó é um ponto ao qunl estão ligados dois ou mais elementos. Um
caiuinho é uma :sequêncin ele elemento!->ligados entre si. na qual nenhum elemento é incluído
mais de umn ,·cz. Um caminho é dito fechado se, ao percorrê-lo a partir de um certo ponto e
em um st'nl ido pré-fixado, C\'cntualmentc \·oltamos ao ponto de partida.
A anáhsr. de circuito~ elétdco.,; é ba-;cada na aplicação judiciosa da primeira e segunda leis
de Kirchhoff 1:l. A primeira lei de Kirchhoff. também conhecida como lei das correntes
ou dos nó~ afirma que, em um no qualquer de um circuito, a soma das correntes elétricas que
entrnm e 19110! à ,oma da_.,con·cntes que saem, de forma que um nó não acumula carga. De maior
intcr~:sc para nós !:leráa segunda lei de Kirchhoff, também conhecida como lei das malhas,
a qual nfirma que. cm todo circmto elétrico. a soma algébrica das diferenças de potencial entre
as e..rtremidadc_,;do:; elementos que compõem ....":'!.caminho fechado é sempre nu.la.
O circuito elétrico ilm,trado na Figura 4.16. conhecido com um circuito R-L, é composto
por: uma fonte de tensão. localizada à esquerda; um resistor de resistência R, localizado na
porção inferior; um indutor de indutância L. localizado na porção superior do circuito; elementos
lineare~ (fios). w1indo a fonte. o re~istor e o indutor, conforme mostrado.
l\essc circuito, uma diferença de potencial U(t) entre os polos da fonte gera uma corrente
elétrica de intensidade i(t). O efeito da presença do indutor, por sua vez, é gerar uma diferença
e tensão L*. resistiYa a variações de corrente. Portanto, percorrendo o circuito no sentido

317
( \1'11 t 111 I t\ l:'\TEC:H,\L DE HIE:\L\:'\~

horário a pnrtir do nó inferior esquerdo, concluímos, com o auxílio de (4.81), (4.82) e da lei das
malhas que U(t) - Ri(t) - L~~ = O.

U(t) --==-

Figura 4.16: circuito RL.

Escrevendo a equação acima como


di R. l
dt + L i(t) = L U(t), (4.83)

obtemos uma equação como aquela em (f 77), com variável t e y(t) = i(t), J(t) = ~. g(t) =
tU(t).
Por (4.80), o fator integrante de (4.83) é
rt Rds Rt ,.
h(t) = eJo r = er.

Portanto, segue de (4.78) que


!( e4\(t)) = ~ U(t)e fl,
1

de formaque, pelo TFC (e tomando o instante inicial t0 = O),


Rt. • l 1t Rs
e"L""i(t)- i(O) = L U(s)eT ds.
O

Se U(t) = sent, então, integrando por partes duas vezes (cf. item (d) do problema 7.1, página
298), obtemos
1 1(ersen n, 1 eTR• R ( t

L1
efi(t) - i(O) = - sds = - • --::---2 -sens - coss) 1,
o L ( ~) + 1 L o
de formaque

318
\I\I" .\1'1.1('\(,'()(·:S \ 1-'hl('\ ,

Portanto, após algum tempo a corrente sobre o circuito é essencialmente igual à corrente
estacionária .

~-----~~-~----
i,(t) ~ L2 1 (1
R2 sen t - cost).
-----------~
A análise matemática do problema físico de que trata o próximo exemplo levar-nos-á a
mergulhar um pouco mais no fascinante mundo das equações diferenciais.

EXEMPLO 4. 78.
Considere novamente as condições descritas no exemplo 81 e suponha que, além da força
restauradora da mola, atua sobre o corpo de massa m uma forma de amortecimento de intensi-
dade proporcional à velocidade do corpo, com constante de proporcionalidade e > O, a qual só
depende dos materiais de que são feitos o corpo e a superfície sobre a qual ele desliza. Então,
nesse caso, a segunda lei de Newton fornece a equação

mx"(t) = -kx(t) - cx'(t),

onde x(t) é a elongação da mola no instante t, k > O é a constante elástica da mola e e é


uma constante positiva, medida em ~s. (Observe que o sinal - na última parcela do segundo
membro deve-se ao fato de que a força de intensidade cx'(t) amortece o movimento do sistema
massa-mola.)
Portanto, a fim de descrever o movimento do sistema, precisamos resolvera EDO
e k
x"(t) + -x'(t) + -x(t) = O. (4.84)
m m

l\r1aisgeralmente do que no exemplo anterior, analisamos a seguir as soluções da equação

y" + ay' + by = O, (4.85)

onde a e b são constantes reais.


Começamos enunciando, para a equação diferencial y" - y = O, um resultado análogo ao do
teorema 79. Para tanto, recorde (cf. problema 6.22, página 287) que as funções seno hiperbólico
e cosseno hiperbólico são as funções senh, cosh : 1R--+ JR, definidas por senh x = ez- 2e-.r e
cosh x = ez+ 2e z. O item (a) do problema supracitado garante senh" = senh e cosh" = cosh, de
sorte que senh e cosh são soluções da equação y" - y = O.
12 Após Joseph Henry, físico americano do século XIX.
13 Após Gustav Robert Kirchhoff, físico alemão do século XIX.

319
, \1·111 11 i 1 _.\ [.'\ ri-:c:H.\L IH·: B1t-::\L\:'\\

De po se da discussão do parágrafo anterior, podemos adaptar facilmente a discussão que


levou ao corolário 80 para caracterizar todas as soluções da equação diferencial y" - >..y= O,
onde>..> O. Mais precisamente, mostra-se (cf. item (c) do problema 9.2) que
y'(O)
y(x) = y(O)cosh( v0:x) + v>.senh ( v>.x),
para todo x E .IR.
Precisamos, agora, do seguinte resultado auxiliar.

LEMA 4.79.
a2
0
Se y'' + ay' + by = O eu= e : y, então u" - !u = O, onde ô= - 4b.

PROVA.
A prova é um cálculo imediato de derivadas. Entretanto, é instrutivo aproveitarmos a opor-
tunidade para perceber o argumento heurístico por trás do fator e ª2.r. Para tanto, faça y = uv,
com u e v deriváveis, de forma que
O = y" + ay' + by = (uv)" + a(uv)' + b(uv)
= vu" + (2v' + av )u' + (v" + av' + bv)u.
Agora, impondo que 2v' + av = O, obtemos (a partir do teorema 76) v(x) = e-ª{ como uma
possibilidade. Com tal escolha, temos
,, , a2 a2 uA
v + av + bv = -v - -v + bv = - -v ..
4 2 4
e, daí, O= v (u" - tu). Mas, como v(x) =/O para todo x E lR, segue que u" -1u = O.

De posse do resultado anterior, podemos caracterizar completamente as soluções de (4.85).

TB0RBMA 4.80.
As soluções da equação diferencial y" + ay' + by = O são da forma
y(x) = e-9f (c1u1(x) + c2u2(x)),

onde c1 e ci são constantes reais e, sendo 6 = a2 - 4b:

(a) u1 (x) = 1 e u2(x) = x, se 6 = O.


(b) 111(z) = coeh( ~x) e u2(x) = senh ( ~x),se 6 > O.

(~) • coe( ~x) e u2(x) = sen ( ~ x),se 6 < O.

320
PROVA.
O lema anterior garante que, pondb u = eª{ y, obtemos y = e-ª{ u e u" - ¾u = O. Portanto,
basta mostrar que u(x) = c1u 1 {x) +c 2 u 2 (x), onde u 1 e u 2 são como em (a), {b) ou (c), conforme
seja 6. = O, 6. > O ou 6. < O.
O que falta segue do problema 4.2, página 166 (no caso 6. = O), do item (c) do problema 9.2
(no caso 6. > O) e do corolário 80 (no caso 6. < O).

Voltando a {4.84), temos (nas notações de (4.85)) a= ~, b = ! e 6. = 2


c -;_:2km, de forma que
há três casos a considerar:

(i) Sem= ~. então 6. = O e (pelo teorema anterior)

quando t ➔ + Nesse caso, o amortecimento do sistema é dito crítico.

(h) Sem< ~~' então 6. > O e (novamente pelo teorema anterior)

x(t) = ,->~( A cosh ( ~ 1) + Bsenh ( ~t))

= c-'i,~ ( Âc~' + ÍJe ~t).

Como - 2: + f < O, ainda temos que x(t) ➔ Oquando t ➔ +oo. Nesse caso, o amortecimento
do sistema é dito supercrítico.

(iii) Sem > 1~,então 6. < O e (uma vez mais graças ao teorema anterior)

x (t) = e- 2mcl ( A cos (v-5.)


2
+t (v-5.
t ))
B sen
2
---t O

quando t ➔ + Nesse caso, o sistema ainda oscila à medida que o amortecimento se processa,
razão pela qual tal amortecimento é denominado subcrítico.

este ponto, observamos que meramente tocamos a superfície do mundo das EDOs. Ao leitor
interessado em saber mais sobre as mesmas, sugerimos as referências (4]ou (211,dentre outras.

321
1 \1 1' 1 1 '1 1 .-\ )\" 1 F<:H \1. Ili·: H11·:\I \\"\

Problemas - Seção 4.9


9.1. A meia-vida de uma substância radioativa é o tempo necessário para que uma determi-
nada. massa dessa substância decaia à metade. Prove que a meia-vida de uma substância
radioativa só depende de sua constante radioativa. Em particular, a meia-vida independe
da. massa inicial da substância.

9.2. * Faça os seguintes itens:

(a) Se J, g : IR ➔ IR são funções deriváveis e tais que f'(x) = g(x) e g'(x) = f (x) para
todo x E IR, então J(x) = J(O)coshx + g(O)senhx e g(x) = g(O)coshx + J(O)senhx,
para todo x E IR.
(b) Se J: IR➔ IRé uma função duas vezes derivável e tal que J"(x) - J (x) = O para todo
x E JR,então
J(x) = J(O) coshx + J'(O)senhx,
para todo x E IR.
(c) Se À> O e f : IR➔ IRé uma função duas vezes derivável e tal que J"(x) - Ãf (x) =O
para todo x E IR, então
.
f(x) = J(O) cosh(Y'Àx)+ f~\enh (v'>:x), (4.86)

para todo x E IR.

9.3. Um corpo de massa m, solto a partir do repouso nas vizinhanças da superfície da Terra,
experimenta, na ausência de ventos apreciáveis, um movimento retilíneo de queda sob a
ação da força de atração gravitacional exercida pela Terra e da força de resistência do ar.
A lei de forças da força resistiva exercida pelo ar sobre o corpo afirma que sua magnitude é
diretamente proporcional à velocidade escalar do corpo. A esse respeito, faça os seguintes
itens:

(a) Mostre que a força resultante F sobre o corpo é tal que F = mg-kv, onde g denota a
aceleração da gravidade, v a velocidade do corpo e k a constante de proporcionalidade
a que aludimos acima.
(b) Use a segunda lei de Newton para concluir que, sendo v = v(t), temos v'(t) + v = g. !
~, .
(e) Mostre que h(t) = em é um fator mtegrante para a equação diferencial do item (b).
{d) Mostre que, partindo do instante inicial / 0 = O, temos

322
l '1;1 >l\1.1 \I \" ~I <,' \( > 1 'l t

(e) Calcule a velocidade limite de queda do corpo.

9.4. A lei de resfriamento de Newton afuma que a taxa temporal segundo a qual um corpo
troca calor com o ambiente é cliretamente proporcional à diferença entre as temperaturas
do corpo e do ambiente. A esse respeito, faça os seguintes itens:

(a) Se y(t) denota a temperatura do corpo no instante t, mostre que existe uma constante
positiva k tal que y'(t) = -k(y(t)-T(t)), onde T(t) denota a temperatura do ambiente
no instante t (suposta conhecida) e o sinal - após a igualdade indica que o calor flui
do corpo mais quente para o mais frio.
(b) Mostre que h(t) = ekt é um fator integrante para a equação diferencial do item (a).
(c) Mostre que, medindo temperaturas a partir do instante t0 = O, temos
y(t) = e-• 1y(O) + ke-• 1 J.'e••r(s)ds.
(d) o caso em que o ambiente é mantido a uma temperatura constante T0 , mostre que
y(t) ➔ T0 quando t ➔ +oo, o que condiz com nossa experiência.

9.5. Dados a, b E 1Re uma função contínua g : I ➔ IR, o objetivo deste problema é aplicar o
método de variação dos parâmetros para resolver a equação diferencial

y" + ay' + by = g(x). (4.87)


Para tanto, faça os seguintes itens:

(a) Se J1 , h: I ➔ 1Rresolvem (4.87), mostre que / 1 -h resolve a equação y"+ay'+by = O


e, portanto, é dada como no teorema 80.
(b) Conclua que as soluções de (4.87) são as funções da forma Yh + Yp, onde Yh : I ➔ 1R
é dada como no teorema 80 e Yp : I ➔ 1Ré uma solução particular de (4.87).
(c) O item (b), juntamente com o teorema 80, reduz o problema de resolver (4.87) àquele
de encontrar uma solução particular dessa equação. Para tanto, sejam u 1 , u 2 : 1R➔ 1R

como no teorema 80, e faça v, = e- ..,{'ui, para i = 1, 2. Se l 1 , l2 : I ➔ 1Rsão funções
duas vezes deriváveis e y = l 1v1 + l2 v2 , mostre que y satisfaz (4.87) se, e só se,

(d) Pelo item anterior, se conseguirmos encontrar l 1 , l2 : J ➔ IR, duas vezes deriváveis e
tais que

323
' < ,1·11, 1( 1 1 ..\ l:\·11-:c;fL\I. ni-: HIE:\1.\:\':\'

teremos sucedido em encontrar uma solução particular y = l1v1 + l2v2 de (4. 7).
Mostre que poderemos encontrar tais funções l1 e l2 se, e só se, a função ll' : IR~ IR,
dada por
W(x) = det [ v1(x) v2(x) ] ,
vi (x) v~(x)
for tal que W =/-O em I. Sendo esse o caso, mostre que

l (x) =
1
-jx g(t)v2(t) dt
W(t)
l (x) =
e 2
jx g(t)vi(t) dt.
W(t)

(e) Para mostrar que W =/-Oem IR (e não somente em I), use o fato de que (cf. lema 79)
Vi e v2 resolvem a equação y" + ay' + by = O para calcular

W'(x) = .!!_det [ vi(x) v2 (x) ] = -aiV(x).


dx v~(x) v~(x)

Então, conclua que W(x) = W(O)e-ax em JR,de forma que vV =/-O em lR se, e só se,
W(O) =/-O. Por fim, analise as várias possibilidades para v1 e v2 (cf. teorema 80) para
concluir que
se~= O
• { 1
vV(O)= ~, se~< O
-/K, se~> O
..
9.6. Resolva, em lR, a equação y" + 4y = sen x, com y(O) = 1r, y'(O) = -1.

324
SÉRIES NUMÉRICAS E
DE FUNÇÕES
Este último capítulo apresenta a noção adequada de convergência para sequências e séries
de funções, com ênfase no estudo de séries de potências. Após discutirmos alguns fatos básicos
·obre séries de números reais, examinamos a série de potências naturalmente associada a uma
função infinitamente derivável, dita a série de Taylor da função. Em seguida, discutimos, nas
seções 5.4 e 5.5, as teorias básicas de séries uniformemente convergentes de funções e séries de
potências. Por fim, na seção 5.6, apresentamos algumas aplicações da teoria desenvolvida.

5 .1 Séries de números reais


Seja (an)n>1 uma sequência de números reais. Pela série Ef:i ai.:,ou simplesmente Lk>l ak,
entendemos a sequência (sn)n>i, onde sn = a 1 + a2 + ••• + an para n > l. O número real sn é
denominado a n-ésima soma parcial da série Lk2:i ªk, e dizemos que tal érie converge se
a sequência (sn)n2:1 de suas somas parciais converge para algum s E IR. Nesse caso, dizemos que
o real s é a soma da série e escrevemos

Lªk = s. (5.1)
k2:l

Em outras palavras, quando escrevemos Ek2:i ak = s, estamos dizendo que as somas finitas
Sn= + a2 + · · · + ªn
a1 aproximam-se mais e mais s, à
do número n➔
real medida que +oo. É

nesse sentido que a igualdade (5.1) deve ser pensada, como um limite.
Por vezes, teremos em mãos uma sequência (an)n2:o de números reais, em cujo caso a série
correspondente será denotada por Ek2:o ªk· Deixamos ao leitor a tar~fa (im~diata) de adaptar
as discussões acima e porvir a tal situação.
Nosso interesse primordial nesta seção é encontrar critérios que permitam decidir se uma dada
série é ou não convergente. Caso não o seja, diremos que se trata de uma série divergente.
Comecemos examinando o caso de uma série geométrica, i.e., uma série da forma Lk2:i qk- 1,
para um certo real não nulo q. Conforme veremos ao longo deste capítulo, séries geométricas
são, sob vários aspectos, fundamentais para o desenvolvimentq da teoria.

PROPOSIÇÃO 5.1.
Dado q e R \ {O},a série geométrica E1:2:i q"- 1 converge se, e só se, O < lql < l. Xesse
6ltimo C880, sua soma é igual a 1~ 9 •

PROVA.
Sendo s" = 1 + q + · · · + q"- 1 , segue da fórmula para a soma dos termos de uma PG que
1 - qn l q"
Sn = --- = -- - --
1- q 1-q 1-q

326
Agora, se O < lql< 1, o exemplo 12 garante que limn-.+ qn = O. Portanto, temos

lim
n-+1"""00
Sn ~ (-1 -
= n-+-'-oo
1
q
- L)
l - q
=--
1-q
1

Por outro lado. se jql> 1, então limn-.-oc lq"I= + , de forma que a sequência (sn)n2:1 não
converge em IR. Portanto, nesse caso a série geométrica em questão é divergente.

Nosso próximo resultado ensina como operar com séries convergentes.

PROPOSIÇÃO 5.2.
Se L1,;2:1 ak e LJ.-, 1 bk são séries conYergentes e e é um número real qualquer, então:

(a) a série Lk ~1 ca1,;com·erge e Lk2:l c.ak = e LA:2:lªk·

(h) a série Lk~ 1 (ak + bk) conYerge e Lk2: 1 (a,. + b,:) = LA:2:la1i:+ E1i:2:ib1c.
---~--~~~~

PHO\,\

(a) Se -"n é a n-ésimc ~oma parcial dn ~ nc LA->1 ak. então a n-ésima soma parcial da série
Lq I cak é e n• Portanto, pelo item (a) da proposição 1 , Lk2:i cak converge, com

~ CQk = lim C n = C n-++


lim Sn = C~ ak.
L- n-++ L-
k2:l k2:l

(b) e -'>n e ln ..ão, re!>pectivamente, as n-é:;imas somas parciais das séries L1i:2:iak e L1i:2:ib1i:,
rntãoan- ima omaparcialda éneLk2: 1(ak+b1i:)ésn+tn. Portanto,oitem(b)daproposição
1 ganrnte a convergencia de Lk2:l (ak + bk) e fornece

EXEMPLO 5.3.
M~tre que a série E1i:2:i( 1.,+!:+3.,) converge e calcule sua soma.

327
'"'01 l! Ç \O.
1,1
Como O <....½, < 1, a proposição 1 garante a convergência das séries geométricas Lk~l (¼)k,
Í:1.:::1 (½)1' e Lk~i(¾)k. Portanlo, aplicando duas vezes o item (b) da proposição 2, obtemos

Dada uma série Lk~l ak, referimo-nos a an como o termo geral ou, ainda, o n-ésimo
termo da série. A proposição a seguir dâ uma condição necessária para a convergência de uma
série em função de seu termo geral.

PROPOSIÇÃO 5.4.
Se a série Ek~l ak é convergente, então limn➔+oo <ln= O.
----~~~-~----------~~-_____J

PROVA.
Dado € > O, queremos garantir a existência de n 0 N tal que n > n 0 ⇒
E lanl< €. Seja
l = Lk~l ªk· Pela definição de convergência de uma série, existe n 0 E..N tal que

> no⇒ 1Sn
n - li <
2,
onde Sn = a1 + a2 + · · · + an, Então, an = Sn - Sn-1 para n > 1, e segue da desigualdade
triangular que, para n > n 0 ,

A recíproca da proposição anterior não é válida, i.e., há séries divergentes Lk~l ak para as
quais lín1n-.+ooª" = O. O exemplo clássico é fornecido pela série harmônica Lk~l ¼,cuja
divergência será estabelecida logo mais. Anles, contudo, precisamos de um critério para a
convergência de séries de termos positivos.
I

328
PROPOSIÇÃO 5.5.
Se (an)n~l é uma sequência de termos não negativos. então Et~l ª• COlffl
eequência (sn)n~l de suas somas parciais é limitada.
---------,----------'

PROVA
Inicialmente, recorde que, por definição, Lk~l ak converge e, e só se, (sn)n~1 converge. Como
jâ sabemos que toda sequência convergente é limitada, concluímos que, se Lk~l ak converge,
então (sn)n~l é limitada.
Reciprocamente, suponha que (sn)n~l é limitada. Como ak > O para todo k > 1, temos que
s1 < s2 < s3 < .... Portanto, (sn)n~l é uma sequência monótona e limitada, logo convergente,

pelo teorema de Bolzano-\Veierstrass.

EXEMPLO 5.6.
A série harmônica Lk~l t dh·crge. Por outro lado. para todo real r > 1, a eêrie Et~l .~
converge.

PHO\A
Sejam Sn - L~ 1 f e tn = L~ 1 /r. Pela propo ição anterior, é suficiente provar que a
!-)<'qucncia
(,.,n)n>l é ilimitada, ao passo que a ~cquenc1a(tn)n~l é limitada.
Pnrn mostrar que ( n)n~l é ilmutada. ob er\'e que, por {4.45), temos¼ > log (1 + ¼)- Por-
tanto,
1 " ( 1)
rl
n- L k > L log 1 + k = Ln (log(k + 1) - log k) = log(n + 1),
k 1 k·-1 k-1

de ...,01tcque " ➔ + quando n ➔ +


Quanto a (tn)n~l, dado n EN, tomem EN tal que 2m > n. Então,

ln< l+ G 1
+ ;,) +···+ C2m~l)' +···+ {2m~ 1)')
1
. m 1 1
<1+2--+4·-+···+2
2r 4r
- •---,-----,-
2(m-l)r
1 1 1
= 1+ -2r-l + 4r-l
- + ... + ----,-----,-
2(m-l)(r-l)
1 1
<L 2(r-l)k =L (2r-l )k •
k~O k~O

l\las, como r > 1, temos o< 1


2r-l < 1, de sorte que 'Ç'""
wk~O
1
(2r-l)k
-
-
2r-l
2r-l_1, pela propos1çao
• - }.

329
Ainda cm relação ao exemplo anterior, apesar de termos garantido que a série Lk>l k1r
converge quando r > 1, não temos a mínima ideia sobre o valor numérico e.xato de sua soma.
Essa é uma situação recorrente para séries, e não deve dar ao leitor a sensação de que a teoria
que está sendo desenvolvida é, de alguma forma, deficiente. Pelo contrário, veremos em vârias
outras partes desse capítulo que a simples garantia da convergência de uma série (que é uma
conclusão meramente qualitativa) pode gerar consequências quantitativas importantes.

ÜBSERVAÇÃO 5.1.
A título de curiosidade, informamos que Lk>l k1r = ((r), onde ( : (1, +oo) ➔ R denota a
2
famosa função zeta de Riemann. Para um cálc~o elementar de ((2) = ~ • referimos o leitor
ao problema 4.7, página 367, ou, ainda, à seção 4.3 de [13J. Observe, contudo, que, parn um
natural m > 1 ímpar, a obtenção do valor numérico exato de ((m) é um problema cm aberto,
i.e., ainda não resolvido.

O teorema a seguir apresenta um critério de convergência para séries de termos positivos


que, de certa forma, pode ser visto como um refinamento da proposição 5. Ele é conhecido na
literatura como o teste da integral para convergência de séries.

TEOREMA 5.7.
Sejam dados no E N e uma função monótona e decrescente f : [no, +oo) ➔ R, tal que
limz-++ooJ(x) = O. Então, ..

J: 00

J(t) dt converge # L J(k) converge.


k2:no

PROVA.
Sejam g, h: [no, +oo) ➔ IRas funções dadas, no intervalo µe,k + l) (para cada k > n 0), por
g(x) = J(k + l) e h(x) = f(k).
As condições sobre / garantem que O < g(x) < f (x) < h(x), para todo x > n 0 . Portanto,
segue do critério de comparação para integrais impróprias (cf. proposição 72) que

+oo /.+oo
J(t) dt converge => g(t) dt converge
/. no no

e
I 1+00 J.+oo
h(t) dt converge => J(t) dt converge.
no no

330
Por outro lado, para um inteiro n > n0 , temos

[
no
g(t)dt = t
k=no+l
f(k) e [
no
h(t) dl = f
k=no
f(k).

Por fim, apliquemos a equivalência (a) <=>(c) do teorema 71, com Xk = k para todo k > no:
00 00
(i) Se Ín: J (t) dt converge, então fn: g(t) dt converge e, daí,

1 +00
g(t) dt = n➔+oo
lim
l" g(t) dt = n➔+oo
lim L- '"°'J(k) = L
n
J(k).
no no k=no+l k~no+l

Logo, Lk>no- J(k) converge, com Lk>n


- o J(k) = J(no) + Lk>n- o+I J(k).

(ii) Se Lk~no J(k) converge, então,

1 +oo
h(t) dt = n➔+oo
lim
l" h(t) dt = n➔+ooL-
lim '"°'
n-1
f(k) = '"°'
L-
J(k),
no no k=no k~no

00 00
de modo que fn: h(t) dt converge. Portanto, fn: J(t) dt converge, pelo que vimos acima.

EXEMPLO 5.8.
O teste da integral permite estabelecer facilmente a divergência da s6tie
convergência da série Lk~l f,., para todo real r > 1. No caso da série IW'IDGU)
f: [l, +oo) ➔ IRdada por f(x) = ~. temos
00

1 1
+ 1
- dt
t
= z➔+oo
lim
/.%

1
1
- dt
t
= z➔lim
+oo
log x = +oo

Analogamente, tomando/: [1, +oo) ➔ R dad~or f(x) =;,.,com r > 1,

i
1
+oo .!_dt = lim
tr z➔+oo
iz
1
.!_dt = lim - 1- (_!__ -
tr z➔+oo 1- r xr-1
--~--
1)
Vejamos um exemplo mais interessante.

ExEMPLO 5.9.
Examine a convergência da série E.~2 u!ct·

331
OLUÇÀO.
Se f : (2,+oo) ➔ 1R é a função dada por f(x) = xl~x' então J é claramente monótona,
decrescente e tal que limx-++ooJ(x) = O. Pelo teste da integral, Lk> - 2 k/ og k converge se, e só se,
Jt J(t) dt também converge.
Para o que falta, observe que a substituição de variável s = logt (cf. problema 8.2, página
310) fornece

12
00
+
f (t) dt = .
1+t 0g t
2
00

l1
log 2 S
dt = l,+oo -1 ds

= lim (log s - log 2) = +oo .


.s-++oo

Portanto, Jt
00
J(t) dt diverge, de sorte que Lk>
-2
1
kl k
og
também diverge.

A definição a seguir isola uma classe de séries cuja convergência, em princípio, é mais fácil
de ser estabelecida.

DEFINIÇÃO 5.10.
Uma série EA:?:lak é absolutamente convergente se a série Ek?:l lakl é convergente.

A utilidade do conceito de série absolutamente convergente é evidenciada no próximo resul-


tado.

PROPOSIÇÃO 5.11.
Toda série absolutamente convergente é convergente.

PROVA.
Seja Lk?:l ak uma série absolutamente convergente e, para cada n > l, sejam sn = E~=l
ak

e tn = E;=1 lakl• Dados inteiros m > n > l, temos


m m

lsm - snl = L
k=n+l
ak < L lak1 = tm - tn.
k=n+l

Como a sequência (tn)n2:1 converge, ela é de Cauchy. Portanto, dado f > O, existe n 0 EN tal
quem> n >no=> tm - tn < t. Com tais feno, segue da desigualdade acima que

m > n >no=> lsm - snl < tm - tn < t,


de sorte que a sequência (sn)n?:l também é de Cauchy. Logo, pelo teorema 28, a sequência.
(sn),i?:J é convergente, conforme queríamos demonstrar.

332
A recíproca da proposição anterior não é Yálida, quer dizer, há séries convergentes que
são absolutamente conYergentes. Antes de podermo apresentar um exemplo, precisao:MJI de
resultado conhecido na literatura como o critério de Leibniz para convergência de séries.

ROPOSIÇÃO5.12. LEIB~IZ

Se (<1n)n2:ié uma sequência não crescente de reais positivos, tal que llm.-++oo
série Lk2'.l (-1)k- 1ak é conYergente.

PROVA.
Para cada n E N. seja sn = a 1 + a2 + · · · + a 11 • A condição a 1 > a2 > a3 > · · · > O garante
facilmente que
1 > 3 > 5 > ... > 6 > -l > 2· (5.2)
Por outro lado, para cada m E N. temo

Por sua. vez, em conjunção com (5.2), , com· rgt>ncingarante claramente que a sequência
(s,,),.2:1 é de Cauchy. Logo, (~n)n2:I l'• com=rg.:>ntl•, conforme de ejado.

5.13.
EXEMPLO
Pelo CI i t éiio dl• Lt i bn iz, a s(•rit.. Lk>
1
l (-l r I
converge. Por outro lado, tal aêrie DãDl.. ...
utai11t'ntt com·l rgt"nh•1 uma vez que a ~rie LA:2:I 1(-it-•
1 1
1é a série hannõnica, que
di\'c:>l'g('llt
l'.

\ oltnndo no c~tudo d~.., n absolutamente convergentes, um ligeiro refinamento dos ar-


gumt'ntos npre~ ntndo. na demo1ll:itraçãoda propo 1ção 11 fornece o seguinte critério útil para
a con\'~rgêncm t\b~oluta de uma éne. Tal critério é conhecido como o teste da comparação
parn n com'l'rgênc1a ab oluta de ene,. •

POSIÇÃO5.14.
Sejam (0n)n~1 e (bn)n~l sequências de números reais tais que < b,.,
lt.1nl
~ 1 b1r converge. então Lt~•a,. é absolutamente convergente e tal que

333
'°'i 1:11" \1 11 1:11 \~ 1 111 11 ,, 111"
'

PROVA.
Sejam Sn = I:~_1 ak e tn = I:;=1 bk. Param> n > l, temos
m m m
lsm - Snl = L ak < L lakl < L bk = tm - tn.
k=n+l k=n+l k=n+l

Agora, como a sequência (tn)n~l é convergente, ela é de Cauchy. Portanto, dado e > O,
existe noE N tal que m > n > n 0 =} tm - tn < e. Logo, segue das desigualdades acima que
m > n > no =} lsm - snl < e, de sorte que (sn)n>l também é de Cauchy. Então, pelo teorema
28, (sn)n>l converge.
Por fim, analogamente às desigualdades acima, concluímos que lsnl < tn para todo n > l.
Fazendo n ➔ +oo e utilizando o resultado do problema 2.1, página 89, obtemos

8KBMPoo5.15.
Existe uma sequência (an)n~1 de números reais positivos tal que ambas as séries Lk~l ak e
~ 1 • . ?
L.,i~l ir.;' CODVU'J&m.

SOLUÇÃO.
Se tal fosse o caso, o item (b) da proposição 2 garantiria a convergência da série I: (ak + k 2~ ) .
k~l "

J
Mas, pela desigualdade entre as médias, temos ak + k 2~1c > 2 ak • k 2~,. = de forma que o teste í,
da comparação garantiria a convergência da série Lk~l í,
o que é um absurdo.

A seguir, apresentamos uma elaboração muito útil do teste da comparação, a qual dá uma
condição suficiente para a convergência absoluta de uma série de termos não nulos. O resultado
a seguir é conhecido na literatura como o teste da razão.

, tal que lim,.~ j 1 1 = l. 4::


~ • l > 1, a série EA:2:i ª• é divergente.

334
SI HII-, l>I '\I \li J:11-, HI \J-.. ,

PROVA.
Provemos inicialmente que. se l < ·l. a série Lk~l ak é absoluta.mente convergente. Sendo
l < 1, podemos tomar um número real q tal que/ < q < 1. Tomando€= q-l > O, a convergência
~::t·~ l garante
1 a existência de Tio E N tal que n > Tio ~ 1lj::t' - li < € = q - l. Em
particular. para n > Tl<J.segue da desigualdade triangular que

lan.....
11 lo.n..ql
-- < -- - l +l <q- l+l = q.
lan1 fanl

Portanto, para n > Tl<J.temos

1°.1= 1°.. 1 fi1~::;1


k=no
< la..,lq"-no.

A~~irn,para n ~ 110, o termo d ~ne í:, ~1 !o -1:-:-."o


majorado· pelos termos da Lk~l lan0 lqn-no,
n qual converge, pelas propo içõ 2 ~ 1. Porumto. ~··~guedo teste da comparação que Lk~l ak
(, nhsolutamcntc convergente.
Se/ > 1, touH~ um renl q tRJ qUt~1 < q < I ~,como dmn. u0 E N tal que n > no ~ > q. 'j:!t'
E11tno, t nu1bém como cima, t •mo 10111> a lq"-' pnrn n > 110 ~las, sendo esse o caso, temos
que a,. /+ O q11n11don ➔ ::,e.ril _ ~ 1 01; clh •rgl', pela propo~ição 4.

n~ not nçõe:. dn proposic;~o m ~rior olliier\'nmosque, l = 1, a série Lk~ 1 ak pode convergir


011 d1,l'rg11. De foto, pnrn On = ~ tc.mo
<lu+l 11 ,,
-- =----+ 1
a 11 n+1 '

mfü:,l\ ::-.éne 1 "f'Jconverge.


-J,.~

VcJamo , ngora, um e.xemplo na direção positiva.

EXE\lPLO 5.17.
1
Dados um natural me um real q > 1, a série Lk~l (-1l- ~ converge ou diverge? Justifique
::.uaresposta.
~~--~----------~-~~----~--~~-----~
335
1

"I 1:11, \1 'li 1:1, \, 1 l•I 1 1 -" 11


1

Sou;ç.,o.
Fazendo an = (-1)"- 1
~; para n > 1, temos

Portanto, pelo teste da razão, a série dada é absolutamente convergente, logo, convergente.

Terminamos esta seção discutindo o produto de duas séries absolutamente convergentes.


Observamos que o resultado a seguir só será utilizado na seção 5.6 e que, portanto, pode ser
omitido numa primeira leitura.

TaôREMA 5.18.
Sejam Li2:l ~ e Li2:l b1 séries absolutamente convergentes. Se
k-1

ck = L
i+j=k
a,b = L a.bk-,
1
1=1

para k > 1, então Lk2:I ck é absolutamente convergente e tal que

PROVA.
É suficiente mostrar que, dado € > O, existe no E N tal que, para n > n 0 , temos

Garantamos a existência de n 0 para que valha a primeira desigualdade acima (a anâlise


validade da segunda desigualdade é inteiramente anâloga).
Para n E N, segue da desigualdade triangular que

336
Denotemos por A, B e C, respectivamente, a primeira, segunda e terceira parcelas do segundo
membro acima, de forma que
n
B = L Cli L bj e e = La, L b)

A sequência (I:;=1 b1) n~l , sendo convergente, é limitada; logo, existe M > O tal que

IE;=lb)1 < M, para todo n > 1. Por outro lado, como as séries Li~I a, e í::1 ~ 1 b1 conver-
gem, temos
n

Lª, = Lª, - Lªi ~ LOi- I:0i = o


e, analogamente, L;>n b1 ---t O quando n ---t +oo. Para estimar C podemos supor, sem perda de
generalidade, que E.~1 a, =/-O. Então, podemos escolher n 1 , n 2 E N tal que

portanto, para n > max{n 1, n2}, temos

l
--:------:- - -

3 IEi>l ªi' 3
Quanto a A, observe inicialmente que

max(,,J}>n
t+j$2n
2n 2n
< L
max(,,J}>n
la,b;I < L Lla,llb1I+
i=n+l j$n
L Llaillbil
t+j$2n

Para estimar A, podemos supor L,~1 la,I =/-O e Í:: 1~ 1 lb1 I =/-O. Agora, como as séries L,~1 lail
e í: 1 ~ 1 lb1 1convergem, as sequências (I:~=I la,l)n~I e ( í::7=
1 lb1 1)n~l são de Cauchy; portanto,

existem n3, n4 E N tais que

337
'-,f f;ff ~ \1 'lf 1:1< \~ f f1I 11 < Ili

Então, para n > max{n 3 , n 4}, temos

Por fim, pondo no= max{n 1 , n 2, n 3 , n 4 } e tomando n > n 0, todas as estimativas anteriores
sã.o vá.lidas, de forma que

f
< A+B +C < 3• = f.
3

Problemas - Seção 5.1


1.1. A sequência (an)n~l é uma PA não constante de reais não nulos. Prove que a série
1
L1c>i - -
- ªA:ª1<+1
converge e calcule sua soma.

1.2. Dado um número real a> 1, prove que



a série L1c>i- ~
a
é convergente e calcule sua soma.

1.3. Seja (Cln)n~l a sequência de números reais positivos definida por a 1 = ½ e an+l = a!+ an,
para n E N. Prove que a série E1c~i ªt~l converge, com E1c~ 1 0 .,~ 1 = 2.
-
1.4. Decida se a série L1c>i
-
J k+v'k7=I
1
converge ou diverge.

15P
• . ''"
rove que a serie 1
L.,k>lOOO ✓1cL1Cxxi1c:i converge.

1.6. Se (an)n~l é uma PA infinita e não constante de termos positivos, prove que:

(a) a série L1c>i


-
..!.
a.,
é divergente.
1
(b) a série L1c> 1 - é convergente.
- ª2k

1.7. * Dada uma sequência (a 1, a2, a3, ... ) de algarismos, prove que existe um único elemento
x E R satisfazendo a seguinte condição: fixado um erro máximo ~ro , com n 0 E N, temos
1
o< X - (ro+ W'+ ... + i°cfn) < l~'ll, para todo natural n > no, Nesse caso, como o leitor
deve sUBpeitar, escrevemos x = O,a1a2a3 ....

1.8. Mostre que todo real x E (O,1) admite uma única expansão decimal da forma x
O,a1iL2a3... , com a,t =/:-O para infinitos valores de n. Em seguida,
construir um exemplo de função sobrejetora f: [O,1] ➔ [O,l] x [O,l].

338
l 1 1i I l ) \ 1 1 \ 1 \ '-- '-i 1 1 \ 1 1 -, I r

1.9. Seja (an)n~1 uma sequência de números reais positivos, Lal que a série Lk~J aiconverge,
Prove que, para todo real r > ½,,a série Lk~l ~ também converge.

1.10. Seja (an)n~l uma sequência de números reais positivos, tal que a série Lk~l a1c converge.
Prove que a série Lk~l ✓akak+l também converge.

1.11. Seja (Fn)n>i a sequência de Fibonacci 1 , i.e., a sequência tal que F 1 = 1, F2 - 1 e


Fk+2= Fk+1+ Fk, para todo inteiro k > l. Mostre que a série Lk~l J.,.converge.

1.12. Examine a convergência das séries abaixo, onde a> O é um real dado:

(a) Lk~2 (log\)º •

(b) Lk~2 k(lo~ k)ª •

(c) Lk~2 k(log k)(l~g log k)º •

1.13. Seja (an)n>1 uma sequência de números reais tal que a série Lk~l ak é absolutamente
convergente, com Lk~l ak = O. Mostre que a série

"""' (-ª-1_ 2 + _a_2_ + ... + _an_-_1)


L.J
n:::1
(n- 1) (n- 2)2 12

converge e calcule o valor de sua soma.

1.14. Seja A um conjunto finito de naturais da forma 2ª3b5C,para algum terno (a, b, e) de inteiros
não negativos. Prove que LxeA ~ < 4.

1.15. Sejam (an)n~l uma sequência de reais positivos tal que ~-+ l.

(a) Se l < 1, prove que a série Lk~l ak converge.


(b) Se l > 1, prove que a série Lk~l ak diverge.
(c) Se l = 1, dê exemplos mostrando que a série Lk~l ak pode convergir ou divergir.

O critério de convergência de séries dado pelo caso l < 1 é conhecido na literatura como o
teste da raiz para a convergência de séries.

O critério de convergência de séries dado pelo próximo problema é conhecido na literatura


2
como o critério de Abel .
1 ApósLeonardo <liPisa, também conhecido como Fibonacci, matemático italiano dos séculos XII e XIII. Parn
uma interpretação combinatória da sequência de Fibonacci, veja o capítulo 1 de ll2J.
2
Após Niels Henrik Abel, matemâtico norueguês do século XIX.

339
1.16. Sejam (on)n~l e (bn)n~l duas sequências de números reais satisfazendo as seguintes condi-
ções:

(i) A sequência (sn)n~l, definida para n E N por Sn = 1=;= 1 ak, é limitada.


(ii) b1 > b2 > b3 > · · · > O e bn ➔ O.
A esse respeito, faça os seguintes itens:

(a) Prove a identidade de Abel:_ E~=l albi = E:i:/(bi - b,+1)si + bnSn-


(b) Conclua, a partir do item (a.), que a série Lk~l akbk converge.

1.17. Mostre que o critério de Abel implica o critério de Leibniz.

1.18. O objetivo deste problema é dar uma outra demonstração da convergência da integral
J/00
~t dt (cf. exemplo 73). Para tanto, faça os seguintes itens:

(a) Mostre que basta estabelecer a convergência da série Lk~l sezk.

(b) Conclua, a partir do critério de Abel (cf. problema 1.16, página 340), que é suficiente
mostrar que a sequência (sn)n~l, definida para n EN por Sn = E~=l sen k, é limitada.
sen !!.se n !!.±!
(c) Aplique o resultado do problema 1.10, página 222, para mostrar que Sn = 2
sen 21
2

e, portanto, que lsnl < se! 1 •


2

1.19. Faça os itens a seguir:

(a) Mostre que pelo menos um dos números lsen nl, lsen (n + i)I ou lsen (n + 2)1 é maior
ou igual a ½-
(b) Conclua que a série Lk~l lse;kldiverge.
(c) Mostre que J/ 00
se~t dt não é absolutamente convergente.

1.20. Se P1 = 2 < P2 = 3 < p3 = 5 < · · · é a sequência dos números primos, um teorema de


Euler afirma que a série Lk>l- ..l..
Pk
diverge. Para demonstrá-lo, faça os seguintes itens:

(a) Mostre que (1 - x)- 1 < e2x.,para O< x < ½-


(b) Para n E N, sejam P1 < P2< · · · < p, são os primos menores ou iguais a n. Mostre
que

(e) Mostre, a partir dos itens (a) e (b), que 1=;= 1 ¼< e E'k= 1 2
Pk.

(d) Conclua q'ue Lk>i- ..l..


Pk
diverge.

340
5.2 Algumas aplicações
Esta seção coleciona algumas aplicações relevantes da teoria de séries de númeroe reail,
conforme apresentada na seção anterior.
Começamos utilizando (4.45) para estabelecer uma estimativa, devida a Euler, para o tama-
nho de logn.

TEOREMA 5.19. EULER


Para n E N, temos

onde , é uma constante positiva.

PROVA.
Inicialmente, observe que
n l n l n-1
L k - logn = L k - L(log(k + 1) - logk)
k=I k=I k=l

~ ( k1 -
= n-l ( 1))
log 1 + k
1
+ n.

Agora, como ¼➔ O quando n ➔ +oo, basta provarmos que a série

(5.5)

converge para uma soma positiva,.


Aplicando a desigualdade (4.45), com k no lugar de n, concluímos que (5.5) é uma série de
termos po itivos, tal que

n-l ( 1 ( 1)) n-l ( 1 1 ) 1


""""
~ k
- - log 1 + -k < """"
~ k
- - -k+l = 1- n- .
k=l k=l

Portanto, a proposição 5 garante que ela realmente converge.

O número 1 definido por (5.4) é conhecido na literatura matemática como a constante de


Euler-Mascheroni, e seu valor com cinco casas decimais corretas é , ~ O,57721. Até hoje

341
- --- ==========================----------------
-·--:..,.;-~--=-=- ·- - ·----------------------

nio se sabe se 'Y é um número racional ou irracional, muito embora suspeite-se que 'Y deva ser
irracional.
Observe que, a partir de (5.4), temos
n 1
log n "' L k - 'Y,
k=l

sendo tal aproximação tanto melhor quanto maior for n E N.


A seguir, utilizamos o material da seção anterior para estudar mais algumas propriedades do
número e.

PROVA.
Uma vez que Ek~o t, é uma série de termos positivos, a proposição 5 garante que, para
estabelecer a convergência dessa série, basta mostrarmos que ela é limitada. Para tanto, como
k! > 2k-l para todo inteiro k > 2, temos, para n > 2 inteiro, que
n. 1 n 1
Sn =2+L k! < 2+ L 2k-1 = 3,
k=2 k=2

onde utilizamos a fórmula para a soma de uma série geométrica na última igualdade acima.
Seja l = Ek~o tr. Para mostrarmos que l = e, o teorema 56 garante ser suficiente mostrarmos
que l = limn-++oo(l+ ¼)". Para tanto, seja o.n= (1 + ~)". Como no exemplo 23, capítulo 2 (cf.
(2.1) e cálculos subsequentes), temos

an = 2 + L (n)
n

k=2
k
1
nk e
(n)2_ __ _.!_. n(n- 1) ... (n- k + 1)
k nk k! nk '

de sorte que

n 1 n 1 1
< 2+ L
k=2
k! =L
k=O
k! < L
k~O
k! = l.

342
.\L<;l .\1 \'-- \l'I H \< 1 li'-- 1

Os cálculos acima também fornecem. para n > m > 2 naturais,

Portanto, segue do problema 2.1, pãgina 89. que

e= lim an > n➔
n-++oo
lim
+oo
(1 - n .!.)(1 - n ... (1 - m n-
~)
1
) (2 + t
k=2
k ,)
1
.

Mas, como m E N foi escolhido arbitrariamente. concluímos que e > 0 para todo z:=;:t,,
m E N. Então, fazendo m ~ +oo e invocando novamente o item (b) da proposição 15, obtemos
finalmente
e> lim
- m➔+

TEOREMA 5.21.
O número e é irracional.

PROVA
Seja sn = L~=O ~!. Para naturais m > n > 1, temos
m 1 1
Sm - Sn = L
k=n+l
k! < L
k::!:n+l
k!

1 ( 1 1 ) (5.6)
= (n + 1)! 1 + n + 2 + (n + 2)(n + 3) + • • •
1 1 1 n+2
< (n + 1)! L (n + 2)k = (n +
k::!'.0
1)! • n + l'

343
onde utilizamos, uma vez mais, a fórmula para a soma de uma série geométrica na última
igualdndc.
Porhmto 1 ainda para naturais m > n > 1, temos a partir dos cálculos acima que
m 1 1 n+2
Sm = Sn +
k=n+l
L k! < Sn + (n + 1)! • n + 1'

e segue do teorema anterior e do item (b) da proposição 15 que


. 1 n+2
e= lim Sm < Sn + --- · --
m-++oo - ( n + 1) ! n + 1

Assim, temos para n > 1 que


1 n+2
Sn <e< Sn + (n + l)! • n + (5.7)
1

1fultiplicando tal desigualdade por (n -1)! e observando que Sn = Sn-l +~'concluímos


n.
que

( 1) 1.Sn-1 + -1 < ( 1)' < ( 1)'.Sn-1 + -1 + (n + 2 )2 · (5.8)


n nn+l
n - n - .e n -

Escrevendo tn = n!sn = n! (1 + fi + fi + • • • + ~i), obtemos tn E N, para todo inteiro n > l.


?\Ias, para n > 3,
1 n+2 1 1 2
-+---- =-+---+~---
n n(n + 1)2 n (n + 1)2 n(n + 1)2~
1 1 2 21
< 3+ 42 + 3. 42 = 48 < l,
de sorte que (5.8) fornece
tn-1 < (n - l)!e < tn-1 + 1,
para todo inteiro n > 3.
Suponha, por fim, que fosse e = ! , com p, q E N. Fazendo n = q +1> 3 nas desigualdades
acima, teríamos
t9 < (q - l)!p < tq + 1,
com (q - l)!p EN. Mas isso, obviamente, é uma contradição.

As desigualdades (5.7) permitem calcular rapidamente boas aproximações para o número e.


Por exemplo (cf. problema 2.1), elas implicam que s 9 aproxima e com erro menor que 10- 6 , de
forma que
e "' 2, 71828,

344
com cinco casas decimais corretas.

Para motivar o que vem a seguir, seja n E N. A desigualdade entre as médias arit
geométrica (cf. exemplo 54, capítulo 4) nos dá

1+2+···+n)"
n l• = 1·2· • • • ·n< ( ------ = (n+l)n
-- 2
n •
Assim, para n suficientemente grande, temos

nl ( 1 )n
(n/~)n < 1+ n ,..__,
e.
Por outro lado, como n + 1 < 2(n - 1) < 3(n - 2) < • • • para n > 3, nós obtemos

(n!)2 = (1 • n)(2 • (n - 1)) ... ((n - 1) · 2)(n · 1)


2 2
> n (n + 1r- = (n: 1)\n + ir.
Então, novamente para n suficientemente grande,

_·nl 2 > -- n ( 1 + -l)n/2 rv Je.


n"/ - n + 1 n
O resultado a seguir refina as estimativas grosseiras acima, dando uma estimativa assintótica
fina para o tamanho de n!. A fim de podermos enunciá-lo de forma adequada, é conveniente
mtroduzirmos duas notações. Dada::,sequencias (an)n~l e (bn)n;?:lde reais positivos, escrevemos
b11 e ª" = o(bn) para denotar os eguintes comportamentos assintóticos (i.e., à medida que
an ,..__,
11---++ ) de t:

Em particular, Gn = o(l) e limn-++ ªn = o.Também, se Q > o eªª" rv bn, então

lim bn - oan = O,
n-++oo an

de forma que ba~~ºº = o(l) e, daí,

bn = ª• + (bn - a0) = (a+ bn ~.ªª")a.= (a+ o(l))a •. (5.9)

O resultado do teorema a seguir é conhecido na literatura como a fórmula de Stirling3, e


sua demonstração pode ser omitida numa primeira leitura.

3 Após James Stirling, matemãtico escocês do século XVTII.

345
, ,, , , S(l<ll·.S :\1 \li· 1<1< \" I·. l>I. 1-'1 .\<.'< >Fs

TEOREMA 5.22. STIRLING

n!"' J27rn
(~)".

PROVA.
Temos de mostrar que limn-++oo( n"e:~v'n) = J; ou, o que é o mesmo, que

• 1og (n"e-"J2irn)
1rm = 1og--. 1
n-++oo • n! ,/2i
Inicialmente, mostremos que o limite do primeiro membro acima realmente existe. Para
tanto, observe que

n"e-"..fii,) n! 1 (n!) 2
log ( n! = nlogn - n - log ..fii,= (nlogn - n) -
2 log ~'

com
n-1

nlogn-n = L [((k + l)log(k+ 1)- (k + 1))- (klogk- k)]


k=l

= L
n-1

k=l
lk+l

k
logxdx -1

(onde utilizamos o resultado do problema 6.5, página 286, na última igualdade) e


2
!2 log (n!)
n
= !2 (log(l • 2) + log(2 · 3) + · · · + log((n .._ l)n)J

= ~ logk + log(k + 1)
~ 2 •
k=l

Portanto,

1og ( n"e-"..fii,)
1
=~
~
(1k+l logx d x- logk + log(k + 1)) -1.
------ (5.10)
n. k=l k 2
1
Para k > 1, faça ak = f11:k+Iogxdx - logk+l~(k+l)_ Pelo problema 3.13, página 244 (com
a= k, b = k + 1 e / = log, que é côncava), temos

ak = 1 k
k+l I d
og x x -
log k + log(k + 1)
2
> O.

Por outro lado, aplicando sucessivamente o resultado dos problemas 6.5, página 286, e 6.25,
página 288, temos

Jog (k + D - 1•+1 log x dx = log ( k + D - (k + 1) log( k + 1) + k log k + 1 > O.


346
.\ 1 < ;1 \I \...; .\1'1.11 • \<, < H" ,

Logo

ak < l g ( k +• 1) - l
2 2
2
=~l (k+½) 1 (1 1 )
2 k( + 1) = 21 g + k(k + 1)
1 1 1 (1 1 )
< 2 •4k(k 1) =- k - k + 1 •

ond u ilizamo o r ultado do probl ma 6.6 página 2 6, na úl ima d igualdad acima.


Concluímo qu Lk~l ak , uma ri d ermo po itivo , tal qu

n-1 n-1 1 ( 1 1 ) 1
Lªk<L- --- =-.
k=l k=l k k +l
rm Lk~l ak , converg nte, segue de (5.10) que xi t

n -n../n) n-1
lirn log ( n n = lirn ~ ak - 1 = ~ ak - 1.
n-++ n. I n-++ ~ ~
k=l k~l

Para o cálculo efetivo do limite eja e > O tal qu Lk~l ak - 1 = log e. Corno

lim log (
n"e-n../n)= log e
n-++ n. 1

terno

e = logc = exp ( nne-n../n))


lim log
n-++ ( n!
nn -n../n)
= n~~ xp log ( n!

= n-++oo
lim
nn -n../n) .
( --- n!
gu d (5.9) (comª"= n" -n..fii,, bn = n! = ¾)qu

R ord agora qu , d acordo om o ex mplo 3 , pí ul


ntão

7
para todo inteiro k > l. Portanto,

I2k ((2k)!)2 2k 7r
I2k-l 24 k(k!)4 2
[(¼+ o(l)) (2k)2ke- 2kv121cr2k. 1r

- 24k [ (~ + o(l)) kke-kJkr 2


21r k 2
= • 2 ----t 21rc.
(½+ o(1))
Então, se mostrarmos que ~ 21<-l
~ 1, seguirá dos cálculos acima que 21rc2= 1 ou, ainda,
e = J;. Por fim, o que falta ê o objeto do problema 2.6.

EXEMPLO 5.23.
Para n EN, o número binomial (2:) é o maior dos 2n + 1 coeficientes da expansão binomial
de (x + y)2". Isto nos dã

(2n+ 1,(2:) > t. e~)=


2'",

!2
de forma que (2:) > 2 ; 1 . A fórmula de Stirling melhora essa estimativa consideravelmente.
Realmente, com a ajuda dela, obtemos

Problemas - Seção 5.2


2.1. * O objetivo deste problema é provar que e rv 2, 71828, com cinco casas decimais corretas.
Para tanto, faça os seguintes itens:

(a) Para n > 10 inteiro, prove que

1 1
-+-+···+-<- 1 1( 1 1 1)
10! 11! n! 10! 1 + -11 + 112 + • •• + 11n-lO •

(b) Use o item (a), juntamente com o fato de que 10! > 2 . 106 , para provar que O <
e - í:!:i ¼r< 10- 6 •

348
(c) Conclua, a partir de (b), que 2,71828 é uma aproximação de e com cinco casM decimais
corretas.

2 . 2 • P rove que e-1 = '°'


.l..Jk>o
(-l)k
kl .

2.3. 1'.Iostre que limn-++ooVni = e.


2.4. Sejam o < Q < 1 um real dado e k, n E N tais que k > an. Prove que (k")
n
rv ((Jl)e)".
2,rn

2.5. a demonstração do teorema 19, vimos que, a fim de garantir a existência de


limn-++oo(I:;=1 ½ - log n), é suficiente garantir a convergência da série

L
k~l
G- log( k + 1) + log k) .

Para tanto, seja f: [1, +oo) ➔ IRa função tal que f(x) = ~ -log(x+ 1) +logx para x > l,
e faça os seguintes itens:

(a) Mostre que/ J(x)


é monótona e decrescente, com lim:r-++oo = O.
(b) Aplique o teste da integral para garantir a convergência da série do enunciado.

2.6. * Sejam n E Z+ e ln = f 0 12(cosxrdx.


1r O objetivo deste problema é mostrar que
1
limn-++oo 1; 1 = l. Para tanto, faça os seguintes itens:

(a) Mostre que ln+l < ln, para todo n > O.


(b) Use o item (a) do exemplo 34, capítulo 4, para concluir que cada uma das sequências
12k- 1 ) e (_!ll;__) é não decrescente.
( l2k-2 k~l l2k-1 k~l
(c) Conclua que as duas sequências do item anterior convergem para limites lo e f 1
digamos, tais que f0, f 1 > O.


1
1
(d) Mostre que 1;' = n~l ( 1:~ 1)- . Em seguida, faça n = 2k +oo para obter
fof1 = 1.
1
(e) Conclua que f0 = l 1 = 1 e, a partir daí, que limn-++oo J;1 = 1.

349
5.3 Séries de Taylor
Começamos esta seção apresentando mais uma generalização do TVM de Lagrange, a. fór-
mula de Taylor 4 com resto de Lagrange. Para seu enunciado, recorde (cf. seção 3.2) que
uma função f : [a, b] ➔ lR é n vezes continuamente denvável em [a.b] se f é 11 ,·eze derivável
em [a,b], com /Cn) : [a, b] ➔ lR continua.

TEOREMA 5.24. TAYLOR


Sejam / um intervalo e f : I ➔ R uma função n vezes derivável em [. Pnrn x0 .. r E /
distintos, existe e entre x 0 e x tal que

n-1 J(k)( ) j(n){ )


/(x) =L ktº (x - xo)4°+ n! e (x - xo)", (5.11)
k=O

PROVA.
Inicialmente, suponha que x 0 < x, e ja g : (x0 . x) ➔ n função dndn por

onde a E lR é escolhido de tal forma que g(x 0 ) = O (e,;dentemente, lHila tal-escolha é possível).
Como fé n vezes derivável em / e (xo,x] C /, é imediato que g é derivável em [x0 , x]. Como
g(x 0 ) = g(x) = O, segue do teorema de RôUe a existência. de e E (x 0 , x) tal que g'(c) = O. Por
outro lado, um cálculo simples fornece

a - f(n>(t)
g'(t) = ---(x- tr- 1,
(n - l)!

de forma que, para e E (x0 , x), temos g'(c) = O se. e só se, a= j<n>(c).Portanto,

O= g(xo) = J(x) - L J<">(xo)


n-1

k=O
k! (x - xo)
k
-
j(n>(c)
n! (x - xof,

conforme desejado.
Agora, suponha que x < x 0 e seja J = xo+x-l = {xo+x-t; t E/}. Então, J é um intervalo
e I = x 0 +x-J, de forma que fica bem definida a função h: J ➔ R, dada por h(s) = f(x 0 +x-s).
4
Após Brook Taylor, matemático inglês do século xvm.

350
Pela regra da cadeia, h ê n vezes derivável em J, com Mk>(s) = (-l)"J(k)(:r 0 + x - s), para
O< k < n. Portanto, aplicando (5.11) .ah e s0 < sem J, obtemos

h(s) = L h(k)(s
n-1

k=O
) h(n)(c)
k! o (s - sot + n! (s - sor,

para algum e E (so, s). Segue, daí, que

n-1 ( )kJ(k)(
Xo + x - so (
) ( l)nf(n)( e) (
f (xo + x - s) = L'"' -1
k'
.
s - s0
)k
+ -
n.1
)n
s - so ,
k=O

para algum e E (s 0 , s). Por fim, fazendo s0 =x e s = x 0 (veja que realmente temos s0 < s nesse
caso), obtemos (5.11) quando x < x 0 .

Se J : I ➔ lR é uma função duas vezes derivável, então, para x0 , x E I distintos, a fórmula


de Taylor com resto de Lagrange garante que

f (x) = f (xo) + J'(xo)(x - xo) + J"~c) (x - x 0 ) 2 , (5.12)

para algum e entre x 0 ex.


o que segue, dada uma função positiva e duas vezes derivável f : [a, b] ➔ JR,utilizamos a
relação anterior para estimar a diferença entre a área da região 'R sob o gráfico de f e a ãrea
da aproximação poligonal de 'R pela união dos trapézios acinzentados, na Figura 5.1. Por tal
razão, o resultado a seguir é conhecido na literatura como a regra do trapézio.

a= x 0 b= Xn

Figura 5.1: a regra do trapézio.

351
TEOkEMA 5.25.
Seja f : (a, bJ➔ 1Ruma função duas vezes derivável (mas não necessariamente positiva). Se
{ n = x 0 < :r1 < · · • < Xn = b} é uma partição equiespaçada do intervalo [a, b), então

/. a
b
f (x) dx - I: 2
n 1 (b -
(J(xk) + f(xk_i)) ~
ª) < (b - a) n
4 2
3
• sup lf"I.
k=l [a,b]

PROVA.
Para 1 < k < n, seja 9k : [xk-l, xkJ ➔ 1Ra função dada por

Denotando por ~ a expressão do primeiro membro da desigualdade do enunciado, segue da


aditividade da integral e do fato de que Xk - Xk-I = b~a, para 1 < k < n, que

n n (5.13)
- L9k(xk)
k=l
< L Wk(xk)I,
k=l

onde utilizamos a desigualdade triangular na última passagem acima.


Agora, como / é duas vezes derivável, o mesmo sucede com 9k·..Aderno.is, 9k(xk-i) = Oe
cãlculos fáceis fornecem

g~(x) = J(x) -1J'(x)(x - Xk-1) -1(/(x) + J(xk_i))


e
9Z(x) = -1J''(x)(x - Xk-1),
de sorte que gHxk-1) = gz(xk-1) = O. Portanto, a fórmula de Taylor com resto de Lagrange
garante a existência de CkE (xk-l, xk) tal que

9k ( Xk ) = 9k ( Xk-1 ) + 9k1 ( Xk-1 ) (


Xk - Xk-1
)
+ 9~( Ck) ( Xk - Xk-1)
2
2
= -~ J"(ck)(ck - Xk-1)(xk - Xk-1)2 .

Mas, como ;r,k - X1t-l = b~a, segue dos cálculos acima que

2
luk(xk)I = }lf''(ck)llck - xk-1l(xk - xk-1)

1 3
< 41/"(C1,;)liXk - Xk-1 13 = 1f "(Ck)1(b -n 3a)
4

352
Por fim, substituindo essa expressão na estimativa (5.13) para Â, obtem01

Em última análise, obtivemos no teorema anterior uma aproximação numérica, com erro
controlado, para o cálculo de uma integral. Conforme evidenciado pelo problema do cálculo do
comprimento de uma elipse (veja o final da seção 4.7), isso é, por vezes, o melhor que podemos
conseguir.
O problema 3.3 discute o emprego da regra do trapézio para o cálculo numérico aproximado
do comprimento de uma elipse. Para saber mais sobre a utilização dos métodos do Cálculo para
a obtenção de tais tipos de aproximações numéricas, sugerimos ao leitor o capítulo 15 de [4].
Voltando ao desenvolvimento da teoria, suponha agora que f : I ➔ lRé uma função infinita-
mente derivável. Para x 0 E /, dizemos que a série

~ f (k)( Xo) ( )k
L.J k! x - xo
k~O

é a série de Taylor de / em xo.


O resultado a seguir dá uma condição suficiente para que a série de Taylor de f em x 0 convirja
para/ em/.

Sejam I um intervalo e f : I ➔ R uma função infinitamente derivivel. Se


nstante e> o tal que 11<n>(x)I
< cn, para todos n > 1 e X E/, então, fixado%o

f (x) = L J(k>(xo)
k! (x - xo) ,
k

k~O

a todo x E/.

PROVA.
Fixados x E/ e n E N, segue da fórmula de Taylor com resto de Lagrange que
n-1 J(k)( ) j(n)( )
J(x) = L k!xo (x - xol + n! e (x - xo)'\ (5.14)
k=O

353
para algum e entre x 0 ex. Agora,
j(n)(c)
-(x-xo)
n (Clx - xolr n o
--
1n. < n.1 --t '

onde utilizamos o resultado do problema 2.3, página 89, na última passagem acima. Logo, basta
fazer n ➔ +oo em (5.14) para obter o resultado desejado.

EXEMPLO 5.27.
Como lsen<21>x1= lsen xi < 1 e lsen<2.1- 1>x1= 1cosxl < 1, segue da proposição anterior que

_ ""'sen{k)Q k _ ""' (-1)1-l 2J-1


sen x - L- k! x - L- (2 . _ l)!x ,
k~O J~l J

para todo x E R. Analogamente,

- ""' ( -1 )1 2J
COSX - L- (2 ')! X ,
J~O J

para todo x E R.
-~-~---------------~~------------
A seguir, utilizamos a fórmula de Taylor com resto de Lagrange p,.aramo~trar que a série de
Taylor da função exponencial converge para tal função em toda a reta.

TEOREMA 5.28.
Para x E R, temos
(5.15)

PROVA.
Fixado x E R, segue do teorema 24 (com x 0 = O) que

e=
:z: I: -+e•-
n-1
X
k
e Xn
(5.16)
k=O
k! n!'

para algum e entre O e x. Utilizando agora o fato de que a função exponencial é crescente,
juntamente com o resultado do problema 2.3, página 89, obtemos

= ec . lxln1 -< max{eº ' e:z:}. lxln1 ~ O


n. ------rn.•

354
Portanto, segue do teorema do confronto que limn-Hoo ec • ~·; = O. Por fim, fazendon
em (5.16), obtemos o resultado de ejado.

rote que o teorema anterior generaliza largamente o teorema 20. Para uso futuro, observamoe
que, trocando x por ax em (5.15), obtemos
k
ax ~ a k
e = ~ k!x , (5.17)
k~O

para todo x E IR.


Por vezes, estimativas simples do erro J<">,<c>
n.
(x - xor em (5.11) não são suficientes para
garantir que a série de Taylor de uma função infinitamente derivável convirja para tal função em
todos os pontos em que poderia fazê-lo. Vejamos um exemplo nesse sentido.

EXEMPLO 5.29.
A função x ~ log(l + x) está definida em (-1, 1) e é infinitamente derivável nesse intervalo.
Denotando f(x) = log(l + x) para lxl < 1, é imediato verificar que J<k>(x)= <-1~:~;~-l)!, para
todo k E N. Portanto, j<k)(O)= (-1)k- 1 (k - l)!, e segue imediatamente que a série de Taylor
de log(l + x), centrada em O, é dada por

(5.18)

I
Como 1(-lr xkl < lxlke Ek~l lxlkconverge quando lxl < 1, o teste da comparação ga-
rante que (5.18) converge em todo x E (-1, 1). Entretanto, estimando o erro em (5.11) como
teriormente, só conseguimos concluir que

a x E[-½, 1). De fato, segue de (5.11) que, para O< !xi< 1, temos

n-1 {-l)k-1 k {-l)n-1


log(l + x) = I: k x + n(l + e)" • x",

k=O

a algum e entre O e x. Se O < x < 1, então O < e < x < 1, e segue que

355
<%< O, então -1 < x < e < O, de forma que 1 + e > 1 + x > O. Logo,

-1 < x < O, concluímos facilmente que kL


l+.r
< 1 <=} -- 21 -< x < O; nesse intervalo, a última
eatimativa acima garante que
(-lt-l 1 n
---- · x" < - --+ O.
n(l + e)" - n
Por outro lado, para -1 < x < -½, temos 11:~ > 1, de modo que

portanto, nesse intervalo não podemos concluir que 1


1~(1 fc~~ .. r I
11
➔ ºquando n ➔ +

Remediaremos situações como a do exemplo anterior na seção 5.5. Por ora, mostremos que
pode mesmo ocorrer que a série de Taylor de uma função infinitamente derivável J : R ➔ 1R,
centrada em x 0 E IR, não convirja para f em nenhum intervalo aberto centrado em .r0 . :\rssc
sentido, o exemplo a seguir é clássico.

ExEMPLO 5.30.
Se / : R ➔ R é a função dada por

e-l/x, sex>O
J(x) = { O, se x < O

então/ é infinitamente derivável, com J(k)(Q) = O para todo k > O. Em particular. a sêne de
•-i1n,r de / centrada em O é identicamente nula.
-------~~---

PROVA.
e-l/x = O, de forma que J é contínua em O, logo, contínua em toda
Evidentemente, lim:r--+O+
a reta. Também evidentemente, f é infinitamente derivável em (O,+ ) e em (- , O), com
JCn>(x)= O se x < Oe n E Z+.
Afirmamos que, para n E Z+, existe um polinomio p11tal que

(5.19)

para todo r / O. V<'rífiquemos a afirmação por indução, sendo o caso n = Oóbvio. Por hipotc..,e
de indução, 1mpo11haqu<', pnnt um certo k > O, exista wn polinomio Pk tal que J'")(:r) ==

356
Pk(l/x)e- 1
/r, para todo x > O. Então, segue da regra da cadeia que, para ~J: > O,

1<•+1>(x)
= : 2 (P• G)- G))
p~ ,-i/, = Pk+i(1/x)e 1I ',

com Pk+1(x) = x 2 (Pk(x) - p~(x)), de forma que Pk+i é, novamente, um polinômio.


De posse de (5.19), façamos mais uma indução para provar que J ê infinitamente derivável
em O, com JCn)(Q) = O para todo n E Z+· Por hipótese de indução, suponha que f ê k ver.les
derivâvel em O,com J(k)(Q) = O. Como fé infinitamente derivável em lR\ {O},para mostrar que
J é k + 1 vezes derivável em O e que J<k+1>(0) = O, é suficiente mostrar que J(k+l)(Q) = O. Para
tanto, temos para h > O que
J(k)(h) - J<k>(o) Pk(l/h)e-l/h -1/h
h = h = Qk(l/h)e ,

onde Qk(x) = xpk(x), um polinômio; então, novamente pelo teorema 60, temos

lim J(k)(h) - J(k)(O) = lim Qk(l/h) = lim Qk(Y)= O.


h-+O+ h h-+O+ e1/h y-++oo eY

= O, uma vez que J(k)(h)


Por outro lado, para h < O, temos limh-+O-i<k>(h)~J<k>(o) = J{k}(Q) = O.
Logo, segue que J<k+l)(O) existe e é igual a O.

Problemas - Seção 5.3


3.1. Sejam I um intervalo aberto e f : J -+ lR uma função convexa, duas vezes derivável em
I. Se x 0 E / e r denota a reta tangente ao gráfico de f no ponto (x 0 , f (x 0 )), mostre que
nenhum ponto do gráfico de / está abaixo de r.

3.2. Sejam I um intervalo aberto, f : / -+ lR uma função n vezes continuamente derivável e


Xo E/ um ponto tal que f'(xo) = f"(xo) = · · · = J<n-l)(x 0 ) = O. Se nê par e J<n>(x
0) > O
(resp. J<n>(x0 ) < O), mostre que x 0 é um ponto de mínimo (resp. máximo) local estrito
para/.

3.3. Seja e(&) o comprimento da elipse E, de eixo maior 2a, eixo menor 2b e distância focal 2c
Use a regra do trapézio para mostrar que

2 b( n-1 2k(k + l)lnr 3


e(&)- : i+ ~ < n
2 J

onde k =f

357
( \l'l 11 1 < 1 -, SÉRIES :\'l":\li::nl<'.\S E DE Ft·xçôEs

3.4. Obtenha as séries de Taylor das funções senh e cosh. Em cada caso. mostre que a :::ienede
Taylor converge para a função correspondente em toda a reta.

3.5. Se I é um intervalo e f : / ➔ IRé uma função n vezes derinh·el e tal que /<11> é con:::.tante,
prove que fé ou identicamente nula ou um polinom10 de grau menor ou igual a n ~ltus
precisamente, se x 0 E /, mostre que
f j)(xo)
=L .
n
J(x) 1
(x - xo)1.
1=0 J.

3.6. Use (5.16) para demostrar o teorema 60.

O próximo problema retoma a análise do método de ewton parn npro,1mnçôcs de


raízes de funções deriváveis (cf. problema 2. , página 14 ).

3. 7. Seja / : [a, b] ➔ IR urna função contínua cm [a. b) e dun'""v "Z ::; cont inunm 'lll ' clcrivi\velcm
(a, b), com!', J" > O em (a. b). Suponhn, ad mni, que f(n) <O< f(b), e s<'jn o 11 únicn
raiz de J no intervalo [a,b] (cf. problema 2."", phginn M ). Fnçn os seguinte~ ill'ns:

(a) Se /3 E (a,b) e 'Y = /3- J}~\,mostr qu o !5-y < fJ.


(b) Se (a,1)n~t é uma sequencio tnl que a 1 E (a, b] e n,.+1 = On - Pc'::\,
pnrn todo n > 1,
mostre que (a 11 ) 11 > 1 com·crg e ª" -+ a quando n ~ +
(e) Refine a análise do item (b), do guinte modo:
(i) Mostre que exi te E.,,E (a, on) tal que /(an) = J'(CLn)(an-
o) - ½J"(f.11)(a11- o )2 .
·1•)
(1 Conclua que O< On+l - O -- 2/"c.
J"({n) 2 L.ilil
,(an - o) < 2/'({n/an
2
- o) .

(iii) Suponha que a < e < d < b são tais que d - e < 1 e J(c) < O < J(d) Se
L...
À = ma.x[c,d]
21, e começarmos com 01 = d, use (ii) para concluir sucessivamente
que O< On+1 - o < >.(an- o) e O<ª" - a < >.n(d- c) para todo n > l.
2
2

",

Ainda em relação ao problema anterior, note que o problema 2.9, página 90, garante que
).."(d - c)2 n ➔ O quando n ➔ +oo. Portanto. o resultado do item (iii) acima estima o
erro com que an aproxima a. Note também que podemos obter resultados anãlogos ao
do problema anterior supondo que /' > O e /" < O em (a. b), bastando começar com
a E [a, o). O mesmo vale se supusermos que f' < O e f" < O (resp. /" > O) em (a. b),
1

bastando começar com o 1 E (a. b) (resp. a 1 E [a. a)).

3.8. O polinômio f(x) = x 3 - 2x - 5 é tal que /(2) = -1 e/(!)=~- Portanto, o TVI garante
a existência de uma raiz a E (2, ~) para/. Aplique os resultados do problema anterior
para estimar o com cinco casas decimais corretas.

35
5.4 Séries de funções
O material desta seção é um prelúdio ao e tudo das séries de potências e estende, a sequêll
e séries de funções. alguns conceitos e resultados das seções 2.2 e 5.1. Comecemos definindo o
conceito de limite de uma sequência de funções.
Em tudo o que segue. salvo menção em contrário, I denota um intervalo.

DEFINIÇÃO 5.31.
Para cada n E N, seja dada uma função fn : I ➔ R. Se, para todo x E 1,
limn •. 00 fn(x), definimos o limite pontual da sequência de funções (/n)n2'.l como a função
J •I ➔ IRtal que
J(x) = lim /n(x), (5.20}
n-++ ,

para lodo .r E / Nesse caso. dizemos que (fn)n>l é uma sequência de funções pontualmente
convergente ou. ainda, que (/n)n 1 converge pontualmente para/.

O limite pontual de uma sequência de fnnçõe fn : / ➔ R, se existir, é único. Realmente,


se (/ n)n 1 converge pontualmente para / : / ~ R, segue de (5.20) que, para cada x E /, a
sequencia de números reais (/,i(.r)),.2:1 com· ~rge pata /(.r.); portanto, a unicidade do limite de
scquencrn.s con\'ergentcs de uumero 1cais {cf. propo!:iiçãol•l) garante que existe um único valor
possível para /(.r).

E:\l·.\IPLO 5.32.
Col<'c101uunos.aqui. dob t•xemplos ilustrando o fato de que o limite pontual de uma sequência
de funçocs pontualmente convergente não é necessariamente uma função bem comportada.

(a) Para cadn 11 E N, seja /,. : (O,l] ➔ R a função tal que J,.(x) = x", para todo x E [O,1).
Segue do exemplo 12 que o limite pontual de (f ,.)n2:1é a função / : (O,1] ➔ R tal que

f(x) = { O. se x E [O,1) .
1, se x = 1

Em particular. esse exemplo deixa claro que o limite pontual de uma sequência de
contínuas pode ser wua função descontínua.

(b) Para cada n E N. seja J,.: [O.l] ➔ 1Ra função tal que fn(x) = nx(l - z2}n, para
[O,l]. Como O< x < l ⇒ O< l-x 2 < 1, segue do exemplo 24 que limn➔+ao n:t(l-r)•
se O < x < 1. Também, como fn(O) = /n(l) = O para todo n E N, conclufmoeiq
o limite pontual da sequência (fn)n?.l, e que esse limite é a função identi,"'4c-~.,~·

359
f: [O,1) ➔ R; em particular, fé integrável, com f01 f(x)dx = O. Observe agora que, pelo
TFC, temos
1 n(l - x2t+l ,x=l n
/.
Ín(:r)d:r= - 2(n + 1) :c=O = 2{n + 1) •
0

Portanto, existe limn➔+oo f0 fn(x)dx


1
= limn-++oo2(n':..i)= ½,mas
1
lim Í fn(x)dx #- [1 lim fn(x)dx.
n➔+oo lo lo --------------
n-++oo
~- ------~---~~-

A seguir, introduzimos um conceito de limite para sequências de funções que é mais restritivo
do que a noção de limite pontual. Contudo, conforme veremos em seguida, sob la! restrição
adicional as patologias ilustradas nos exemplos anteriores não terão mais lugar.

DEFINIÇÃO 5.33.
Uma sequência (fn)n~l de funções fn: I ➔ R converge uniformemente para uma função
J : I ➔ R se a seguinte condição for satisfeita: dado € > O, existe n0 E N tal que
n > no => IJn(x) - f (x) 1 < €. V x E J. (5.21)

Em palavras, uma sequência {fn)n~1 de funções reais com domínio J converge unjformemente
para/ : / ➔ R se pudermos tornar o erro lfn(x)- f (x)I menor que€ simultaneamente para todo
x E /, bastando, para tanto, tomar o índice n suficientemente grande (n > n 0 , nas notações da
definição anterior).

EXEMPLO 5.34.
Para n E N, seja Ín : R ➔ R a função dada por fn(x) = ( 1.;.r:2)". Então, (/ n)n~I converge
uniformemente para a função identicamente nula.
----~~----~---~--------~
PROVA.
Como 1);;2 < ½ para todo x E R, temos lfn(x) - OI= l/n(x)I < 2~, para todo x E R. De
posse dessa desigualdade, e dado€> O, escolha no E N tal que 2~ < f para n > n 0 . Então, para
n > no, temos lfn(x) - OI< f para todo x E R, conforme desejado.

O corolário a, seguir é uma consequência imediata da definição 33.

360
COROLÁRIO 5.35.
Se uma sequência (/ n)n'?.l de funções fn : / ➔ lR converge unitor:me
então_a sequência (fn)n'?.l converge pontualmente para/. ________ __._ ...........

O conceito de convergência uniforme tem a seguinte interpretação geométrica: como

lfn(x) - f(x)I < € {:} f (x) - € < fn(x) < f(x) + €,


temos que (fn)n'?.l converge uniformemente para / : / ➔ lR se, dado arbitrariamente € > O,
existir no E N tal que, para n > no,o gráfico de fn está contido na faixa de largura 21: do plano,
centrada no gráfico de/ (a região acinzentada, na Figura 5.2).
y

ln

a b X

Figura 5.2: interpretação geométrica da convergência uniforme.

A imporlancia do conceito de convergencia uniforme repousa nos teoremas de convergência 36


e 37, apre enlados a seguir. Em particular, coligindo tais resultados com as sequências de funções
elencada.':ino exemplo 32, concluímos que a recíproca do corolário 35 não é necessariamente
verdadeira. De outra forma, concluímos que existem sequências (! n)n'?.l de funções, definidas
em um intervalo J, tais que (/n)n'?.l converge pontualmente, mas não uniformemente, para uma
função f : I ➔ R.

TEOREMA 5.36.
Se uma sequência (fn)n'?.l de funções contínuas fn : I ➔ R converge
uma função f : I ➔ lR, então f é contínua.

PRO\'A.
Para x 0 E / e n E N, segue da desigualdade triangular que

lf(x) - f(xo)I < lf(x) - fn(x)I + lfn(x) - fn(xo)I + lfn(Xo) - f(xo)I,

361
Agora) pnrn f > O dado, a convergência uniforme de (/ n)n'?.l para / garante a existencia
de no E N tnl que n > no ::::} lfn(x) _ J(x)I < t, para todo x E /. Portanto, escrevendo a
dc:-.igualdade anterior para n = n 0 , obtemos

IJ(:r) - f(xo)I < 2 • ~ + l/n 0 (x) - fn 0 (xo)I = ~ + lfno(x) - fno(xo)I.

Por outro lado, a continuidade de fno garante a existência de ô > O tal que
é
x E J, lx - xol <ô=} lfn0 (x) - fno(xo)I <
2-
Portanto, para x E / tal que lx - x 0 1 < ô, temos
é é é
IJ(x) - f (xo)I < + lfn0 (x) - fno(xo)I < 2 + 2 = é,
2
de forma que fé contínua em x 0.
Por fim, como x 0 E I foi escolhido arbitrariamente, segue que a função J é contínua em I

TEOREMA 5.37.
Seja (fn)n'?.l uma sequência de funções contínuas f n : [a, b] ➔ IR,que converge uniformemente
para uma função J : [a, b] ➔ IR. Sendo 9n, g: [a. bj ➔ R as funções definidas por

9n(x) = J.:c fn(t)dt e g(x) = J.:r J(t)dt,


temos que (gn)n'?.I converge uniformemente para g. Em particular,

b f(x)dx = lim 1b fn(x)dx. (5 22)


J.a n-++oo a

PROVA.
Para x E [a, b], segue da desigualdade triangular para integrais (cf. proposição 13) que

lu.(x) - g(x)I = [ Un(t) - J(t))dt < [ lfn(t) - J(t)ldt-

Agora, a convergência uniforme de (fn)n'?.l para/ garante que, dado é > O, existe no E N tal
que
é
n >no::::} lfn(t) - J(t)I < b _a,
para todo t E [a, b]. Portanto, para n > no ex E [a, b], segue das desigualdades acima que
e
1
.r é é
l_q,,(,r)- _q(.r)I< -b- dt = -b- • (x - a)< -- • (b - a)= é.
11 -a -a b-a

362
Para a segunda parte. lembre-se de que convergência uniforme acarreta convergênciapont~
Portanto, 9n(b) ➔ g(b) quando n ➔ +oo. o que é o mesmo que (5.22).

OBSERVAÇÃO 5.2.
Nas notações do enunciado do teorema anterior, suponha que as funções /n são
mas que (f n)n'?.l converge pontualmente para uma função integrável a Riemann / : (a, bJ
Se existe L > O tal que
lfn(x)I < L. V x E [a.b]. n EN, (5.23)
é possível mo::;trar que (5.22) ainda é ,·erdadeira. Esse é o conteúdo do teorema da convergên-
cia dominada (abreviamos TCD) de Lebesgue. mas sua demonstração foge ao escopo destas
notas. Para o leitor intercs ado. sugerimo!:>16Iou 1271
Por fim, note que o item (b) do exemplo 32 mo:-.traque a condição (5.23) é necessária para a
validade <loTCD. Por outro lado, o problema 4.4 deixa claro que mostrar a validade de (5.22)
cm urn caso particular. na au:-.ênciad• conY~rgência uniforme mas sem o auxílio do TCD, pode
tart>fa difícil.
..,l'I' 11111a

Voltemo nos, agora, no ~ludo dn:-.,t r·t<, de funçõ~.

DEFINIÇ,\O5.3'"'.
Dada uma ~cqu(~ncia(fn)n'?,l dt• fu11çõ• /n : J ➔ IR. definimos a série de funções

como n ~cqnência (,,n)n"?.I de funçõc~ ~,. : J ➔ IR, tal que sn = E~=l Ík, para todo n > 1.

:\a~ notnçõc da definição acima. dizemos que Ek'?.l Ík converge pontualmente (resp. unifor-
memente) cm/ para uma função J: / ➔ IR se a sequência (sn)n"?.l converge pontualmente (resp.
uniformemente) para f. Xes e caso, escrevemos

e ob~en·amo que J(x) = Ek~l fk(x). para todo x E /.


O mai das yeze . aplicaremos os teoremas 36 e 37 a séries uniformemente convergentes de
funçõe . Ne e entido, o corolario a eguir é o resultado de nosso interesse. Observe que,
em palavras. eu item (b) garante que séries uniformemente convergentes de funções contínuas
podem er integradas termo a termo.

363
COROLÁRIO 5.39.
Para cada n > 1, seja f n : [a, b] ➔ IR uma função contínua. Se a série Lk~l A converge
uniformemente em [a, b] para uma função f : [a, b] ➔ IR, então:

(a) f ê contínua.

PROVA.
(a) Se Sn = I:;=1 fk, então Sn é uma soma finita de funções contínuas, de sorte que sn é contínua.
Mas, como sn ➔ f uniformemente, o teorema 36 garante que f é uma função contínua.

(b} Como sn ➔ f uniformemente, segue do teorema 37, juntamente com a aditividade da mtcgral,
que

10
b
J(x}dx = lim
n-++oo
1O
b
sn(x}dx = lim /.
n-++oo 0
b n
L !1.(x)dx
k=l

L1
n b b
= n~~oo
k=l o
J,.(x)dx = L J.fk(x}dx.
k~l a

O corolário anterior somente será útil se tivermos uma maneira eficiente de discernir, nos casos
de interesse, quando uma série de funções é uniformemente convergente. O teorema a seguir.
conhecido na literatura como o M-teste de Weierstrass, preenche essa lacuna, apresentando
uma condição suficiente simples para a convergência uniforme de uma série de funções. Observe
que o" M' no nome M-teste de Weierstrass refere-se a maJoração.

~MA 5.40. M-TESTE DE WEIERSTRASS


Seja E,~1 f 1c uma série de funções definidas em um intervalo I e satisfazendo as seguintes
condições:

(a) Para cada n > 1, existe Mn > O tal que lfn(x)I < Aln, para todo x E J.

(b) A lérie E1c~i M1c converge.

Então, a série de funções Lk~l !k converge uniformemente em I.

364
PRO\'A.
Fixado x E J, como lfn(x)I < Aln para todo n > l e Ek2:I M1c converge, o teste da coa~
ração para séries de números reais (cf. proposição 14) garante a convergência absoluta da .eíie
Ek2:I Ík(x). Portanto, fica bem definida a função f : I ➔ IR, tal que

f (x) = L !k(x), V x E J.
k2:l

Para estabelecer a convergência uniforme de Ek2:l Ík para f, seja Sn = E;= 1 !,,. Para x E/
e n EN, temos

(5.24)
k>n k>n k>n

Agora, dado € > O, a convergência da sequência (E;= 1 Mk)n2:l para Ek2:l Mk garante a
existência de n 0 E N tal que
n
L Mk - L A1k < t, V n > n 0.
k2:l k=l

Portanto, 1Ek>n Mk 1 < €, para todo n > n 0.


Voltando a (5.24), concluímos que, para n > n 0, tem-se

1/(x) - sn(x)I < L Mk < €,


k>n

para todo x E J. Portanto, (sn)n2:1 converge uniformemente para/, e isso é o mesmo que dizer
que Ek2:l Ík converge uniformemente para f.

EXEMPLO 5.41.
A série de funções Ek2:l -bsen(kx) converge uniformemente em R. De fato,
1 para todos k > l ex E R, temos

1 1
2 sen(nx) < 2n ,
n

para todos n > 1 ex E IR. Mas, como Ek2:I -b


converge, basta aplicar o M
para obter a convergência uniforme desejada. ---~~~----....;..;:;::..::..;____,,

Acidentalmente, o exemplo anterior mostra que, dada uma série Ek2:l Ík de funções deriváveis
fn : I ➔ R, uniformemente convergente em I, não necessariamente a série Ek2:l fÍc ê conver-
gente, mesmo pontualmente, em todo o intervalo I. Para um exemplo, veja que L1c2:i j,sen(kz)

365
conYcrgc uniformemente em lR, mas

não converge em .t algum da forma x = 2f.7r,com eE Z.

Problemas - Seção 5.4

4.1. Prove que a série Lk~l ( 1;:r 2 )k converge uniformemente em 1Re calcule sua soma.

4.2. Prove que as séries de Taylor das funções ex, sen x e cos x convergem uniformemente para
tais funções, em cada intervalo da forma (-a, a], com a > O.

4.3. Para n E N, seja fn : [O,1] ➔ .IRa função dada por fn(x) = nxe-nx. Prove que (J11 )n~1
converge pontualmente, mas não uniformemente, para O Iostre também (sem apelar para
1
f
o teorema da convergência dominada) que 0 fn(x) dx ➔ O quando n ➔ +

4.4. Para n > O inteiro, seja fn: [O,1] ➔ 1Ra função dada por fn(x) = x"e-x. Faça os seguintes
itens:

(a) Mostre que (fn)n>o converge pontualmente para a função f [O,1] ➔ lR, tal que
J (x) = { O~se x-::: 1 _
e 1 , se x = 1

(b) Mostre que, para cada n > O inteiro, existem naturaisª" e bn tais que f01 fn(x) dx =
ª" - bne- 1 , com ao = bo = 1 e ª" = nan-1 e bn = nbn-1 + 1, para todo n E N.
(c) Use o resultado do item anterior para concluir que ª" = n! e bn = n! I:;=0 ~!' para
todo n EN.

(d) Use o resultado dos dois itens anteriores para mostrar que, quando n ➔ +oo, temos
J0 fn(:r) d.e ➔ O.
1

4.5. Seja (/n)n?l uma ~cquência de funções deriváveis Ín : [a, b] ➔ lR, tal que IJ~(x)I < 1'1n
para todos n E N e x E [a, b), onde Lk~I Ah é uma série convergente. Se existe x 0 E [a, b)
tal que a série L1.->i J,..(To) converge absolutamente, mostre que a série de funções Lk~i Ík
c:or1vc•rgc?
1mifon11cmc11te no intervalo [a, b].

366
4.6. Seja J: R ➔ R wna função seccionalmente contínua e periódica, de perfodo211'.A ..
de Fourier 5 de / é a série de funções

onde
ak(!) = 1r1 /_71'
-11' J(x) cos(kx)dx e bk(!) = 1r1171'
-11' f (x) sen (kx)dx

para todos os valores possíveis de k.

(a) Se J é continuamente derivável e k > l, prove que

A partir daí, conclua que ak(f), bk(f) ➔ O quando k ➔ +oo.


(b) Se J é duas vezes continuamente derivável e k > l, prove que

A partir daí, conclua que a série de Fourier de f converge uniformemente em R e,


portanto, define uma função contínua em R.
(c) Ainda supondo que fé duas vezes continuamente derivável, seja g: R ➔ R a função
contínua definida por

g(x) = ªº;!) + L (ak(f) cos(kx) + bk(f) sen (kx)).


k~l

Prove que at(g) = at(f) para l > O e b1(g)= b1(J) para l > 1.
4. 7. O teorema de convergência de Fourier afirma que, se f : R ➔ R é contínua, periódica de
período 21r e derivável, com derivada seccionalmente contínua, então a série de Fourier de
f converge para/ em toda a reta. Admitindo a validade dessa afirmação, faça os seguintes
itens:

(a) Se f : IR ➔ IR é periódica de período 21r e dada no intervalo [-,r, 1r]por f(x) = x~,
calcule a série de Fourier de f.
{b) Use o teorema de Fourier para mostrar que Ek~l tJ= ~ 2

5
Após Joseph Fourier, matemático e físico francês do século XIX.

367
Para uma prova. do teorema de convergência de Fourier, juntamente com urna discussão
de muitas outras propriedades interessantes de séries de Fourier, sugerimos ao leitor as
referências [19] ou [27).

4.8. O teorema de aproximação de Weierstrass 6 afirma que toda função contínua f :


[a, b) ➔ IR é o limite uniforme de uma sequência de funções polinomiais Pn : [a, b) ➔ IR.
Admitindo a validade desse resultado, se J : [a, b) ➔ IR é uma função contínua e tal que
J:f(x)xk dx = O para todo k E Z+, prove que fé identicamente nula.

4.9. Em relação à série de funções Lk;::i fsen (kx), faça os seguintes itens:

(a) Use o critério de Abel {cf. problema 1.16, página 340), juntamente com a discussão
dos itens (b) e {c) do problema 1.18, página 340, para concluir que a série dada
converge pontualmente no intervalo (O,21r).
{b) Revisite a prova do critério de Abel, conforme delineada no problema 1.16, página
340, e nas sugestões ao mesmo, para mostrar que a identidade de Abel garante que
a convergência do item (a) é uniforme em todo intervalo da forma [ó,21r- ó), para
o< ó< 1í.

tJPara uum provu C>)erncutnrdo teorema de aproximação de Weierstrass, sugerimos ao leitor as referências 17],
[20J ou [27J.

368
5.5 Séries de potências
.
a seção 5.3, vimos exemplos de funções infinitamente deriváveis, definidas em um •
aberto e que coincidem com suas séries de Taylor em tal intervalo.
A série de Taylor de uma função infinitamente derivável é um caso particular de uma série
de potências, i.e., uma série de funções da forma

L ak(x - xo)\ (5.25)


k~O
onde ao, a1, a2, ... são números reais dados. esse caso, como no caso de séries numéricas,
dizemos que an(x - xot é o n-ésimo termo da série de potências (5.25), e que an é o n-ésimo
coeficiente da série.
esta seção, desenvolvemos os aspectos mais básicos da teoria de séries de potências, come-
çando com o seguinte resultado central.

TEOREMA 5.42.
Dada a série de potências Lk~o ak(x - x 0 )k, existe O< R < +oo tal que a série:

(a) Converge absolutamente no intervalo (x 0 - R, x 0 + R) e diverge em R \ (x0 - R, Xo + R).


(b) Converge uniformemente no intervalo [xo - r, xo + r], 'v'O < r < R.

PROVA.
ote primeiramente que a série Lk~o ak(x - xo)k converge absolutamente no intervalo (x 0 -
R, x 0 +R) se, e só se, a série Lk~O akxk converge absolutamente no intervalo (-R, R). Da mesma
forma, a série Lk~O ak(x - xo)k converge uniformemente no intervalo [x0 - r, x 0 + r) se, e só se,
a série Lk~o akxk converge uniformemente no intervalo [-r, r). Portanto, podemos supor que
Xo = Ü.
Analisemos primeiramente o item (a).

Afirmação 1: se a série Lk~o akxk é convergente quando x = a =/O, então ela é absolutamente
convergente, qualquer que seja x E (-a-, a-).
De fato, para um tal x, temos

(5.26)

Mas, como Lk>o akok converge, a proposição 4 garante a existência de no E N tal que n > no *
lano-nl < 1. Po;tanto, segue de (5.26) que lakxkl 1! lk < j; jk para k > no. Então, uma ve-J que a

369 Â
:-iéricgeométrica Lk~no l!lk
é convergente (posto que l!I
< 1), o teste da comparação garante
que o mesmo se passa com Lk~no lakxkl e, logo, com Lk~o lakxkl-

Afirmação 2: se a série Lk~o akxk é divergente quando x = /3 =/:-O, então ela é divergente,
qualquer que seja x E 1Rtal que lxl > 1/31-
Realmente, para um tal x, se Lk~o akxk convergisse, então, pela afirmação anterior, tal con-
vergência acarretaria a convergência absoluta (e, logo, a convergência) da série Lk>O ak/3k, o que
é um absurdo.

As afirmações 1 e 2 garantem que, em relação à série Lk~O akxk, uma das tres possibilidades
a seguir ocorre: (i) ela só converge quando x = O; (ii) ela converge absolutamente, para todo
x E lR; (iii) existem a, /3 =/:-O tais que lol < 1/31e a série em questão converge , bsolutamcnte
quando lxl < o e diverge quando lxl > 1/31-
Se um dos casos (i) ou (ii) ocorrer, nada mais há a fazer Se (iii) ocorrer, fica bem definido
o real positivo

R = sup{r > O; Lªkuk com·erge ab olutamente quando lul< r}.


k~O

Afirmamos que Lk~o akxk converge absolutamente quando !:ri< R e diverge quando lxl > R
Para tanto, consideremos dois casos separadamente:

(i) Se lxl < R, tome r tal que lxl < r < R e L1.~o akuk converge absolutamente quando lul < r.
Então, em particular, Lk~O akxk converge absolutamente.

(ii) Se R e Lk~o akxk convergisse, então, tomando R tal que R < R < lxl, seguiria da
lxl >
afirmação 1 que Lk~o akuk converge absolutamente quando lul < .R.Mas isso claramente con-
tradiz a definição de R. Logo, Lk~O akxk diverge.

Para o item (b), dado O < r < R, segue do que fizemos acima que Lk~O lakrkl converge.
Então, fazendo Mk = lakrkl, temos, para lxl < r, que lakxkl < lakrkl = Arlk. Portanto, pelo
M-teste de Weierstrass, a série Lk~o akxk converge uniformemente no intervalo [-r, r].

Nas notações do teorema anterior, dizemos que O < R < +oo é o raio de convergência da
sé>ricde potências (5.25).
O corolário do teorema anterior demonstrado a seguir mostra que, em certas circunstâncias,
podc•1110scalcular facilrncnlc o raio de convergência de uma série de potencias. ~esse ponto.

370
sugerimos ao leitor reler a demonstração do teste da razão (cf. proposição 16), o
ela continua válida se supusermos que { = + ~

COROLÁRIO 5.43.
SeJa dada uma sequência (an)n>O de reais não nulos. Se existe O < R <
limn-.+oo1~ 1 = R. então a série d; potências Lk>O ak(x - xot tem raio de coo
n+I -
---~-~----~---------
PRO\A.
Como
an.J-1 (x - Xo)n+l lx - xol n lx - xol
an(X - xo)" - ,~1 --t R

o teste da razão garante que a séne Lk~o ak(x - x 0)k converge absolutamente se ix;ol < 1 e
diverge se 1.r Rxoi > 1. De outra forma. L1.>o ak(x - x 0 )k converge absolutamente se lx - xol < R
e diverge se l.r.- xo 1 > R Portanto. o teorema anterior garante que o raio de convergência da
:,é11eLk~o ak(x - :rol é igual a R.

EXE~1PLO 5.'14.
(a) A sede de potência-, Lk 1k
! .rk tem raio de conYergfmcia1, uma vez que I 1/(n+l)
l/n 1 = !!±!
n
➔1
quando 11 ➔ +

(h) A sénc ~
~A ~1
kl••rk tem mio de convergência O' pois 1____!!1_
(n+l)!
1 = -n+l1- ➔ O•
(c) A série Lk'>I b(:r - 2)" tem raio de convergência +oo, pois l 1A~~'
1,, 1 =n+1 ➔ +oo
quando 11 ➔ +
(cl) O C'orolárioanterior não se aplica a série I:,.~1 i,x 2 k, uma vez que ela tem uma infinidade
de termo~ com coeficientes iguais a O.

(e) Dado a -=/=O. o corolário anterior garante que a série Lk~ 0(ax)k tem raio de conver~
1
!
pois 10 ~: 1 1 =
1
,

1
!
para todo n > O. Pela proposição 1, temos, para lxl < for,
1
, que

1
"(ax)k
L,_
= 1- QX
.
k~O

A eguir, colecionamos outra consequência do teorema 42, a qual será de crucial importância
para a demonstração do resultado subsequente.

371
PROPOSIÇÃO 5.45.
Se a série de potências Lk~o ak(x - x 0 l tem raio de convergência R > O. então:

(a) A função/: (x 0 - R,x 0 + R)-+ IR, dada por /(x) = Lk~oak(x - .rol, é contínua.

(b) Para todo x E (x 0 - R, x 0 + R), temos J~J(t)dt = Lk~O ,.":f.


1 (x - .r0 )>.-+i
_ Em particular,
a série do segundo membro tem raio de convergência maior ou igual a R.

PROVA.
Seja Ík : (xo - R, Xo + R) -+ IR a função dada por fk(x) = ak(x - .r.0 l. Pelo tcorcmn 42.
temos J = Lk~I !k em (xo - R, x 0 + R), sendo tal convergência uniforme cm [.i.:o
- r . .1o+ r], pnrn
todo O < r < R. Como Ík é contínua, os itens (a) e (b) da propo ição gu m. imcdintnmcntc,
dos itens (a) e (b) do corolário 39.

O próximo teorema é o segundo rc':iultado ccntrnl cln tcorin de ~ ~rir" cl, potêncins.

TEOREMA 5.46.
Se a série de potências Lk~o ak(x - x 0)k tem raio ele convergência R > O, então:

(a) A função / : (xo - R, Xo + R) -+ R, dnda por J(x) = Lk~O ak(.r - .r0)\ e infinitamente
derivâvel.

(b) Para todo n EN, temos J<">(x) = Lk~n (k~~),-ak(x-xo)k-n, para todo x E (.r0 R r.0, R).
e a série que define /Cn) também tem raio de conYergência R.

PROVA.
Como na demonstração do teorema anterior. podemos supor. sem perda de generalidade 1 que
x 0 = O. Provemos primeiro que, sendo R o raio de convergência da série Lk>l kakxk- 1 , temos
R = R e f'(x) = Lk~I kakxk-i, para todo x E (-R, R). - '
Observe inicialmente que lkakxkl > lakxkl, para todos k > 1 inteiro ex E R. Portanto, se a
série Lk>l kakxk-l converge absolutamente, então o mesmo se passa com Lk>l ka,,_xk e, logo,
com Lk;o akxk. Assim, R < R. -
Agora, para O < x < R e O < h < R - x, temos

372
Surn-:s ()(·. ('()( l.'\<"I \S /

Por outro lado, pelo TVM de Lagrange, para cada k > 1 existe Ck E (:e,:r+ h) tal que (:r+l•r-;rl,=
kct- 1 . Portanto,
f (X + h) - f (X) ""' k k-1
h = ~ akck ,
k~l

de forma que a série Lk~i kakc~- 1 converge absolutamente e, como lkakxk-ll < lkakcZ- 1 1,o teste
da comparação garante que Lk~o kakxk-l também converge absolutamente. Analogamente, tal
série converge absolutamente se -R < x < O, de forma que R > R.
A discussão até aqui garante a boa definição da função g : (-R, R) ➔ JR,dada por g(x) =
Lk~o kakxk-l. Pelo item (b) da proposição anterior, temos

Portanto, o TFC garante que, para !xi< R, temos


J'(x) = g(x) = L kakxk-l.
k~O

Para o que falta, suponha, como hipótese de indução, que já mostramos que f é m vezes
derivável, com J<m>(x)= Lk~m (k~~>'• ak(x - xo,)k-m, e que essa última série também tem raio
de convergência R. Então, pela primeira parte, /Cm)é derivável e a série de potências que define
(/Cm))'= J<m+l) tem raio de convergência Reé tal que

k!
J'm+l)(x) = "°' --- · (k - m)ak(x - xot-m-l
~ (k-m)'
k~m+l •
k•
_ ""' ' . ( _ )k-(m+l)
- ~ (k _ (m + l))! ak x xo •
k~m+l

ras notações do item (b) do teorema anterior, observe que a expansão em série de potências
de J<n>é obtida derivando termo a termo, n vezes, a expansão de f em série de potências.
Realmente, denotando a n-ésima derivada de uma função por fxn», um cálculo imediato fornece

dn ( )k k! ( )k-n
dxn x - Xo = ( k - n) ! x - Xo •

A seguir, colecionamos duas consequências úteis do teorema anterior, a primeira das quni
refina a análise do item (b) da proposição 45.

373
COROLÁRIO 5.47.
Se a série de potências Lk~o ak(x - x 0 )k tem raio de convergência R > O. então a série de
potências Lk>O k°ti (x - x 0 )k+l também tem raio de convergência R.

PROVA.
Como Lk~O ak(x - xo)k = Í:rLk~O k°.ti(x - xo)k-Ll,o item (b) do teorema anterior garnnle
que ambas as séries do enunciado têm um mesmo raio de com·ergêncit1.

COROLÁRIO 5.48.
Suponha que a série de potências L1.:~oak(.i::- .r0 )k tem mio ele convcrgêncin R > O, e' ::-cja
J : (xo - R, xo + R) ➔ lR a função dada por f (.r) = I::,..::ook(:r - .rot. Então:

(a) an = fC">(xo)
n! , para todo n >
- O•

PROVA.
O item (b) segue prontamente de (a). Pnrn (a), auc cio item (b) do teorema antcrio1 que
f (n) (x O) = _,i!_
(n-n)!
. an = n !an,. portanto 1
an = f(n)(zo).
11 ,

A seguir, veremos que os resultados discutidos até aqui nesta seção permitem reobter, por
meio de uma abordagem unificada, algumas das e.'\.-pansõesem séries de Taylor discutidas na
seção 5.3. A esse respeito, veja também o problema 5.2.

EXEMPLO 5.49.
O item (e) do exemplo 4-1garante que i.!x = Lk~ 0 (-l)kxk. com raio de convergência igual
a 1. Portanto, o item (b) da proposição 45. juntamente com o corolário 47, garante que

log(l - 1-dt = /.:e


+ x) = J..:r:
o 1+t k~O o
(-1tt L dt = L (-ll
k~O k + l
xk+l
l

também com raio de convergência igual a 1.

374
EXEl\IPLO 5.50.
A respeito da função a.rctg : lR ➔ •( - ~, Í), observe que, para lxl < 1,

1: x2 = 1- .r2 + :r 1 - .rü + ... .

Portanto, segue da proposição 45 que, para l,z:I< 1, temos

arctg .r. =
;r 1
dt =L J.:r
( - 1l t2k dt
2
/.O 1 + t k~OO
x3 xs x1
=x--+---+ .. ·.
3 5 7
~o problema 5.5. mostraremos que a igualdade acima continua verdadeira para x = 1.

EXEl\tPLO 5.51.
Segue facilmente do corolário 43 que a série Lk~o f!xk tem raio de convergência +oo. Se
J(x) = Lk>o f,xk para x E JR,o item (b) do teorema 46 fornece

! '(x ) = ~ 1
L- (k _ l)!x
k-1 ~ 1 k
= L- k!x =
j( )
x •
k>l k2:'.0

Portanto,
d
dx(e-xf(x)) = c-:r(-f(x) + J'(x)) = O,
de sorte que a função x H e .xf(x) é constante. Por fim, como f(O) = 1, segue que e-:rf(x) = 1
e, daí, f (.r) = ex.

Terminamos esta seção aplicando o teorema 46 para desenvolver a função f (J·) = (1 + x )º,
a i- O, em série de potências quando lxl < l. Para tanto, dados a- E lRe n > O inteiro. definimos
o número binomial generalizado (~) pondo (~) = 1 e, para n > 1,

(ª) =
n
a-(a-l)(a-2)
n!
... (a-n+l)_
(5.28)

O lema a seguir estabelece algumas propriedades úteis dos números binomais generalizados. A
propriedade do item (a) também é conhecida como a relação de Stifel.

LEMA 5.52.
Dados a E lR e n EN, temos:

375
(b) *(~) = (~=D,para todo o# O.
(e) 1(~) 1 ~ 1, para lol < l.

PROVA.
O item (a) é um cálculo fácil:

1
= 1n. o(o - l)(o - 2) ... (o - n + 1)
1
- 1 (o- l)(a-2) ... (a -n)
n.
= 1n.1 (o - l)(a - 2) ... (o - n + l)(a - (a - n))
1
(n _ l)! (a - l){a - 2) ... (o - 11 + 1)

= (ª -1).
n-l
O item (b) segue imediatamente de (5.28)
Por fim, para o item (c), sendo lal < 1, egue de (5 28) e da de igualdade triangular que

(º)
n -
< lol(lal + l)(lal + 2) ... (lal + n - 1) < 1 • 2 • ..
n! - n!
·71
= l.

O resultado a seguir é conhecido como o teorema da série binomial e é devido a Newton.


Observe que (5.29) generaliza a fórmula usual do binômio de Newton, uma vez que (~) = O se
o EN e k > o.

TEOREMA 5.53. NEWTON


Para a~ O e lxl < 1, temos
(5.29)

PROVA.
Suponhamos inicialmente que O < lal < l. Como

n+l n
----+ 1,
1o-n 1

376
o corolário 43 garante que a série ~k~o (~)xk tem raio de convergência igual a 1. Portanto,
teorema 46, a função f: (-1, 1) ➔ JR,.dada por f(x) = ~k~o (~)xk, é derivável, com

onde utilizamos o item (c) do lema anterior na última igualdade. Segue daí e do item (a) do
referido lema que

= a/(x).

Logo, se g(x) = (1 + x) J(x), então, para lxl < 1, temos


0

g'(x) = -a(l + xtº- 1


J(x) + (1 + x)- 0 J'(x)
1
= (1 + x)- 0 - (-a/(x) + (1 + x)J'(x)) = O,

de sorte que g é constante no intervalo (-1, 1). Mas, como g(O) = 1, segue que (1 +x)- 0 f(x) = 1
para lxl < 1, conforme desejado.
Para o caso geral, suponha que (l+x)º = ~k~o (~)xk para um certo a-:/ Oe todo x E (-1, 1).
Iostremos que valem fórmulas anâlogas para a - 1 e a + 1 (e para todo x E ( -1, 1)):

(a) Para a - 1: o teorema 46, juntamente com o item (b) do lema 52, garante que, para lxl < 1,
temos

377
(b) Para o+ l: o item (a) do lema 52 dá-nos

Por indução, concluímos que (5.29) vale para todos o-=/-O e lxl < 1.

COROLÁRIO 5.54.
Para o, /3 f:. O, temos

para todo x E R tal que lxl < à


11
.

PROVA.
Basta aplicar (5.29), com {3x no lugar de x. observando que IJ3xl
< 1 <=>lxl < l~I"

EXEMPLO 5.55.
Como aplicação do corolário anterior, observe que, para lxl < ½,temos

com

(~
2
) =~,(-D(-~_
1) ___(-~ -n+ 1)
( - 1t 1 3 5 2n - 1
- n! 2 2 2 2
(-1)" (2n)!
- n! 2" • 2 • 4 • • • (2n)
(-1)" (2n)! = (-1)" (2n)
n! 2" · 2"n! 4" n •

378
Portanto, para lxl < ½,temos

Problemas - Seção 5.5


5.1. Calcule os raios de convergência das séries de potências dos itens a seguir:

5.2. Utilize a abordagem do exemplo 51, juntamente com o resultado do lema 78, para mostrar
que
1
" (-1l- 21.·-l "(-1/ 2k
senx = L (2k _ l)!x e cosx = L (2k)! x •
k~O k~O

5.4. ~lostre que, para lxl < 1, temos

;(o4k(2k1 + l) (2k)
arcsen x -- " k x 2k+l •

5.5. ovamente a respeito da função arctg : IR ➔ (-~, ~), faça os seguintes itens:

(a) tvlostre que, para todo x E IR, temos

1 (-l)"x2n
-- = 1- x2 +x4 -x6 + ... + (-1r-1x2n-2 + ---.
2
1+ x 1 + x2

(b) Integre ambos os membros da igualdade do item (a), de Oa 1, para deduzir a fórmula
de Leibniz para 1r:
1r 1 1 1
-=1--+---+ ....
4 3 5 7
5.6. A fórmula do exemplo 49 pode ser modificada de modo a calcular log a, para todo a > O.
Para tanto, faça os seguintes itens:

379
(a) Mostre que, para x E (-1, 1), temos

1 (l+x)
-log -- ~ ~
=x+-+-+-+···. ~
2 1-x 3 5 7

(b) Mostre que x M ~!~ define uma bijeção de (-1, 1) em (O,+oo).


(e) Utilize a fórmula do item (a) para calcular log 3 com quatro casas decimais corretas.

380
5.6 Mais aplicações
Nesta seção, discutimos brevemente a utilização de séries de potências em Álgebra e COIUM-
natória, utilização essa que consiste na aplicação do poderoso método das funções gerrul.om,.

DEFINIÇÃO 5.56.
7
A função geradora (ordinâria) de uma sequência (an)n~o de números
potências

A definição anterior sugere que a grande diferença entre a teoria de séries de potências e
o método das funções geradoras é de enfoque. o primeiro caso, estamos primordialmente
interessados em examinar as propriedades da função f, definida pela série (5.31); no segundo
(conforme veremos logo mais), queremos utilizar as propriedades de f para inferir conclusões
sobre os termos da sequência (an)n~o- Comecemos apresentando duas aplicações interessantes à
Combinatória.

EXEMPLO 5.57.
Calcule o número de soluções inteiras e não negativas da equação

onde k e m são naturais dados.

SOLUÇAO.
otc que, para x real não nulo, temos a 1 + a 2 + · · · + ak = m se, e só se, xª 1 xº 2 ... xº" = xm;
portanto, há tantas soluções (a 1 , a 2 , ... , ak) da equação do enunciado quantas forem as maneiras
de obter uma parcela xm no produto

J(x) = (1 + x + x 2 + · · · )(1 + x + x 2 + · · ·) ... (1 + x + x 2 + • ••).


k

(De outro modo, tomando xª 1 no primeiro fator, xª 2 no segundo fator, ... , xº" no k-ésimo
fator, obtemos xm no produto.) Mas, para lxl < 1, segue do item (e) do exemplo 44 que

J(x) = (-1 )k = 1 = ~ (k+ n- l)xn,


1 -x (1 - x)k ~ n
n>O

7
Em contraposição às funções geradoras exponenciais, cf. capítulo 4 de l12j.

381
onde utilizamos, na última igualdade, o resultado do problema 5.3, página 379. Logo, a resposta
de no so problema é o coeficiente de xm na série acima, i.e., (k+;;:-1).

EXEMPLO 5.58.
O conjunto dos naturais é particionado em m progressões aritméticas infinitas e não cons-
tantes, de razões d 1 , d 2 , ... , dm. Prove que

PROVA.
Se f(x) = Ek~l xk e a 1 é o termo inicial da PA de razão d,, então a condição do cnuncindo,
juntamente com o item (e) do exemplo 44, garante que, para l:rl< 1, temos

Como 1-xd, = (1-x)(l+x+x 2


+- • •+xd•- 1 ), multiplicando ambos o membros da igualdade
anterior por 1 - x, obtemos
m
."tº'
X=~ 2 d l' (5 32)
L.,
t=l
1 +X+ X + ···+X ,-

para lxl < 1. Agora, note que ambos os membros dessa igualdade definem funções contínuas
em (O,1], as quais coincidem em [O,1); portanto, tais funções coincidem também para x = 1, de
sorte que, fazendo x = l em (5.32), obtemos E:: 1 = 1. J

O uso de funções geradoras é particularmente útil no estudo de sequências (a,.Jn>oque sa-


tisfaçam uma relação de recorrência conhecida. A ideia é considerar a função geradora cor-
respondente, Lk~o akxk, e cumprir as várias etapas descritas a seguir, as quais compõem o
procedimento padrão para muitos problemas similares:

1. Utilizar os valores iniciais da sequência, bem como a recorrência por ela satisfeita, para
concluir que a função geradora converge em algum intervalo da forma ( -r, r).

II. Novamente com o auxílio dos valores iniciais e da recorrência satisfeita pela sequência,
realizar operações apropriadas com a igualdade f (x) = Lk~o akxk para obter uma forma
fechada (i.c., uma fórmula) para f (x).

382
III Desenvolver a forma fechada obtida no item II em série de Taylor.

IV. Usar a unicidade da representação em série de potências, dada pelo corolé.rio 48,
concluir que an é igual ao coeficiente de x" na expansão obtida na etapa III.

Para o cumprimento da etapa I acima, o resultado a seguir é frequentemente útil.

LEMA 5.59.
Seja (an)n~o uma sequência de números reais. Se existirem reais positivos e e 1,1;
lanl < cAI" para todo n > O, então a série de potências Ek>O akxk converge, para
(- lf. lt) · -

PRO\'A.

Como lanx"I = lanllxln < c.t\lnlxl" = cl.Alxl" e a série geométrica Ek~O IMxlk converge
1
quando lxl < 111 (cf. item (e) do exemplo 44), o critério de comparação para séries garante que
a série de potencias Ek~O a,_xk converge quando lxl < ;, .

O exemplo a seguir utiliza o procedimento composto pelas etapas I a IV para obter uma
fórmula posicional para o n -ésimo número de Fibonacci.

EXEl\lPLO 5.60.
Seja (Fn)n~l a sequência de Fibonacci, i.e., a sequência dada por F 1
2 = Fk+l + Fk- para todo inteiro k > 1. Calcule Fn em função -~---~--..__,...__
F1.:+ de n.

~OLL'Ç\O
eja J(x) = Ek~i Fkxk a função geradora correspondente à sequência de Fibonacci, e cum-
pramo as etapas I a IV de critas anteriormente.

Etapa I. Observe inicialmente que, por uma fácil indução, temos Fn < 2n para todo n > 1.
Portanto, o lema 59 garante que, para todo x E (-½,½),a série Ek~l Fkxk converge.

383
Etapa II. Para x E ( -½, ½), podemos escrever

= x +x 2 L
• F1,;_1 + F1.:-"1)x"
k~3

= x + x2 + L x F1,;-1Xk-1 + ;r·.: L F1--2Xk-2


k~3 k~3

= x + x2 + .r.(J(x) - F1x) + x2 J(x)


= x + (x + .r.2 )f (x).
Então, para x E ( -½, ½), temos
f(x) = l - X 'l •

Etapa III. Escrevendo 1 - .r. - x2 = (l - ox)(l - fJx), m o, E ,t m o+ fJ = 1, n/J - -1


e
X
J(x) = --~ = x)"
l - x - x2 (l - QX )( 1-
Impondo, sem perda de gen rnlidnd , qu o > , u qu o=~ fJ= .!.::.i§
2 , d "' lll8 lll'I rn
que o - /3= J5 e, portnnto,

l ( 1 1 )
f(x) = "5 l - X - l - X •

Aplicando o dcscn\'olvimeuto em ~é.ric(5.27) (primeiro com a e, em uicla, oom f3no lugar


de a), obtemos

contanto que l,rl< min { ~ . J1} = ¼-l\Ia.s, como ½ < ¼,segu que

(5.33)

para todo .r.E (-½, ½)-

Etapa IV. Por fim. pelo corolário 4 . a . érie de potências que representa f é única, de forma
que, a partir da igualdade acima. obtemo.::,Fn = º";{', para todo n > 1.

3 -l
Continuando, utilizaremos uma abordagem similar à do exemplo anterior para
seguinte resultado geral.

TEOREMA 5.61.
Dados u. v E 1R,com v -=/=O, seja (an)n~I wna sequência tal que Ot+2 = U4
todo k ~ l Suponha, ademais, que a equação de segundo grau x 2 - ux - v = O
o. e {3.

(a) Se o. -=/=/3.então ª" = Ao."- 1 ,- B{3"- 1 para n > l, onde A e B são as soluções
_
deequaço~ { A+ B = a1 .
o.A+ {3B = a2

(b) Se a, = {3, então ª" = (A,- Bn)o."- 1


para n > l, onde A e B são as soluções do m·~ma
A+ B = a1
de equaçõe!>
{ (A+ 2B)a = a2

PROVA.
Comecemos mostrando que existe q > O tal que lan1 < q", para todo n > l. Realmente, se
lakl < qk e lak+il < qk+i, então, pela desigualdade triangular, temos

de fo1ma que lak+2l < qk+ 2 se lulqk•'-1+ lvlq" < qk+2 ou, equivalentemente, se lulq + lvl < q2.
~Ias, como a maior raiz de x 2 - lulx - lvl = O é Xo = ½(!ui+ Ju 2 + 4lvl), temos lulq + lvl < q2
se q > x 0. Portanto, teremos lanl < q" para todo n > l se lail< q, la21 < q2 e q > x 0, para o
que basta tomarmos q > max{lail, Jia;T,xo}.
Fixado um tal q, segue de lanl < q" para n > 1, via lema 59, que a série Lk>l akxk
converge no intervalo(-¼,¼)· Portanto, podemos definir uma função f: (-¼, ¼)➔ 1R-pondo
J(x) - Lk~i akxk, para lxl < ¼·
Para obter uma fórmula fechada para f(x), utilizemos a recorrência satisfeita pela sequência,
nos molde do que fizemos no exemplo da sequência de Fibonacci:

J(x) = L akxk = a1x + a2x2 + L akxk


k~l k~3

= a1x + a2x 2 + L(uak-1 + vak-2)xk


k~3

= a1x + a2x 2 + ux L ªk-1Xk-t + ux 2 L ªk-2Xk- 2


k~3 k>3

= aix + a2x 2 + ux(f(x) - a1x) + u.c2f(x).

3 5
Logo, (1 - vx 2 )J(:c) = a 1x + (a
11:r - ua )x para todo x E
2
-
1
2

, (-¼,¼).
Agora, como a e /3 são as raízes de x 2 - ux - v = O, segue do item (b) da proposição 16 que
a+ /3 = u e o/3 = -v. Portanto, 1 - ux - vx 2 = (1 - ox)(l - f3x) e, daí,
2
J(x) = a1x + (a2 uai)x
-
(1 - ox)(l - f3x)
• 1x 1 < mm
prua • {l'q, lol'
l 1}
IPI .
Consideremos os casos (a) e (b) separadamente:

(a) Se a -::f./3, podemos decompor a expressão que define f em frações parciais, obtendo f(x) =
-1 A:r + com A e B tais
Bz . que { A + B = ª1 . ('T"
.1al sistema
. tem uma so1uçao
- umca,
, •
_ 1 _ 13 ,
oz x {3A + aB = ua1 - a2
uma vez que a -::f./3.)
Expandindo 1_10 x e 1_1Pz em séries geométricas, conseguimos

J(x) = Ax L(axl + Bx L(f3xl

= L(Aak + B/3k)xk+1
k>O

= L(Aak-1 + B/3k-1)xk.
k~l

Comparando essa última e>-.1)ressãocom o fato de que f (x)


Aak-I + B/3k-i, para todo inteiro k > 1.

2
(b) Se o = /3, temos 2a = u, a 2 = -v e J(x) = ª 1r~f~;x)~i)x • De acordo com o teorema 64,
podemos decompor a expressão que define f em frações parciais da forma f(x) = 1 ~~ + ci!o-:)2,
A+ B = a1 .
com A e B tais que { . (Novamente, o sistema tem uma solução única, uma
aA = uai - a2
vez que v -::f.
O=} o -j. O.)
Expandindo 1_10 x e (l ~r):1 em séries de potências (para a segunda, com o auxílio do resultado
do problema 5.3, página 379), obtemos

J(i·) = A.r L(axt + Bx L(k + l)(axl

k>O

= L(A + Bk)ak-Ixk.
J,. 1

386
Por fim, comparando essa última expressão com o falo de que f(x) -
ak = (A+ Bk)ak-i, para todo inteiro k > l.

O teorema anterior pode ser visto como uma versão discreta de um resultado similar da teoria
de EDOs, o qual utiliza séries de potências para resolver EDOs da forma

y" = p(x)y' + q(x)y,


onde p, q : I ➔ 1Rsão funções que admitem expansões em séries de potências.
De uma forma um pouco mais precisa, suponha, por simplicidade, que O E I e que p(x) =
Lk~o ukxk e q(x) = Lk~o vkxk para x E (-R, R) e I. Dados a0 , a 1 E JR, é possível refinar a
demonstração do teorema anterior, com o auxílio da proposição a seguir, para mostrar que o
problema de valores iniciais
y" =
p(x)y'; q(x)y
{ y(O) = ao, y (O) = a1

admite uma única solução y = J(x), com f (x) = Ek~o akxk definida no intervalo (-R, R).
Para mais detalhes sobre a demonstração desse resultado, bem como para aplicações interes-
santes, referimos o leitor às seções 6.17 a 6.24 de [4J ou, ainda, ao Capítulo VI de (25J.
Nosso último exemplo fará uso do resultado a seguir.

PROPOSIÇÃO 5.62.
Se as funções /, g : (xo - R, Xo + R) ➔ IR têm expansões em séries de potências /(x) =
Ek~O a1,;(x-xol e g(x) = E,~0 b,(x-xo) , então a função J g: (xo-R, xo+R) ➔ R tem expansão
1

em série de potências (J g )( x) = Ln~o Cn(x - Xot, onde Cn = Lk+l=n akbl = E~ a1:b..-1:-

PROVA.
Pelo teorema 42, as séries de potências que definem f e g convergem absolutamente no
intervalo (x 0 - R, x 0 + R). Portanto, pelo teorema 18 e para x E (xo - R, x 0 + R), temos

J(x)g(x) = (L
k~O
ak(x - xot) (L
/~O
b,(x - xo)')

=L (L ak(x - xo)k • b,(x - xo)')


n~O k+l=n

= L (L
n~O k+l=n
akb,) (x - xot = L en(x -
n>O
i·oY',

387
s 'ndo que (também por aquele resultado) a convergência da última série é também absoluta no
intcrwlo (,r0 - R, x 0 + R).

EXEMPLO 5.63.
Seja (an)n~o a. sequência tal que a0 = 1, a 1 = -1 e

para k ~ 2. Calcule 0n em função de n.

SOLt:ÇÀO.
Sendo f(x) = Ek~O akxk a função geradora da sequência, cumpramos novamente as etapas I
a IV anteriormente descritas.

Etapa I. Tentaremos novamente aplicar o critério de comparação para séries de potências, na


forma do lema 59: impondo que lak-21 < ak- 2 , lak-il < ak-i, e aplicando a desigualdade
triangular à recorrência satisfeita pela sequência, obtemos

1
portanto, teremos lakl < ak se 0
:- + 2ak- 2 < ak, ou, equivalentemente, se

Uma vez que tal desigualdade é verdadeira para a = 2 e todo k > 2, e laol < 2°, la 1 1 < 21 ,
segue por indução que lanl < 2n, para todo n > 1. Portanto, o referido lema, juntamente com o
teorema 46, garante que a função f está definida e é derivável no intervalo ( -½, ½).

Etapa II. Escrevendo a recorrência do enunciado como kak = -ak-l + 2kak_ 2, para k > 2, temos
J'(x) = L kakxk t = a1 + L kakxk-t = a1 + L(-ak-l + 2kak_ 2 )xk-l
k~I k?2 k~2

.,....a1 - La,_ ixk-t + 2x L kaA.--2xk- 2


k~2 k~2

a1 - (J(.r) - ao)+ 2x (L(k - 2)ak-2Xk-


2
+2 L ak_ xk- 2
2
)
k~2 k~2

a1 l oo - J(.r) + 2.r(J'(x) + 2/(x)).

388
11as. como a1 +ao= O, obtemos f'(x) = (4x - 1)/(x) + 2x/'(x) ou, ainda,

(2x - 1)/'(x) = -(4x - 1)/(x).

Para integrar a equação diferencial acima, note inicialmente que / é positiva num intervalo
(-r, r), para algum O < r < ½ (uma vez que /(O) = a0 = 1 > O e / derivável ⇒ / contínua,
logo, positiva numa vizinhança de O). Portanto, para lxl < r, podemos escrever
1
f (X) = _ 4x - 1 = _2 _ 1
f ( x) 2x - 1 2x - 1
e, daí, para lxl < r < ½,

log / (x)
t-x
= log J(t) lt=O =
1:rJ'(t)
!( ) dt
o t
=- (x (2 + 1 ) dt
J0 2t - 1
1
= -2x - - log{l - 2x).
2
Assim, para lxl < r < ½,temos
(5.34)

Etapa III. Inicialmente, note que o desenvolvimento em série de potências de e- 2:r é dado por
{5.17), com a= -2, e vale em toda a reta:

Portanto, segue do exemplo 55 e da proposição 62 que, para f dada como em (5.34) e lxl < r < ½,
temos

389
Etnpn TV Comparando a expansão acima com J(x) = En~o anxn, segue que

an =
2
n n
L 41(n -
(-l)n-l (2l)
l) ! l •
l-0

Para mais aplicações de funções geradoras à Álgebra e à Combinatória, recomendamos ao


leitor as referências [12] e [14].

Problemas - Seção 5.6


6.1. Dados k, m E N, calcule, com o uso de funções geradoras, o número de soluções inteiras
da equação a1 + a2 + · · · + ak = m, tais que a, > 1, para 1 < i < k.

6.2. Calcule, com o auxílio de funções geradoras, o número de soluções inteiras e não negativas
da equação a1 + a2 + a 3 + a4 = 20, tais que a1 > 2 e a3 < 7.

6.3. Generalize o resultado da proposição 62, mostrando que, e a função f : (x 0 - R, x 0 + R) ➔


IRtem expansão em série de potências f(x) = En~o an(x - x 0 )n, então, para k EN, temos
J(x)k = En~o Cri(X - xo)n, onde

e a soma acima se estende a todas as k-uplas (i1, ... , ik) de inteiros não negativos tais
que i1 + · · · + ik = n.

6.4. Nas notações do exemplo 58, se a 1 , a2, ... , am são os termos iniciais das progressões, prove
que

6.5. Resolva a recorrência a1 2 e ak+l = ak + (k + 1), para k > 1, por meio de funções
geradoras.

6.6. A sequência (o,,),.~o é dada por ao = 1 e an 1-1 = 2an + n, para n > O. Para calcularª" em
função de n, faça os seguintes itens:

(a) s,,t1,, .,,, n 11 1 mo1:,Lrcque a,.+1 < on+i, contanto que an(o - 2) > n; conclua que
fJ 11 .,,, ;111 . parn todo n > O.

390
{b) l\Iostre que a função geradora de (a,,) 11~ 0 convcrgl' no int.erw1lo (-!i, ~),.é dadr.1,r,or
f( 'l·) -_
2
l-2:rf2.r
(l-.r) 2 {l-2.r)'

(c) Ache constantes reais A, B e C lnis que

1 - 2.r + 2.r2 A B C
------ - --- + -- + --.
(1 - .,;)2(1- 2x) (l - .r,) 1 :,: l - 2.r
2

(d) Expanda em série de potências cada uma das funções do segundo membro da. igual da.de
acima para concluir que an = 2n+1 - (n + 1), para n > O.

6. 7. Seja (an)n~o a sequência tal que ao= 1, a 1 = -3, a2 = 5 e ak+3 = ak+2+ ½ak+1-½ak, para
todo inteiro k > O. O propósito deste problema é mostrar que (an)n>O converge e calcular
seu limite. Para tanto, faça os seguintes itens:

(a) Mostre que lanl< 5n, para todo inteiro n > O. A partir daí, conclua que a série de
potências Lk~o akxk tem raio de convergência maior ou igual a ½..
{b) Se J : ( -½1 ½)➔ R é dada por J (x) = Lk>O akxk, use os valores iniciais e a recorrência
satisfeita pela sequência para obter f(x) - (~s~:)c!t~i),para lxl < 1.
{c) Aplique o método de decomposição em frações parciais para obter
A B C
J(x)=-+x-1 x- v'2+
2 x+ /2'
2

com A= -9 e B, CE lR. Em seguida, expanda 1 _:x,1_ 1~ e 1/ em séries geomé-


72
tricas para obter

B+(=__lr-1c , n
(d) Conclua que an = -9 - (J2)n+i para n > O e, dai, que a 11 ---+ -9.

391
ALGUNS PRÉ-
REQUISITOS
Este primeiro apêndice coleta, por completude e para a conveniência do leitor, fatos essenciais
n praticamente todos os desenvolvimentos do livro. As demonstrações que omitiremos podem
ser encontradas nas referências [9], [10] e [13].
Para além dos assuntos discutidos nas próximas seções, assumimos do leitor certa familia-
ridade com o uso de argumentos lógicos, com as definições e conceitos básicos sobre conjuntos
e com fatos elementares de geometria euclidiana. Para maiores detalhes, sugerimos ao leitor as
referências [10], [17] e [29].

A.1 Números reais


Nesta seção, recordamos alguns fatos básicos sobre números reais, utilizados com frequêncin
ao longo do texto. Assumimos conhecidos o conjunto N = {l, 2, 3, 4, ... } de numeros naturais,
71,= {O,±1, ±2, ±3, ... } de números inteiros e

Q = {~- o. b E Z. b :/;O}

de números racionais. Assumimos também conhecidas as operações aritméticas elcmenlmcs


entre elementos de tais conjuntos.
Particularmente sobre números inte1ros1 , para o, b E Z, com o :/;O, dizemos qu a divide b se
existe um inteiro c tal que b = ac; nesse caso, denotamo~ a I b. Equivalentemente, dizer que a
!
divide b é o mesmo que dizer que o racional é wn inteiro. Por exemplo, 13 divide 52, uma vez
que 5213 -- 4 •

Se a divide b, dizemos ainda que a é um dnnsor de b, ou que b é divisível por a; nesse caso,
denotamos a I b. Se a não divide b (ou, equivalentemente, se ~ (/:.Z), denotamos a f b Um
inteiro n é par se 2 1 n; caso contrário, o inteiro n é ímpar. Assim, O, ±2, ±4, ±6, ... são os
inteiros pares, ao passo que ±1, ±3, ±5, ... são os inteiros ímpares.
Dados naturais a e b, com a:/; O, é bem sabido que existem únicos inteiros q e r satisfazendo
as seguintes condições:
b = aq + r e O < r < a. (A.1)

A relação acima é conhecida como o algoritmo da divisão, e os inteiros q e r são denominados, res-
pectivamente, o quociente e o resto da divisão de b por a. As condições (A.l) são frequentemente
resumidas no diagrama

b~
r q
1
Rcnwternos o leitor no capftulo 1 de ll3j para uma discussão sistemática do que segue.

394
m particular para a b na ur • n no açõ irna a I b qui al rmo r = O q = !-
Doi m iro não nulo a b mpr ·m um maior ctivi or po i i o omum u máximo divisor
comum (d no ado mdc (a b))· a b ão primo ntre i quando mdc (a b) = 1. um r ion 1
não nulo r admi -o fracionária r = mn com m n int ir n ão implificando
o fator, comu m i.. c c lando o fa or mdc (m n) m m m n), obt mo uma
r pr n ação fr ionári irredutível para r. Por • mplo o r cional -;12 t m a r pr ntação
fr 10nãria irr d í el -/ ob ida can elando o f or 6 = mdc ( -12 1 ) do numerador do
d no minador.
m int ir p > 1 • primo úni itiv for m 1 p· d outro modo, um
in iro p > l ' primo par E

EE =} a = 1 ou p.
a

m nh id ( f. r m 1.37 [1 1)qu o onjun o d núm ro primo ' infim o


. nd u lm n m qu 1 O núm r

2 11, 1 1 1 2 1 7 1 3 7, 53

7. 71 7 . 7 . 97.

m inl 1r 111 u ll it compo to.


1.41 d n ur 1 n > 1 pod r
{ u fator, primo ) , qu
e l llH 11 úni d ai potAnci . Por ex mplo
d mp 1 in iro m f or primo . propri ad •
nh 1d1 m t o ma fundam ntal da a ·tmética.

.1.1 n m m
tum m n ar núm r racionai m notação decimal. Para o racional ! por
.• mpl , r v mo ! = O 125 orno uma abr vi ção para a igualdad

diz m qu O 1-5 r pr n ação d imal d l.

2par princip • propried d d pot'nci d um núm ro, v j d


o .1.2.

39
Racionais há, entretanto, com representações decimais mais complicadas. Tomemos como
~-cmplo o racional /2 , para o qual usualmente escrevemos
1
12 = 0,08333 ....

O que significa a igualdade acima? Imitando o caso do racional ½,somos tentados a dizer que a
igualdade acima é uma abreviação para

(A.2)

Esse é, de fato, o caso, contanto que interpretemos corretamente a soma com um número infinito
de parcelas do segundo membro. Rigorosamente falando, a igualdade (A.2) significa que, fixado
a priori um erro máximo 1~" = o,po..:.. 01,,,temos
n

1 (8 3 3 3) 1
O < 12 - 102 + 103 + 104 + ••• + lOk < 10n
para todo natural k > n; de outro modo, a igualdade (A.2) significa que todos os números
¾ + ~ + ~ + · · · + ~, com k > n, são aproximações por falta de 112 com erro menor ou
igual que O,po..;...
01,,.De fato, seguirá da proposição 27 que
n

e, portanto, o erro na aproximação por falta acima de 112 será 3 _;0 ,., o qual é menor ou igual
que o erro máximo 1~", sempre que k > n. É nesse sentido que devemos pensar na igualdade
1~ = 0,08333 ....

De posse da discussão acima, uma pergunta que se coloca naturalmente é a seguinte: dada
uma sequência 3 qualquer (a 1 , a2 , a 3, ... ) de algarismos, podemos pensar em

como a representação decimal de algum número racional, nos moldes da discussão acima?
É possível provar que a resposta a tal pergunta será sim se, e só se, a sequência (a 1 , a2, a3, .. .)
for, a partir de um certo ponto, periódica, i.e., da forma

(A.3)
p p 'P

'fConfomic ,wrá. visto no capítulo 1, uma sequência de números reais é uma função/: N ➔ R. Entretanto, para.
11<>8808propósitos atl! lá, podemos pensá-la como uma lista ordenada de números reais, i.e., uma lista de números
reais na qual c.>Bpt-cifica1110H
quem é o primeiro número da lista, quem é o segundo, o terceiro etc. Discutiremos
1tlg11111M H<>quôncias
HÍJ3temati<.:auwut,• clcmcnlares importantes na seção A.3.

396
(Em particular, para o racional / 2 a sequência em questão é (O,8, 3, 3, 3, ... ), a qual6, clM
periódica.)
Entretanto, há sequências de algarismos que não são periódicas a partir de ponto algum.Vii
exemplo é fornecido pela sequência

(O,1, O,1, 1, O,1, 1, 1, O,... ),

na qual há infinitos algarismos Oe a quantidade de algarismos 1 após cada algarismo O é sempre


igual à quantidade anterior de algarismos 1 mais um.
Informalmente, podemos resumir a discussão acima evidenciando a seguinte deficiência do
conjunto dos racionais: todo racional admite uma representação decimal, mas nem toda re-
presentação decimal representa algum racional. Nesse ponto, a fim de suprir tal deficiência,
postulamos 4 a existência de um conjunto, o qual denotaremos JR, contendo Q e possuindo as
seguintes propriedades:

(I) As operações de adição, subtração, multiplicação e divisão em Q estendem-se a operações


homônimas entre elementos de JR,gozando em 1Rdas mesmas propriedades que gozam em
Q.

(II) A ordenação dos elementos de Q estende-se aos elementos de JR,também gozando, em R,


das mesmas propriedades que goza em Q; em particular, todo elemento de 1Ré negativo,
igual a zero ou positivo.

(III) A toda sequência (a1, a2, a3 1 ••• de algarismos corresponde um único elemento x E JR,
)

no sentido da discussão do início desta subseção, o qual será denotado x = O,a 1a2 a3 .. ..
Reciprocamente, a todo x E 1Rcorrespondem um único inteiro m e uma única sequência
(a1, a2, a3, ... ) de algarismos, tais que x = m + O,a1a2a3 ... , onde+ representa a operação
de adição em JR.

Os elementos de 1Rsão denominados números reais, e o conjunto 1Rcomo um todo é o conjunto


dos números reais 5 .
4 Um axioma ou postulado em uma certa. teoria é uma propriedade imposta. como verdadeira. Uma das
ca.ra.cterfsticas fundamentais da. Matemática como ramo do conhecimento humano é a utilização do método
axiomático, i.e., a a.ceita.çãodo fato de que nem toda propriedade matemática. pode ser deduzida a partir de
propriedades matemáticas previamente estabelecidas, sendo necessária a adoção a prion de um coajunto de
axiomas. De outro modo, há um ponto em que precisamos admitir que certas propriedades (os axiomas) aejam
vá.lidas per si, i.e., sem justificativa embasada na validade de outras propriedades.
5 Há maneiras de certa forma mais construtivas de se introduzir o conjunto dos números reais (veja, por

exemplo, l26J, ou, para urna abordagem mais profunda, 1161}.Nestas notas, optamos por uma abordagem que
fosse a mais próxima. possível da experiência prévia do leitor.

397
l o que segue, detalhamos o que cada um dos itens (I), (II) e (III) realmente significa.
Começando por (I), postulamos que o conjunto R dos números reais é munido com duas
operações, denotadas + e • e respectivamente denominadas (por analogia com as operações
correspondentes em Q) adição e multiplicação, as quais satisfazem os axiomas ( 1) a (7) a seguir:
(1) Consistência: para a, b E Q, o resultado a+ b da adição de a e b é o mesmo, quer
consideremos a adição usual de Q ou a operação correspondente em Ilt Analogamente,
o resultado a • b da multiplicação de a e b é o mesmo, quer consideremos a multiplicação
usual de Q ou a operação correspondente em R.

(2) Comutatividade: as operações + e · são comutativas, i.e., são tais que a + b = b+ a e


a • b = b · a, para todos a, b E IR.

(3) Associatividade: as operações+ e• são associativas, i.e., são tais que a+(b+c) = (a+b)+c
e a· (b • e) = (a· b) · e, para todos a, b, c E R.

(4) Distributividade: a operação de multiplicação é distributiva em relação à de adição, 1.c. é1

tal que a· (b + e) = (a· b) + (a· e), para todos a, b,e E IR.

(5) Existência de elementos neutros únicos: os números racionais O e 1 são elementos neutros,
respectivamente, para as operações de adição e multiplicação em IR, i.e., tem-se O+ a= a
e 1 • a = a, para todo a E IR.

(6) Lei do cancelamento: se a, b E IRsão tais que a· b = O, então a= O ou b = O.

(7) Existência de inversos aditivo e multiplicativo: se a E R, então existe b E IR tal que


a + b = O. Se a E IR, a f:. O, então existe b E R tal que a • b = l.

Como decorrência das propriedades acima, temos as seguintes consequências importantes:


(i) Unicidade do inverso aditivo: para a E R, se b, b' E R são tais que a+ b = O e a+ b' = O,
então a associatividade e a comutatividade da adição, juntamente com o fato de O ser
elemento neutro para tal operação, dão-nos
b = b + O = b + (a + b')
= (b + a) + b' = (a + b) + b'
=O+ b' = b'.
Portanto, o real a possui um único inverso aditivo, o qual será, doravante, denotado por -a,
como feito usualmente para os racionais. Segue de a+ (-a) = O que a é o inverso aditivo
de -a; portanto, de acordo com a notação estabelecida acima para inversos aditivos, temos
-(-a)= a.

398
(ii) unicidade do inverso multiplicativo: para a E IR,a -::/O, se b, b' E IRsã.o tais q,w a· b = 1
e a· b' = 1, então b = b'. A prova deste fato é totalmente análoga à do item anterior
(cf. problema 3, página 400). Doravante, denotaremos o inverso multiplicativo de a E R,
a# O, por a- 1 , como é usual para os racionais.

(iii) Para a E lR, tem-se a· O = O: a fim de verificar essa igualdade, denote a · O = e. Pela
distributividade da multiplicação em relação à adição, temos

e = a • O = a • (O+ O) = a • O+ a • O = e + e.

Por outro lado, usando o fato de que O é elemento neutro da adição e que tal operação é
associativa, obtemos

e =e+ O= e+ (e+ (-e))


=(e+e)+(-e)
=e+ (-e)= O,
i.e., a· O = e = O.
De posse das propriedades acima para as operações de adição e multiplicação de números
reais, convencionamos (como é praxe em Q) omitir o sinal • da multiplicação, escrevendo sim-
plesmente ab para denotar a· b. Observe, agora, que a associatividade e a comutatividade da
adição e da multiplicação em lR nos permitem adicionar ou multiplicar urna quantidade finita
qualquer de números reais, sem nos preocuparmos com quais parcelas ou fatores devemos operar
inicialmente; o resultado final será sempre o mesmo 6 .
Também, definimos as operações de subtração (-) e divisão (-;-) em IR assim como em Q:
para a, b E lR,pomos

com b-::/O neste último caso; ainda nesse caso, sempre que não houver perigo de confusão com
a representação fracionária do racional i escrevemos a/b ou %como sinônimos de a-;- b.

Problemas
1. Estabeleça as seguintes propriedades de proporções: se a, b, e e d são inteiros não nulos,
tais que i = á,então
a e a± e
b-d-b±d"
6 Rigorosamente falando, a validade dessa afirmação deveria ser demonstrada como um tcorcmn, o que pode
ser feito com o auxilio do princípio de indução finita (cf. subseção A.3.3).

399
2. Dada uma. sequência (a 1 , a2 , a3, .. .) de algarismos, prove que o número real 0,a1a2a 3 ...
representa a expansão decimal de um racional se, e só se, a sequência (a 1 , a 2 , a 3 , .. . ) é
periódica, no sentido de (A.3).

3. Prove a unicidade do inverso multiplicativo em lR. Mais precisamente, prove que se a =J. O
é um número real e b, b' E IRsão tais que a· b =a• b' = 1, então b = b'.

400
A. 1. 2 A relação de ordem em IR
Voltemo-nos, agora, à discussão detalhada do item {II).
Em IR, postulamos também a existência de uma relação de ordem (i.e., uma ma
comparar elementos de IR), denotada, por analogia com a relação correspondente em Q, > (
maior ou igual que) e satisfazendo os axiomas {1') a {5') a seguir:

(1') Consistência: se a, b E Q e a> bem Q, então a> bem IR.

(2') Reflexividade: a > a, para todo a E IR.

{3') Antissimetria: se a, b E IR são tais que a > b e b > a, então a= b.

(4') Transitividade: se a, b, e E IRsão tais que a> b e b > e, então a> e.

(5') Dicotomia: para todos a, b E IR, tem-se a 2: b ou b > a.


o que segue, se a, b E IRsão tais que a > b e a=/=b, denotamos a > b (lê-se a maior que b).
Escrevemos ainda a < b (lê-se a menor ou igual que b) como sinônimo de b > a, e a < b (lê-se
a menor que b) como sinônimo de b > a. Para a E IR, se a > O dizemos que a é positivo; se
a < O, dizemos que a é negativo.
Impomos, ainda, à relação de ordem> em IR,os axiomas (6') e (7') a seguir, os quais garantem
- assim como em Q - sua compatibilidade com as operações de adição e multiplicação em R:
(6') a> b {::}a - b > O.

(7') a, b > O =>a+ b, ab > O.


A proposição a seguir coleciona mais algumas propriedades úteis da relação de ordem em
IR, as quais podem ser deduzidas a partir dos axiomas (1 ') a (7') acima. Para o enunciado da
mesma diremos, doravante, que dois números reais não nulos têm um mesmo sinal se forem
ambos positivos ou ambos negativos.

PROPOSIÇÃO A.1.
Sejam a, b, e, d E IR.

(a) Se a > O, então -a < O, e vice-versa.

b>O => ab > O


(b) Se a > O, então {
b<O => ab < O

_ { b > O => ab < O


(c) Se a < O, entao
b<O ⇒ ab > O

401
(d) a > b =>a + e > b + e.

(e) a > b, e > d =>a + e > b + d.

c>O => ac>bc


(f) Se a > b, então {
c<O => ac<bc
(g) a ~ O =>a 2 > O.

(h) a> O# ¼> O.

(i) Se a e b têm um mesmo sinal e a> b, então ¼ < ¼-


-----~~----

PROVA.
Provemos somente alguns dos itens acima; os demais ficam como exercícios (veja o problema
1, página 404).

(a) Segue do axioma (6') acima:

O> -a# O- (-a)> O# a> O.


(d) Novamente pelo axioma (6'), temos que

a > b =>a - b > O=> (a + e) - (b + e) > O => a + e > b + e.


(e) Usemos (d) e a transitividade de >:

a>b => a+c>b+c} b


=> a+c > +d.
c>d => b+c>b+d
(f) Suponha e> O (o caso e< O é análogo). Segue de (6') e (7') que
a > b => a - b > O=> (a - b)c > O=> ac - bc > O=> ac > bc.
(h) Suponha a > O. Se fosse ¼ < O, teríamos, pelo item (b), que 1 = a• ¼ < O, o que é um
absurdo (uma vez que a relação de ordem em IR estende a relação correspondente em Q).

(i) Como a e b têm um mesmo sinal, segue de (b) e (c) que ab > O. Assim, ;b > O pelo item (h)
e, de b - a< O, segue de (b) ou (c) que
1 1 b-a 1
- - - = - = (b- a)· - < O.
a b ab ab
Por sua ve-.t,(6') garante que a relação acima equivale a ¼< ¼-

402
Os itens (b) e (c) da proposição anLcrior são colcLivamculc conhecicloHeomo as regru de
sinal para a multiplicação de números reais.
Para o que segue, recorde que, dado r E IR, definimos o quadrado de r, de>notado r 2 (lê-se
também r ao quadrado), como o número real r 2 = r · r, e o cn bo de r, denotado r3 {lê-se também
r ao cubo), como o número real r 3 = r • r · r. Mais geralmenlc, para um dado n E N, definimos
a nª (lê-se n-ésima) potência der, denoLada r 11, como sendo r, Ca.'30 n = 1, ou o número real
obtido multiplicando-se r por ele mesmo n vezes, caso n > l:

Tn = '---v--'
T • T • • • • • T.
n

Aqui, cumpre chamar a atenção do leitor para o fato de que a associatividade da multiplicação
de números reais, juntamente com o princípio de indução finita (cf. subseção A.3.3), permite
provar que o resultado do segundo membro da igualdade acima independe da ordem em que
efetuemos as multiplicações, de sorte que o número rn é bem definido. Os números reais r _ r2.
r 3 , ... são conhecidos coletivamente como as potências de expoentes naturais do número real r.
O problema 4, página 404, lista algumas propriedades operatórias úteis de tais números reais.
Uma consequência importante das propriedades da relação de ordem em IR elencadas na
proposição anterior é aquela constante do corolário a seguir.

COROLÁRIO A. 2.
Sejam r um real positivo e m e n naturais, com m > n. Então:

PROVA.
(a) Como ré positivo, multiplicando por r ambos os membros da desigualdade r < 1, obtemos
r 2 < r. Multiplicando ambos os membros dessa última desigualdade novamente por r, segue que
r 3 < r 2 e, daí, r 3 < r 2 < r. Prosseguindo dessa maneira, chegamos ao resultado desejado, i.e.,

• • • < r 4 < r 3 < r 2 < r.

(b) A demonstração é essencialmente a mesma que aquela do item (a), com a. diferença de que,
inicialmente, temos r > l.

Terminamos esta subseção apresentando outra consequência bastante simplt's dn proposi,üo


anterior.

403
COROLÁRIO A.3.
Para a, b E JR,tem-se a 2 + b2 > O, ocorrendo a igualdade se, e só se, a = b = O.

PROVA.
Pelo item (g) da proposição 1, temos a2,b2 > O. Portanto, segue do item (d) daquela
proposição que a 2 + b2 > O. Suponha, agora, que a # O. Novamente pelo item (g) da proposição
referida, temos a 2 > O. Por outro lado, como b2 > O ainda é verdade, o item (e) da proposição
garante que a 2 + b2 > O.

Problemas
1. Prove os itens (b), (c) e (g) da proposição 1.

2. Sejam dados n E N e a, b reais positi,·o~ qunbqucr. Prove que:

(a) a< b se, e só se, a 2 < b2 .


(b) a < b se, e só se, an < b11

3. Generalize o corolário 3, mostrando que, se a, b, e E JR,então

ocorrendo a igualdade se, e só se, a = b = e = O.

4. Dados r, s E 1Rem, n EN, prove que7 :

(a) (rs)n = rnsn.


(b) rm+n = rmrn.

(c) (rm)n = rmn_


(d) (;) n = :: , se s =/:-O.
7
Rigorosamente, para provar as propriedade::. dos itens (a) a (d) temos que recorrer ao princípio de indução
finita (cf. subseção A.3.3) Todavia, nossa intenção aqui é simplesmente que o leitor, utilizando argumentos
heurísticos corno os apontados na sugestão ao problema, se convença da validade de tais propriedades.

404
5. Parar real não nulo e n natural, estendemos a noção de potências de expoentes naturais
definindo r-n = /n. Por exemplo. r- 1 = ¼,r- 2 = ~ etc. Definindo também r 0 = 1, prove
que, para todos m, n EN. tem-se r:r = rm-n_

6. Se x =/=O é um número real e n EN. proYe que x" ê positivo se n for par, ex" tem o mesmo
sinal de x se n for ímpar.

405
A.1.3 A completude do conjunto dos reais
Postulamos, por fim, que a toda sequência (a 1 . a 2 , a3 ... . ) de algarismos corresponda um único
elemento x E R, no seguinte sentido: fixado um erro má..ximo1~", n E N. temos
a1 a2 ak ) 1
O< X - ( 10 + 102 + ••• + l0k < l0n 1

para todo natural k > n. Em particular, segue da condição acima que


a1 a2 l ak
X < 10 + 102 + ••• + l0k + 1011 '
para todo k > n. Tomando k =n e lembrando que a; < 9 para todo J, obtemo::,

de sorte que O < x < l.


Resumimos o postulado acima dizendo que o conjunto R dos número reais é completo A
esse respeito, veja também a discussão da seção 2.1 e o problema 1.7, página 338.
Reciprocamente, postulamos também que a todo número real positivo x corresponda um
inteiro (não negativo) me uma lista (ai, a2, a3, ... ) de algarismos, tal que x = m + O,a1a 2 a 3 ...
no sentido acima. Sem> O em = bn ... b1bo,com os b, sendo seus algarismos, escrevemos

e dizemos que bn ... b1 b0 ,a1 a2a3 ... é a representação decimal de x.


Conforme vimos no problema 2 da página 400, um número real é racional exatamente quando
sua representação decimal for finita ou infinita e periódica. Por outro lado, números reais que
não são racionais são denominados irracionais. Desse modo. os números irracionais são aqueles
números reais que não podem ser escritos como quocientes de dois números inteiros ou, ainda,
aqueles cuja representação decimal é infinita e aperiódka.
Até o presente momento, o único exemplo de número irracional que conhecemos é o número
0.0101101110 .... De certa forma, tal exemplo é um tanto frustrante, uma vez que tal número
irracio11alé difícil de ser manipulado (i.e., é difícil fazer contas com ele). Remediamos um pouco
<>stasituação no que &cgue.

406
1'IH>BLE\I.\S

Do ponto de ,ista aritmético, uma grande vantagem do conjunto dos números reais, em
comparação com o conjunto dos racionais. é a possibilidade de extrairmos raízes de números
reais positivos. 1'Iais precisamente. dados x > O real e n E N, é possível provar (e o faremos
no teorema 9) que existe um único real positivo y tal que yn = x. Tal real positivo y será,
doravante, denotado y = .ifx e denominado a raiz n-ésima (lê-se enésima) de x ou, ainda, a
raiz de índice n de x. Assim.

Os casos n = 2 e n = 3 ocorrem com tanta frequência que merecem nomes especiais. Quando
n = 2 (e x > O), escrevemos simplesmente Jx, em vez de ?;'x, e dizemos que Jx é a raiz
quadrada de x; quando n = 3. dizemos que ifx é a raiz cúbica de x.
Intuitivamente, podemos entender por que existem raízes de números reais positivos exa-
2
minando um exemplo simples. Por definição, temos ./2 = 2. Assim, como 12 < 2 < 22 ,
segue do problema 2, página 404. que 1 < ./2 < 2; como 1,42 < 2 < 1,52 , segue novamente
do referido problema que 1.4 < ./2 < 1.5; analogamente. como 1,412 < 2 < 1,422 , temos que
1,41 < ./2 < 1,42 e, prosseguindo de<. a maneira. obtemos uma única lista (4, 1, 4, ... ) de algaris-
mos, tal que~= 1.414 .. (\'eJa também o problema 7. página 447). Denominamos radiciação
à operação de obtenção de raízes de um real po iti\'o.
Voltemo-nos, agora. a potêncins de números naturais. Um quadrado perfeito é um número
nntmal que pode r escrito nn forma m 2 . para algum m E N; assim, os quadrados perfeitos
sno Ol:>númcrol:>12 = 1. 22 = 4, 32 =- 9, •t'2 = 16 etc. Um cubo perfeito é um natural que
pode ser escrito na forma m 3 . para algum m E N; os cubos perfeitos são os números 13 = 1,
23 - , 33 = 27, 4 3 = 61 etc 1Iais geralmente, um natural n ê uma potência perfeita se
ex1stirem k > 1 int c1ro e m E N tais que n = mk. 1csse caso, dizemos que n é urna k-ésima
potência perfeita, i.e., um dos naturai& 11', 2" 1 3", 4k etc. Equivalentemente, dizer que n E N
é unrn k-ésima potência perfeita é o mesmo que dizer que sua raiz k-ésima, -1-n, é um número
naturnl
O rc ultado a seguir fornece inúmeros exemplos de números irracionais. Para uma prova do
mesmo, referimo o leitor ao exemplo 1.23 de [13J.

PROPOSIÇÃO A.4.
Dados números naturais n e k, com k > 1, ou n é uma k-ésima potência perfeita ou ~ é
um número irracional.

De acordo com a proposição acima, números como v'2,--YJ,q'1Õ etc. são todos irracionais
(posto que 2 não é um quadrado perfeito, 3 não é um cubo perfeito e 10 não é uma quinta
potência perfeita).

407
Parn terminar nossa discussão sobre números racionais e irracionais, note que o conjunto
dos números irracionais não é fechado em relação às operações aritméticas. De fato, dado r
irracional, temos que -r também é irracional, muito embora r + (-r) = O, um número racional.
Por outro lado, fazendo r = ~ temos r • r = r 2 = 2, também racional. Finalmente, se r -=/:O,
então o quociente de r por si mesmo é igual a 1, novamente um número racional.

Problemas
1. Sejam a e b números racionais e rum número irracional. Se a+br = O, prove que a= b = O.

2. Seja r um real positivo e k um inteiro maior que 1. Se r é irracional, prove que os números
~ e ifF também são irracionais.

3. Assumindo a validade do teorema fundamental da aritmética (cf. último parágrafo da


página 395), prove que ~ é irracional.

4. Seja p E N um número primo e k > 1 um natural. Prove que o número ifjJ é irracional.

408
A. 1.4 A representação geométrica
Uma maneira bastante útil de pensar geometricamente no conjunto dos números racionais é
a seguinte: escolhemos uma reta r e marcamos sobre ela um ponto O; em seguida, escolhemos
uma das semirretas que O determina sobre r, a qual chamamos positiva, sendo a outra a
semirreta negativa, e um segmento f. como padrão de comprimento. Agora, associamos cada
número racional a um ponto de r do seguinte modo: primeiro, associamos O ao ponto O; em
seguida (cf. Figura A.l), dado um racional i, com a, b E N, marcamos, a partir de O e sobre
a semirreta positiva, um segmento OA de comprimento ae (i.e., OA é obtido justapondo-se,
consecutivamente, a segmentos-padrão). Se b = l, associamos I = a ao ponto A. Se b > l,
particionamos O A em b segmentos iguais, marcando b - l pontos sobre O A; sendo B o ponto da
partição mais próximo de O, associamos i ao ponto B. Não é rufícil mostrar que a construção
descrita acima é consistente, no sentido de que, trocando i por outra fração equivalente, obtemos
um mesmo ponto B sobre r (a esse respeito, veja o problema 1, página 411). Uma construção

O B A r

Figura A. l: racionais sobre a reta.

análoga pode ser feita para os racionais negativos, marcados sobre a semirreta negativa.
Ocorre que, ao assim fazermos, sobram muitos pontos sobre r, os quais não estão associados
a nenhum número racional. Para exemplificar, considere o ponto A associado ao número 1 e
construa um quadrado OABC, como na Figura A.2. Em seguida, marque, com um compasso,

C.---_B

[2h
o AE r

Figura A.2: um ponto que não representa racional algum.

um ponto E sobre a semirreta positiva, tal que OE= OB. Como OA = 1, segue do teorcnm
de Pitãgoras 8 que OE= OB = ../2.Mas, como ../2é irracional (cf. proposição 4 ou problcmn
3, página 408), segue que E não está associado a nenhum número racional.
8 Pitágoras de Samos foi um dos maiores matemáticos da escola grega.. O teorema quo le,1\ seu nome, e

409
'
\1·1 \1111 1 \ :\ua·~s P1<i::-HEQl·1s1Tos

Duas perguntas naturais colocam-se neste ponto: é possível marcar sobre r todos os números
reais? Supondo que a resposta à pergunta anterior seja sim, após marcarmos todos os pontos
de IR sobre r ainda sobram em r pontos não marcados? Um dos a.xiomas da construção da
Geometria Euclidiana plana 9 , enunciado a seguir, garante que as respostas a tais perguntas são
respectivamente sim e não.

AXIOMA A.5.
Existe uma correspondência biunívoca entre os pontos de uma retare o conjunto dos numeros
reais, a qual fica totalmente determinada pelas seguintes escolhas:

(a) Um ponto O sobre r para representar o número real O.

(b) Uma semirreta, dentre as que O determina sobre r, onde são marcados os reais positivos.

(c) Um ponto A sobre a semirreta do item (b), ao qual corresponderá o número real 1.

Se fixarmos sobre uma reta r escolhas como as e pecificada pelo axioma acima, diremos que
ré a reta numerada (cf. Figura A.3).
Para uso posterior, estabelecemos a definição a egmr.

DEFINIÇÃO A.6.
Para reais dados a < b, definimos10 :
(i) [a,b]={xEIR;a<x<b}.

(ii) [a,b) = {x E IR; a< x < b}.


(iii) (a,b]={xEIR;a<x<b}.

(iv) (a,b)={xER;a<x<b}.

(v) [a,+oo)={xEIR;a<x}.

(vi) (a, +oo) = {x E R; a< x}.

(vii) (-oo,a]={xER;x<a}.

(viii) (-oo,a) = {x E R; x < a}.


------
que afirma que, em todo triângulo retângulo, o quadrado do comprimento do maior lado é igual à soma dos
quadrados dos comprimentos dos outros dois lados, já era conhecido dos babilônios pelo menos mil anos ante:;
de: fi<?U nru;cimcnto; no entanto, Pitágoras foi o primeiro a conseguir prová-lo. É também atribuída a Pitágoras a
priuwira prova du irrncionalidnde de ./2.
9
P:u-a uma c·onstruçiio nxiomálica da Geometria Euclidiana plana, referimos o leitor a 15]ou, ainda, a 124].

410
l 1IH >BLl-:\1.\S

Um intervalo em lRé o próprio conjunto lRou um conjunLo de um elos HCt.ctipos acima.


Observe que, na reta numerada, um intervalo corresponde a um segmento ou semirreta (contendo
ou não pelo menos uma das e>...1.remidades
correspondentes), ou mesmo à reta inteira.

ÜBSER\'AÇÕES A. 7.
1. É importante frisar que os símbolos +oo e -oo (lê-se, respectivamente, maia infinito e
menos infinito) não representam números reais. Tais símbolos servem, meramente, para
significar que um intervalo de um dos tipos (v), (vi), (vii) ou (viii) a.cimaengloba todos 08
números reais maiores ou iguais (resp. maiores) ou menores ou iguais (resp. menores) que
a.

ii. Consoante a definição anterior, denotaremos lR= (-oo, +oo).


~--~~--~-~---'

Dados números reais a < b, dizemos que a e b são as extremidades e que b - a é o


comprimento de cada um dos intervalos dos itens (i) a (iv) da definição acima. Nesse caso,
dizemos também que tais intervalos têm comprimento finito. Analogamente, o número real a é a
(única) extremidade de cada um dos intervalos dos itens (v) a (viü), os quais têm comprimento
infinito (i.e., não finito). Um intervalo em lR é finito ou limitado se tiver comprimento finito;
caso contrário, o intervalo é dito infinito ou ilimitado. Em particular, os intervalos limitados
de 1R.são precisamente aqueles dos itens (i) a (iv) da definição acima.
Também, classificamos um intervalo finito, de extremidades a< b, como fechado, fechado
à esquerda, fechado à direita ou aberto quando tal intervalo for respectivamente igual a
[a, b], [a, b), (a, b] ou (a, b) (note a correspondência das nomenclaturas utilizadas com o fato de
o intervalo conter ou não conter as extremidades a ou b). Alternativamente, dizemos que [a, b)
é aberto à direita e (a, b] é aberto à esquerda. Por fim, aplicamos extensões óbvias dessas
nomenclaturas aos intervalos infinitos. A Figura A.3 mostra um intervalo aberto à direita [a, b),
marcado em negrito sobre a reta numerada.

o a b lR
Figura A.3: o intervalo aberto à direita [a,b).

Problemas
1. Em relação à interpretação geométrica dos números racionais, discutida nesta sul>sc~no,
sejam dadas frações positivas i e ~. com a, b, e, d E N. Se i = ~. explique por que n
10Chamamos a atenção do leitor para as notações menos comuns [a, b[ em vc~ dr [a, b), Jo, bJ no lugt\r dl' ]o, b],
]a, b[ em vez de (a, b), [a, +oo[ em vez de [a, +oo) e] - 00 1 a[ 110 lugur de (- , o).

411
construção dada no texto associa a tais frações um mesmo ponto da reta numerada.

412
A.2 Álgebra elementar
Esta seção revisa diversas ferramentas de Álgebra elementar, utilizadas livremente ao lodp
do texto.

A.2.1 Identidades algébricas


Denominamos variáveis números reais fixados, mas arbitrários. Variáveis serão, em geral,
denotadas por letras latinas minúsculas, como por exemplo a, b, e, x, y, z etc. (uma exceção
importante é mencionada no próximo parágrafo).
Uma expressão algébrica, ou simplesmente uma expressão, é um número real formado
a partir de uma quantidade finita de variáveis reais, possivelmente com o auxilio de uma ou
mais dentre as operações de adição, subtração, multiplicação, divisão, potenciação e radiciação
(sempre que os resultados tiverem sentido em .IR),as quais denominamos operações algébricas.
Assim, por exemplo,
2
x+~-x z
----- yz + 3{!x 2yz 3 - x4

é uma expressão algébrica que tem sentido para todos os reais x, y, z tais que y > Oe z ::/: O. Em
particular, toda variável pode ser vista como uma expressão algébrica. Denotaremos expressões
algébricas em geral por meio de letras latinas maiúsculas, por exemplo E, F etc.
Dizemos que uma expressão algébrica E é um monômio se E for o produto de um número
real não nulo dado por potências de expoentes inteiros não negativos de suas variáveis. Por
exemplo, os monômios em x, y são as expressões da forma axkyl, onde a ::/: O é um real dado e
k, l > O são números inteiros (convencionamos aqui que xk = 1 caso k = O, adotando convenção
análoga quanto a yl - veja o problema 5, página 405). Para um monômio qualquer, o real não
nulo dado que faz o papel de a em axkyl é o coeficiente do monômio. Assim, os monômios em
x, y com coeficiente 2 são aqueles da forma

2, 2x 2y, 2x 2

, 2xy, 2y 2

, 2x 3

, 2x y, 2xy
2 2

, 2y
3

, etc

(lembre se de que, em axkyl, podemos ter k = O ou l = O).


Uma expressão polinomial ou polinômio é (uma expressão que é) uma soma finita de
monômios, como, por exemplo,
2 + 3xy - V5x2yz.
Os coeficientes de um polinômio são os coeficientes de seus monômios.
Sejam E e F expressões algébricas. Dizemos que a igualdade E = F é uma identidade
algébrica se for verdadeira para todos os valores reais possíveis das variAveis envolvidas. Para

413
exemplificar, consideremos a expressão algébrica E= (x + y) 2 . Pelas propriedades básicas das
operações de adição e multiplicação de números reais (i.e., comutatividade e associatividade da
adição e multiplicação, distributividade da multiplicação em relação à adição) temos
E =(x+y)(x+y)=x(x+y)+y(x+y)
= (x2 + xy) + (yx + y2 )
= x 2 + 2xy + y 2 ,
para todas as variáveis reais x e y. Portanto, pondo F = x2 + 2xy + y 2 obtemos a identidade
algébrica E= F, i.e.,
(x + y) 2 = x 2 + 2xy + y2, (A 4)
à qual nos referiremos doravante como a f 6rmula para o quadrado da soma de dois números
reais.
A proposição a seguir coleciona algumas identidades algébricas importantes, as quais dc,·em
ser guardadas para uso futuro.

PROPOSIÇÃO A.8.
Para todos os x, y, z E R., temos:

(a) x 2 - y2 = (x - y )( x + y).
2 2 2
(b) (x±y) =x ±2xy+y .

(e) x 3 ± y 3 = (x ± y)(x 2 =f xy + y2).


(d) (x ± y)3 = x 3 ± y 3 ± 3xy(x ± y).
(e) (x + y + z) 2 = x2 + y 2 + z2 + 2xy + 2xz + 2yz.

PROVA.
Deixamos as provas dos itens (a), (b) e (c) como exercícios, observando que a identidade do
item (b), com o sinal +, foi estabelecida em (A.4) (veja o problema 1, pâgina 416). Provemos
em (d) a identidade para (x + y)3, sendo aquela para (x - y) 3 totalmente anâloga: utilizando a
distributividade da multiplicação em relação à adição e a identidade (A.4), obtemos
(x+y) 3
= (x+y)(x+y) 2
=x(x+y)2 +y(x+y) 2

= x(x 2 + 2xy + y2) + y(x 2 + 2xy + y2 )


= (x3 + 2x2 y + xy2) + (x2 y + 2xy 2 + y 3 )
= x 3 + y 3 + 3x 2 y + 3xy 2
= x 3 + y 3 + 3xy (x + y).

414
Para (e), apliquemos (b) com x + y no lugar de .r e z no lugar de y:
(x + y + z)2 = [(x + y) + z]2
= (.r + y) + 2(x + y) z + z 2
2

= (x 2 + 2xy + y 2) -t 2(xz + yz) + z2


= x 2 + y 2 + z2 + 2xy + 2xz + 2yz.

ÜBSERVAÇÂO A. l.
Alternativamente, é costume denominarmos produto notâvel a uma identidadeE= Fl
que E é um produto de (pelo menos dois) polinômios e F a soma dos monômios resultantesda
expansão desse produto. Assim, frequentemente referir-nos-emos às identidades dos itens (b),
(d) e (e) da proposição acima como produtos notáveis. Por outro lado, uma identidade E= F
na qual E é um polinômio e F é um produto de (pelo menos dois) polinômios é usualmente
denominada uma fatoração. As identidades dos itens (a) e (c) da proposição acima são exemplos
de fatorações. Doravante, utilizaremos as nomenclaturas alternativas acima para identidades
envolvendo polinômios sem maiores comentários.

O exemplo a seguir utiliza (A.4) para deduzir uma desigualdade simples, mas importante.

EXE~1PLO A.9.
Identidades algébricas também podem ser utilizadas para estabelecer algumasdesigualdades
úteis. Por exemplo, dados x. y E IR sabemos que (lxl - IYl) 2
> O, com a igualdade ocorrendo
se, e só se, lxl = IYI-Se desenvolvermos a expressão entre parênteses, chegamosà d~
lxl2 + IYl2 > 2lxyjou, o que é o mesmo,
x2 + y2
2 > lxyl,
com a igualdade se, e só se, lxl = IYI.
Por outro lado, partindo de dois reais positivos quaisquer a e b e fazendo x =-..fã~ O?.
y = ./b > O, segue de (A.5) que
a+b r-;b
-- 2 >- vao 1

com a igualdade ocorrendo se, e só se, Jã = ./b,i.e., se, e só se, a= b.


A simplicidade da desigualdade acima esconde sua importância. Ela é um caao
uma desigualdade bem mais geral, que será discutida na seção 3.5 (cf. exemplo55)
denominada a desigualdade entre as médias aritmética e geométrica.

415
Um produto notável por vezes útil, mas não contemplado pela proposição 8, é o dado pela
igualdade
(x - y)(x - z) = x2 - (y + z)x + yz. (A.7)
Observe que no segundo membro aparecem a soma S = y + z e o produto P = yz de y e z. Uma
expressão do tipo x Sx + P, onde S e P representam respectivamente a soma e o produto
2

de dois números ou expressões, é denominada um trinômio de segundo grau em x. Assim,


podemos ver (A.7) também como um produto notável que fornece a fatoração de um trinômio
de segundo grau:
x S +p = (
2

- y) (
X Z), X - X - (A.8)
onde S = y + z e P = yz. A fatoração acima é por vezes denominada fórmula de Viête 11 .
Uma variante útil da fórmula de Viête, de verificação imediata, é a fatoração para a expressão
2
x + Sx + P, onde, como antes, S = y + z e P = yz:

2
x + Sx + P = (x + y )( x + z). (A D)

Trocando, em (A.8), S, y e z respectivamente por -S, -y e -z, vemos imediatamente que (A.9)
é realmente equivalente àquela fatoração, de maneira que é bastante fácil lembrarmo-nos de mais
essa identidade.

Problemas
1. Prove os demais itens da proposição 8.

2. Para números reais positivos x e y, prove que:

(a) x + ~ > 2, ocorrendo a igualdade se, e só se, x = 1.


4
(b) !z + !y-x> + li , ocorrendo a igualdade se, e só se, x = y.
3. Para x, y, z reais positivos, prove que x 2 + y 2 + z2 > xy + xz + yz, ocorrendo a igualdade
se, e somente se, x = y = z.

4. Para reais positivos e distintos x e y, prove que são válidas as seguintes racionalizações12:

11
François Viête, matemá.tico francês do século XVI. Por seu pioneirismo na utilização de letras para representar
variáveÍIJ, Viête l! por vezes conhecido como o pai da Álgebra moderna.
11
De uma rnan<>irainformal, podemos pensar numa operação de racionalzzaçàocomo o emprego de arguroeoto:>
de á.lgchra clemeutflr parn retirar ra(zcs do denominador de uma fração.

416
i>H<>HI.I.\I ,.._

1
(b) ~±W _ ~=F~+w
- x±y
() 1 _ ~±W
e ~=F~+W - :dy •

5. Dados um inteiro n > 1 e a, b E IR,prove que são válidas as seguintes fatorações:

(a) a" - b" = (a - b)(an-I + a"- 2 b + a"- 3 b2 + • •• + b"- 1).


(b) a"+ b" =(a+ b)(a"- 1 - a"- 2 b + a"- 3 b2 - • •• + b"- 1 ), se n é ímpar.

417
A.2.2 Módulo e equações modulares
Dadas expressões algébricas E e F, em uma mesma variável x, digamos, a equação a uma
variável E = F é o problema de encontrar todos os valores reais de x para os quais as expressões
E e F tenham sentido e a igualdade E = F seja verdadeira. Se E = F é uma equação na variável
x, dizemos que x é a incógnita da equação; os valores reais de x que resolvem a equação E= F.
i.e., que tornam E= Fuma igualdade verdadeira, são as raízes da equação; por fim, o conjunto
solução é aquele formado por todas as raízes da mesma.
No restante desta seção, discutiremos alguns tipos importantes de equações a uma variá-
vel. Para começar, dados números reais a e b, com a :/- O, podemos considerar a equação de
primeiro grau ax + b = O. Como a:/- O, temos
b
ax +b = O ç:::>ax = -b ç:::>x = --,
a
de maneira que - ~ é a única raiz da equação.
A fim de examinar nossa segunda classe de equações, necessitamos inicialmente da seguinte

DEFINIÇÃO A.10.
Para x E R, o módulo de x, denotado lxl, é definido por

lxl = { x, se x > O .
-X, se X< 0

Para exemplificar a definição acima, urna vez que -5 < O temos 1 - 51 = -(-5) = 5;
analogamente, 1- J31= -(-J3) = J3 etc. Mais geralmente, uma consequência imediata da
definição é que lxl > O para todo real x, ocorrendo a igualdade se e só se x = O. Ademais, tem-se
sempre
x < lxl = 1- xi,
com igualdade se, e só se, x > O. Note, ainda, que

lxl = ~ = max{x, -x}. (A.10)

Representando os números reais como pontos da reta numerada, é fácil ver que o módulo de
um número real x ê simplesmente a distância de x a O (cf. Figura A.4). Mais geralmente, dados
x, y E R, podemos olhar lx - YI como a distância entre os pontos x e y na reta. De fato, como
lx - YI = IY- xi, podemos supor que x < y. Então,

lx - YI = y - x = distância de x a y na reta.

418
lxl
X o IR
Figura A.4: módulo de um número real.

A equação modular mais simples é a equação

lx - ai= b,
com a e breais dados. Como lx - ai > O, tal equação não admite raízes quando b < O. Quando
b > O, segue da definição de módulo que deve ser x - a= b ou x - a= -b, donde temos as raízes
x = a+ b,a- b.
O exemplo a seguir mostra como resolver uma equação mais elaborada envolvendo módulos.

EXEMPLO A.11.
Resolva a equação lx +li+ lx - 21+ lx - 51 = 7.

SOLUÇÃO.
ole primeiro que
x+ 1, se x > -1
lx + 11= {
-X -1, se x < -1
x-2 , se x > 2
lx-21 ={ -x+2, se x < 2
e
x-5, se x > 5
lx - 51= { -x+5, se x < 5
Agora, uma vez que a interseção das condições x < -1 ou x > -1, x < 2 ou x > 2, x < 5 ou
x > 5 particiona a reta nos intervalos (-oo, -1), [-1, 2), [2, 5) e [5, +oo), faz-se mister considerar
separadamente x em cada um de tais intervalos, a fim de simplificar o primeiro membro da
equação. Procedendo dessa maneira, obtemos
-3x + 6, se x < -1
-x + 8, se - 1<x<2
lx + li + lx - 21 + lx - 51 =
x + 4, se 2<x <5
3x - 6, se x >5
Por fim, note que

419
•-3x + 6 = 7 {::::}
x =-½;como a condição-½ < -1 não ê satisfeita, não há soluções neste
caso.

•-x + 8 = 7 # x = l; como a condição -1 < 1 < 2 ê satisfeita, x = l ê solução da equação

•x + 4 = 7 # x = 3; como a condição 2 < 3 < 5 ê satisfeita, x = 3 é solução da equação.


1
•3x - 6 = 7 {::::}
x= ;; como a condição ~3 > 5 não ê satisfeita, não há soluções neste caso.

Logo, as raízes da equação são 1 e 3.

Problemas
1. Dados números reais a e b, com a =/:-O, mostre que

(-!! +oo) se a > O


{x E IR; ax + b > O}=
1 1
ª
{ (-oo , -k)
a ' se a< O

Faça o mesmo para ax + b > O, ax + b < O e ax + b < O.

2. Dados números reais a e b, mostre que

0, se b < O
{x E IR; lx - ai < b} = {a}, se b = O
(a - b, a+ b), se b > O

Faça o mesmo para lx - ai < b, lx - ai > b e lx - ai > b.


3. Prove que, para todos x, y E IR, tem-se lxyl = lxl · IYl-
4. Resolva, para x E R, as seguintes equações:

(a) x 2 + Slx - li + 11 = O.
(b) lx2 - 3xl =x - l.

(c) x + 1x!1 - 31= 6.


(d) xixi+ 4x + 3 = O.
(e) lxl = x - 6.
(f) lx + li + lx - 21+ lx - 51= 4.

420
A.2.3 A desigualdade triangular
Uma inequação é uma sentença de uma das formas E> F, E> F, E< F ou E S
E e F são expressões em uma ou mais variáYeis. Resolver uma inequação significa en
todos os valores da(s) Yariável(is) que a tornem uma desigualdade verdadeira.
Uma inequação não necessariamente torna-se uma desigualdade verdadeira para todos 08
valores reais possíveis das variáveis. Para exemplificar, considere a inequação

X + X 3 + 1 > 5x 4 .

Ao atribuirmos à variável x o valor real 2, a desigualdade resultante 11 > 80 é falsa. Por outro
lado, conforme o corolário 3, a inequação x 2 + y 2 > O se torna uma desigualdade verdadeira
quaisquer que sejam os valores reais atribuídos a x e y.
Doravante, sempre que não houver perigo de confusão, denominaremos desigualdades (al-
gébricas) às inequações que se tornam desigualdades verdadeiras para todos (ou quase todos 13 )
os valores possíveis das variáveis
A desigualdade da proposiçao a seguir é conhecida como a desigualdade triangular.

PROPOSIÇÃO A 12.
Para todos o::;reais não nulo:s a e b, temo:s

(A.11}

ocorrendo a igualdade se, e só se, a e b tiverem um mesmo sinal.


~-------------
PRO\\
Como la+ ble lal+ lblsão ambos não negativos, temos

la+ bl< lal+ lblç:> la+ bl2 < (lal+ lbl)2


ç> (a+ b) < lal + lbl + 2labl
2 2 2

ç> 2ab < 2labl,

o que é claramente verdadeiro. Segue também dos cálculos anteriores que la+ bl= lal+ lblse,
e 6 e, ab - jabl, o que por sua vez ocorre se, e só se, ab > O. Mas como a, b # O, teremos
igualdade e, e 6 e, ab > O.

13 0 s1gnificado da expressão quase todos nesse contexto ficarâ claro à me<Hdaque prosseguirmos nosso estudo.

421
COROIJÁRIO A.13.
Para todos a e b reais, temos
llal - lbll < la - bl,
ocorrendo a igualdade se, e só se, a e b têm um mesmo sinal.

PROVA.
Aplicando a desigualdade triangular com a - b no lugar de a, obtemos

lal = l(a - b) + bl ~ la - bl + lbl,

e daí lal - lbl < la - bl. Repetindo agora o argumento acima, trocando os papéis de a e b, segue
que lbl - lal < la - bl. Segue, então, que

la - bl > max{lal - lbl, lbl - lal} = llal - lbll,

onde, para a última igualdade, utilizamos (A.10).


Por fim, teremos a igualdade se, e só se, a ti\'ermos em pelo menos uma das desigualdades
triangulares lal < la - bl + lbl ou lbl < lb - ai+ lal, digamos na primeira delas. Mas para que
laJ = la-bl + lbl, a condição de igualdade da proposição 12 garante que deve ser (a -b)b > O, ou.
o que é o mesmo, ab > b2 . Em particular, deve ser ab > O. Reciprocamente, suponha que ab > O
e mostremos que a igualdade ocorre. Há duas possibilidades: a, b > O ou a, b < O. Suponha
que ocorre a primeira possibilidade (a primeira pode ser tratada analogamente). Então, lal = a,
lbl = b, e temos daí que llal - lbll = la - bj.

Dados números reais não nulos a, b e e, podemos aplicar a desigualdade triangular duas vezes
para obter
(A.12)
i.e., para obter a desigualdade para três números reais

(A.13)

análoga àquela obtida em (A.11), e que, portanto, chamaremos também de desigualdade trian-
gular.
Se a igualdade ocorre na desigualdade acima, devemos ter também a igualdade em todas
as desigualdades em (A.12), donde em particular em la+ bl < lal + lbl. Portanto, segue da
proposição 12 que a e b têm sinais iguais. Ocorre que podemos também escrever

122
de maneira que, se a igualdade ocorre em (A.13), então os números b e e também devem ter
sinais iguais. Reciprocamente, se a, b e e têm todos um mesmo sinal, digamos a, b, e < O (o C880
a, b, e> O é análogo), então a+ b +e< O, de maneira que

la+ b+ cl =-(a+ b+c) =(-a)+ (-b) +(-e)= lal + lbl + lei-

tvlostramos, pois, que há igualdade em (A.13) se, e só se, a, b e e têm todos um mesmo sinal.
Analogamente, uma fácil indução (cf. subseção A.3.3) permite estabelecer a seguinte genera-
lização da discussão acima, também conhecida na literatura como a desigualdade triangular.

TEOREMA A.14.
Para números reais não nulos a 1 , a 2 , ... , an, temos

{A.14)

Ademais, a igualdade ocorre se, e só se, a 1 , a 2 , ... , antiverem todos um mesmo sinal.

Problemas
1. Faça os seguintes itens:

(a) Se O< x < y, prove que 1:x < ~-


(b) Sejam a, breais quaisquer, prove que

lal lbl > la+ bl


1 + lal + 1 + lbl - 1 + la+ bl •

2. Prove que, para todo x E IR, tem-se

lx - li+ lx - 21+ lx - 31+ • • • + lx - 1001> 502 .

423
A.2.4 Equações polinomiais
Consideremos inicialmente a equação de segundo grau

ax
2
+ bx + e = O, (A.15)

onde a, b e e são reais dados, sendo a :/- O. Por motivos que ficarão claros mais adiante (veja
a discussão que culmina com a fatoração (A.18)), o primeiro membro de (A.15) também é
denominado o trinômio de segundo grau associado à equação (A.15), e a, b e e são seus
coeficientes.
Dada uma equação de segundo grau como acima, denotamos por 6 (leia-se delta) o número

6 = b2 - 4ac.

Tal número é o discriminante da equação (ou do trinômio associado) (A.15), e, conforme


veremos, será usado para discriminar (daí o nome!) quando a mesma possui raízes. Para tanto,
comecemos com o seguinte resultado auxiliar.

LEMA A.15.
Dados a, b, e E R, com a :/- O, tem-se
2
2
ax + bx + e = a [ (x + ~) - ~] • (A.16)
2a 4a 2

A identidade algébrica.a.cimaé a forma canônica do trinômio de segundo grau ax 2 + bx + e.

PROVA.
Basta ver que

ax
2
+ bx + e = a ( x 2 + ~x + ~)
=a b 2 b b
( x + -xa + -4a2 - -4a2 + -a
2 2
e)
2 2
=a [ (
X
2 b
+ -X+ -b 2 ) - -b 2 + -4acl
2
a 4a 4a 4a
2

=a[(x+~) 2a -~]-
4a 2

424
.2.
id i , mar u tr ir rt t rm uma x
quadrado, gunda i uald d na prov d
d\. m ip d truque alg brico
p b m orno n r l

.1 .
om ai, O.

(a) qu ç-o ax 2 + bx +e= O m raíz r a1 e só ~>O. caso,


d d ____,....a...= -b ,/3.
2a

( ) ~ O não a om o produto P d raízes da equação do item (a) -


_!!.
a
p = ~.
a

PR V.
(a) gu d ( .1 ) qu

a
2
+ bx + =Ü<=>( b )2
+-2a -- ~ ( .17)
- 4a 2 •

m (x 2~)2 > Op r d E qua r raíz r ai ntão d r 6 > O.


id n d ( .17) (v ja am 'mo pr bl m 1 p gina 427) qu X+.!_=2a ±-Aa'
d n m ( ).

-b - -/E -b + -/E b
----
2a 2a a

(
-b-
2a
-/E)
(-b+-/E)
2a
= (-b)
2

a2
.- ~ = -.
a

- - ...... -

ÜB ER AÇÕE A.17.
~~,..
1. Quando ~ > O, a fórmula -bi/A para raíz da equação d ; ., •
-
1 • •
'
' í.'

-
' ' /

conh ida como a fórmula d Bh kara 14 - ·,.


------- .

1
11. As fórmulas do item (b) da proposição acima são também conhecidas como fórmulas de
Viete.

iii. Nas notações do item (a) da proposição acima, se ô = O, diremos que a equação ax 2 +
bx +e= O tem duas raízes iguais.
~----

Terminemos nossa discussão sobre equações de segundo grau com a seguinte observação: se
a f:. O e ax 2 + bx +e= O tiver raízes reais a e /3 (não necessariamente a f:. /3),então teremos a
fatoração
ax 2 + bx +e= a(x - a)(x - /3). (A.18)

De fato, segue do item (b) da proposição 16 que, para todo x real,

a(x - a)(x - /3) = a[x2 - (a+ {3)x + n/3]


2
= a [x - ( -D+ ~] x
= ax 2 + bx + e.
É instrutivo comparar o resultado de (A.18) com (A.7). O segundo membro de (A.18) é deno-
minado a forma fatorada do trinômio ax 2 + bx + e.
Neste ponto é natural perguntarmo-nos sobre como resolver a equação resultante da genera-
lização natural das equações ax + b = O e ax 2 + bx + e = O, qual seja, a equação polinomial
de-grau n
(A.19)

onde n > l é inteiro e a 0 , a 1 , ... , an são reais dados, com an f. O. Observe que, quando n =
1 ou n = 2, voltamos respectivamente às equações de primeiro e segundo graus discutidas
anteriormente.
Aqui, contentar-nos-emos em discutir alguns casos particulares e tecer alguns comentários
que, acreditamos, serão úteis ao leitor. Para um estudo mais sistemático, recomendamos [14].
Primeiramente, para equações polinomiais de graus n = 3 ou 4, fórmulas há, construídas em
termos dos coeficientes ao, a1, ... , an da equação e que fornecem as raízes da mesma. Conforme
o Capítulo 4 de (30], tais fórmulas derivaram dos trabalhos dos matemáticos italianos Scipione
del Ferro, Girolamo Cardano, Niccolõ Tartaglia e Lodovico Ferrari. Elas são, porém, demasia-
damente complexas para serem úteis e, portanto, não as discutiremos aqui.
14
Uma homenagem ao matemático indiano do século XII Bhaskara II, também conhecido como Bhaskaracharya
(Báskara, o prof~or).

426
Para equações polinomiais (A.19) de grau n > 5. Abel e o matemático francês Évariste
Galois 15 . ambos do século XIX. prm-aram independentemente que não existe fórmula, construída
em termos dos coeficientes da equação. que forneça as soluções reais da mesma. Bem entendido,
não importa quão inteligente alguém seja; eles prm-ararn que é impossível descobrir uma tal
fórmula, simplesmente porque a mesma não e.'\iste!
Alguns tipos particulares de equações de grau -1ou 6 são suficientemente simples para me-
recerem certa atenção. especialmente porque substituições de \'a.riável apropriadas as reduzem
imediatamente a equações de segundo grau. Para equações biquadradas, por exemplo, i.e.,
equações do tipo
ax 4 + bx2 + e = O,
c:om a =/,O, a sub tituição de YariâYely = x 2 a transforma na equação de segundo grau ay 2 +
ÓIJ r e = O. Portanto. para cada raiz real não negati\'a y = a dessa última equação, resolvendo
2
c1 equação .r =a, obtemo~ o par de raíz•· rcai x = ::r:../Õpara a equação biquadrada original.
Hec1procame11le. .e: = f3 é uma rniz da equação b1quadrnda original, então é imediato que
y /Jl é urna rnfa real não ncgntivn d equaçilo d , •gundo grau ay2 + by +e= O.

Probl n1as
l . D11doum , c~ru
a # O, cnrout Ir todo XE tais que .r 2 = a 2, sem recorrer à fórmula

2. l'Jt\lll o. b • e reab dndo::-. 5e ac < O, mo t 1 e que a equação ax 2 + bx + e = O tem duas


rn1zt''> ll't\1:-i d1 t intn~.

3. Con:.,..1d'rc n qunçÃo polinomic l de terceiro grau x 3 + ax 2 + bx + e = O, onde a, b e e são


numero::,,r 'fü' dado::,,,com e =I O Se .r. = a é uma raiz real (não nula) da mesma, prove
(1\ll' e,1st cm numero reais {3 e I ta1 que vale a fatoração

x 3 + ax 2 + bx + e = (x - a) (x 2 + ux + v).

Condun. a partir daí. que a equação polinomial em questão tem no máximo três raízes
reni::. X ~> caso. ·endo o. f3 e 1 tais raízes, mostre também que:

(a) e a, f3 e I forem duas a duas distintas, então

x 3 + ax 2 + bx + e = (x - a) (x - {3)(x - , ) .
15 A d~peito de sua morte prematura. Galois é considerado um dos maiores matemáticos que já existiu. Seus
trabalho ~obre equações polinomiais de grau n ~ 5 e teoria dos grupos constituíram a base do que hoje é
conhecido como a Teona de Galois. sub-ramo da ÁJgebra com aplicações em vârias partes da matemática.

427
(b) Se o = /3-/-'Y, então x:1 + ax 2 + bx +e= (x - a)2(x - ,).
(e) Se o= /3= 'Y, então x 3 + ax 2 + bx +e= (x - a) 3 .

Esse rc~u}tado generaliza a forma fatorada (A.18) de uma equação de segundo grau e é um
caso particular do algoritmo da divisão para polinômios 16 .

4. Estabeleça as relações entre coeficientes e raízes para uma equação polinomial do


terceiro grau: se os reais a 0 , a 1 , a 2 e a 3 (a3 -/- O) são tais que a equação polinomial de
terceiro grau

tem raízes reais x 1 , x 2 e x 3 , então

f"Para 1JlJ1ÍOre1
dt~talheH acerca deRRCponLo, nt1Rim como pnrn a generalização do resultado do problema -1,
remeterOO#o leitor at> Capitulo 4 ,fo IJ.-11.

'128
A.3 Sequências e indução
Esta seção revisa alguns resultados sobre sequências elementares e apresenta o princf.,~,"""'
indução e a fórmula do desenvolvimento binomial de Newton. Como antes, seu conte6do '1
utilizado livremente, ao longo do texto.

A.3.1 Recorrências elementares


Dizemos que a sequência (ak)k~l está definida por uma fórmula posicional se os valores
ak E IR forem dados por uma fórmula em k.

EXE\tPLO A 18.
. ..
.•
·-
:
A sequencia (ak)k~l dos quadrados perfeitos é a sequência (12 ,2 2 ,3 2 , >.

-·-
.....
a1 = 12 , a2 '- 22 , a 3 = 32 e, mais geralmente, ak = k2 para k > 1 inteiro.

Para sequencias definidas por fórmulas posicionais, é frequentemente útil listar os termos da
mesma a partir de lero, i.e., denotar a sequência por (ak)k>O· Tal notação pode parecer estranha
a princípio. uma vez que o primeiro termo da sequência seria a 0 , o segundo seria a 1 etc. No
enlanto, vezes há em que, a fim de snnplificar a fórmula posicional que define os valores dos
termos da sequencia, é desejãvel fazer-se assim. Com tal notação alternativa, a sequência dos
quadrados perfeitos, por exemplo, ::,eriadada por ak = (k + 1)2 , para todo k > O.
Uma alternativa a fórmulas posicionais para os termos de uma sequência é uma definição
recursiva, ou por recorrência, dos mesmos. Tal procedimento consiste em especificar um ou
mais lermos iniciais da sequência, bem como uma receita para calcular certo termo em função
dos (i.e., recorrendo aos) lermos anteriores a ele.

EXE~IPLO A.19.
Considere a sequência (ak)1,:~
1 definida recursivamente por a 1 = 2, ai=

Fazendo k = 3 na relação acima, obtemos a3 = 2a2 - a 1 = 2 · 5 - 2 ~ =


obtemos a4 = 2a3 - a2 = 2 • 8 - 5 = 11, e assim por diante. A relação (A.
de recorrência, ou simplesmente a recorrência satisfeita pela sequên •

É importante notar que só fomos capazes de calcular o valor do termo a3 porque conhecfamoe
de antemão, além da recorrência (A.20), os valores dos dois primeiros termos, a 1 e a 2 ; conhecer
somente o valor de a 1 não bastaria, uma vez que (A.20) calcula cada termo em função dos c:lou

429 Â
termo· imediatamente anteriores. Por outro lado, se mudássemos os valores de a 1 e a2 (mas
mant iYésscmos a recorrência acima), em geral mudaríamos os valores dos termos subsequentes
da sequência (faça o 1 = 1 e a2 = 2 para a recorrência acima, por exemplo, e calcule o novo valor
do terceiro termo).
Observe também que há outras formas equivalentes de escrevermos a recorrência anterior.
De fato, chamando k - 2 de j em (A.20), obtemos k = j + 2, e daí

(uma vez que k > 3). Esse procedimento evidencia que o nome que damos ao índice de uma
sequência (i.e., j, k, etc.) não é relevante para sua definição; poderíamos mesmo escrevê-la como

Uma pergunta natural a esta altura é a seguinte: se uma certa sequência está definida
recursivamente, como podemos obter uma fórmula posicional para seus termos? Responderemos
essa pergunta em dois casos simples a partir de agora, remetendo o leitor interessado à seção 5.6,
ao Capítulo 3 de (12] ou, ainda, às seções 3.5 e 6.2 de 114],para a análise de casos mais gerais

DEFINIÇÃO A.20.
Uma sequência (ak)k~l de números reais é uma progressão aritmética (abreviamos PA)
se existir um número real r tal que a recorrência

(A.21)

seja satisfeita para todo inteiro k > 1.


----~-~--------------~~
Na definição acima, o número real r, diferença comum entre dois termos consecutivos quais-
quer da PA, é denominado a razão da mesma. Ademais, note que, para uma PA estar comple-
tamente determinada, é necessário, além de sua razão r, conhecermos seu termo inicial a 1 . Por
exemplo, a sequência (ak)k~1 dada por a1 = 2 e ak+l = ak + 3 para k > l é uma PA de termo
inicial 2 e razão 3, totalmente determinada pela recorrência que deve ser satisfeita e pelo valor
2 do termo inicial. No entanto, se soubéssemos apenas que ak+i = ak + 3 para todo k > 1, não
teríamos uma só PA, pois não saberíamos como começá-la.
Uma outra caracterização recursiva útil para PAs é aquela dada na proposição a seguir.

~POIIÇAO A.21.
Umatequência (al)l~t de números reais é uma PA se e só se

(A.22)
------~-~~-~--------
430
PRO\A.
Por definição, a sequência é uma PA se e só se a 2 - a 1 = a3 - a 2 = · · ·, i.e., se e só se,
para todo k > 1 inteiro, tivermos ak+2 - ak+l = ak+I - ak, que é uma maneira equivalente de
escrevermos (A.22).

O próximo resultado ensina mais algumas propriedades interessantes e úteis de uma PA; em
particular, ele ensina como obter uma fórmula posicional para os termos de uma PA. O leitor
deve se esforçar para guardar as fórmulas nele constantes.

PROPOSIÇÃO A.22.
Se (ak)k>I é uma PA de razão r, então:

(a) ak = a 1 + (k - l)r, para todo k > 1.

PROVA.

(a) Para chegar a ak a partir de a 1 , são necessários k - 1 passos, onde cada passo resume-se a
somar r a um termo, a fim de obter o próximo _termo. Logo, para obter ak temos de somar, ao
lodo, (k - l)r a a 1 , de maneira que ak = a 1 + (k - l)r.

(b) É imediato que a1 +an = (a2 -r) + (an-1 +r) = a2 +an-1, a2 +an-1 = (a3 -r) + (an-2 +r) =
a3 + an-2 etc. Logo, sendo S = a1 + a2 + • • • + ª", temos

2S = 2(a1 + a2 + a3 + • • • + ªn-2 + ªn-1 + an)


= (a1 + an) + (a2 + ªn-d + (a3 + ªn-2) + • • • + (an + ai)
= (a1 + an) + (a1 + an) + (a1 + an) + • • • + (a1 + an)
n parcelas

e a fórmula segue.

As fórmulas dos itens (a) e (b) da proposição anterior são conhecidas respectivamente como
a fórmula para o termo geral e a fórmula para a soma dos k primeiros termos de uma PA.
Vejamos um exemplo de aplicação das mesmas.

431
EXEMPLO A.23.
Calcule a soma dos k primeiros inteiros positivos ímpares.

SOLUÇÃO.
Os inteiros positivos ímpares formam a PA 1, 3, 5, 7, ... , de razão 2. O k-ésimo termo da
mesma (o k-ésimo inteiro positivo ímpar!) é, pela fórmula para o termo geral de PAs, igual a
1 + (k - 1) · 2 = 2k - 1. Logo, a soma dos k primeiros inteiros positivos ímpares é, pelo item (b)
da proposição anterior, igual a k[l+(;k-I)! = k 2 .

Outra classe bastante útil de sequências é a formada pelas progressões geométricas, de aco1do
com a definição a seguir.

DEFINIÇÃO A.24.
Uma sequência (ak)k~l de números reais é uma progressão geométrica (abreviamos PG)
se existir um número real q tal que a recorrência

(A.23)

seja satisfeita para todo inteiro k > 1.

Assim como com PAs, o real q que aparece na definição de uma PG é a razão da mesma.
Observe que, se q = O,então ak = Opara todo k > 1. Por outro lado, se q = 1, então ak = a 1 para
todo k > 1. Também como com PAs, uma PG (ak)k~l só estará completamente determinada se
dela conhecermos o primeiro termo a 1 e a razão q.

EXEMPLO A.25.
Fixado um real não nulo q, a sequência (ak)k~i, dada para k > 1 por ak = qk (i.e., a
sequência formada pelas potências de q com expoentes naturais) é uma PG de razão q. Se q < O,
o problema 6, pégina 405, garante que ak = qk é positivo se e só se k for par; se O < q < 1, o
corolário 2 garante que a1 > a2 > a3 > · · · > O; se q > 1, novamente aquele resultado garante
que O< a1 < ~ < a3 < • • •.
-------------

Uma outra caracterização recursiva útil para (quase todas as) PGs é a dada na proposição a
seguir, cuja prova deixamos ao leitor.

432
PROPOSIÇÃO A.26.
Uma sequência (ak)k~l de números reais não nulos é uma PG se, e só se,

Mantendo nosso paralelo com o desenvolvimento da teoria das PAs, o próximo resultado traz
as fórmulas para o termo geral e para a soma dos k primeiros termos de uma PG. Também
corno com PAs, tais fórmulas devem ser guardadas para uso futuro.

PROPOSIÇÃO A.27.
Se (ak)k>l é uma PG de razão q, então:

(a) ak = a1 • qk-i, para todo k > 1.

(b) Se q =/=1, então a 1 + a 2 + · · · + an = a..;:~º 1


, para todo n > 1.

PHO\A.
(a) Para chegar a ak a partir de a 1 • são necessârios k - 1 passos, onde cada passo resume-se a
mulLiplicar um termo por q, a fim de obter o próximo termo. Logo, ternos de multiplicar a 1 por
q um lolal de k - 1 vezes, e dai ak = a 1 • qk-i.

(b) Denote por Sn a soma desejada, i.e., Sn = a1 + a2 + · · · + an; segue, pois, de (A.23) que

qSn = q(a1 + a2 + ''' + ªn-l + an)


= Qa1 + Qa2 + . ' ' + Qªn-l + QOn
= a2 + a3 + ' •• + an + ªn+ 1,

Portanto,

(q - l)Sn = qSn - Sn
= (a2 + a3 + • •• + an + ªn+1) - (a1 + ª2 + • •• + an)
= (a2 + a3 + · · · + an) + On+I - a1 - (a2 + · · · + an)

onde, na última passagem, cancelamos as duas ocorrências da parcela a2 + a3 + · · · + a 11 • Basta


agora dividir ambos os membros da igualdade (q - l)S = an+l - a1 por q - 1.

433
Problemas
1. :'+
cja (an)n>l uma sequência de reais posiLivossatisfazendo a recorrência ak+l = 30 1 , para
k > 1. Se (bn)n~l) é a sequência definida para n > 1 por bn = 01n, obtenha uma recorrência
satisfeita pela mesma.

2. Escreva uma recorrência para cada uma das sequências a segufr:

(a) (1, 1, 1, 3, 5, 9, 17, 31, 57,105, 183, ... ).


222
(b) (1,2,2 2 ,2 22,2 , ... ).

3. Seja (ak)k~1 uma PG de razão q. Prove que, para n > 1 inteiro, temos

4. A sequência (ak)k~1 é uma progressão aritmético-geométrica se, para cada inteiro k > 1,
tivermos ak = bkck, onde as sequências (bk)k~l e (ck)k~l são respectivamente uma PA e
uma PG. Se q =/=1, calcule, em função de n, b1 , c1 e das razões q e r, respectivamente da
PA e da PG, o valor da soma dos n primeiros termos de uma tal sequência (ak)k>l·
)'H<)BU-\1 \" \

A.3.2 Somatórios e produtórios


Nesta subseção. re,-isamos as notações L {lê- e sigma) para somas e TI(lê-se pi) para
produtos, as quais e reYelam mwto úteis no conte.'\.""to
de sequências.

DEFINIÇÃO A.28.
Dada urna sequência (ak)k~I- escre,·emos L]=ia; para denotar a soma a 1 + "2 +··•+~e
lemo~ o somatório dos a,, para 1 < j < n. Assim.

,sen=l
se n > 1

Corno c,iso particular d dcfiniçiio ~cima , (a )k>I for umn "equência constante, digamos
<<JIII(11 = e pai" todo k > 1 h•n•mo clnrn.m m

\ J e= nc.
JD 1 •

111,1 dn:s 111ilidach d 11ot1ç.ão}: d , o r to ddn tornar fi\cil n manipulação de somas


t 0111 g.11111d(• 11ü111t'H> ( l' J>•1( 1 ~ nmd m i qu ndo d p.u-ccla for, ela mesma, uma soma.
l'rn ,~:x1~111plo dnd, .... 4111 fü in (a )t~m (b,) ~l ei ,t ividndc e ,\ comutatividade da adição
d1• l\'flis g1111llltt li\ <)Ih

c 1\111 l> \IM) da 1101 n,~ o _ ,.,:-.ntgunldnde pod ,cr ,.,cri(,1 d forma bem mais compacta, como

(A.25)

l\lr out 1o Indo, dndo e E R. n di nibuth·idade da multiplicação em relação à adição fornece

i~unldndl• qlll' , ~cn.!,,. com a notação L· como

<I::= L
"

J=l
ª1
J=l
n

caj. (A.26)

Em ontrfk> pahwra~. e po 1vel partir um somatón.o de somas em do1.s outros somatórios, bem
como pôr uma con,tante em evidência em um somatório.

435
\ 11 1, 1, 1 \ . \ 1 <, 1 \...., 1 • l l l - 1{ I· < l l f-.;1 1 < >s

EXEMPLO A.29.
Calcule o valor da soma E;=1(2k + 1) em função de n EN.--~--------~---.

SOLUÇÃO.
Aplicando as propriedades acima, obtemos
n n n n
L(2k+l) = L2k+ I:1=2I:k+n
k=l k=l k=l k=l
n(n
= 2 • ---
+ 1) + n 2
= n + 2n,
2
onde a penúltima igualdade segue da fórmula do item (b) da proposição 22.

A notação E é particularmente útil para fazermos cancelamentos em somas. Mais precisa-


mente, dada uma sequência (ak)k~1, efetuando os cancelamentos intermediários na soma

obtemos an - a1 como resultado. Com o uso da notação E, podemos escrever a igualdade acima
como
n-1

L(a3+1 - aJ) = an - a1. (A.27)


j=l

Uma fórmula equivalente (obtida da fórmula acima escrevendo n + 1 no lugar de n), que serã
por vezes utilizada no lugar de (A.27), é
n
L(ªJ+l - a}) = ªn+i - ª1• (A.28)
J=l

Uma qualquer das fórmulas (A.27) ou (A.28) é conhecida como a fórmula para uma soma
telescópica. A ideia por trás do nome é a seguinte: assim como olhando num telescópio
encurtamos a imensa distância de um corpo celeste a nossos olhos, a fórmula acima encurta o
caminho entre uma soma inicial de muitas parcelas e o cálculo do resultado da mesma.
A fórmula da soma telescópica é uma das principais vantagens da notação E. Vejamos dois
exemplos.

para o termo geral de uma PA utilizando a fórmula da soma telescópica.

436
SOLUÇÃO.
Se a sequência (ak)k~l é uma PA de razão r, então (A.28) fornece
n-1 n-l

an - a1 = I:)a 1+1 - a1 ) = L r = (n - l)r,


j=l j=l

e segue que an = a 1 + (n - l)r.

EXEMPLO A.31.
Dados inteiros positivos k e n, prove que:
(1 +l)n+l n+l
(a) Jn < n+l-J < (j + l)n, para todo j EN.

PROVA.
1
Para o item (a), fatoramos (j + 1r+ - j"+l com o auxílio do resultado do problema 5, página
417, obtendo
n+l
(j + 1r+I - jn+l = L(j + 1r+l-if-l
i=l
n+l
< I:u + ir+1-,(j + 1)'-1
i=l

= (n + l)(j + lt.
Analogamente, (j + 1r+ 1 - jn+l > (n + l)jn.
Quanto ao item (b), segue de (a) e da fórmula para somas telescópicas que

+ l)n+l
k-1
LJn < I:
k-1 ( •
J
n+l
-1
·n+l
----<--.
kn+l -
n+l
1 k"+l
n+l
3=1 3=1

~k ·n kn+J
Ana 1ogamente, ~;=l J > n+I •

Também podemos introduzir uma notação bastante útil para representar produtos, conforme
ensina a definição a seguir.

'137
DEFINlÇÂO A.32.
Dada uma sequência (akh~1, escrevemos TI;=l aj para denotar O produto a1a2 · · · an, e lemos
o produtório dos ai, para 1 < j < n. Assim,

se n = 1
rrªi =
i=l
n { a1

a1a2 •.. an
,

se n > 1

Assim como com somatórios, a utilidade da notação TIreside no fato dela comutar Jormal-
mente com os símbolos de multiplicação e divisão. De fato, dados um número real e e sequências
(ak)k~l e (bk)k~l, temos

a1a2 • • • an a1 a2 an
----=--···-
b1b-i• • • bn b1 b2 bn
e

(a segunda igualdade acima desde que os b1 sejam todos não nulos). Escrevendo ambos os
membros dessas identidades usando produtórios, obtemos as igualdades

n n
e cn IIªi = IT(cai).
j=l J=l

Vejamos um exemplo de aplicação de tais fórmulas.


Analogamente ao caso de somatórios, a notação TIé particularmente útil para a realização de
cancelamentos em produtos, de acordo com a fórmula para produtos telescópicos, colecionada
na seguinte

PROPOSIÇÃOA.33.
Se (at)t~l é uma sequência de reais não nulos, então

(A.29)

438
PI« >BLE\I.\S ,

PRO\A.
Como com somas telecópicas, basta observarmos que os fatores intermediários do produto
do primeiro membro acima se cancelam. Em súnbolos,

EXEMPLO A.34.
Para simplificar n;=l (2+ f), observe inicialmente que

n ( + k1)= (")
TI 2 1 g gn( 2 • 1)
1+ k TI
= 2" n(k+l)
-k- •

Agora, definindo a sequência (ak)k~l por ak = k, segue de (A.29) que

Problemas
1. Seja (ak)k~ 1 a sequência definida por a1 = 1 e an+l = an + 3n-1 para todo inteiro positivo
n. Calcule, em função de n, o n-ésimo termo dessa sequência.

2. A sequência (an)n~l é dada por a1 = 1 e an+l = an + 8n para n > 1. Calcule an em função


de n.

3. Calcule, em função de n EN, o valor da soma E;=2 (k~l}k'


4. Prove que, para todo inteiro n > 1, tem-se
1 1 1 1 1
-12 + -22 + -32 + ... + -n2 < 2- -.
n

5. Calcule, em função de n E N, o valor da soma I:;:~(4k-i{( 4k+J).


6. Dado n > 1 natural, calcule em função de no valor de n:=2 (1 - b),
7. Considere a sequência (ak)k~l dada por a1 = 1 e ak+1 = 1:fal para todo inteiro positivo
k. Calcule an em função de n.

439
A.3.3 Indução finita
Várias são as maneiras de demonstrarmos uma proposição. Podemos fazer uma prova direta
ou uma prova por contradição, por exemplo. O princípio de indução será, para nós, mais uma
ferramenta para demonstrações, ademais muito útil.
Para entender como ele funciona, considere um conjunto A C N tal que 1 E A. Suponha
ainda que saibamos que toda vez que um certo natural k estiver em A, então k + 1 também está
em A. Então, 1 E A assegura que 2 E A. Por sua vez, 2 E A nos permite concluir que 3 E A.
Assim por diante, concluímos que A contém todos os naturais, ou seja, A = N. A discussão
intuitiva acima pode ser formalizada no seguinte axioma de indução, também conhecido como
o primeiro princípio de indução.

AxlOMA A.35.
Seja A e N um conjunto satisfazendo as seguintes condições:

(a) 1 E A.

(b) Se k E A, então k + 1 E A.

Então, A= N.

Uma pergunta natural nesse momento seria: como aplicar o princípio de indução para de-
monstrar algo em Matemática? Para responder esta pergunta, suponhamos dada uma propne-
dade P(n) do natural n, a qual queremos provar ser verdadeira para todo n E N. Definimos um
conjunto A pondo
A= {k EN; P(k) é verdadeira}
e observamos que
A= N ~ (P(n) é verdadeira para todo n EN).

Assim, a fim de mostrarmos que P(n) é verdadeira para todo n E N, basta mostrarmos que
A= .N, ou ainda, pelo primeiro princípio de indução, que

• 1 E A;

• kEA ~ k + 1 E A.

Por sua vez, a definição de A garante que mostrar os dois itens acima é o mesmo que mostrar
que

• P(l) é verdadeira;

440
• P(k) verdadeira ⇒ P(k + 1) verdadeira.

A discussão acima pode ser resumida na seguinte receita para demonstração por

PROPOSIÇÃO A.36.
Dada uma propriedade P(n) do natural n, temos P(n) verdadeira
se, as duas condições a seguir forem satisfeitas:

(a) P(l) é verdadeira;

(b) P(k) verdadeira ⇒ P(k + 1) verdadeira.


-----------------'~

Para entender na prática como funciona uma demonstração por indução, vejamos os dois
exemplos a seguir.

EXEMPLO A.37.
Para cada n EN, a soma dos n primeiros naturais ímpares é igual a n 2 .
--------~~

PRO\",\.
Como o k-ésimo natural ímpar é o número 2k - 1, a propriedade P(n) é, nesse caso,
n
P(n) : L(2J - 1) = n 2 .
1-l

Para fazer uma demonstração por indução, temos de verificar que:


i.P(l) é verdadeira.

ii.P(k) verdadeira ⇒ P(k + 1) verdadeira.

A verificação dei. é imediata: o primeiro natural ímpar é 1, o mesmo que 12 . Para provarmos
ii., supomos que P(k) é verdadeira, i.e., que

1 + 3 + · · · + (2k - 1) = k2

e queremos deduzir que P(k + 1) também é verdadeira, i.e., que

1 + 3 + · · · + (2k - 1) + (2(k + 1) - 1) = (k + 1)2.

Mas, uma vez que estamos supondo a validez de P(k), segue que

1 + 3 + · · · + (2k - 1) + (2k + 1) = k 2 + (2k + 1) = (k + 1)2.

Portanto, por indução, P(n) é verdadeira para todo n EN.

441
EXEMPLO A.38.
Para cada n EN, a soma dos n primeiros quadrados perfeitos é igual a
1
n(n + 1)(2n + 1).
6

PROVA.
Como o k-ésimo quadrado perfeito é o número k2 , a propriedade P(n) é, nesse caso,
n 1
P(n) : Lj2 = n(n + 1)(2n + 1).
;=l
6
Como antes, para fazer uma demonstração por indução temos de verificar que:

i.P(l) é verdadeira.

ü.P(k) verdadeira =>P(k + 1) verdadeira.


Verificar i. é novamente imediato: 12 = I(I+1)g 2 •1+1) Para verificar ii supomos que P(A) é
verdadeira, i.e., supomos que
k

Li= ~k(k + 1)(2k + 1),


J=l

e queremos deduzir que P(k + 1) também é verdadeira, i.e., que

Como estamos supondo a validez de P(k), podemos verificar a igualdade acima do seguinte
modo:
k+l k l
2
El = El + (k + 1) = 6k(k + 1)(2k + 1) + (k + 1)2
J=l J=l
1 1
= 6(k + l)[k(2k + 1) + 6(k + 1)] = 6(k + l)(k + 2)(2k + 3).
Portanto, por indução P(n) é verdadeira para todo n EN.

Uma forma ligeiramente mais geral do primeiro princípio de indução pode ser enunciada
como abaixo.

442
l 'H< >BLl-:\1 \ '-

AXIOMA A.39.
Sejam a EN e AC {a, a+ 1. a+ 2, ... } um conjunto satisfazendo as •

(a) a E A.

(b) Se k E A, então k + 1 E A.

Então, A= {a.a+ 1, a+ 2, ... }.

Essa variante do princípio de indução dá mais versatilidade a sua aplicação como método de
demonstração. Mais uma vez, suponhamos dada uma propriedade P(n) do natural n, a qual
queremos demonstrar ser verdadeira para todo natural a partir de um certo a (ou seja, para todo
natural n > a). Para isso definimos o conjunto

A= {k EN: P(k) é verdadeira}

e observamos que
A= {a.a+l,a+2, ... }
t
P(n) é verdadeira para todo n > a natural.
Obtemos, assim, a seguinte variante mais geral da receita de demonstração por indução.

PROPOSIÇÃO A.40.
Dados a E N e uma propriedade P(n) do natural n, temos P(n) verdadeira para todo natural
11 ~ a, se, e só se, as duas condições a seguir forem satisfeitas:

(a) P(a) é verdadeira;

(b) P( k) verdadeira :::}P( k + 1) verdadeira.

E a forma mais geral de demonstração por indução é, por vezes, realmente necessária, e o
próximo exemplo ilustra esse ponto. Para o enunciado do mesmo, para n EN denotamos por n!
(lê-se n fatonaQ o produto de todos os inteiros de 1 até n; assim, 1! = 1, 2! = 2, 3! = 6, 4! = 24
etc.

EXEMPLO A.41.
Para todo natural n > 4, temos n! > 2n.

443
PHO\'\.
Observe primeiro que temos realmente de começar com pelo menos n = 4, pois a desigualdade
não é válida para n = 1, 2, 3. A propriedade P(n) que desejamos provar é:

Para uma. demonstração da mesma por indução, temos de provar que P( 4) é verdadeira e
que P(k) verdadeira=> P(k + 1) verdadeira. A validade de P(4) segue de 4! = 24 > 16 ==24.
Suponhamos agora que P( k) é verdadeira para um certo k E N, ou seja, que

Queremos deduzir a veracidade de P(k + 1), i.e., que (k + 1)! > 2k+ 1. Para isso veja que, pela
veracidade de P(k), temos

(k + 1)! = (k + 1) · k! > (k + 1) • 2\

por outro lado, segue de k > 4 que

(na verdade, essa última desigualdade vale para todo inteiro k > 1). Portanto, combinando as
duas últimas desigualdades acima, obtemos que (k + 1)! > 2k+1, i.e., que P(k + 1) é verdadeira.
Logo, por indução P(n) é verdadeira para todo inteiro n > 4.

Antes de apresentar outro exemplo, façamos uma pequena observação quanto à terminologia.
numa demonstração por indução, o passo P(k) => P(k + 1) é, em geral, denominado passo de
indução. Para executá-lo, supomos que P(k) é verdadeiro (o que constitui nossa hipótese
de indução) e, então, deduzimos que P(k + 1) também é verdadeiro. Assim, uma prova por
indução nos moldes da proposição 40 pode ser resumida do seguinte modo:

• identificação da propriedade P(n) a ser provada;

• caso inicial: verificação da validade de P(a);

• hipótese de indução: suposição da validade de P(k).

• passo de indução: dedução da validade de P(k + 1) usando a hipótese de indução.

444
Uma \·ez que a propriedade P(n) e tá, em geral, bastante clara no enunciado de
blema, urna pro\·a por indução utiliza. via de regra, o últimos três itens do esquema IMIIIAI
se fazendo menção e>..-plicitaa P(k) ou ao passo P(k) :::} P(k + 1).
Há, ainda. uma outra forma importante de indução, o segundo principio de ind
(também chamado princípio de indução forte). que pas amos a descrever agora.

AXI0\1A A.42.
"e a A e N wn conjunto satisfazendo as seguintes condições:

(a) 1 E A.

(b) Se {1, ... , k} e A então k + 1 E A.


Então. A= N.

ponto, o uso do segundo princípio de indução em demonstrações deve estar claro para
N<>~..,l<'
o lc•itor \ tJamos mais dob exemplos, à guisa de ilustração.

EXEt-.tPLO A.43.
~[o!Stn'que, para todo n E N, o número (i -t -1\/'3)"+ (7 - 4J3)n é um inteiro pau

PHO\''\.
Se u = 7 -t •ht3 e 11 = 7 - -h/J. então u + u = 14 e uu = 1. Segue, pois, que u e v são as
nuzcs da t>quaç,wde segundo g1ãu J· 2 - 14:.r+ 1 = O Segue daí que u2 = 14u - 1 e v 2 = 14v - 1,
clP modo que, para todo k > 2 inteiro.

cndo ., = u' + 1.,J e '>ornando as duas relaçõe::.acima, obtemos, para todo inteiro k > 2, que

Agora, .s0 = 2 e s 1 = u + v = 1--lsão inteiros. Suponha, por hipótese de indução, que s1ce Z
para todo 1 < k < n. Então, a recorrência acima fornece

Sn = 14Sn-l - Sn-2,

donde concluímo que n E Z, por ser a soma de dois números inteiros.


Para o que falta. note que u, v > O garante que sn = u 11 + vn é positivo para todo n. Por
fim, a recorrenc1a para a sequência (s,Jk~1 também garante que sk e Sk-2 têm mesma paridotk

445
(i.c., ou são ambos pares ou ambos ímpares). Mas, uma vez que s 0 e s 1 são ambos pares, segue
novamente por indução que Sn é par para todo n natural.

EXEMPLO A.44.
Todo número natural pode ser escrito de uma única maneira como soma de potências de 2
com expoentes inteiros não negativos e dois a dois distintos, dita sua representação binária.

PROVA.
PrQvemos por indução forte que, para cada n natural, existe uma única representação binária
de n. Para n = 1, temos 1 = 2°, e obviamente essa é a única representação possível. Suponha
agora que o resultado desejado seja verdadeiro para todo natural menor que n.
Mostremos inicialmente que existe uma representação binária de n. Para tanto, tome a maio1
potência de 2 menor ou igual a n, digamos, 2k. Então,

- n < 2k+l,
2k <

de maneira que O< n - 2k < 2k. Se n - 21.= O, nada mais há a fazer. Senão, 1 < n - 2k < n, e
por hipótese de indução existem inteiros não negativos O < ao< a 1 < · · · < a1 tais que

Mas, como também temos n - 2k < 2k (veja acima), segue que 2°0 + 2°1 + ••• + 2ª 1 < 2k, e daí
a1 < k. Portanto,
n = 200 + 2°1 + · · · + 2°1 + 2\
com O < ao< a1 < · · · < a, < k.
Mostremos, agora, que a representação binária é única. Para tanto, suponhamos que

com O < ao< a 1 < · · · < a1 e O < bo< b1 < · · · < b,. Então,

2°1 < 2ª 0 + 2°1 + ... + 2°1


= n = 2bo + 2b1 + ... + 2b'
< 2º + 21 + ... + 2b1
= 2b1+1 - 1
'

446
de modo que 2°, < 2b,+l e, portanto, a1 < b1 + 1, i.e., a1 < b1. Trocando os papéis de a1 e b, na
discussão acima, concluímos analogamente que b1 < a1 , e daí aj = b1. Denotando a 1 = bt = k,
digamos, segue que

Utilizando agora a parte de unicidade da hipótese de indução, segue de n-2k < n que j-1 = l-1
e ao = bo, a1 = b1, ... , a1 -1 = b,_1 , como desejado.

Problemas
1. Prove por indução que a soma dos n primeiros naturais é igual a n(n~t1>.
2
2. Prove que, para n EN, temos 13 + 23 + ... + n 3 = ( n(~+ 1>) .
3. Mostre que, para cada inteiro n > 1, temos
1
1 • 2 + 2 • 3 + · · · + (n - l)n = -(n
3
- l)n(n + 1).

4. Mostre que, para cada inteiro n > 1, temos


1
12 + 32 + 52 + •• • + (2n - 1)2 = 6(2n - 1)2n(2n + 1).

5. A sequência (an)n;:: 1 de reais é definida por a1 = 2 e, para n > 1 inteiro, an+l = a~ - ª" + 1.
Prove que, para todo inteiro n > 1, temos:

(a) an+l = 1 + a1· · · an.


1
(b) "" ak
=2-
.l,.;k-1 ...!.. 0102 .. a,, .

6. Seja x um real não nulo tal que x + x- 1 E Z. Prove que xn + x-n E Z, para todo inteiro n.

7. Fixado um número real a > 1, seja (xn)n>l uma sequência tal que va< X1 < va+ 1 e
Xk+I = ½( xk + xªA:),para iodo k > 1. Pro~e que, para todo n ~ 1, temos

1
Ja < Xn< Ja + 2n-l •

447
A.3.4 A fórmula do binômio
Comecemos relembrando a definição de fatorial. estendida ao mte1ros não negat1\·os

Dado um inteiro não negativo n, o fatorial de n é o número

1_ { 1, se n = O
n. - n;=l
j, se n > 1 •

Em princípio poderia parecer mais razoável definirmo O!= O, mm ns rnzõc:-.pnrn n com· •nçiio
O!= 1 logo ficarão evidentes.

DEFINIÇÃO A.46.
Dados inteiros n e k, com O< k < n, definimo~ o número bino111ial (~) por

(11) n!
k = k!(n - k)!.

É de fâcil verificação que, parn todo n E . t m-s (~) = l, (';) = 11e (~) -
n(n 1)
2 Por
outro lado, para lodos o inteiros n e k tnb (111 O < Á: < n, tem-

Observe que (~), (';) e G) (e· e último em \ irtude do fato ele que o produto de dois intei-
ros consecutivos é par) ão todo numeros naturais Por outro lado, a igualdade de números
binomiais acima garante que (~) = (~).C _ ) = {i) e (n: ) = (;) também são todos naturais.
1 1 2
Cumpre, pois, perguntarmo-no se (~) é natural para todas as escolhas de inteiros n e k, tais
que O< k < n. Tal é de fato o caso. e erá deduzido mab adiante como consequência da relação
(A.31) a seguir. conhecida como a relação de tifel1-

PROPOSIÇÃO A.47.
Se n e k são inteiro::;tais que O < k < n, então

(n)= (n - 1)
+ (n - 1)
. (A.31)
k k k-1

17Após Michael Stifel. matemâtico alemão do :-.éculoXVI.

4-!
PRO\\

Basta aplicar a definição de número binomial ao segundo membro da igualdade acima:

(n -k
1)+ (n - 1)
k-1
(n - 1)! (n - l)!
k!(n - 1 - k)! + (k - l)l(n - k)!
(n-1)! (1 1 )
= (k - l)!(n - 1 - k)! k + n - k
(n - l)! n
- (k - l)!(n - 1 - k)! k(n - k)

- k!(nn~ k)! - (;) •

Com os números binomiais acima definidos construímos uma tabela numérica triangular, o
triângulo de Pascal 18 , do seguinte modo: Contamos a..c.linhas e colunas a partir de O, sendo
as linhas numeradas de cima para baixo e as colunas da esquerda para a direita; a entrada (i.e.,
o número) da nª linha e kª coluna é o número binomial (:). Mais especificamente:

• As entradas da coluna O, lidas de cima para baixo, são respectivamente iguais aos números
binomiais (~), (~), (~), (~), .... Como já vimos, todos esses números são iguais a l.

• A linha zero é formada somente pelo número binomial (~) = 1. A linha 1 é formada pelos
binomiais (~) e G), ambos também iguais a l.

• Em geral, as entradas da linha n, lidas da esquerda para a direita, são respectivamente


iguais aos binomiais (~), (7),G),... , (:) •

Mostramos, abaixo, as linhas iniciais do triângulo de Pascal, consoante a construção ncium


descrita:

18
Após Blaise Pascal, matemático francê~ do século XVII.

449
\1 1 \1111 1 \ .\1,C,1 '\'-, l'l!l·.-H1.q1 l'-ill<>S

Triângulo de Pascal
(~)

(~) G)
(~) e) (;)
(~) (~) (~) (~)
(~) (1) (~) (;) (!)
(~) G) (~) G) (!) (!)
(~) (~) (~) (~) (!) (:) (~)
Em relação ao triângulo de Pascal, a relação de Stifel diz que, ao somarmos, na linha n - 1,
as entradas da coluna k - l e da coluna k, obtemos a entrada da linha n e coluna k. Isto é
mais difícil de dizer do que entender e verificar, e permite obtermos recursivamente os valores
numéricos dos números binomiais (:) . A tabela a seguir mostra os valores numéricos do números
binomiais G) para O < n < 6, obtidos com o auxílio da relação de Stifel.

Valores numéricos das entradas do triângulo de Pascal

1
1 1
1 2 1
1 3 3 1
1 4 6 4 1
1 5 10 10 5 1
1 6 15 20 15 6 1

Mais geralmente, desde que para todo inteiro n > O temos (~) = 1 e (~) = 1, não é difícil
o leitor 6C convencer de que, para todos os inteiros n e k tais que O < k < n, temos G) E N.
Damos a seguir uma prova formal desse fn.to no corolário a seguir.

~ A,48,
P•• todas OI inteir01 n e k ta.is que O< k ~ n, temos (~) EN.
) 'H< lBLI \I \"

PROVA

Façamos indução sobre n > O, sendo o caso n = O óbvio: o único número binomial nessas
condições é (~) = 1.
Suponha. por hipótese de indução, que (n;-1)é natural para todo O < j < n - 1, e consi-
deremos um número binomial da forma G). Há dois casos a considerar: se k = n, então já
observamos que (~) = L um natural; se k < n - 1, então, por hipótese de indução, temos que
(";1) e G=!)são ambos naturais, e a relação de Stifel garante que (;) = (n; 1) + (:=D
EN.

A seguir, obtemos a fórmula do binômio de Newton, ou seja, da expansão da expressão


(x + y )" em monômios.

TEOREMA A.49.
Para n E N, temos
{A.32)

PRO\\.
Façamo· uma demonstração por indução sobre o e>..-poenten do binômio. Para n = 1, temos
(.r + y)l = .t + y =(~)xi+ G)y•.
Suponha, por hipótese de indução, que {A.32) é verdadeira quando n = k, i.e., suponhamos
que (.r.+ Yl = E~=o (:)xk-JyJ. Então, para n = k + 1, temos

Façamos, na última expressão acima, as seguintes trocas nos índices dos somatórios: no
primeiro omatório troque j por l e no segundo somatório troque j + 1 por l; desse modo, no

451
egundo omatório temos .7 = l - l e O < j < k - l 1
<=:>- < l < k. Assim procedendo, obtemos

onde na última igualdade utilizamos a relação de Stifel. Por fim, desde que (kt1) = (!!!)= 1,
podemos escrever a última linha acima como

( o 1)
k+
X
k+l ~
+L.J
(k+l 1)
X
k+l-1 t
y+
(k+ 1)
k+l y
k+l
'
l=1

ou, o que é o mesmo, E7:/ (k·t)xk+ 1- 1y 1, exatamente a expressão que desejávamos obter. Logo,
temos por indução que (A.32) é verdadeira para todo n E N.

No que segue, colecionamos alguns exemplos de aplicação da fórmula do binômio, bem como
algumas consequências importantes da mesma.

COROLÁRIO A.50.
Para n EN, temos (x - y)n = E;=
0 (-l)
1 (;)xn-,y7.

PROVA.
Basta aplicar (A.32), trocando y por -y.

O item (a) do corolário a seguir é conhecido como o teorema das linhas do triângulo de
Pascal.

COROLÁRIO A.51.
Para n e N, temos

(a) Ej-o(j) = 2n•


(b) Lo~"(j) • Eº\JJ"
{;)= 2n-1 _
~---~~~--~----~-~~-----

452
f>B<>BLl-.\1 \-.;

PROVA.
Para o item (a), basta fazer x = y = l na fórmula do desenvolvimento de (x + y)n. Quanto
a (b). fazendo inicialmente x = l e y = -l na fórmula do binômio, obtemos (verifique!)

O= L
0:SJ:Sn
(n)- L (n)·
J 0:SJ:Sn J
211 21ó

Denotando A= Í:o:SJ:Sn (n)


1
e B = Í:osJ:Sn (n),
1
segue do item (a) e da relação acima que
211 216

A+B = 2n
{ A-B = O

Portanto, temos que A= B = 2n- 1 .

Problemas
1. Prove o teorema das colunas do triângulo de Pascal: na coluna n, a soma das entradas
das linhas n, n + 1, ... , n + k - l é igual à entrada situada na coluna n + 1 e linha n + k.
Em símbolo ,

2. Calcule, para cada n E N, o valor de 12 + 22 + • • • + n 2.

3. Para n > O inteiro, a diagonal n do triângulo de Pascal ê formada pelos números binomiais
(~), (n; 1), (n;2), (n;3), . . .. Prove o teorema das diagonais do triângulo de Pascal:

4. Se O< q < 1 e n EN, prove que qn < q+nrl-q)'

453
A.4 Geometria Analítica e Trigonometria
Recordamos, aqui, alguns fatos elementares sobre Geometria Analítica e Trigonometria, os
quais são utilizados com relativa frequência ao longo do texto. Para uma discussão mais apro-
fundada, sugerimos ao leitor a referência [l0J.

A.4.1 O plano cartesiano


Trace em um plano duas retas perpendiculares x e y, que se intersectam no ponto O. Con-
sidere, em seguida, x e y como cópias de R, escolhendo uma mesma unidade de medida para
ambas e fazendo O corresponder a O em ambas. Ficam, assim, determinadas sobre cada uma de
tais retas duas semirretas, uma positiva e outra negativa, com a convenção de que, em cada uma
delas, a semirreta positiva ê indicada por meio de uma pequena seta (cf. Figura A.5). As retn~
x e y dividem o plano em quatro regiões, denominadas quadrantes, os quais são numerados
de 1 a 4, conforme a convenção da Figura A.5; em particular, o ponto A marcado na mesma
encontra-se no segundo quadrante.

: y
A' Ay
---~----------
1
1
1
1
: 22 Quad. 12 Quad.
1

Â:rl Q X
, 32 Quad. 4 Quad.
2
1

Figura A.5: construção do plano cartesiano.

Dado um ponto qualquer no plano (A, por exemplo), trace por A uma reta perpendicular
à reta x e outra perpendicular à reta y, as quais intersectam tais retas respectivamente nos
pontos Ax e Ay. Reciprocamente, escolhidos arbitrariamente sobre x e y pontos Ax e Ay, as
perpendiculares traçadas a x por Ax e a y por Ay intersectam-se em um único ponto A do plano.
Portanto, dar um ponto A no plano é o mesmo que dar suas projeções ortogonais Ax e Ay sobre
as retas x e y, respectivamente.

454
C:1-:0:--.11-:'11<1.\.\:\\I.ÍII<"\ I·. THICl>"\1>\II 11,1\

Por outro lado, uma vez que as retas x e y estão sendo consideradas como cópias de R, às
projeções Â:r e Ay do ponto A sobre x e y correspondem números reais XA e YA, respectivamente,
os quais determinam completamente o ponto A (haja vista XA e YA determinarem os pontos Az
e A 11). Nesse caso, convencionamos escrever A = (xA, YA) ou, ainda, A(xA, YA). Na Figura A.6,
marcamos os pontos A(-3, 2), B(2, 1), C(-4, -3/4) e D(l, -J2).

y
A(-3, 2)
~---------
B(2, 1)
------,

1 o 1
X

C(-4, -3/4)
~------------- 1
1
--~
D(l, -/2)

Figura A .6: alguns pontos no plano cartesiano.

Em geral, fixadas em um plano retas r e y, perpendiculares em um ponto O, e escolhidas


em cada uma delas uma emirreta pos1twa de origem O, dizemos que o plano está munido de
um sistema de coordenadas cartesiano 19 xOy ou, ainda, que o plano euclidiano tornou-se
em um plano cartesiano. Para um ponto A(xA, YA) do mesmo, dizemos que XA e YA são as
coordenadas cartesianas do ponto A. Nesse contexto, o número real XA ê a coordenada-X

Figura A. 7: René Descartes, matemático e filósofo francês. Descartes é


considerado o criador do método analítico, também dito cartesiano, por
ter sido quem primeiro utilizou sistema de coordenadas na análise de
problemas de geometria euclidiana.

19 Após René Descartes, matemático e filósofo francês do século XVII.

455
\ 1 •1 , 1 11< 1 \ .\ 1( ,1 '\" 11 1: 1 1{ 1 ( JI 1-.,1 1 1 >-...

ou a abscissa de A, ao passo que o real YA é a coordenada-y ou a ordenada 20 do ponto A. As


retas x e y são respectivamente denominadas eixo-x ou eixo das abscissas e eixo-y ou eixo
das ordenadas do sistema cartesiano ern questão. Em particular, os pontos situados sobre
os eixos x e y têm coordenadas cartesianas respectivamente da forma (x 0 . O) e (O,Yo); o ponto
O, que representa O em ambos os eixos, tem, no sistema cartesiano sob consideração, ambas as
coordenadas iguais a zero.
Doravante, sempre que nos referirmos às coordenadas de um ou mais pontos do plano, salvo
menção explícita em contrário suporemos fixado no plano um sistema de coordenadas cartesiano
xOy.
Nosso primeiro resultado calcula as coordenadas de um ponto P que, situado sobre um
segmento AB, o divida em dois segmentos cujos comprimentos estejam, entre si, numa certa
razão dada.

PROPOSIÇÃO A.52.
São da.dosum real t E (O,1) e os pontos A(xA, YA) e B(xs, Ys). Se P(xp, yp) é o ponto sobre
o segmento AB tal que AP = t • AB, então

xp = (1 - t)xA + txs e 1/P = (1 - t)yA + tys. (A.33)

PROVA.
Provemos que Xp = (1 - t)xA + txB, sendo a prova da outra relação do enunciado totalmente
análoga. Se XA = x 8 , então o segmento AB é vertical e, dai,

Se XA # XB, suponha, sem perda de generalidade, que XA < XB (o caso XA > x 8 é análogo).
Sendo A', P' e B' as projeções ortogonais respectivamente de A, P e B sobre o eixo das abscissas
{cf. Figura A.8), segue do teorema de Thales da Geometria Euclidiana que
A'P' AP
==-==t.
A'B' AB
Mas, como A'(xA, O), P'(xp, O) e B'(xB, O), com P' E A' B' (posto que P E AB) temos A' P' =
Xp - XA e P' B' = x 8 - xp. Substituindo tais igualdades na relação acima, obtemos xe-xA
Xs-XA
=t
ou, o que é o mesmo, xp - XA = t(xB - XA)- Logo,

20 Não confundir ordenada com coordenada;a ordenada de um ponto é uma de suas coordenadas.

456
(;EO'.\IETHI.\ .\:'\.\I.Íll<'\ 1-: TBIC(l'\(>\11 llil\

y
B(xB, YB)

X
A' O P' B'

Figura A. : dividindo um segmento numa certa razão.

COROLÁRIO A.53.
Dados pontos A e B no plano, as coordenadas do ponto médio M do segmento AB são as
médias aritméticas das coordenadas respectivas de A e de B. Mais precisamente, se A(xA, YA) e
B(xB, YB), então AI e·6~ 7 Bl ~ ).

PRO\\
Sendo 1\/ o ponto médio de AB, temos A1\/ = ½AB. Portanto, basta fazer t = ½em (A.33)
para obter as coordenadas do ponto .A/.

as notações da proposição 52, é sugestivo escrever

P = {1 - t)A + tB {A.34)

como abreviação das duas relações em (A.33). Em particular, abreviamos as coordenadas do


ponto médio 1\/ do segmento AB escrevendo M = A1B. Doravante, faremos uso de tal abreviação
sempre que necessário.
Para o exemplo a seguir, recorde que, em um triângulo, uma mediana é um segmento que
une um vértice ao ponto médio do lado a ele oposto. Em particular, todo triângulo possui
e,xatamente três medianas.

EXEMPLO A.54.
Mostre que, em todo triângulo, as medianas concorrem no baricentro do triângulo e que o
mesmo divide cada mediana, a partir do vértice correspondente, na razão 2 : 1.

457
PROVA.
Seja ABC um triângulo qualquer, Mo ponto médio do lado BC e C o ponto sobre a mediana
Ali/ tal que AC= 2 CM. Então, AC= j AJvl e segue de (A.34) (com t = ~) e do corolário 53
que
(A.35)

Cálculos análogos com as demais medianas mostram que todas elas passam por esse mesmo
ponto.

DedUzimos, agora, uma fórmula extremamente útil para o cálculo da distância entre dois
pontos do plano em função de suas coordenadas, conhecida como a fórmula da distância.

PROPOSIÇÃO A.55.
Para pontos A(xA,YA)e B(xs, YB) do plano, temos

(A.36)

PROVA.
Temos que considerar quatro casos separadamente: XA ~ XB e YA < YB; XA < XB e YA > YB;
XA > XB e YA < YB; XA > XB e YA > YB· Contudo, uma vez que a análise de cada um de

tais casos é essencialmente equivalente à dos demais, concentrar-nos-emos no caso XA < x 8 e


YA < YB (cf. Figura A.9). Por simplicidade de notação, seja A(a, b) e B(c, d), de maneira que

y B(c, d)
1
B(c, d) T 1
1 1
1 1
1 A(a, b) - - - - - - _. C (e, b)
A(a, b) t
o X

Figura A.9: distância entre dois pontos no plano cartesiano

458
a< e e b < d. Se a= e (possibilidade à esquerda, na Figura A.9), então temos claramente

Como o caso b = d é análogo, suporemos, então, que a < b e e < d (possibilidade à direita
na Figura A.9). Trace, por A, uma paralela ao eixo das abscissas e, por B, uma paralela ao
eixo das ordenadas, e marque o ponto C de interseção das mesmas. Como C tem a mesma
ordenada que A e a mesma abscissa que B, temos C(c, b). Ademais, como os eixos cartesianos
são perpendiculares, o triângulo ABC é retângulo em C. Portanto, pelo teorema de Pitágoras e
pelos dois casos acima, temos
2 2
AB = AC + BC 2 = (e - a)2 + (d - b)2 = (a - c)2 + (d - d)2 ,

como queríamos provar.

Problemas
1. Dados números reais a e b, prove que os pontos (a, b) e (-a, -b) do plano cartesiano são
simétricos em relação à origem.

2. Dado um sistema cartesiano xOy, considere um outro sistema cartesiano x'O'y', onde o
novo eixo das abscissas é a reta y = y0 do sistema antigo e o novo eixo das ordenadas é
a reta x = x 0 do sistema antigo. Se um ponto A do plano tinha coordenadas (x, y) no
sistema antigo, prove que suas coordenadas no sistema novo são (x', y'), com x' = x - x 0
e y' = y- Yo-

459
A.4.2 Retas no plano cartesiano
Como frisado anteriormente, em tudo o que segue, supomos fixado um plano e um sistema
cartesiano de coordenadas no mesmo. Estudamos, a partir de agora, o problema de como repre-
sentar retas do plano em um tal sistema; nesse sentido, temos o seguinte resultado fundamental.

TEOREMA A.56.
Toda reta do plano cartesiano pode ser vista. como o conjunto dos pontos (x, y) do mesmo
que satisfazem uma equação da forma

ax + by +e= O, (A.37)

# O ou b # O.
onde a, b e e são números reais tais que a
~-~~---~--~~~--------
PROVA.
Seja O a origem e r uma reta do plano cartesiano. Suponha, inicialmente, que O ~ r. Se

Figura A.10: reta do plano cartesiano não passando pela origem.

A(a, b) é o pé da perpendicular baixada de O ar (cf. Figura A.10), é imediato que um ponto


B(x, y) do plano está sobre r se, e só se, OÂB = 90º. Portanto, o teorema de Pitágoras e sua
recíproca garantem que
2 2 2
(x,y) E r {:} OA + AB = OB
{:} (a 2 + b2 ) + [(x - a) 2 + (y - b)2)= x 2 + y 2
{:} ax + by - (a2 + b2 ) = O,
com {graças a O ~ r) a# O ou b # O. Segue quer é o conjunto das soluções (x. y) da equação
ax + by +e= O, com e= -(a 2 + b2).

460
, PHOBLE\1.\~

Suponhamos, agora, que O E r (cf. figura A.11) e, sobre a reta perpendicular a r traçada
por O, marquemos um ponto qualquer A(a. b). Como no primeiro caso, um ponto B(x, y) do
plano está sobre r se, e só se, AÔ B = 90º. Portanto, novamente pelo teorema de Pitágoras e
sua recíproca, temos

Figura A.11: reta do plano cartesiano passando pela origem.

2 2 2
(x,y) E r # OA + OB = AB
# (a2 + b2) + (x2 + y2) = [(x - a)2 + (y - b)2]
# ax + by = O,
com (graças a A f/.r) a -:f.O ou b -:f.O. Segue quer é o conjunto das soluções (x, y) da equação
a.r + by + e = O, com e = O.

Nas notações do teorema anterior, dizemos que (A.37) é a equação da reta r, o que indi-
camos escrevendo
r : {ax + by + e = O}.

OBSERVAÇÃO A.3.
Para todo real k i=O, também é lícito dizermos que

(ka)x + (kb)y + (kc) = O

é a equação da reta r; de fato, não é difícil verificar que toda equação da reta ré d~ forma.

461
Doravante, utilizaremos tal observação sem maiores comentários.

COROLÁRIO A.57.
Se r é a reta do plano cartesiano com equação ax + lYy+ e = O e s é a reta perpendicular a r
e passando pela origem, então s tem equação -bx + ay = O.

PROVA.
De acordo com o teorema anterior, a retas tem equação da forma ex+ dy = O, para certos
e, d E lR, não ambos nulos. Entretanto, como r tem equação ax + lYy+ e = O, segue da prova do
teorema anterior que (a, b) E s. Portanto, ca+ db = O e, pela observação acima, podemos tomar
e= -b,.__d= a.

Provemos, agora, a recíproca do teorema 56.

COROLÁRIO A.58.
Se a, b e e são reais tais que a :/:Oou b :/:O,então o conjuntersolução da equação ax+lnJ+c = O
representa uma reta no plano cartesiano.

PROVA.
Suponhamos que e =/-O, sendo a análise do caso e = O totalmente análoga. Escolha uma
solução arbitrária x = xo, y = Yo da equação ax + by +e= O, de maneira que ax 0 + 1Yy
0 +e= O
Se s é a reta passando por (O,O) e (a, b), então s tem equação -bx + ay = O. Seja r que

Figura A.12: toda equação linear em x e y representa uma reta.

passa por (xo, Yo) e é perpendicular a s, digamos no ponto (ka, kb) (cf. Figura A.12). Pelo

462
corolário anterior, r tem equação da forma (ka)x + (kb)y +d= O, para algum d E R, ou, ainda,
f
ax + by + d' = O, onde d' = (como e f O, temos que (O,O) ~ r, de sorte que k f O). M88,
como (xo, Yo) E r, temos ax 0 + by0 +d'= O e segue do que fizemos acima que

e" = -axo - bYo = e.

Assim, a equação da reta ré ax + by +e= O.

Para o que segue, observe que uma reta ax + by + e = O é vertical no sistema cartesiano
escolhido se, e só se, b = O. Portanto, se uma reta r de equação ax + by + e = O não for vertical,
então podemos escrever sua equação equivalentemente como y = -i x - f ou, mais sucintamente
(escrevendo a no lugar de -! e b no lugar de -f),

y = ax + b.
De posse do comentãrio acima, o próximo resultado apresenta a condição necessária e sufici-
ente usual para o paralelismo de duas retas em termos de suas equações.

PROPOSIÇÃO A.59.
Duas retas não verticais r e s são paralelas se, e só se, têm equações da forma 'li= ~ + b e
y = ax + b', para certos números reais a, b e b', com b =/:f/.

PROVA.
Suponha, inicialmente, que r 11 s, e que r tem equação y = ax + b ou, o que é o mesmo,
ax - y + b = O. Pelo corolário 57, a reta t, de equação x + ay = O é perpendicular ar. Mas,
como r li s, também temos s..Lt e, daí, mais uma aplicação do corolário 57 garante que s tem
equação da forma ax - y + b' = O, para algum b' E IR.
Reciprocamente, se r e s têm equações da forma y = ax + b e y = ax + b', com b f b', então
é imediato que o sistema de equações

y = ax + b
{ y = ax + b'
não possui soluções, de sorte que r li s.

Continuando nosso estudo de retas, a seguir derivamos a condição necessária. e suficiente


usual para o perpendicularismo de duas retas em termos de suas equações cm um certo sistemn

463
cartesiano. Concentramo-nos somente no caso não trivial em que nenhuma das retas em questão
é vertical ou horizontal em relação ao dito sistema.

PROPOSIÇÃO A.60.
Duas retas não verticais r e s, de equações respectivamente y = ax +be y = a'x + b', são
perpendiculares se, e só se, aa' = -1.
-------------------~----~
PROVA.
Sabemos, pelo corolário 57, que a reta t, de equação x + ay = O, (ou, equivalentemente,
y = -¾x) é perpendicular ar. Por outro lado, como r ..Ls<=> t li s, segue da propo ição anterior
que r l_s se, e só se, a equação de t é da forma y = a' x + e, para algum e E IR. Por fim,
comparando as equações y = -¾x e y = a'x + e para t, concluímos que a equação de t lcm n
forma y = a'x + e se, e só se, e= O e a'=-¼, i.e., se, e só se, aa' = -1.

Em relação a um sistema cartesiano fixado, se r é uma reta não vertical, de equação y - a.z:+b,
dizemos que a é seu coeficiente angular. Daremos uma mterpretação geométrica do coeficiente
angular na proposição 69 (veja também o parágrafo subsequente à demonstração da mesmA)
Por ora, observamos que, de posse desse conceito a propo ·ição anterior diz, em palavras, que se
1

r e s são retas não verticais do plano cartesiano, então r e s são perpendiculares se, e só se, o
produto de seus coeficientes angulares é igual a -1.
Ainda em relação à discussão do parágrafo anterior, seja y = ax + b a equação de uma reta
não vertical r. Então, se A(x 1 ,y 1 ) e B(x2,Y2) são pontos distintos sobre r, temos y1 = ax 1 +b e
Y2 = ax2 + b, com X1 # x2. Daí,

de maneira que
Y2 -y1
a=---. (A.38)
X2 - X1

De posse da discussão acima, terminamos esta subseção com o seguinte resultado.

PROPOSIÇÃO A.61.
No plano cartesiano, sejam ruma reta de equação ax + f,y +e= O. Se P(x 0, y 0) é um ponto
quàlquere d é a distAncia de P a r, então

d = laxo + i,yo + cl
(A.39)
✓a2+b2 •

464
l 'IH >BLE:'\I.\S

PRO\A
Suponha quer não é vertical nem horizonLal (para os casos em quer é vertical ou horizontal,
veja o problema 7). Então, a, b =/=O, de maneira que podemos escrever a equação der da forfna
y = -ix - ~- Ses é a vertical ar passando por P, então s também não é vertical, de forma que
podemos escrever sua equação como y = ax + (3, para certos a, (3 E IR.
Como r ..ls, a proposição anterior garante que (-i)a = -1. Como P E s, temos também
que Yo = axo + (3. Portanto, a= ~ e (3 = y 0 - ax 0 = ªYo~bxo.
Se Q(x1, yi) é o ponto de interseção de r e s, então x = x 1 e y = y1 é a única solução do
sistema
a e
y = -bx - b '
{ y = QX + (3
de forma que
_ e+ (3b _ b2x 0 - aby0 - ac
X1 - ----- ------
a+ ab a 2 + b2
e
-ac + (3a -abxo + a2 y 0 - bc
Yi = a+ ab - a 2 + b2
Portanto,
a(axo + byo + e) b(axo + byo + e)
Xo - X1 = a2 + b2 e Yo - Y1= a2 + b2
e, daí,
-2 2 2
PQ = (xo - xi) + (Yo - Y1)
a2 (axo + byo + c)2 b2 (axo + byo + c)2
=-------+-------
(a2 + b2)2 (a2 + b2)2
2 2
(axo + byo + c) . (a 2 + b2 ) = (axo + byo + c)
(a2 + b2)2 a2 + b2

Problemas
1. Dados números reais a e b, prove que os pontos A(a, b) e B(b, a) do plano cartesiano são
simétricos em relação à bissetriz dos quadrantes ímpares.

2. São dados os pontos distintos A(a, b), B(c, d) e a reta vertical r, formada pelo conjunto
dos pontos de abscissa x 0 . Prove que A e B são simétricos em relação a r se, e somente
se, b = d e Xo = ªIc.

465
\ i 1 \ 1 1 1, 1 \ '\ 1 ' ' 1 '\"' l 11, 1 - 1{ 1 ( 11 '"' 1 1 ( )'-i

3. ào dado' O' pontos A(a, b), B(c, d) e a reta horizontal r, formada pelo conjunto dos pontos
d' ordenada y = y0 . Prove que A e B são simétricos em relação a r se, e somente se, a = e
"Y o --ili 2 •

4. t um sistema cartesiano de origem O, temos pontos A(a, b) e B, tais que A está no primeiro
quadrante e B no segundo. Se AÔB = 90º e OA = OB, prove que B(-b, a).

5. r o plano cartesiano, mostre que a equação da reta que passa pelos pontos (xo, O)e (O,Yo),
com xo, y 0 =/:O, é 3..
xo
+ JL
!/O
= 1.
6. Dados, no plano cartesiano, os pontos A e B, seja Pt = (1 - t)A + tB, com t E R.
+--+ +--+
(a.) Prove que Pt E AB e que para todo P E AB existe um ú~ t E R tal que P = Pt.
(b) Discuta a posição do ponto Pt sobre a reta ÃÊ em funçã6 dos valores de t.
/
+--+ +--+
A descrição acima dos pontos da reta AB é denominada a equação paramétrica de AB,
com parâmetro t. ,

7. Complete a prova da proposição 61, analisando os casos em quer é vertical ou horizontal.


A.4.3 Seções cônicas

Podemos examinar círculos do pont.o de vista analít.ico como aplicação direta da fórmula
(A.36). Para tanto, lembre-se de que o círculo r(C; R) do plano é definido como o conjunto dos
pontos A do plano cuja distância ao centro C é igual a R.

y
r~-.A(x, y)

C(xo, Yo)
o X

Figura A.13: a equação de um círculo no plano cartesiano.

Fixado no plano um sistema de coordenadas cartesiano como na Figura A.13, sejam C(x 0, y 0)
e A(x, y) um ponto qualquer. Segue de (A.36) que

2
AE r # AC = R # AC = R 2
# (x - xo)2 + (y - Yo)2 = R 2
2 2 2
# x + y - 2xox - 2yoy + (x6 + yi - R ) = O.

Dizemos, então, que


(A.40)

é a equação do círculo de centro C e raio R.


Reciprocamente, será que toda equação da forma

x2 + y 2 + ax + by +e= O, (A.-11)

tem como solução, no plano cartesiano, o conjunto dos pontos de um círculo? Veremos que, snlvo
nos casos em que tal equação não possuir solução alguma ou possuir no mA.ximoumn soluçúo
(x, y), a resposta é sim! Para tanto, comecemos completando quadrados em x 2 + a:r e y 2 + by,

467
\
\ 1 1 '1' 1 \ .\ 1 ( '' '\" l '1 \ 1 1: ' ( J' , ...,' 1 ( )'-,

obtendo
2
x + y 2 + ax + by .+ e =

Portanto, (A.41) equivale à equação

(
Sendo C (- ~, -! ),
segue daí e da fórmula (A.36) para a distância entre dois pontos quc o
conjunto dos pontos A(x, y) do plano cartesiano que satisfazem (A.41) coincide com o conjunto

{
A· AC2
1
= ª2 +4 b2 - e} •

Há, então, três possibilidades:

, • ª
2

!be < O: nesse caso, é imediato que não há ponto A tal que AC
2

-
2
=ªt 2 2

- e, de
sorte que o conjunto de pontos procurado é vazio.

• ª e= O: uma solução qualquer A(x,y) da equação deve satisfazer


2 2

tb -

e já sabemos que a única possibilidade nesse caso é x = -~, y = -!, i.e. A= C. Portanto,
o conjunto de pontos procurado tem um único elemento.
2 2
• ª tb - e > O: os pontos A procurados são aqueles tais que

i.e., o conjunto-solução de (A.41) é o círculo de centro C (-~, -~) e raio R = Jª 2


~b2 - e.

A discussão acima provou o resultado a seguir.

468
TEORE:\fA A.62.
No plano cartesiano, a equação

x + y2+ ·ax + by + e = O
2

representa o conjunto vazio. um conjunto com um único elemento ou


2
ª !&2- e seja negativo, nulo ou positivo, respectivamente. Ademais, no 61
círculo é o ponto ( - ~. - ~) e seu raio é Jª 2
~b
2
- e.

A seguir, estudamos brevemente elipses, hipérboles e parábolas. Juntamente com os círculos,


tais curvas são exemplos de seções cônicas, o nome cônicas vindo do fato de que as mesmas
podem ser obtidas como interseções de um cone de revolução com planos que não passam pelo
vértice do cone (cf. 1 J, por exemplo).

ÜFFINIÇÃO A.63.
SeJam dados, no plano. pontos F 1 e F2 e um segmento de comprimento 2a, com
A elipse de focos F 1 e F2 e eixo maior 2a é o lugar geométrico dos pontos P d~

PF1 + PF2 = 2a.

esse caso. F 1F2 é a distância ___________________


focal da elipse. ._.__

A Figura A.l.J esboça a elipse da definição acima. A fim de justificar seu formato, comecem~\l
\i~ (i-t
y (){+Gl

Figura A.14: elipse de focos F1 e F2 e eixo maior 2a .

.J.69
pr vand prop 1 ao guir a qu l iden ifica a lip da d finição an rior m coord nad
um i t m car iano d coord nad particular.

Consid re a. elip de focos F 1 e F 2 , eixo maior 2a e distância focal 2c. Em relação ao ma


rDTTooiano
tal que F 1 (-e O) e F 2 (c, O), a equação da elip é dada por

x2 y2
a2 + b2 = 1, ( .43)

ond b = Ja2 - c2. Noc:i,cu,


caso, dizemo que (A.43) é a equação can nica da lip

PRO A.
endo P(x,y) gue da fórmula (A.36) pra adi t~n i ntr doi p n qu

PF1 + + c)2 + y 2 = 2a - J( - e)
PF 2 = 2a ç::> J(x
=> (x + c)2 + y 2 = (2a - J( - c)2
X1_+:) ti ~
~ i I~ -1
ló_ ...'>-- 0+
~ l # aJ( - c)2 + y2 = a2 - ex
/'=> a2((x - c)2 + y2) = (a2 - cx)2 /, \)"

e.,'I , y, \ _ O'- - , ç> (a2 _ e ) 2 + 0 2y2 = 0 2(a2 _ 2)


,..., 't \'Y - ')..:...
{ L , ç> b2x 2 + a2y =a b2
:tl, 2 y2
f\.l
~\_-\-/
o- 0-
,.
..
1 ç::> a2 + b2 = 1.
Portanto, todo ponto P que p rt nc à elip m que tão ati faz ( . 3).
Reciprocament 1 ja P(x, y) um pon o ati faz ndo ( . 3). Conclwr mo qu P F 1 P F2 =
2a form capaze d mo trar qu n quência d implicaçõ qwval ~n i im . a
duas implicaçõ ão de fato equivalênci Para tanto gue d ( . 3) qu 1 1 < a , d í
2 2
a - ex > a - ca = a(a - e) > O. Portanto

a2[(x - c)2 + y2] = (a2 - ex)2 ç::> aJ(x - c)2 + y2 = la2 - exl
=> aJ(x -c) 2 +y 2 = a2 - ex

e conseguimos transformar a gunda implic ção em equivalência. fim de garantir qu a


prim ira implicação é uma equivalência, not inicialm nte que a igualdade ( + c)2 y 2 -
(2a - J(x - c) 2 + y2 ] 2 equival nt a

J(x + c)2 + y2 = 12a - J(x - c)2 + y21.

70
PHOl3LE~I.\S

Portanto. se mostrarmos que, estando P(x. y) sobre a elipse 1, tem-se


2a - J(x - c)2 + y 2 >O.seguirá da última igualdade acima que

conforme desejado. Para o que falta. basta observar que J(x - c)2 + y = PF 2 < PF 1 + PF2 =
2

2a.

De acordo com (A.43), a elipse em questão passa pelos pontos A 1 (-a, O), A2(a, O), Bi(O, -b)
e B2(0. b) e está inteiramente contida no retângulo de lados paralelos aos eixos coordenados e
+--+ +--+
passando por A1, A2, B 1 e B2 . Também, ela é simétrica em relação às retas A 1A2, e B1B2 e ao
ponto médio de A A2. Realmente, como (A.43) permanece inalterada ao trocarmos x por -x ou
1

y por -y, o problema 2, página 465, garante que a elipse é simétrica em relação à reta x = O, a
+--+
qual coincide com B1B2, e o problema 3. página 466, garante que a elipse é simétrica em relação
+--+ +--+
à reta y = O, a qual coincide com A 1 A 2 . Por fim, sendo simétrica em relação à reta A 1A 2 e à
+--+
sua mediatriz (que é a reta B B a elipse deve, necessariamente, ser simétrica em relação ao
1 2
),

ponto médio O de A1A 2 . Graças a esse resultado, dizemos que a elipse é uma cônica central,
de centro O, eixo maior A 1 A2 e eixo menor B 1 B 2 .

DEFINIÇÃO A.65.
SeJam dados, no plano, pontos F1 e F2 e um segmento de comprimento 2a, com 2a < F1 F2 .
A hipérbole de focos F 1 e F2 e eixo maior de comprimento 2a é o lugar geométrico dos pontos
P do plano tais que
1 PF1 - PF2I = 2a. (A.44)

Na Figura A.15, a hipérbole em questão é a união das duas curvas contínuas que passam por
A 1 e A 2. A fim de justificar seu formato, escolha um sistema cartesiano de coordenadas tal que
F 1 = (-e, O) e F = (e, O). Um raciocínio análogo ao executado no caso de uma elipse garante
2

que a hipérbole ê o conjunto dos pontos (x, y) do plano tais que


x2 y2
a2 - b2 = l, (A.45)

onde b2 = Jc 2 - a 2 . Em particular, ela passa pelos pontos A 1 (-a, O) e A 2 (a, O), mas, diferente-
mente da elipse, não passa pelos pontos B1(O,-b) e B2(0, b). Por outro lado, assim como ocorre
com a elipse (e com uma demonstração análoga), a hipérbole da definição anterior é simétrica em
+--+ +--+
relação às retas A A e B B logo, também é simétrica em relação ao ponto médio O de A A
1 2 1 2
,
1 2
.

Então, assim como com a elipse, dizemos que a hipérbole em questão é uma cônica central de

471
y

Figura A.15: hipérbole de focos F 1 e F2 e eixo maior 2a.

centro O, eixo maior A 1A2 e eixo imaginário B 1B 2 (observe que, no caso de uma hipérbole,
não necessariamente temos b < a).
Como não há ponto algum da hipérbole sobre a reta x = O, concluímos que a hipérbole
consiste de dois pedaços, denominados seus ramos. Ainda nas notações da Figura A.15, observe
que as retas r e s que contêm as diagonais do retângulo tracejado têm equações respectivamente
iguais a y = ~x e y = -~x. Portanto, um cãlculo fácil garante que tais retas não intersectam a
hipérbole, e o problema 1.12, página 137, quantifica a suspeita de que tais retas aproximam-se
mais e mais dos ramos da hipérbole, à medida que lxl aumenta. As retas r e s são as assíntotas
da hipérbole. Se a= b (ou, equivalentemente, se as assíntotas são perpendiculares), dizemos que
a hipérbole é equilâtera.
A definição a seguir fornece um modelo comum para o estudo de elipses e hipérboles, bem
como introduz as parábolas.

DEFINIÇÃOA.66.
Sejam dados um ponto F e uma reta d, tais que F (Í. d. Para e > O também dado, definimos
a c6nica de foco F, diretriz d e excentricidade e como a curva formada pelos pontos P do
plano tais que
PF =e· dist(P; d), (A.46)
iW.fliat.•P;d)denota a distãncia de P à reta d. O parAmetro da cônica é a distância p de F

A Figura A.16 esboça uma porção da cônica de foco F, diretriz d e excentricidade e, em

472
'
l 11« IBl.l·.\I \'-,

relação a um sistema cartesiano de coordenadas tal que F(c, O) e d: {x = x0 }, com x 0 >e> O.


Então, p = xo - e, para um ponto P(x, y) do plano cartesiano, temos

y d

Po(xo, y)

o V X

Figura A.16: cônica de foco F, diretriz d e excentricidade f.

PF =e· dist(P;d) <=>J(x - c)2 +y 2 = elx - xol


<=>(x - c)2 + y2 = e2(x - xo)2
(A.47)
x2
<=> - 2cx + c2 + y2 = e2(x2 - 2xox + xl)
<=>(1 - e )x + 2(e x
2 2 2

0
- c)x + y 2

= (ex0 ) 2 - c
2

A última equação acima deixa claro que a cônica que estamos descrevendo é simétrica em
relação à reta e que passa por F e é perpendicular a d (e que, no sistema de coordenadas
escolhido, coincide com o eixo das abscissas), e passa pelo ponto V O). Doravante, diremosU~~,
que e é o eixo e V é o vértice da cônica em questão.
Se e = 1, a cônica de foco F e diretriz d é denominada uma parábola. Nesse caso, pelos
cãlculos acima (e substituindo p = xo-c), concluímos que sua equação no sistema de coordenadas
escolhido é y 2 = -2px+p(x 0 +c). A partir daí, trocando os papéis dos eixos x e y (o que equivale
a uma mudança de sistema de coordenadas), obtemos a equação y = - 2~ x 2 +~+e para a parábola
em questão. Invertendo o sentido positivo no eixo-y (o que equivale a trocar y por -y, na última
equação acima), obtemos finalmente a equação

1 2 p
Y = 2px - 2 - e,

473
para a p r bola, em r lação à qual s u foco é o ponto F(O, -e) e sua diretriz é a reta d : {y ==
-xo}, com xo >e> O. A ess respeito, veja também o teorema 49.

oltando a (A.47) suponha, doravante, que E =J1. Transladando o eixo d ord nada da
po ição atual para a po ição da reta x = a, egue do problema 2, página 459 qu a nova quaçào
da cônica é
(1 - E
2
)(x + a) 2 + 2(rxo - c)(x +a)+ y 2 = (Exo)
2
- c2 .
Escolhendo a= _ E:~1:? (a fim de anular o coeficiente de x) e substituindo p = Xo-c concluímo
facilmente que a equação acima se torna
E2p2
(l _ E2)x2 + Y2 = --.
1- f.2

Ademais, nes e novo i tema carte iano, temo F ( 1~;:2 O) e d: {x =


1
!i2}.

É imediato que a última equação acima pode ser escri a como

(A. )

com os sinais + ou - e colhidos conform ja, re pec ivam nte O < < 1 ou E> 1.
Denotando a= 11~F21, b =
E
p
ll-i 2
e verificando, separadamente, o casos O < < 1
J
>l 1

conclua que a2 =Fb2 = c2 , onde e= 1~~ é a nova abscissa do ponto F. Portanto a quação da
cônica em questão assume uma das formas
x2 y2
a2 ± b2= 1,
com os sinais + ou - escolhidos conforme seja, respectivamente, O < E < 1 (elip es) ou >1
(hipérboles).

Problemas
1. Prove que a excentricidade da elip e ~ + ~ = 1 mede seu achatamento na direção y ou,
equivalentemente, seu alongamento na direção x. fais precisamente, prove que, fixado
o eixo maior A 1A2 1 quando E se aproxima cada vez mais de 1 (por valor menores que
1), a elipse fica cada vez mais parecida com o segmento A1 A2 , ao passo que, quando E e
aproxima cada vez mai de O, a elipse fica cada vez mais parecida com o círculo de diâmetro
A1A2.

47
[ >B<>BI.L\I \ ..._

A.4.4 Trigonometria
o plano cartesiano, o ciclo trigonométrico é o círculo r da Figura A.17, centrado na
origem 0(0, O), com raio 1 e comprimento 21r.

y
B

o X

B'

Figura A.17: seno e cosseno de um arco.

Dado um número real e, medimos sobre r, a partir de A, um arco de comprimento lcl, no


sentido anti-horário (dito trigonométrico) se e > O e no sentido horário se e < O. Sendo P a
extremidade final desse arco, dizemos que o arco AP (de comprimento possivelmente maior que
21r) mede e radianos, e definimos o seno e o cosseno de e, abreviados respectivamente sen e e
cose (cf. Figura A.17) por

cose = abscissa de P; sen e = ordenada de P.

Como as abscissas e ordenadas dos pontos de r são menores ou iguais a 1 em módulo, temos

-1 < sen e < 1 .


{ -1 <cose< 1

Reciprocamente, fixado um real a E [-1, 1], a reta paralela ao eixo das abscissas traçada pelo
ponto {O,a) intersecta r em pelo menos um ponto P; sendo P(sen e, cose), é imediato que
senc = a. Em outras palavras, todo número real no intervalo [-1, 1] é o seno (e, analogamente,
o cosseno) de algum arco.
Para k E Z, é imediato que a extremidade final de um arco de 2k1r radianos coincide com o
ponto A de r. Mais geralmente, fixado e E lR, a extremidade final de um arco de comprimento

475
e+ 2k1rcoincide com aquela de um arco de comprimento simplesmente igual a e, de maneira que

sen (e + 2k7r) = sen e


{ cos(c+2k1r)=cosc '

para todo k E Z.
A proposição a seguir é conhecida como a relação fundamental da trigonometria.

PRC>PóSIÇÃO A.67.
Para todo e e R, temos
sen 2 c + cos2 e = 1. (A.50)
-----------------
PROVA .
.......
Seja AP = e (cf. Figura A.17). Como P(cosc,senc) e 0(0,0) segue da fórmula (A.36) e de
AP = 1 que
J(cos e - 0)2 + (sen e - 0)2 = l.

Se e E Ré tal que cose-::/:O, a tangente de e, abreviada tg e, por


senc
tgc = --.
cose
Nas notações
.......
da Figura A.17, observe que cose= O exatamente quando a extremidade final
P do arco AP = e coincidir com B ou B'. Por outro lado, é imediato que tal ocorre precisamente
quando e = ~ + 2k1rou e = 3; + 2k7r,para algum k E Z. Isso é o mesmo que dizer que e = i + k1r,
para algum k E Z. Assim:

tg e está definida se, e só se, e -::/:i + k1r para todo k E Z.

A tangente de um arco tem uma interpretação geométrica bastante útil, explicitada na pro-
posição a seguir.

......
,,aura A.18, se AP= e é um arco do primeiro ou terceiro quadrantes, então
6 UPl arcodo segundo ou quarto quadrantes, então tg e = - AP'.

476
PRO\A.
Façamos a prova no caso em que e é um arco do segundo quadrante (a prova nos demais C8808
é completamente análoga). Seja P" o pé da perpendicular baixada de P ao eixo das absci8888.
Pela semelhança dos triângulos PP" O e P' AO, temos ~;,~ = ~ . Mas, como PP" = sen e,
P"O = - cose e AO = 1, ao efetuar essas substituições na igualdade acima, obtemos
-- PP"· AO
P'A = --==-- = -tgc
P'A '
que é precisamente a relação do enunciado.

y
B

A' A X

P" O

P'
B'

Figura A.18: interpretação geométrica da tangente.

Uma consequência imediata da proposição anterior é o fato de que, se dois arcos diferem por
um múltiplo inteiro de 1r, então suas tangentes são iguais. Em símbolos, tg (1r + e) está definida
se, e só se, tg e também estiver; nesse caso, temos ainda

tg (1r + e) = tg e. (A.51)

Para o que segue, estabelecemos a seguinte convenção: dado um ângulo (), com 0° < () < 360º,
definimos o seno, o cosseno e a tangente de() como sendo respectivamente iguais ao seno, o cosseno
e a tangente do arco correspondente a () em radianos, i.e., do arco e = 21r • ~. Por e.xemplo,
para () = 20º, o arco correspondente é e = i radianos. Assim, temos, por definição, que
7T" 7T" 7T"
sen 20º = sen , cos 20º = cos e tg 20º = tg .
9 9 9

477
A proposição a seguir encerra um importante corolário do resultado anterior.

PROPOSIÇÃO
A.69.
Dada, no plano cartesiano, uma reta não vertical de equação y = ax + b, o número real a =/O
é igual à tangente do ângulo trigonométrico que o eixo das abscissas forma com a reta. Mais
precisamente, nas notações da Figura A.19, temos a= tgo.
~~-~~~-----------

PROVA.
Na Figura A.19, o círculo representa o ciclo trigonométrico. Pela Proposição 59, a rela
paralela à reta y = ax + b e passando pela origem tem equação y = ax. Como o ângulo
trigonométrico que o eixo das abscissas forma com tal reta também é o, egue da proposição
anterior (e das convenções acima) que a tg o é igual à ordenada do ponto comum às relas de
equações y = ax ex= 1 da Figura A.19. Para tal ponto comum, temos x = 1 e y =a· 1 = o.

y
x=l

y = ax

Figura A.19: coeficiente angular de uma reta.

de sorte que tg o = a, como desejado.

Graças à proposição acima, se uma reta não vertical tem equação y = ax + b, dizemos que a é
o coeficiente angular da reta. Colecionamos, no exemplo a seguir, uma aplicação importante
da noção de coeficiente angular.

478
EXEl\tPLO A. 70.
Em um certo sistema cartesiano, temos um ponto (x0 , y 0 ) e um ângulo o tal que 0'-
180º. Encontre a equação da reta que passa pelo ponto dado e forma ângulo trigononl6t
com o eixo das abscissas.

PROVA.
Se o= 90º, então a reta é vertical, logo, tem equação x = x 0 . Se a=/:- 90º (cf. Figura A.20),
podemos supor que sua equação é da forma y = ax + b, para certos a, b E IR. Pela proposição 69,
temos a= tgo. Por outro lado, como o ponto (x0 , y0) pertence à reta, devemos ter

y = ax + b
X

Figura A.20: reta com coeficiente angular e ponto prescritos.

Yo = axo + b = ( tg o)xo +b
e, daí, b = y0 - ( tga)x 0. Portanto, a equação da reta ê

y = ( tga)x + Yo - ( tga)xo.

as notações do exemplo acima, vale frisar que é costume escrever a equação obtida para a
reta em questão na forma
y-yo
--- = tgo. (A.52)
X- Xo
Continuando nossa revisão de Trigonometria, deduzimos, a seguir, as fórmulas de adição
de arcos.

479
PROPOSIÇÃO A. 71.
Para a, b E R, temos:

(a) cos(a±b) = cosacosb~senasenb.


(b) sen (a± b) = sen acosb ± cosasen b.

(e) tg (a ± b) = ;~:;f.bb, sempre que tg a, tg b e tg (a ± b) estiverem definidos.

PROVA.
Mostremos, inicialmente, a fórmula para cos(a + b). Suponha, sem perda de generalidade,
b > Oe marque, sobre o ciclo trigonométrico, os pontos P, Q e R tais que AP = a, AQ = -b e
a,,...,.
AR= a+b. Então, P(cosa,sena), Q(cos(-b),sen (-b)) = (cosb, -senb) e R(cos(a+b),. n (n f-
b)).
........ ........
Como os arcos AR e QP (medidos no sentido anti-horário) são ambos iguais a a+b radianos,
as cordas subentendidas AR e PQ têm comprimentos iguais Portanto, as igualdades
2
AR = (l - cos(a + b)) + scn 2 (a + b) 2

e
-2 2
PQ = (cosa ?
- cosb)· +(sena+ senb)
dão-nos
( 1 - cos(a + b)) + sen a + b) = (cos a - cos b) + (sen a + sen b)
2 2

(
2 2

Desenvolvendo ambos os membros dessa relação e observando as igualdades cos2 a+ sen 2 a = 1,


cos b + sen 2 b = 1 e cos (a + b) + sen (a + b) = 1, obtemos
2 2 2

-2 cos(a + b) = -2 cos acosb + 2 sen asen b.

conforme desejado.
Quanto às demais fórmulas, veja que, pelo problema 1, temos

cos(a - b) = cos(a + (-b)) = cosacos(-b) - sen asen (-b)


= cosa cos b + sen a sen b
e

sen (a + b) = cos (; - a - b) = cos ( ~ - a) cos b + sen (; - a) sen b


= sen acos b + cosasen b.

480
Ademais, uma dedução análoga nos permite obter a fórmula para sen (a- b) (veja o problema2,
página 482). Por fim,

sen (a + b) sen a cos b + sen b cosa


tg(a+ b) - -
cos(a + b) cosacosb - sen asen b
1
_ cosacos b (sen a cos b + sen b cos a)
1
- cos 0 005 b (cosa cos b - sen a sen b)

_ rosa
sena+ côs'b _ t ga + tgb
senb
-----
- 1- ~senb - 1 - tgatgb'
cosa cosb

valendo, também aquj, uma dedução anãloga para tg (a - b) (cf. problema 2, página 482).

Fazendo b = a nas fórmulas com sinal + da proposição 71, obtemos as fórmulas do corolário
a seguir, as quis são conhecidas como as fórmulas de arcos duplos.

COROLÁRIO A. 72.
Para todo real a: temos:

(a) cos 2a = cos2 a - sen 2 a.

(b) sen 2a = 2sen a cosa.


(c) tg 2a = 1:_tfg2a
, sempre que tg a e tg 2a estiverem definidas.

O próximo resultado coleciona as fórmulas de transformação em produto.

PROPOSIÇÃO A. 73.
Para todos os a, b E R, temos:

(a) sen a ± sen b = 2 sen ( ªib) cos ( ªWb)


.
(b) cosa+ cosb = 2cos (ª;b) cos (ª 2b).
(c) cos a - cos b = -2 sen ( ~) sen ( ª 2b) .

(d) t g Q ± t g b = cosaCOlb
sen(a±b).
---~~-~-~~--------------------!
PROVA.
Façamos as deduções das fórmulas para sen a + sen b e tg a + tg b, sendo os demais casos
totalmente anãlogos (cf. problema 5, página 482).

481 â-
Fazendo x = ªt e y = ª 2 b, temos a= x + y e b = x -y, de maneira que

sen a + sen b = sen (x + y) + sen (x - y)


= (sen x cos y + sen y cos x) + (sen x cos y - sen y cos x)

2 2
(ª-
= 2senxcosy = 2sen ( -a+ -b) cos - -b) •

Por outro lado,

sen a sen b sen a cos b + sen b cosa sen (a + b)


tga+ tgb= --+-- = ------- = b.
~a ~b ~a~b ~a~

Problemas
1. Para todo e E IR, prove que:

(a) sen (-e) = -sen e e cos(-c) =cose.


(b) sen (~ - e)= cose ecos(~ - e)= senc.
(c) sen (1r- e) = sen e e cos(1r- e) = - cose.

2. Prove as fórmulas da proposição 71 para os desenvolvimentos de sen (a - b) e tg (a - b).

2 cos2 a - 1
3. Para a E IR, prove que cos 2a = .
{ 1 - 2 sen 2 a

4. * Mostre que sec2 a= 1 + tg 2 a, para todo a E IR\{~+ k1r;k E Z}.

5. Obtenha as demais fórmulas àe transformação em produto listadas na proposição 73.

6. Sejam xOy e x'Oy' sistemas cartesianos de coordenadas com mesma origem O, tais que o
semieixo positivo das abscissas do sistema x'Oy' faz (nessa ordem) um ângulo trigonomé-
trico de(} radianos com o semieixo positivo das abscissas do sistema xOy. Se um ponto P
do plano tem coordenadas (xo, Yo) no sistema xOy e (x~, y~) no sistema x'Oy', prove que

x~ = Xocos (}+ Yosen 0 e y~ = -xo sen fJ + y0 cos 0.

7. Dada a elipse de equação ~ + ~ = 1, faça os seguintes itens:

482
(a) Se (x, y) é um ponto sobre a mesma, prove que existe um único 0 E [O,2,r) para o
qual x = acos0, y = asen 0, e reciprocamente. As equações x = acos(J, y = asenO,
0 E [O,21r),são as equações paramétricas da elipse, e podemos interpretá-las como
uma descrição da traJetória de uma parLícula que parte no instante 0 = O do ponto
A, percorre a elipse no sentido anti-horário e volta ao ponto A no instante 0 = 2,r.
(b) Defina os círculos diretores da elipse como os círculos de equações x 2 + y 2 = a 2
e x 2 + y 2 = b2 . Pela origem O, trace um raio de tais círculos, formando um ângulo
trigonométrico /3 com o eixo das abscissas. Suponha que tal raio intersecta os círculos
diretores acima nos pontos Se Q, respectivamente. Se P(x, y) é o ponto obtido como
a interseção da paralela por Q ao eixo das abscissas com a paralela por S ao eixo
das ordenadas (cf. Figura A.21), prove que P pertence à elipse (note que o processo

A(a, O)

Figura A.21: equações paramétricas da elipse.

acima permite construirmos com régua e compasso tantos pontos da elipse quanto
queiramos).

483
SUGESTÕES AOS PRO-
BLEMAS
1.1
1. m •t r - 1 ~ O 3 - x > O d forma qu x E [l, 3).

2. m t r x ~ O, 3 - .fi, ~ O, ~ - J3 - -Ji ~ O ½- J-
~ J3 - .fi,. O domínio maximal d
f o conjunto form do p l int r çõ do conjunto - olução de cada uma de as in qu çõ .

P o i m ( ), v j qu /(1) = f (½) = li~~i~I = O, /(10) = f (1~) = 1 i~~~!~


1= i 9
1 1
J n:)= J (~) = l;~~~~I = Quanto a (b) 5
la2 - b2 1 = 55 a 2 + b2 = 73, ent-o (a b) = ( , 3)
13 .
ou (3, ), d form qu / (ª) = / ( 3) = ~~- la2 - b2 1 = 32 e a 2 + b2 = 257, ntão 2a 2 = 2
ou 2a 2 = 225 d forma qu a f1.N; m , colocando a 2 - b2 = 32k a 2 + b2 = 257k, om k E ,
ob m 2a 2 = 2 9k 2b2 = 225k, d forma qu k = 2 forn a = 17 b = 15 ( nalo m nt
upondo b a = 32k obt mo a = 15 e b = 17); logo f n~) = /
2

-
2

= P r fim ui 3

57
-

2 2 2 2 2 2
a - b = 101k e a + b = 9k, para algum k E ntão a = 95k b = -6k d f rm u
2 2 2 2
(ab) = -570k ; b - a = 101k a 2 +b 2 = 9k, concluímo an l am n qu (ab) 2 = -570A·2
m qualquer o, eh gamo a um ab urdo, um z qu (ab)2 >O> -570k 2 .

4. Basta v r qu /(x) = x(x 2 - 2x + 5) = x((x-1) 2+ ), d form qu f(x) = 2 (( -1) 2 ) > O.

5. Comec faz ndo x = 1 e y = v12na rei ção d d par calcular f(l + v12).
6. Ob rv qu ªk+l = ak + r calcul J(ak+i) = J(ak + r) = J(ak)f(r).

Para o quo i n ,fr pomo ,., = {x E • g(x) :/aO} e definimo ; : ' ➔ por L(
g
) =M
g(x)'
para todo x E '

9. Por definição, lxJ E Z, de forma que imagem d l·J é um ubconjunto d Z. Por outro 1 do,
para n E Z, é claro qu o maior int iro menor ou igual a n • o próprio n d forma qu ln J = n
e, as im, n E Im(l·J). Logo Im(l·J) = Z.

10. lxJ = n, então nê o maior int iro menor ou igual a x, de forma que n $ < n 1. Portanto
O$ x-n < 1 , daí, {x} = x- lxJ = x-n E [O 1). egue qu Im({·}) e (O,1). R ipro am nt,
O$ o< 1, ntão loJ = O d modo que {o} = o - loJ = o. Logo E Irn({·}) p todo
oE(0,1),d formaqu [O l)clrn({·}). Então Im({·})=[0 1).

11. Para o it m (b), us ( ); p ra (e), onsid re inicialm nte o e o k E , i indução; por fim,
apliqu o r ultado d (e) p ob r (d) o re ult do d ( ) ( ) para b er ( ).

ÃO 1.2

1. m bk+l - bk = J(ak+d - J(ak) = (aak+l + b) - (aak + b) = a(ak+l - ak) = ar.


2 J\Iostre que a imagem pedida é (O,+oo).

3 Observe inicialmente que x + ½~ 2, para todo x > O, ocorrendo a igualdade se, e só se, x = 1.
Em seguida, conclua que a imagem pedida é lR\ (-2, 2).

4. Analisemos o caso em que fé crescente em (-oo, a] n J e decrescente em [a, +oo) n J (o outro


caso é totalmente anâlogo). Para xo E I tal que xo < a, temos f(xo) > /(a) e, assim, / não
atinge seu valor mínimo em x 0 . Também, para xo E J tal que xo > a, temos J(xo) > f(a) e, uma
vez mais, xo não é ponto de mínimo para /. Como / não atinge seu valor mínimo em nenhum
elemento xo E I \ {a}, concluímos que a é seu único ponto de mínimo.

5. Para Yo E Y, tome xo E X tal que J(xo) = YO· Então, pela definição da função /+e, temos
(f + c)(xo) = J(xo) +e= Yo + e, de sorte que Yo + e E Im (J + e). Mas, como Yo+ e ê um elemento
típico do conjunto Y + e, concluímos que Y + e e Im (J + e). Agora, se Y1 E Im (J + e), então,
por definição, existe x 1 E X tal que (J + c)(xi) = y 1 ou, o que é o mesmo, f(x1) +e= Yt· Então,
J(xi) = Y1 - e, de forma que Y1 - e E 1m (/) = Y. Assim, Y1 = (Y1 - e)+ e E Y + e e, como isso
é válido para todo Y1 E 1m (J + e), segue que Iro (J + e) e Y + e. Logo, Im (/+e)= Y + e.

6. Argumente como na sugestão dada ao problema anterior.

7. Consideremos o caso em que a> O, sendo o caso a< O totalmente análogo. Para y E R, temos
2
J(x) = y se, e só se, (x + ;0 ) - {:r = ~ ou, ainda, se, e só se,
2
b) y 6 4ay + t!1
(x + 2a = ~ + 4a2 = 4a 2 •
---
2
Como (x + 2~) ~ O, essa equação tem solução se, e só se, 4ay + 6 ? O; mas, como a> O, tal
condição equivale a y ? - fa,.Portanto, existe x E IR tal que f(x) = y se, e só se, y ? - de t,
sorte que 1m (!) = [- fa, +oo).

8. Use a forma canônica de J (cf. problema anterior) para obter, no caso em que a> O,por exemplo,

f(x) < O ~ a [ ( x + ;a)'


- 4~2] <O ~ (x + ;J- ~ <O
4 2

# (x + ~)
2a
2
< ~2
4a
~~-...,....2ª
lx + ~, < ./E#
2a
- ../"K< x +
2a
~
2a
< ./E
2a
-b- -/E J-b+ó
~ <x< 2 .
2a a

9. Para mostrar que 6 > O, use a forma canônica de f para obter

af(xo) = a
2
[(xo
+ ..!!.._)
2a
2
- ~i
4a 2 '

487
Mas, como (xo + 2~) ~ O, devemos ter
2 2
de forma que af(xo) < Ose, e só se, (xo + ;0 ) -~<O.
O$ (xo + ;0 ) < ~ e, daí, ó.> O. Para a segunda parte, use o resultado do problema anterior.
2

10. Se um retângulo de perímetro 2p tem dimensões x e y, então x + y = p. A partir daí, mostre que
sua ârea depende de x de acordo com a função quadrãtica f(x) = -x 2 + px e apllque o resultado
da proposição 24.

11. Sendo l a largura e h a altura do caminhão (medidas em metros), é imediato que o volume de
carga que o mesmo pode transportar é igual a 18lhm 3 ; portanto, temos de ma.xnnizar o produto
lh. A fim de que o caminhão possa entrar no túnel, sua seção reta ( um retângulo de lnd~ medmdo
l e h) deve ter diagonais de comprimento d, com d$ 5m, mas, como d= J/ 2 + h 2, temos que

Pondo x = l 2 , concluímos que basta ma.ximizru-a função J(x) = 25.r.- 2 • com O< x < 5.
:..r

12. Mostre, inicialmente, que devemos procurar o menor real po:;itivo o tnl que f('J·) $ "' parn
todo x E R; equivalentemente, I.S::íOé o m mo qu, procumr o menor real pos1t1vo ci tnl que
ax 2 - 5x +(a+ 1) ~ O para todo :r E R.

13. Para par, digamos n = 2k, utilizando n de:-igunldnd trinngulnr. obtemos

f(x) = (lx - ad + ·· · + l:r.- OA:I) + (lok•H - xi+···+ la2A-- xi)


k U D •
~ I)x - a1 ) + L (aj - x) = L a1 - L ªJ•
J=l J=k+l J=k-t-1 1 l

Agora, mostre que todo x E (ak, Ok+1) satisfaz J(x) = L;:k+l a 1 - L;=l a 1 . Por fim, o caso em
que n é ímpar pode ser tratado de modo análogo.

14. Para o item (a), aplique a desigualdade entre as médias aritmética e geométrica ao denominador
da expressão que define a função. Para o item (b), escreva f(x) = 2-,- :r~~ 1 e, em seguida, proceda
como em (a).

15. Para o item (a.), escreva

(x + lO)(x + 2) ((x + 1) + 9)((x + 1) + 1) ( ) . 9


------ = ---------- = X+ 1 T -- + lQ
x+l x+l x+l

e, em seguida, aplique a. desigualdade entre as médias para obter f(x) ~ 2/§-+- 10 = 16. Para
(b), aplique a desigualdade entre as médias ao denominador para obter x 4 Ta ~ 2x2va e, daí,
f(x) $ 2 7a-
488
16 Comece escrevendo
4x2 4(x2- l) 4
f(x) =--+k=---+--+k
x+l x+l x+l
4 1
= 1) + - - + = 4
4(x

x+l
- + 1 + - -)
x+l
k (x + (k - 8).

Portanto. sendo g(x} = x+l+ z_: 1 temos f(x) = 4g(x)+(k-8). Agora, o resultado do problema
2.3, Juntamente com o fato de g ser ímpar. implica que Im (g) =IR\ (-2, 2), de modo que (pelo
resultado do problema 2.6) Im (4g} = lR\ (- . 8). Agora, o resultado do problema 2.5 garante
que 1m (/) = \ (k - 16. k). de sorte que de,·emos ter k - 16 = -L e k = L.

SEÇÃO 1.3

1 >
1 Observe primeiro que, para todo .::E R. temos l.:: .::se, e só se, z < O. Portanto, queremos que
g(f(r)) < O. Fazendo y = f(x). ~egue do problema 2.8, página 34, que g(y) <O<=>y 2 - ¼ <O<=>
-! < y < ½·M . como y = f(x), queremo que-½ < x - ~ < ½ou, ainda, 3 < x < 4.

2. Com e ob~ervando que (f o g)(x) = f(g(x)) = 2g(x) + 7 e, daí, que 2g(x) + 7 = x 2 - 2x + 3.


2
3 P,1çn g(:r) = y para ob1e1 x = "t3 • diú /(y) = f(g(x)) = 2x2 -4x+l !
= 2 ( 11 3 ) -4 (J/!
3
) +l.

41. Como f o g o g têm do111fnio cont1 domínio. iguais, temos f o g = g o g <=>(f o g)(x)
g =
(g o/){ r.) ~ /(g(r.)) = g(f(.r.)). D ~rn, o orn, calcular f(g(x)) = (ac)x + (ad + b) e g(f (x)) =
(ac).r.+ (bc + d) e igualar o~ r ·ultndo .

5. Pnrn .l\ f(x) ,j. -b, temos

/(/(:i·)) = f(x) + a = ~ + a = (a+ l)x + (ab + a)


f(x) + b :t6+ b (b + l)x +(a+ b2).

pnrtir• d m,• cone 1ua que d evemos t er (a+l ab+a = x para t od o x E IR, exceto por no máximo
r+ a+t,:i
b+l x~ •
quatro ,-n.loresde x. Portanto, e..xcetopor tais valores de x, temos (a+ l)x + (ab + a) = (b +
1):r.2 +(o+ b2 )x Conclua, então, que b + 1 = O.)

6 Supondo que f é crescente (o caso em que f é decrescente pode ser tratado de modo análogo),
bnsta observar que, se X1,x2 E/, então x1 =fix2::::}/(xi) =fif(x2). Por sua vez, a fim de verificar
a validade dessa 1mphcação, note que x1 =fix2 implica X1 < x2 ou x 1 > x2. Caso seja x 1 < x 2 ,
segue do fato de f ser crescente que /(xi) < f(x2) e, então, /(x1) =fif(x2). Argumente de modo
análogo no caso em que x1 > x2.

7. Analisemos o caso em que f e g são crescentes, deixando ao leitor a análise dos outros dois
caso . Para x 1 < x 2 em I. segue do fato de / ser crescente que f(x 1) < f(x 2). Mas, como
/(x 1), f(x 2 ) E J e g também é crescente, temos g(f(xi)) < g(f(x 2 )).

489
Comece supondo que f = 9 + h, onde 9, h : I ➔ IR são tais que 9 é par e h é ímpar. Em
::;eguida, use as definições de função par e ímpar para concluir que 9(x) = ½U(x) + J(-x)) e
h(x)½(f(x) - f(-x)).

9. Inicialmente, mostre que /(1) = O. Em seguida, faça a = 1 e b = -1 para calcular f (-1). Por
fim, faça b = -a.

10. Basta calcular (J o f)(-x) = J(J(-x)) = J(- J(x)) = - J(f (x)) = -(! o f)(x), onde utilizamo~
o fato de que f é ímpar segunda e terceira igualdades.

11. Tome f(x) = -9(x) para x :5 O e J(x) = -x para x ~ O.

12. Para o item (a), comece mostrando que todo x E IRpode ser escrito da forma x = kp + o, com
k E Z e o E [O,p). Em seguida, faça indução sobre k para mostrar que f(x) = J(x - kp) Para
o item (b), observe primeiro que 9 ( x + far)
= f(ax ± p) = f(ax) = g(x), para todo x E IR;em
seguida, note que, se p' > O satisfaz 9(x + p') = 9(x) para todo x E R, então f(ax + ap') = f(ax)
para todo x E JR.A partir daí, conclua que lalp' = lap'I ~ p.

13. A fim de mostrar que J é ímpar, comece observando que /(20 + x) = /(10 + (10 + x)) =
J(lO - (10 + x)) = J(-x) e, analogamente, /(20 - x) = f(x). Em seguida, use o fato de que
J(20+x) = - !(20-x). Para a segunda parte, calcule J(40+x) = J(20+ (20+x)) = - f(20+x) =
f(x).

14. Fazendo 9(x) = f(x) +½,mostre que 9(x +a)= J~- g(x) 2 e, daí, que g(x + 2a) = 9(x), para
todo x E JR.

15. Encontre uma partição de N da forma N = A1 U A2 U A3 U ... , com An infinito, para todo n EN.
Em seguida, defina/ pondo f(x) = n, para todo x E An.

16. Supondo que x1 < x2 são pontos fixos de/, segue do fato de f ser decrescente que f(xi) > f(x2).
Mas, como /(xi)= xi e J(x2) = x2, temos uma contradição.

17. Se existissem x1 < x2 em I tais que /(xi) = 9(xi) e J(x2) = 9(x2), então, como J é decrescente
e 9 é crescente, teríamos /(xi) > f(x2) = 9(x2) > 9(xi) = /(xi), um absurdo.

18. Observe inicialmente que x = 4 é uma solução dessa equação. Em seguida, veja ambos os membros
da equação como funções de (O,+oo) em si mesmo e aplique o resultado do problema anterior
para concluir que x = 4 é a única solução.

SEÇÃO 1.4
1. Para o item (a), use a proposição 36. Para o item (b), suponha que 91,92 : Y ➔ X satisfazem
g, o f = ldx e f o g, = ldy, parai = 1, 2. Use a proposição 31 para obter 91 = 91 o Idy =
91 ° (/ 0 92) = (91 ° f) 0 92 = Idx O 92 = 92·

490
2. Use repetidas vezes o resultado da proposição 31, juntamenle com o falo de que 1- 1 o f = Idx,
Jof- 1 = ldy.909- 1 = Idz eg- 1 09= ldy.

3. Aplique o resultado do exemplo 41.

4. Veja a sugestão ao próximo problema.

5. Para a primeira parte, note que a equação ~~ = y tem, para todo y f. ~. a solução única
X= -dy+b_
cy-a

6. Para a primeira parte, basta ver que, para O $ x 1 < x2, temos x2 - x 1 > O; portanto, pelo item
(a) do problema 5, pâgina 417, temos

X2 - X1 = (x2 - X1)(xr-l + xr- 2x2 + · • · + X2-l) > Ü,

de maneira que J(xi) < J(x2) e f ê crescente, logo injetiva. Para a segunda parte, dado y E
[O,+ ), como estamos assumindo que Im(J) = [O,+oo), existe x E [O,+oo) tal que y = xn;
então, obtemos x = r:Jypor definição da raiz n-ésima de y, de sorte que 1- 1 (y) = ylij.

7. Se J(x) = 2x, temos 1- 1(x) =!,de forma que (J + 1- 1)(x) = 5{ e (J - 1- 1)(x) = 3;,as quais
são duas bijeções de R em IR.

8. Suponha que f ê crescente (o caso em que f ê decrescente é inteiramente análogo). Dados y 1 < Y2
em J, tome os elementos x1,x2 E J tais que J(x1) = Y1 e J(x2) = Y2· Se x 1 = x2, então
Y1 = f(x1) = J(x2) = Y2, o que não é o caso; se x2 < xi, então segue de f ser crescente que
Y2 = J(x2) < /(xi) = Y1, o que também não ê o caso. Logo, x 1 < x2 ou, o que é o mesmo,
J 1(yi) < 1- 1(y2). l\1as, como os elementos Y1< Y2 em J foram escolhidos arbitrariamente, isso
garante que / 1 também é crescente.

SEÇÃO 1 5

1. Para a segunda parte, mostre que as abscissas dos pontos comuns aos gráficos são as soluções da
equação J(x) = x. Em nosso caso, a única solução é x = ~-

2. Se lf(x)I $ l'vl para todo x E J e (xo, Yo) E GJ, então IYol = 1/(xo)I $ M, de sorte que
-1'1 $ y0 $ M. Portanto, (xo, Yo) pertence à faixa horizontal do plano cartesiano, delimitada
pelas retas paralelas y = - Jvf e y = M.

3. Como a bissetriz dos quadrantes ímpares do plano cartesiano é o conjunto dos pontos (x, y) do
mesmo tais que x = y, concluímos que xo E J é ponto fixo de f se, e só se, f(xo) = xo, i.e., se, e
só se, (x 0 ,x 0 ) E G 1 . Portanto, os pontos fixos de J são exatamente as abscissas dos pontos onde
o grâfico de J intersecta a bissetriz dos quadrantes ímpares do plano cartesiano. a Figura B.l,
supondo que J é crescente para x < x1 e decrescente para x > x3, temos que os pontos fi..xosde
J são x 1 , x 2 e x 3 , os quais são as abscissas dos pontos A, B e C, respectivamente.

491
X

Figura B.l: pontos fixos de uma função f.

4. Um ponto (x, y) ê comum aos grâficos de f e g se, e só se, x E / e y = f(x) = g(x). Portanto,
para identificar tais pontos, ê suficiente resolver, para x E J, a equação f(x) = g(x).

5. Façamos a prova do item (a), sendo a prova do item (b) totalmente análoga. Se f ê par, temos
J(x) = f(-x), para todo x E /. Portanto, para x E J, temos (x, y) E Gf {=} y = J(x) {:} y =
f(-x) {:} (-x, y) E Gf, Mas, como os pontos (x, y) e (-x, y) são simétricos em relação ao eixo
das ordenadas, concluímos que o mesmo sucede com Gf.

6. AI, funções h e f 4 são sempre não negativas e só se anulam em x = O. Também, é claro que
são funções pares, de forma que, pelo problema anterior, seus gráficos são simétricos em relação
ao eixo vertical do sistema cartesiano. Por outro lado, à medida que lxl aumenta, é evidente
que h(x) = x 2 e f 4 (x) = x 4 tornam-se cada vez maiores e, eventualmente, ultrapassam qualquer
valor predefinido. Por último, lxl < 1 ::::}x 4 < x 2 , lxl > 1 ::::}x 4 > x 2 e lxl = 1 ::::}x 4 = x 2 = 1,
justificando o fato de o gráfico de /4 estar situado abaixo do gráfico de h no intervalo ( -1, 1)
e a.cima fora do intervalo [-1, 1] (veja a Figura B.2). As funções h e / 5 são positivas para
x > O, negativas para x < O e se anulam em x = O. Também, ê claro que são funções ímpares, de
forma que (novamente pelo problema anterior) seus gráficos são simétricos em relação à origem do
sistema cartesiano. Agora, à medida que lxl aumenta é evidente que 1/J(x)I = lx3 1e 1/s(x)I = lx5 1
tornam-se cada vez maiores e, eventualmente, ultrapassam qualquer valor prefixado. Por último,
lxl < 1 ::::}lx5 1< jx3 1,lxl > 1 ::::}lx5 1> lx3 1e lxl = 1 ::::}lx5 1= lx3 1,justificando o fato de o
gráfico de /s estar, no intervalo ( -1, 1), mais próximo do eixo horizontal do que o gráfico de h e
mais distante de tal eixo fora do intervalo [-1, 1] (veja a Figura B.3).

7. Observe que fé a inversa da função g: IR-+ IR, tal que g(x) = x 3 , para todo x E IR. Em seguida,
aplique o resultado da proposição 50 ao esboço do gráfico de g = h, construído no problema
anterior.

492
y

x2

o X

Figura B.2: grâficos de h e Í4.

8. Verifique que tal gráfico é a união dos conjuntos (n, n + 1) x {n}, quando n varia em Z.
9. Para o item (a), observe inicialmente que, para x E [O,p), a periodicidade de f garante que
J(x + kp) = f(x), para todo k E Z. Portanto, (x, y) E G I <=? y = f(x) <=? y = f(x + kp) <=?
(x + kp,y) E e,. Então, sendo F = e,
n ((0,p) x JR),temos G1 = Ukez(F + k), onde F + k
denota a translação de F segundo o vetor (k, O). Para (b), note que {x} = x para x E (O,1).
Como {·} é periódica de período 1, segue de (a) que a Figura 2.1, página 92, esboça o gráfico de
{·}.

10. Para o item (a), basta mostrarmos que (x, y) é um ponto do gráfico de f se, e só se, (x - a, y)
é um ponto do gráfico de g. De fato, se (x, y) pertence ao gráfico de J, então f(x) = y e, daí,
g(x -a) = J((x - a) +a) = y, quer dizer, (x - a, y) pertence ao gráfico de g; provamos a recíproca
do mesmo modo. Para o item (b), basta mostrarmos que (x, y) é um ponto do gráfico de f se,
e só se, (x,y + a) é um ponto do gráfico de g. De fato, se (x,y) pertence ao gráfico de J, então
J(x) = y e, daí, g(x) = J(x) +a= y + a, quer dizer, (x, y + a) pertence ao gráfico de g; provamos
a recíproca do mesmo modo. Os demais itens podem ser analisados de forma similar.

11. Em princípio, pode parecer que não podemos usar o resultado do problema. anterior, haja vista que
a função em questão não tem por domínio o conjunto dos reais. Contudo, como 2,:x = .r 2 2 = ,:!
-1- x.:. 2
, podemos raciocinar como naquele problema, esboçando o gráfico de f do seguinte modo
(deixamos ao leitor a tarefa de esboçar as figuras correspondentes aos passos descritos a eguir):
primeiro, traçamos o gráfico de x i--+ i; em seguida, transladamos o gráfico anterior duas unidach:.
para a direita, obtendo o gráfico de x i--+ :z:~2; agora, alongamos o gráfico anterior na dir~oo
vertical, pelo fator 2, obtendo o gráfico de x i--+ x.:. 2 ; refletimos o resultado no eixo horizontal,
obtendo o gráfico de x ._. - x.:. 2 ; por fim, transladamos o gráfico assim obtido umn unidade para

493
\ 1 1,1, 1 1: SI LI ..,l<>I.., \<>.., i>H<>III.I.\I.\S

y x3

Figura B.3: gráficos de /J e /s-

baixo, obtendo o gráfico de x H 2 ,:x.

12. Para a primeira parte, mostre que, de posse do gráfico de f, obtemos o gráfico de g refletindo, ao
longo do eixo das abscissas, a porção do gráfico de f situada abaixo de tal reta. Para os itens de
{a) a (c), aplique esse procedimento às funções f(x) = x 2 -4, f(x) = xii e f(x) = x 2 - lx+21 + 2,
respectivamente. Observe que, de acordo com o problema 5.10, o gráfico de f(x) = x~I pode
ser obtido transladando-se o gráfico de x H ½ uma unidade para a esquerda. Quanto ao gráfico
de f(x) = x 2 - lx + 21+ 2, para esboçá-lo analise separadamente os casos x $ -2 e x > -2,
observando que lx + 21 é respectivamente igual a -x - 2 ex+ 2.

13. Aplicando uma rotação de -¾ radianos ao sistema cartesiano original (o que equivale a rotacionar
a hipérbole x 2 - y 2 = 4 de f radianos), obtemos, pelo resultado do problema 6, página 482, a
2 2
curva de equação ( x72 ) - ( :x:J?) = 2/2 ou, ainda, xy = l.

SEÇÃO 1.6

a primeira parte do item {a), sejam


l, Para ,...... A = (1, O) e P o ponto sobre o ciclo trigonométrico tal
que AP = x. Como scn x ê a ordenada de P, se x1, x2 E [-;,;],com x 1 < x 2 , temos claramente
sen x 1 < sen x2, de sorte que a rest,rição da função seno ao intervalo [-;, ~] é crescente, logo,
jnjetiva. Como tal reBtrição tem imagem [-1, 1], a função arco-seno fica bem definida e, pelo

494
problema 4. , página 47, é crescente. A primeira parte do item (b) pode ser tratada de modo

fato de que, para x E (-i, i), tg x ê a ordenada do ponlo de interseção da semirreta OA com a
reta vertical passando por A.)
-
análogo, observando que cos x ê a abscissa de P. Por fim, a primeira. parte do item (e) segue do

2. Para o ilem (a), observe que ctg x = - tg (x + i) e utilize os resulLados do problema 5.10, página
59. Para os itens (b) a (d), utilize o resultado da proposição 50.

3. Para os itens (a) e (b), referimos o leitor à discussão do exemplo 55, mais precisamente à equação
(1.9). Quanto ao item (c), sugerimos ao leitor rever os itens (a) e (e) do problema 5.10, página
59.

4. Argumentando como no exemplo 55, obtenha f(x) = J a2 + b2 cos(.Xx- a:), onde cosa:= ✓a.f+&'i
e sena= ✓a.f+i/1. Agora, mostre que f tem período m·
5. Para o item (a), comece utilizando as fórmulas do arco duplo para obter J(x) - -2sen 2; +
2 sen x + 1. Em seguida, observe que a função quadrática y ......+-2y 2 + 2y + 1 assume seu valor
máximo em y = ½,sendo crescente em [-1, ½]e decrescente em [½,1]. O item (b) segue da
expressão acima obtida para/, juntamente com o fato de que a função seno tem período 21r. Por
fim, para o item (c), esboce os grâficos de x ......+2 sen x ex......+cos(2x) no intervalo [O,21r)e em um
mesmo sistema cartesiano; em segwda, some as ordenadas dos pontos desses dois grâficos para
obter o esboço do gráfico de f.

6. Use o resultado do problema 5.4, página 58.

7. Opere a mudança de variável x = ✓5 cos 0, onde O $ 0 $ 1r. Em seguida, use a discussão do


exemplo 55 para concluir que f(x) = 25 cos(O- a:), com a: tal que cosa:= i
e sena:= ;.

8. Suponhamos, por contradição, que f fosse periódica, de período T > O. Então, deveria ser
j(T) = J(O) = 2. Mas, aí, 2 = f(T) = cosT + cos(a:T) $ 1 + 1 = 2, uma vez que o cosseno de
qualquer número real é no máximo 1. Assim, deve ser cos T = cos(a:T) = 1. Para tanto, devem
existir inteiros (não nulos) k, l tais que r = 2k1r e a:r = 2l1r. Dividindo membro a membro essas
duas igualdades, chegamos a a:= t E Q, o que é uma contradição.

SEÇÃO 2.1

1. Para o item (a), veja que ( Vl/x)


= t1x
= x. Para (b), argumente de maneira análoga. O item
n

(c) segue imediatamente dos resultados dos problemas 4.6 e 4.8, pâgina 47. Por fim, parn (d),
segue do problema 4.6, página 47, que v'ã > v'ã se, e só se, a11 > a 111; então, o resto segue do
corolário 2.

495
2 Para o item (a), suponha que temos a validade do resultado quando a ~ O. Se b ~ O, então
-b ~ O e, por (a), existem r E Q e a E lR\ Q tais quer, a E (-b, -a). Portanto, -r, -a E (a, b),
com -r E Q e -a E lR\ Q. Se a ~ O < b, mostre que basta aplicarmos o resultado que supomos
conhecido ao intervalo (O,b). Para o item (b), basta tomar n E N tal que n > 61 • Por fim,
quanto a (c), note que m > nb => mp > ';: > b; mostre em seguida que, se m for o menor
natural satisfazendo mp> b, então m > 1 e (m-~)v'2 E (a, b).

3. Adapte os itens (b) e (c) do problema anterior ao caso presente.

4. Use o exemplo 2, juntamente com o axioma 3.

5. Use o item (b) do problema 1.2.


2
6. Para a= k e b = k 2 + 1, temos lva- Jbi = ~~)b= ~+# < 7ft·Como todo elemento de
X é positivo, segue que inf X = O.

7. Observe que, para k EN, todos os números da forma i,,


e 1- i,,pertencem a C. Conclua o que
se pede a partir daí, juntamente com o fato de que f,ê< Í·

8. Adapte a prova da proposição 6 ao caso presente.

9. Use a proposição 6, juntamente com o resultado do problema 1.8, para concluir pela existência
de Xn E X e Yn E Y tais que a - 2~ < Xn < a e a < Yn < a + 2~.

10. Para o item (a), mostre que, se a (resp. {J) é uma cota superior para X (resp. para cX), então
ca (resp. ~) é uma cota superior para cX (resp. para X). Os outros casos podem ser tratados
de modo análogo.

11. Para o item (a), comece mostrando que, se a e /3 forem cotas superiores respectivamente para
X e Y, então a+ /3 é uma cota superior para X+ Y; a partir daí, conclua que sup(X + Y) ~
supX +sup Y. Em seguida, mostre que se fosse sup(X + Y)-supX < sup Y, então Y teria uma
cota superior menor do que sup Y.

13. O resultado do problema 1.11, página 23, garante que f(x) =x para todo x E Q. Agora, se
x,y E JR,com x < y, e a =/ O for tal que y - x = a 2 , aplique (a) e (b) para concluir que
f(y) - J(x) = J(y) + J(-x) = J(y- x) = f(a 2 ) = J(a) 2 > O, de sorte que fé crescente. Por fim,
supondo que existe a E lR tal que /(a) < a, use a densidade de Q em lR {cf. problema 1.2) para
escolher r E Q tal que /(a) < r < a; em seguida, use o caráter crescente de f para chegar a uma
contradição. Conclua, analogamente, que não se pode ter /(a) > a.

SEÇÃO 2.2

L Adapte o argumento da demonstração do item (a) da proposição 15, mostrando que, se fosse
a > b, terfamOR,eventualmente, ª" > ªIb > bn,

496
2 Basta ver que IFn - vai = ~~J;i < 7alan - ai.
3. Para n > 16a2 , mostre que I~~ < (~~q < (~)n< 2~.

4. Escrevendo l = (1 - tn)l + tnl, use a desigualdade triangular para obter len- li = 1(1- tn)(On -
l) + tn(Cn - l)I $ (1 - tn)lan - li+ tnlbn - li, de sorte que lan - li, lbn - li < l impliquelen- li ~
{1 - tn)f+ tnl = f.
5. Note primeiro que bn = (1 - tn)an + tnCn, com tn = ~ Cn-On
E (O,l], se an < Cn, e tn = O, se
an = Cn- Em seguida, aplique o resultado do problema anterior.

6. Para o item (a), escreva "2


n
ft = ,/n
n+~
1
1 < --¼-
vn
Para (b), escreva

✓ 2 an+b a+!
n + an+ b-n = - --;==="==---
Jn 2 + an + b + n j 1+ ~ + ~ + 1
Para o item (c) observe que, se an = y'l + qn, entãoª"> 1 e, pelo item (a) do problema 5, página
1
417, O < ª" - 1 = Onn-r +an:32-+ ··+an+1 < Ç-.Por fim, para (d), escreva v'a" + b" = a ~l + (~)"
e use o resultado do item anterior.

7. Adapte, ao presente caso, os argumentos da demonstração da proposição 18.

8. Para a primeira parte, escreva q = 1 + a., com o > O,e note que an = (1 + o)" ~ 1 + no, de modo
que an > M para n > M;; 1 .

9. Seja b = ½,de sorte que b > 1. Use a primeira parte do problema anterior para concluir pela
existência de k E N tal que bk > a. Em seguida, mostre que 2" > kn para todo n suficientemente
grande e conclua, a partir daí, que anq2n = (p;)" · b2n:kn
➔ 0 quando n ➔ +oo.

10. Para k E N, use a desigualdade triangular para mostrar que Iam - anl < m;~t:2
+ n?~t2 . Em
seguida, faça k ➔ +oo.

11. Observe que an+l = Jl + an, para todo n ~ 1. A partir daí, conclua sucessivamente que
ª~+l - a~ = ª" - an-1 e que (an)n~1 é crescente. Em seguida, note que ª~+l = 1 + an implica
an+l = ilia.
On+I
< l+an+t
On+l
< 2, e aplique o teorema de Bolzano-Weierstrass para garantir a existência
de l = limn--++ooan. Agora, faça n ➔ +oo na relação an+l = Jl + ª" e utilize o item (a) do
problema 2.2, juntamente com o item (b) da proposição 18, para obter a equação l = ✓f+I.

12. Use o fato de tk= -to - t1 - • • • - tk-1 para escrever


an = to( vn- Jn+k) + t1 (Jn + 1 - Jn + k) + ...
+ tk_ 1(Jn + k - 1 - Jn + k)
kto kt1 _ ... _ ktk-1
= - vfn+ Jn+k Jn + l + Jn + k Jn + k - 1 + Jn + k •

497
Por fim, faça n ➔ +oo.

13. Faça a substituição trigonométrica an = 2 cos 0n, com 0n E [O,~], e use um pouco de Trigonometria
para concluir que 0n+l 2::20n, para todo n 2::1. A partir daí, mostre que 0n ~ ",l,.2'"
~ 2,'"+1, para
8

todo k 2::1, de sorte que 0n = O para todo n 2::1.

14. Mostre, sucessivamente, que a~ = k + ªn-l e a~+l - a~ = an - an-l, e conclua, a partir daí, que a
sequência (an)n2:l é crescente. Use agora que a~ < k + an para concluir que an < ½(1+ J4k + 1)
para todo n 2::1, de sorte que a sequência em questão é convergente. Por fim, faça n ➔ +oo em
a~ = k + ªn-1 para mostrar que an ➔ ½(l + J4k + 1) quando n ➔ +oo. A partir dai, os itens
(b) e (c) são relativamente imediatos.

15. Comece usando indução para concluir que ªk+l 2::Ja,, para todo k 2::1. Agora, faça k ➔ +oo na
recorrência do enunciado para mostrar que, se (an)n2:i for convergente, então seu limite é igual a
Ja,. Para o que falta, use a desigualdade triangular para obter

para k > 2. A convergência de (an)n2:1 e a estimativa para lan - vai seguem do exemplo 29.

16. Sejam a1 = 1 e, para n > 1,


1
an = 1 + -----=-1-
1+ ---
•. + 1
(com n - 1 barras de fração). Então, an+l = 1 + a:, e uma fãcil indução permite concluir que
J < an < 2, para todo n > 1. A partir daí, mostre que

e use o resultado do exemplo 29 para concluir pela existência de l E IR tal que an ➔ f quando
n ➔ +oo. Por fim, fazendo n ➔ +oo em an+l = 1 + ...L, mostre que l = 1+2/s.
ª"

SEÇÃO 2.3

L Paraa segunda parte do item (a), observe que, se a < O < b, então a imagem do intervalo [a, b]
por /éa união (-oo, ¾)U {O}U {¾, +oo). Para o item (b), comece calculando a imagem do
intervalo [-!,O] por f,

498
2 Suponha que g é contínua, e seja g(xo) = e. Para x E / \ {:ro},segue do dcsig1mldodc tríangular
que lc - /(xo)I $ lc - g(x)I + lg(x) - /(xo)I = lg(xo) - g(x)I + IJ(.r) - J(:ro)I, Agora, use o fato
de que f e g são contínuas para concluir que lc - /(xo)I < e, para todo e> O.

3. Para os itens (a) e (b), use o resultado do problema anterior. Para os itens (c) e (d), use a
definição de continuidade para concluir que não existe valor de e que torne contínua a função
dada - esboçar o gráfico da função em questão pode ajudar.

4. Comece observando que o conjunto (xo - ó,xo + ó) x (f (xo) - t, f(xo) + e) é um retângulo aberto
centrado em Po(xo,J(xo)) e de lados paralelos aos eixos. Em seguida, lembre-se de que todo disco
centrado em Po contém um retângulo aberto centrado em Po e de lados paralelos aos eixos, e
vice-versa.

5. Observe que, em D, a função tg é o quociente das funções contínuas sen ecos.

6. Use a regra da cadeia para funções contínuas.

7. Idem à sugestão ao problema anterior.

8. Para o item (a), adapte, ao presente caso, a demonstração da continuidade da função raiz quadrada
em xo = O. Para (b), faça y = y'x e Yo = yÍXÕ e aplique o item (a) do problema 5, pãgina 417
para obter
IYn- YÔI
1y-yo 1 = .
yn-1 + yn-2yo + ... + YÔ-1
Por fim, para o item (c), use o resultado de (b) para adaptar, ao presente caso, a demonstração
da continuidade da função raiz quadrada em xo.

9. Use o resultado do problema anterior, a regra da cadeia e o exemplo 39.

10. Use a regra da cadeia para mostrar que J é contínua em xo, para todo xo I O. Em seguida, use
a definição de continuidade, juntamente com o fato de que lx sen ½I $ lxl, para mostrar que f é
contínua em O.

11. Fixado xo E [O,1], tome e= ½e suponha, por contradição, que f ê contínua em xo. Pela definição
de continuidade, existe ó> O tal que x E (O,1] e lx - xol < ó implicam lf(x) - f(xo)I < ½•Agora.
considere separadamente os casos xo racional e xo irracional, e use o resultado do problema 1.2,
pâgina 74, para chegar a uma contradição.

12. Para mostrar que fé descontínua em todo racional do intervalo [O,1), argumente como na sugt.'Stuo
dada ao problema anterior, utilizando o resultado do problema 1.2, página 74. Se i·o E [O.1) é
um número irracional e no E N, mostre que existe ó > O tal que as rcpresentn~õc-~frndon1\rias
de todo racional pertencente ao intervalo (xo - ó,xo + ó) têm denominadores mniorcs que o 0 ; t'm
o,
seguida, conclua que 1/(x) - /(xo)I < ~o, para todo x E [O,1] n (J·o - .i:o + ó).

499
SEÇÃO 2.1

1. Inicialmente, recorde que, pelo problema 1.11, pâgina 23, temos J(x) = f(l)x, para todo x E Q.
Em seguida, fixado x 0 E IRirracional, use o resultado do problema 1.2, pâgina 74, para garantir a
existência de uma sequência (an)n~I em Q, tal que an --+ x 0 quando n--+ +oo. Por fim, aplique
o teorema 44.

2. Comece utilizando a segunda hipótese do enunciado para mostrar que f é injetiva; para tanto,
tome x, y E lR distintos e considere a sequência divergente (an)n~l tal que a2k = x e a2k-1 = y,
para todo k ~ 1. Por fim, use as hipóteses sobre f para mostrar que 1- 1 transforma sequências
convergentes em sequências convergentes.

3. Dado f > O e sendo e a constante de Lipschitz de J, mostre que podemos tomar ô= ~ na defimçào
de continuidade uniforme.

4. Note que, para todo ô> O, existem x, y E IRtais que lx-yl < ô mas lf(x)- J(y)I ~ 1; por exemplo,
tomando x > O e y = x + ô, segue da fórmula do binômio que lf (x) - J(y)I = (x + ô)n - .e" ~
nôxn- l ~ nôx.

5. Se / fosse uniformemente contínua, ao f > Odo enunciado corre::iponderia um ô > Ocomo em (2. 7)
Agora, como lan - bnl --+ O quando n--+ +oo, poderíamos escolher n E N tal que lan- b,.I < ô,
portanto, deveríamos ter 1/(an) - J(bn)I < e, o que é uma contradição.

6. Use o resultado do problema anterior.

7. Para o item (c), considere g : [a, b] --+ IR tal que g(x) = M-~(z), para todo x E [a, b].

8. Basta tomar /: (a, b) --+1Rtal que /(x) = (r-a) 1(:r-b), para todo x E (a, b).

9. Inicialmente, observe que o problema de calcular a distância de P a Gf é equivalente ao problema


de minimizar a função contínua g : (a,b) --+ JR, dada por g(x) = J(x - x 0 ) 2 + (J(x) -y 0 ) 2 .
Agora, seja Q(xo, /(xo)) e, para O< ô< b-a, sejam Ró= (a+ô, J(a+fJ)) e S5 = (b- ô, /(b- ô)).
Use a condição do enunciado para mostrar que existe ôo > O tal que, para O < '5 < '50 , temos
RõP, SóP > PQ. A partir daí, conclua que basta minimizar g no intervalo (a+ 60 , b - '50 ]. Por
fim, aplique o teorema de Weierstrass para concluir o que se pede.

10. Para o item (a), aplique a desigualdade triangular para obter

Para (b), observe que !xi ~ 1 implica lxl $ lxl2 $ · •· $ lxln-l_O item (c) segue de (b). tomando
A = max { 1, a~,E"]:;la1 1}e observando que xn > O para x =/=O, uma vez que n é par. Por fim,
para o item (d), use o resultado de (e), em conjunção com o teorema 53 e o fato de que a0 = J(O).

500
Su; \< > '2.--,

11. O item (a) segue imediatamente de (2.11). Quanto a (c), por um lado temos

/(a) = f( n-++oo
lim Xn) = lim /(xn)
n-++oo
= n-++oo
lim Xn+l = a;

por outro, se {3 E IR for tal que / (/3)= {3, então as desigualdades la- .BI= 1/(a)- / (.B)I $ ela- PI,
juntamente com O <e< 1, garante que la - /31 = O, conforme desejado.
12. Pondo /(x) = ½(x + Jx 2 + 1), temos claramente f(x) < x para todo real x. Ademais, aplicando
a desigualdade triangular, obtemos

1 x2 - y2
lf(x) - J(y)I - X - y + --;::::::;==----;::=:===
2 J x2 + 1 + J y2 + 1
= !lx -yl 1+ x +y
2 Jx 2 + 1 + J y2 + 1
< !lx _ YI
- 2
(l +
✓x2
lxl+ IYI )
+ 1 + J y2 + 1
< lx-yl.

SEÇÃO 2.5

1. Para que o TVI fosse aplicável, deveríamos ter [O,2] contido no domínio de/, o que não é o caso.

2. Use o TVI em cada caso. Esboçar os gráficos das funções envolvidas pode ajudar.

3. Se y = ax+b é uma reta não vertical, i.e., tal que a-::/; O, basta provarmos que a função g: R.--+R.,
dada para x E IR por g(x) = x 3senx - ax - b, é tal que g(x) = O para infinitos valores reais de x.
Para o que falta, suponhamos a > O (o caso a < O pode ser tratado de modo análogo). Mostre
que, para todo k E N suficientemente grande, temos g(2kn) < O < g(2kn + ;); em seguida,
aplique o TVI.

4. Por contradição, suponha que existisse x E [O,1] tal que f(x) < O. Utilize o lema 48, juntamente
com o problema 1.3, 74, para chegar a uma contradição.

5. A prova desse resultado é uma adaptação do argumento da prova do teorema de Bolzano. :Ma.is
precisamente, suponha / = [a, b] (os outros casos podem ser tratados de modo análogo), fixe
a E X e defina A = {x E [a, b]; [a.,x] C X}, de sorte que A -::j;0. Se e = sup A, tome uma
sequência (an)n2!l em A tal que an --+ e e use a condição (ii) para mostrar que e E A. Se e < b,
use a condição (i) para escolher O < o < b - e (correspondente a x 0 = e) para concluir que
e+! E A, o que é uma contradição. Por fim, mostre, analogamente, que [a,xo]e X.

6. Como x = O não é solução, podemos reescrever a equação cm questão como f(x) = O, onde
f : (O,+oo) --+ IR é a função dada por f(x) = I:~ 1 J-fi
+ 7 - n. Assim, devemos mo.,trnr que

501 ~
J p~sui exatamente uma raiz positiva. Para tanto, como a função x H /ti+ 7 é contínua e
decrescente, e como J é uma soma finita de funções desse tipo, segue que f ê também contínua e
decrescente, o que implica dizer que J tem no máximo uma raiz positiva. Por outro lado, ternos
f(l) = E:=l Jl + a, - n > O. Também, /ti+ 7 < l se x > max{ ./2,2a,}, de modo que
f(x) < O se x > max{ ./2,2a 1, ... , 2an}- Assim, segue do TVI que J possui exatamente uma raiz
positiva.

7. Seja f : [O,1] ---+IR a função dada por J(x) = ¼I:~ 1 lx - x1 I. Basta garantirmos a existência de
x E [O,1] para o qual f(x) = ½- Para tanto, note que J(0) = ¾I:~ 1 Xi e J(l) = 1 - ¾I:~ 1 x,,
de modo que /(O)+ /(1) = 1. Se /(O) = /(1) = ½,nada hâ a fazer. Senão, podemos supor, sem
perda de generalidade, que /(O) < ½ < /(1). Nesse caso, como / é contínua, segue do TVI a
existência de O< x < 1 tal que f(x) = ½-

8. Primeiro, suponha que /(O) = O. Com x = O, obtemos J(l) = f(0)/(2) + J(l) = O. Fazendo,
agora, x = 1, obtemos /(2) = /(1)/(3) + /(2) = O. Prosseguindo do mesmo modo, concluímos
que f(n) = O, para todo inteiro positivo n. Suponha, pois, que f(0) > O. Então, segue de
/(0)/(2) + /(1) = O que f(l) e /(2) devem ter sinais contrârios. Mas aí, o TVI garante que deve
haver um real a E (1, 2) tal que J(a) = O. Argumentando como no caso /(O) = O, conclua que
f(a + n) = O, para todo inteiro positivo n.

9. Faça x = 1000 e, em seguida, use o TVI para concluir que a imagem de f contêm o intervalo
[~, 999]. Use novamente o TVI para garantir a existência de xo E R tal que f(xo) = 500 e, por
fim, faça x = xo na relação do enunciado.

10. Use a relação satisfeita por / para obter uma contraclição ao TVI.

11. Para o item (a), o fato de f ser uma bijeção segue do exemplo 37; então, o teorema 61 garante que/
é crescente ou descrescente. Para o item (b), note primeiro que, por (a), temos f(0) = Oe /(1) = 1.
Em seguida, se existe x E (O,1) tal J(x) < x, então o fato de / ser crescente, juntamente com a
lúpótese do problema, garante que x = J(J(x)) < f(x), o que é uma contradição. Analogamente,
não podemos ter J(x) > x, de forma que f(x) = x é a única possibilidade. Por fim, para (c),
note primeiro que, por (a), temos J(0) = 1 e J{l) = O. Em seguida, fixe a E (O,1) e defina J
pondo /(a) = a e tal que suas restrições aos intervalos [O,a] e [a, 1] coincidam com funções afins.
É imediato checar que J(J(x)) = x, para todo x E (O,l].

12. Defina a função auxiliar g : IR ---+IR tal que g(x) = E;- 1 f(x + jr), com r > O a ser escolhido
posteriormente. Em seguida, se f(xo) < O < J(xl), aplique o lema de permanência do sinal para
/ para mostrar que r pode ser escolhido de forma tal que g(xo) < O < g(x 1). Por fim, aplique o
TVI.

13. Pelo exemplo 58, a funçfw g tem um ponto fixo xo. Se x 11 = g(x 11_ 1) para n ~ 1, mostre que a

502
sequência (xn)n~l é não decrescente e tal que J(xn) = Xn, para lodo n ~ 1. Se Xn 4 a, use o
teorema 44 para mostrar que J(a) = g(a) = a.

SEÇÃO 3.1

1. A ideia é adaptar, ao presente caso, a demonstração da unicidade do limite de uma sequência,


dada na proposição 14. l\Iais precisamente, suponha que L e M são reais distintos e que /(x)
converge simultaneamente para L e A/ quando x 4 xo. Como E= IL - Ml/2 > O, pela definição
de limite existiriam reais 61,62 > O tais que x E J, O < lx - xol < 61 =} lf(x) - LI < E e
x E I, O< lx - xol < ô2 =} lf(x) - Ali< e. A partir daí, faça 6 = min{61,62}, tome x E J tal
que O< lx - xol < 6 e use a desigualdade triangular para chegar a uma contradição.

2. Suponha que fosse L > AI, e faça e= L-/ 1 > O. Tome 6 > O tal que x E J e O < lx - xol < ô
implique simultaneamente IJ(x) - LI < E e lg(x) - MI < e. Por fim, aplique a desigualdade
triangular para chegar a uma contradição.

3. Faça indução sobre n. O caso inicial n = 2 é exatamente o conteúdo dos itens (a) e (b) da
proposição 7.

4. Para limites laterais à direita, por exemplo, copie o enunciado e a prova da proposição 7, trocando.
quando conveniente, limx-+xopor limx-+xo+e O < lx - xol < 6 (resp. O < lx - xol < 61,62) por
xo < x < xo+6 (resp. xo+61, xo+62).

5. A implicação =} e a última parte seguem do fato de que tanto xo - 6 < x < xo quanto xo < x <
xo + ô implicam O < lx - xol < 6. Reciprocamente, se limx-+xo+ J(x) = limx-+xo- J(x) = L e
e > O é dado, tome 61, 62 > O tais que as condições xo - 61 < x < xo e xo < x < xo + 6 impliquem
lf(x)-LI < e; tomando ô= min{61,62} > O, conclua que O< lx-xol < 6 implica lf(x)-LI < E.

6. Suponha que lim f(x) = L > Oe lim g(x) = +oo (os demais casos são completamente análogos),
x-+xo x-+xo
e seja dado M > O. Pelo lema de permanência do sinal, existe 61 > O tal que x E J, O< lx-xol <
61 =} J(x) > ½;pela definição de limite infinito, existe 62 > O tal que x E J, O< lx - xol < 61 =}
g(x) > ~- Então, tome 6 = min{61,62} e conclua que, para x E J tal que O< lx - xol < 6,
temos f(x)g(x) > M. Para o que falta, comece observando que o resultado análogo para. limites
no infinito (em +oo, por exemplo) diz que, se lim J(x) = L > O e lim g(x) = +oo, então
x-++oo :t-++oo
lim f(x)g(x) = ±oo. Para sua demonstração, copie a demonstração do caso inicial acima,
:r-++oo
trocando, quando conveniente, O< lx - xol < 6 por x > A.

7. Suponha, inicialmente, que a, b, L, M E IR (os casos em que a = -oo, b = + , L = - ou


J..1= +oo são totalmente anãlogos). Dado e> O, tome 6 > O tal que a< x < a+6 e b-6 < y < b
impliquem, respectivamente, f(x) < L + E e g(x) > Aí - €. Em seguida, aplique o TVI pnrn
concluir que [L + e, M - e] C Im (!).

503
Em todos os itens você terá de utilizar o limite fundamental da Trigonometria após executar

1 + cos x; para o item (c), escreva =~


alguma operação auxiliar. Para os itens (a) e (b), multiplique o numerador e o denominador por
g~~ = ~. seo2~:c) • seo3<3x); para (d), multiplique o numerador
e o denominador por J; para (e), use (d), juntamente com o fato de que cos (;x) = sen {; - 11{);
por fim, para (f), use a identidade 1 - y = 1 !~!:
2 com y = ~cosx e aplique, em seguida, o
resultado do item (b).

9. Para a primeira parte adapte, ao presente caso, a demonstração da proposição 10. Para cal-
cular os limites pedidos, utilize o resultado da primeira parte, juntamente com o fato de que
lsen xi, 1cos xi $ 1, para todo x E IR.

10. Dado E:> O, tome ô> O tal que x E J e O< lx - xol < ô impliquem lf(x) - LI < e Em scgmda,
tome no E N tal que n > no ⇒ lan - xol < ô. Então, para n > no, ternos O < la,i - xol < ô
(uma vez que an =f.xo, para todo n ~ 1), de sorte que IJ(an) - LI < E: Para a segunda pmtc,
analise o comportamento de f ao longo das sequências (an)n~l e (bn)n~l, tais que a,i = 2,~1r e
bn = 2n1r~1r 72 , para todo n ~ l.

11. Para o primeiro caso, se limx-++ooU(x) - (ax + b)) = O, então

O= lim (l(x) - (ax + b)) = lim (J(.r.) - a - b) = lim J(.r) - a.


x-++oo X ..r.-.+"0 X .l' ..t-++oo X

e
O= lim (J(x) - (ax + b)} = lim (J(x) - ax) - b.
x-++oo .r-++oo
O segundo caso (i.e., aquele de uma função J: (-oo, A) ➔ IR) é totalmente análogo ao primeuo,
e será deixado ao leitor.

= ax + /3é assíntota oblíqua do gráfico da função em questão quando x ➔ +oo,


12. No item (a), se y
então a= limx-++oo f~x) = limx-++oo{1 + -1,r)= 1 e /3= limx-++ooU(x) - ax) = limx-++oo½= O.
Analogamente, y = x também é assíntota oblíqua do gráfico da função do item (a) quando
x ➔ -oo. No item (b), se y = ax + /3é assíntota oblíqua do gráfico da função em questão quando
x ➔ +oo, então a = limx-++oo/(xx) = limx-++oo -º- 1 - ~ = -º-e
Cl
J X Cl

lim (J(x) - ax) = lim ~ (Jx


/3 = x-++oo 2 - a2 - x)
x-++oo a
2
= - lim -b ( ---;=::;;:::==:;;:---
a ) = O
x-++oo a Jx2 - a2 + x •

Analogamente, y = -~x é assíntota oblíqua da função do item (b) quando x ➔ -oo. Para o
item (e), observe que limx-++oox sen ½ = 1. Por fim. em todos os itens, deixamos a análise da
existência de assíntotas horizontais e verticais a cargo do leitor.

504
13. E cr va uc i am n

✓ r= r= X+ Jx + ,Ji- X
✓l + ~1
X + +VX - V X' = - --;==~::.::.::.'=.'=..:---
✓X + ✓X + ,Ji + .fi ✓l + l:r
1
+ '7z' +l

Em guida faça x

14. Podemo upor que > 2 d forma qu 1}f:,)> ~~2:f.


Por ou ro lado k E é tal que 21c> n,
n ão /(é))
/ :r
< /((
/ :z:
r = ílk-l
J=l
1(2'+1:r)
/(2J:r) •
Por an o ,

J(2x)
J(x)
< f(nx) <
f(x)
IT f(2 1 +lx)
J(Vx)
J=l

b t faz r x ➔ + n d igualdad cima u ilizando o teor ma do confronto para concluir


o qu p de.

15. p t do i m ( )
induç-o. Para o i m (b) para a primeira parte de (c),
2 ( om Bn no lugar de 2a), juntam nte com o fato d qu
an = 2nJ2 - bn. P r fim, p nd p d (c), calcul 81, cr va an = 201 · sen9 1f}") e ~j
pliqu limi tri nom tri o fundam n al .

.2
1. P r o i m ( ), ob rv qu g(:rJ=!~:ro) = e· /(:rJ=~~:ro). Para o item (b), v ja qu h{:rJ=~i:ro) -
/(:r)-/( o)
.r.-.ro

2 P r x no d rnínio g g(.r)-g( ~) = e • /(ex)- /(:ro). Agora fazendo y = y(x) = ex t m


m :r-~ e
ex-~ '
u y xo quando x ~ d modo qu

lim g( ) - 9:r(~) = e lim f(y) - f(xo) = cf'(xo).


:r-+!ile - :it
e
y xo y - xo

3. função are n, om x E (-1,1) j y E(-;,;) l qu ny = Como n'y =


y f. O, gu do t r ma 27 ( om o p p i d x y ro a.d ) qu funç- n d riv
m x, om
, 1 1 1 1
ar D = -- = -- = --;:=====;= - ---;::==;
n 'y co y J1 - n 2y ✓1 - x 2 -

argum n o para função arcco é inteiram nt anâlogo, rá d ix o l it r.

o r ultado do probl m an rior .


5. Para a. segunda parte de (a), veja que J'(l) = n e g'(l) = m, de forma que /'(1) = g'(l) se, e
só se, m = n. Para o item (b), temos f(xo) = 1- 1 (xo) = Yo e J'(xo) = 1
u-
)'(xo) (uma vez que
as tangentes aos grâficos de / e 1- 1 em A coincidem). Por outro lado, segue do teorema 27 que
(/- 1)'(xo) = /'(~o) e, daí, J'(xo) = /'(~o). Logo, J'(xo)f'(yo) = l.
6. Expresse o quociente de Newton de x ....+f(x) em x 0 em função do quociente de ewton de/ em
XQ.

7. Comece escrevendo
f(bn) - f(an) _ J(bn) - J(xo) . bn - Xo + f(xo) - f(an) . xo - an
bn - an - bn - Xo bn - an Xo - an bn - an •
Então, use a desigualdade triangular para obter
f(bn) - f(an) _ LI $1 J(bn) - f(xo) _ LI . bn - Xo+
1 ~-~ ~-~ ~-~
+ 1/(xo) - J(an) -LI. xo - Gn
Xo - ªn bn - Gn
$1 J(bn) - J(xo) _ LI + 1f(xo) - J(an) _LI.
bn - Xo XO - an

8. Para o item (a), note que a reta tangente ao gráfico de / no ponto (an, f(an)) tem equação
y - f(an) = /'(an)(x - an); em seguida, faça y = O e calcule o valor correspondente de x. Para
o item (b), faça f(x) = x 2 - a e use o resultado do item (b) do problema 2.1 para concluir que
an+l = an - ( ª~:ª) = ½( an + O:) •
0

9. Para o item (b), escreva o quociente de Newton de f em xo como


JR 2 - x 2 - JR 2 - xi l (R 2 x 2 ) - (R 2 - x~)
-
x-xo - x - xo • J R 2 - x 2 + J R 2 - xi
x+xo .

em seguida, faça x ➔ xo. Para (c), seja r a reta que passa por A e é perpendicular ao raio OA. A
proposição 60 ensina que o coeficiente angular de r é igual a - ,!i,onde m é o coeficiente angular
de ÕÃ. Mas, como m = /(zo),
xo
temos que o coeficiente angular de r é - ~,x.
/(zo)
Por outro lado,
segue imediatamente de (b) que - if:o) = f'(xo), conforme desejado.
.
10. Para a primeira parte, como o ponto (xo, yo) pertence ao grâfico de f, devemos ter y 0 = J(x 0 ).
Agora, como r tangencia o grâfico de/ em (xo, Yo), seu coeficiente angular deve ser igual a J'(xo);
por outro lado, como r também passa por (a, b), seu coeficiente angular deve ser igual a ~- xo-a
Para o que falta, se uma reta passa por (1, -1) e é tangente à parábola dada no ponto (x 0 , y 0 ),
segue da primeira parte e do problema 2.1 que
%2
Yo= ~
{ Yo - ( -1) = ~ (xo - 1)

506
Portanto, =}+ 1 = !f (xo - 1) e. daí, Xo = 1 ± v'5,Yo= 3
±/ 5.
11. Escolha um sistema cartesiano tal que F(O, ~) e d seja a reta y = -~. Como Q(x,y) E P se, e s6
se, FQ = dist(Q; d), um câlculo simples fornece y = ;; como equação da parábola em questão.
+--+
Agora, sendo P(xo, -~) e Q(x, ;;), conclua, a partir do problema anterior, que PQ é tangente a
P se, e só se,
~ (~x2)
= ~ - (-~)
dx 2p x - xo
ou. o que é o mesmo (cf. problema 2.1), x 2 - 2x 0 x - p 2 = O; sendo o e /3 as raízes dessa
equação de segundo grau, temos A(o, ;;) e B(/3, ~). Por outro lado, o coeficiente angular de
02
+--+ -(-P) 2 2 2.%
AF é m = 2P o-O 2 = ~ ~ 2po = ~.
2 2
2po • ~las • como o - 2xoo - p = O' segue quem= p
+--+ +--+ +--+
Analogamente, o coeficiente angular de BF também é igual a~, de forma que AF = BF ou, o
que é o mesmo, A, B e F são colineares.

SFÇÃO 33
1 Aplique as fórmulas de derivação de somas. produtos e quocientes, em conjunção com os resultados
dos exemplo desta seção e da cção anterior.

2. Para x E (O,+ ), defina h(x) = ifx, de sorte que, pelo exemplo 28, h é derivável em todo x > O,
com h'(x) = ¼x¼- 1 Como g(x) = (ho /){x) e/ é derivável em xo, a regra da cadeia garante que
9 é derivável em xo, com g'(xo) = h'(f(xo))f'(xo) = ¼/(xo)¼- 1J'(xo).

3. Aplique as fórmulas de derivação de somas, produtos e quocientes, em conjunção com a regra da


cadeia e com o resultados dos exemplos desta seção e da seção anterior. Para os itens (e) e (f},
revise o resultado do problema 2.3, página 148.

•I Se r = 1;:,com m, n E Z♦ , então /(x) = f/xm. Para x > O, seja g(x) = y'x, de sorte que g é
derivável (pelo exemplo 28). Como x ~ xm é derivável (para x =/=O se m < O) e f(x) = g(xm),
a regra da cadeia garante a derivabilidade de /, com /' (x) = g' (xm) • mxm- l. Então, novamente
pelo exemplo 28, temos

! '( ) = n-1 • (
X X
m)l/n-1
• mx m-1 = m
- • m(1-1)+m-l
n
n
X = T • X
~-1
n = TX
r-1
.

5. Para a primeira parte, derive ambos os membros de (3.22} e, em seguida, divida ambos os membros
do resultado por J(x). Para a segunda parte, derive ambos os membros da igualdade f(x) =
a(x - o 1 )(x - o 2 ) ... (x - an) e, em seguida, divida ambos os membros do resultado por f(x).

6. Escrevendo J(x) = 2+ 7(x-3)- 1 , obtemos, com o auxílio da regra da cadeia, J'(x) = -7(:r-3)- 2,

f"(x) = 14(x -3)- JC3)(x) = -42(x -3)-


3

, JC )(x) = 168(x-3)-
4

, e JC )(x) = -8-tO(x -3)-


4
5 5
6

Portanto, JC5 )(Q) = -840 • 3- 6 = - ~g.


507
• \ , 1 1 , 1, 1 1: S 1 <; 1 -.. I < >1 " \< ,-.. 1,!{( >111I· \ I \ ...._

7. Ponha fi(x) = v'x+l, h(x) = Jx + Jx+T e, mais geralmente, para n ~ 2 defina

com n raízes quadradas. Para n ~ 2, temos fn(x) 2 = x + fn-i(x). Portanto, segue da regra da
cadeia que 2fn(O)J~(O) = 1 + /~_ 1 (O) ou, ainda, 2J~(O) = 1 + f~-l (O), uma vez que fn(O) = 1.
Agora, corno Jf(O) = ½,pode-se utilizar a recorrência acima nove vezes para obter Jf0 (0) = ~g~~
Alternativamente, observando que J~(O) -1 = ½(!~_1 (0) -1), obtemos J~(O) - 1 = 2 n~I (/f (O) -
1) = - 2~, de forma que /~(O) = 1 - 2~.

8 • F~ra
~
indução sobre k. Para o passo de indução, derive x ~ P.1.(:r)
(l+x2)k utilizando a fórmula de
derivação de um quociente, juntamente com a regra da cadeia.

9. Para o item (a), derivando em relação a x com o auxílio da regra da cadeia, obtemo· 4.r3 +.iy3~ =
O; em seguida, faça x = xo e y = Yo para obter J'(x 0 ) = ~(xo)
,..,.
= -~llo = -!, uma vez que .co = 1
e Yo = 2. Para os itens (b) e (c), argumente de forma análoga.

10. Suponha que a reta y = ax + b tangencia as parábolas do enunciado nos ponto de abscissas o r
/3, respectivamente. Então, o-2 =ao+ b e 2 + (/3- 3)2 = a/3 + b, no passo que a= f;.x 2 L=o = 2o
e a = -iz,x2 j (2 + (/3 - 3)2) = 2(/3 - 3)
x=/3
Portnnto 1 o = /3 - 3 e b = a2 - 2o • o = -o 2,

b = 2 + (/3 - 3)2 - 2(/3 - 3) • /3 = -{3 2 + 11, de forma que o 2 = /32 - 11 Resolvendo o sistema
formado pelas duas equações em o e /3 assim obtidos, chegamos a o = ½e f3 = 13 A partir daí, °
calculamos facilmente a = } e b = -!,
de forma que a tangente comum às parábolas é a reta
_ 2X l
Y - 3 - g·

11. Para o item (b), resolva.o sistema formado pelas equações ~ + ~ = 1 e~ - fi = 1, conclumdo
que o mesmo tem soluções graças ao resultado do item (a). Para (c), analisemos o caso em
que P(xo, Yo), com xo, Yo > O (os demais casos podem ser tratados de modo anâlogo). Mostre
(utilizando a regra da cadeia, derivação implícita ou diretamente o resultado do problema 3.2)
que os coeficientes angulares das tangentes a E e 1{ em P são dados, respectivamente, por -~a . ~
yo
b'2
e ã!'1• ~. Conclua, então, que basta mostrarmos que õ'T"ã11•
t,2br2
7a= 1. Por fim, use o resultado
x2
de
(b) para mostrar que essa igualdade realmente é verdadeira.

12. Para o item (b), tome g igual à soma das funções x ~ J(x - xk), onde f ê como em (a).

13. Adapte, ao presente caso, a discussão do exemplo 40.

14. Inicialmente, observe que a1 + 2a2 + · · · + nan = J'(O). Em seguida, utilize o fato de que
1/(x)I $ lsenxl, juntamente com J(O) = Oe o limite trigonométrico fundamental, para provar que
1/'(0)I $ 1.

508
15. Sendo p > O o período de J, mostre que J(k)(p) = J(k)(O), para todo inteiro k 2: O. Conclua que,
para O $ k S n- 1, tais equações fornecem um sistema linear de equações nas incógnitas ccJe(jp),
1 $ J S n, cuja única solução é cos(jp) = 1, para todo 1 S j S n.

16. Inicialmente, verifique que, para x E (a, b), a discussão que precede o lema 35 nos permite escrever,

J(x) J'(a) + ~
g(x) = g'(a) + =~l'
COm

llffix-+a ~
x-a e
liffix-+a ~
:r-a = o·

SEÇÃO 3.4

1. Como J' = g' se, e só se, (J - g )' = O, basta mostrarmos que, se J' = O, então f é constante.
Suponha, pois, que J' = O em I, e fixe a < bem I. Como J é derivável em I, temos que J é
contínua em (a, b] e derivável em (a, b). Portanto, pelo TVM de Lagrange, existe e E (a, b) tal que
f(bt~(a) = J'(c). Mas, como J'(c) = O, temos J(b) = J(a), e f é constante em J.

2. Se g(x) = J"(xo)(x - xo), use o resultado do problema anterior para concluir que J' - g é
constante e, daí, que J'(x) = J'(xo) + J"(xo)(x - xo), para todo x E J. Em seguida, se h(x) =
J'(xo)(x - xo) + /"ro> (x - xo)2 , use novamente o resultado do problema anterior para mostrar
que J - h é constante e, daí, que J(x) = J(xo) + h(x) = f(xo) + J'(xo)(x - x 0 ) + f"~xo) (x - x 0 ) 2 ,
para todo x E J.

3. Use a desigualdade do enunciado para concluir que fé derivável, com f'(x) = O para todo x E R.
Em seguida, aplique o resultado do problema anterior.

4. Aplique o teorema de Rôlle ao polinômio ~x + Tx 2 + • • • + :.; 1 xn+l.

5. Use o TVI para concluir que f(x) = x 2 - xsenx - cosx tem pelo menos duas raízes reais, uma
no intervalo (-i,0) e outra no intervalo (O,~). Em seguida observe que a derivada de f tem
somente um zero, e aplique o teorema de Rôlle para concluir que a equação J(x) = O não pode
ter uma terceira raiz real.

6. Aplique o teorema do valor médio de Lagrange.

7. Em cada um dos itens, aplique a regra de l'Hôpital para §.


8. Aplique a regra de l'Hôpital para §.
9. Aplique duas vezes a regra de l'Hôpital para §.

509
10 Para o item (a), use o TVM de Lagrange duas vezes para obter J(x)-(x-a)f'(x) = - J"(ç)(x-a)2.
para algum ç E (a, x); agora, use a continuidade de J", juntamente com o fato de que J"(a) # O,
para garantir a existência de e E (a, b) tal que J" # O em [a,e). Para o item (b), denote
-(x-a)/'(x) e use a regra d e L'H~
g (x ) = J(x ~x-a)/(x) • l d uas vezes para obt er limz➔a g (x ) = - 2/"(a)
op1ta l'(a) •

11. Aplique a versão usual da regra de l'Hôpital às funções F, G: (O,¾) ➔ IR,dadas por F(x) = !(¾)
e G(x) = g(½)-

12. Aplique n vezes a versão da regra de l'Hôpital dada pelo problema anterior.

13. Inicialmente, observe que ,f;(~/(x))


i[iX
= '!;(xf(x)) = f(x)
+ xf'(x). Agora, por nossns h1p6tcscs e
pela extensão da proposição 7 a limites no infinito, temos que limx-++ooU(x) + xf'(x)) existe
Portanto, pela versão da regra de l'Hôpital dada pelo problema 4.11, temos

lim J(x) = lim xf(x) = lim -Í;(:J(x)) = lim (J(x) + xf'(.r))


x-++oo x-++oo x :r➔ +oo c[iX x-++oo

= x-++oo
lim J(x) + lim (xf'(x)).
x➔+oo

Logo, limx-++oo(xf'(x)) = O.
14. Use o TVM de Lagrange, em conjunção com o resulta.do do problema 4.11, pâgina 111.

15. Tome, em I \ {xo}, uma sequência {xn)n~l, com Xn ➔ xo. Se Xn > xo {resp. Xn < xo), então,
aplicando o TVM de Lagrange ao intervalo [xo, xn) (r~p [xn, xo]), conclua pela existência de
_ Yn E (xo, Xn) (resp. Yn E (xn, xo)) tal que

(B.l)

Mas, como Yn ➔ xo e Yn # xo, temos por hipótese que f'(Yn) ➔ L. Portanto, segue de (B.l) que
/(xn)- /(:ro) ➔ L. O raciocínio acima mostrou que
Zn-%0

Mostre, argumentando por contradição, que isso é suficiente para garantir que limx-+xo/(z)-/(:ro)
x-xo
existe e é igual a L.

16. Supondo que uma tal / exista, comece escrevendo a relação do enunciado como f(z+~ f(x) <
• 1n,;f>.A partir daí, conclua que J'(x) :5:-1, para todo x > O. Em seguida, use o TV1I de
Lagrange para mostrar que f(x + 1) :5:f(x) - 1, para todo x > O. Por fim, use essa última
desigualdade para mostrar que / deveria assumir valores negativos, o que não é o caso.

510
17. Comece usando o TV1I de Lagrange para mostrar que existe O< ó< 1 tal que 1/(x)I< lxl, para
lxl < ô. Então, obsen·e que 1/'(x) ~ lxl. para lxl <ó.e use novamente o TVM de Lagrange para
mostrar que 1/(x)I ~ lx12 , para lx < ó Itere esse argumento, mostrando que (com o mesmo 6)
1/(x) ~ lxln, para lxl < ô e todo n E N. A partir daí, conclua que J(x) = O, para lxl < ô. Em
seguida. estenda o argumento acima, mostrando que, se f(xo) = O, então existe ô > O tal que
/(x) = O. para lx - xol < ô. Por fim. use o resultado do problema 5.5, página ll9, para concluir
que /(x) = O, para todo x E IR.

SEÇÃO 3.5
1. Considere a função quadrática /(x) = ax 2 + bx -4- e, tal que a > O (o caso a < O é totalmente
análogo). Como /'(x) = 2ax + b e a > O. temos J'(x) < O se, e só se, x < -; 0 e J'(x) > O se, e
só se. x > -fa. Logo.fé decre.-;cente para (-00,-; 0 ) e crescente em (-fo,+oo), de sorte que
.r = - ,! é o único ponto de miiúmo de /, com J (- 2~) = - t.
2. Em todos o itens, aplique o te::;teda primeira derivada. em conjunção com o estudo da primeira
variação. Para o item (a), por exemplo, veja que, para x > O, temos J'(x) = 2 }x~; 3
2~~ 6 ) 2 .
Po1Lanto. x = 73 é o único ponto critico de /. com / crescente em [O, 73]
e decrescente em
[73, + ). Logo, x = 7:i é o único ponto de mâ..ximode f, com / (
73)
= 32 ¼. Agora, observe
3

que /(O) -= O e j('l·) > O pnro .r. > O dt• formn que :r = O é o único ponto de mínimo para/. Os
clc1rnrn,1ten.:;podem :;c1 trntndo~ de modo nnúlogo.

3. lnicinlmente, ob::-crv' que f niio tem ponto:, de máximo ou mínimo global, uma vez que (cf.
exemplo 16) limz-+- /( r) - - e lim.z:-++ f(x) = +co Agora, temos J'(x) = 3x 2 + 2ax + b,
com 6. - 1(a2 - 3b) Se 6. < O, então J'('i:) > O para todo x E IR, de forma que / é crescente em
todn n rct,1. e~= O. então J'(x) =O~ x = -J, e J'(x) > O para x =/-~;portanto, nesse caso
-1 é o úmco ponto crítico de J, mas J ainda é crescente em toda a reta. Por fim, se~ > O, então
J' tem exatamente duru; raízes reais, X1 < x2; ademais, para x < x 1 ou x > x 2 temos J'(x) > O,
enquanto J'(.--c)< O para x1 < x < x2, portanto, f é crescente em (-00 1 x 1], decrescente em
[.r1, .r.2Ie no,·a.mente crescente em [x2, +oo), de forma que x1 e x2 são, respectivamente, pontos
de má."11110 e mim mo local para /.

:J Pru-a a primeira parte, seja f: [O,-roo)➔ lR dada por f(x) = senx-x.


Então, f'(x) = cosx- l $
O de -.orte que fé não crescente em [O,+oo). Portanto, para x ~ O temos f(x) ~ J(O) = O. Para
a ~egunda parte, seja f: [O.+oo) ➔ IRdada por J(x) = cosx-1 + x;. Pela primeira parte, temos
J' (x) = - cn x + x ~ O. para todo x ~ O, de forma que f é não decrescente em [O,+oo). Assim,
pru-a x ~ O, temos f(x) ~ f(O) = O.

5 Verifique que APQ ~ DQR, e conclua a partir daí que, se AQ = x, então DR = x(2 - x).
Então. calcule PQ = Jx 2 + l e QR = (2 - x)Jx 2 + 1, de forma que PQR tem área igual a

511
½PQ • QR = ½(2- x)(x + 1). Por fim, estude a primeira variação da função f : (O,2] ➔ IR, tal
2

que /(x) = (2 - x)(x


2
+ 1), para concluir que o máximo de f ê obtido quando x = ~-

6. Se A(a:, J(a:)) e A'(x, então, graças a (A.36), a: é o ponto de mínimo da função d: (a, b) ➔ IR
J(x)),

dada. por d(x) = (x - xo)2 + (J(x) - yo)2 . Pelo teste da primeira derivada, temos O= d'(a:) =
2(0: - xo) + 2(/(a:) - Yo)f'(a:), de modo que ou a: = xo e J'(a:) = O (uma vez que P ~ GJ ), ou
a::/: xo e J'(a:) ( 1 ~2~;'º)
= -1. Use, agora, o resultado da proposição 60.
+--+
7. Segue do problema. anterior que AB é perpendicular às tangentes traçadas ao gráfico de f em A
e ao gráfico de g em B; então, tais tangentes são paralelas.
2
8. Se A(a,a
2
) e B(b, 1-(b-3) então AB = (a-b)2+(a 2 -l+(b-3)2) 2 . A condição f'(a) = g'(b)
2
),
2
dá-nos b = 3 - a, de modo que AB = 4a 4 - 12a + 10; portanto, temos que minimizar a. função
h(x) = 4x4 - 12x + 10. O teste da primeira derivada garante que a= ~ é o único ponto de
mínimo de h, e um cálculo fácil garante que, nesse caso, AB =j 10 - 9 ~-

9. Sejam AH= h, BH = l, H ÂB = a: e Bê=


O. Aplique o item (c) da proposição 71 para obter
ar+8 -t ar - E 'd2hl 'Uai O al ul J'(l) -- 2h.(h2_3,2)
3t2 para > , c c e
8 - t
tg - l+tg ar+ ·tgar - m segw a, se J(l) -- h2+
11.2+317•
l
(h.~+Jl 2)2
e conclua. que o ponto crítico l = 73
de f ê o ponto de máx.imo global.

10. Se 8 = BÂD = ADC, ê imediato que O< O< i· Também, AD= a+ 2bcos0 e, sendo h a. altura
de ABCD, temos h = bsen0. Portanto, A(ABCD) =½(AD+ BC)h =(a+ bcos8)bsen8, e
basta. maximizar a. função/ : (O,i) ➔ R, dada por J(8) = (a+ bcosO)senO. Para tanto, use
a relação fundamental da. Trigonometria, juntamente com o item (a) do corolário 72, para obter
f' (8) = 2bcos2 8 + a cos 8 - b e f" (O) = -2b sen (28) - ab sen O. Agora., note que f" (8) < O para
todo 8 E (O,~), e verifique que 8 = arccos (a+Ja52!.@+a) é o único ponto crítico de f. Por fim,
a.plique a versão do corolário 56 a funções estritamente côncavas.
- - OP 2 sen sen{J -2
11. Nas notações do exemplo 57, temos PX • PY = senBseo(:+P+B)· Como OP sena:sen/3 é cons-
tante, concluímos que P X • PY ê mínimo se, e só se, f (8) = sen 8 sen (a:+ /3+ 8) é máximo, onde
/ : (O,,r - a: - /3) ➔ R. Como f ê derivável, o teste da primeira derivada garante que, se existir
um ponto de máximo para f, ele será crítico. Mas, como

/'(8) = cos 8sen (a:+ /3 + 8) + sen0 cos(a: + /3 +O)= sen (a:+ /3 + 28),

temoe que (J é crítico se, e só se, a:+ /3+ 28 = 1rou, o que é o mesmo, se, e só se, 8 = 1r- a: - /3- 8,
quer dizer,se, e só se, OXY é isósceles de base XY. Resta verificar que, sendo OXY isósceles de
bMe XY (ou, o que é o mesmo, sendo 8 = ½(1r- a: - /3) (que é o único ponto crítico de!), temos
..iroeote que/ atinge seu valor máximo. Para tanto, basta notar que, se O< 8 < ½(1r- o - /3),
_..O< a+fJ+28 < 1r, de sorte que !'(8) = sen (o+/3+28) > O;se ½(1r-0:-/3) < 8 < 1r-a-/3,
hl•le 211< G + fJ + 2IJ< 2,r, de sorte que J' (8) = sen (a: + /3 + 28) < O. Logo, a proposição 53

512
garante que fé crescente em (O, ½(rr- o - /3))e decrescente em (½(1r- o - /3),1r - o - /3), de
sorte que 0 = ½(1r - a - /3)é o único ponto de máximo global de /.

12. Fixe um sistema cartesiano de coordenadas tal que A(O,a) e B(l, b), e seja P(x, O), com x E R.
Conclua, com o auxílio da proposição 55, que é suficiente mostrar que a função f: R-+ R, dada
por f(x) = (x 2 +a 2 )((x-l) 2 +b 2 ), tem um único ponto de mínimo global quando l :=;J2(a 2 + b2).
Calcule /(x) = x 4 - 2ix 3 + (a 2 + b2 + l 2 )x 2 - 2a2 lx + a 2 (b2 + l 2 ), J'(x) = 4x 3 - 6lx 2 + 2(a 2 + b2+
l 2 )x - 2a 2 l e f"(x) = 12x2 - l2lx + 2(a 2 + b2 + l 2 ). Em seguida, mostre que f' tem uma raiz no
intervalo (O,l) e examine sob que condições temos J"(x) ~ O, para todo x E IR.

13. Conclua que é suficiente mostrar que a função f(x) = (.r+;r+l, x > o,atinge seu valor mínimo
em x = n. Para tanto, estude a primeira variação de /.

14 Mostre, inicialmente, que y = ½( k( J1 + x 2 - x) + ¼(J1 + x 2 + x)). Em seguida, estude a vari-


ação da função f: IR-+ R tal que f(x) = x + y, onde y é dado em função de x como acima.

15 Use a desigualdade entre as médias para dois números reais (cf. (A.5)) para concluir que a
f
expressão acima é maior ou igual que f(x + y), onde f(t) = t + + ffepara t > O. Em seguida,
estude a primeira variação da função /.

4
16. Fazendo o - sen e /3 = sen f,
mo.·tre que basta analisar a igualdade /(o) - f(/3), onde
f(x) = (l-~:) 312 . para x E (O,1). E:stude, então, a primeira variação de f.

17. Escreva f(x) - J'(x)- J"(x) + f 111(x) = (f(x)- J'(x)) - :fxU'(x) - J"(x)). Em seguida, aplique
repelidas vezes o seguinte fato: se g é uma função polinomial de grau positivo e k e IR é tal que
g(x) + kg'(x) ~ O para todo x E lR, então g(x) ~ O para todo x E IR. Para provar tal fato, comece
utilizando o resultado do exemplo 16 para mostrar que g tem grau par e coeficiente líder positivo.
Em :seguida, tome (pelo problema 4.10, página 111) xo E IR tal que g atinge seu valor mínimo em
.r.0 e aplique aplique o teste da primeira derivada (cf. proposição 50).

Su;',\O 3 G
1. Para a <bem I e O < t < 1, segue do fato de g ser estritamente convexa que g((l - t)a + tb) <
(1- t)g(a) + tg(b). Agora, como/ é crescente, temos (J o g)((l -t)a + tb) = J(g((l -t)a + tb)) <
/((1 - t)g(a) + tg(b)). Por fim, use o fato de que/ também é estritamente convexa para mostrar
que o mesmo sucede com f o g.

2. Inicialmente, observe que, como f é uma bijeção contínua, o teorema 61 garante que / é crescente
ou decrescente. Suponha que fé crescente (o caso em que fé decrescente é totalmente anâlogo).
Para a< bem I e O< t < 1, temos f((l-t)a+tb) < (1-t)f(a)+tf(b). Faça a= f(a) e /3= f(b),
de sorte que a= J- 1 (a) e b = 1- 1 (/3). Aplicando 1- 1 em ambos os lados da desigualdade acima
e invocando o resultado do problema 4.8, página 47, mostre que (1 - t)J- 1 (o) + t/- 1 (/3) <
/- 1 ((1 - t)o + t/3). A partir daí, conclua que J- é estritamente côncava.
1

513
3. Use diretamente as definições de função estritamente convexa ou côncava, nos moldes das sugestões
dadas aos dois problemas anteriores.

4. Escreva /(.e)= (✓l)+l = -Jf=x+~ e mostre que cada uma das funções t H -,/t, e t H ~t'
1-x vl-x V[

t E (O,1), é estritamente convexa. Em seguida use o resultado do problema 6.3. Alternativamente,


calcule J"(x) = 4 ( 1~~) 512 e estude a variação de seu sinal.

5. Como J"(x) = 6x + 2a, temos J" > O em (-J, +oo) e f" < O em (-oo, -J). Portanto, J é
estritamente convexa em ( -J, +oo) e estritamente côncava em ( -00 1 -J), de sorte que x = -~
é seu único ponto de inflexão.

6. Calculando derivadas, obtemos, com o auxílio da regra da cadeia, f" (x) = (2 - )sen ½- ~ cos ¾. tz
Agora, se n E N e Xn = ..1..,
n,r
então ..1..
Xn
= mr I de modo que J"(xn) = -2mr cos(mr) = ( - 1r+ 1 2mr.
Portanto, J"(xk)f"(xk+1) < O, e o TVI garante a existência de Yn E (xn+l, Xn) tal que f''(Yn) = O
(observe que não precisamos invocar o teorema de Darboux 58, uma vez que f" é contínua em
(O,72)), Verifique que Yn é um ponto de inflexão de/.

7. Veja que g também é duas vezes derivável, com g"(x) = 2/'(x) +xf"(x) ~ O. Em seguida, aplique
o item (a) do teorema 64.

8. Suponha que f é convexa. Fixados x, y E J, mostremos primeiramente que /((1 - t)x + ty)) $
(1 - t)J(x) + tf(y) (*), para todo racional diâdico t E (O,1] (cf. problema 1.3, página 74); para
tanto, façamos indução sobre k ~ 1. Para k = 1, já temos a validade de (*) para t = O, ½ e 1.
Para t = ¾,aplicando a convexidade de / duas vezes, obtemos

f (x~3y) =f ('1\+y)(~~ + </ f(y)

J(x)~f(y) + f(y) 1 3
< 2 = 4f (X) + 4f (y),

conforme desejado. Por fim, para t = ¼basta trocar x por y no argumento acima. Suponha,
agora, que (*) seja válida para um certo k E N, todos x, y E / e todo O $ n $ 2k inteiro. Se
t= 2
r+
1 , com O $ m $ 2k+l inteiro, distingamos dois subcasos: (i) m é par, digamos m = 2n:
então t =~'e a validade de(*) segue da hipótese de indução. (ii) m é ímpar, digamos m = 2n+l:
então
m 1 (m-1 m+l) 1 (n n+l)
t= 2k+l = 2 2k+l + 2k+l = 2 2k +~ .

Pondo s = ffe eu= "it1 , temos s, u E [O,lJ e t = ½(s + u). Portanto, segue da convexidade de /

514
e da hipótese de indução que

/((1 - t)x + ty) = f ( ( 1 - S; U) X+ ( S; U) y)

= f ( ((1 - s)x + sy) ~ ((1 - u)x + uy))

/((1 - s)x + sy) + /((1 - u)x + uy)


< 2
1
$
2[((1 - s)J(x) + sf(y)) + ((1 - u)f(x) + uf(y)))
= ( l _ S ; U) /(X) + ( S ; U) f (y)
= (1 - t)f(x) + tf(y).
Agora, dados x, y E I, a continuidade de f garante, via regra da cadeia, a continuidade da função
g: [O,l] -+ IR, tal que

g(t) = (1 - t)f(x) + tf(y) - J((l - t)x + ty)),

para t E [O,l]. Por outro lado, mostramos acima que g(t) 2'.:O, para todo t E [O,1} que é um
racional diádico, de sorte que a densidade dos mesmos em [O,1} garante, mediante o problema 5.4,
página 119, que g(t) 2'.:O para lodo t E [O,1}, conforme desejado.

9. Suponha que fé estritamente convexa e fixe a <bem I e O< t < 1. Se e= (1 - t)a + tb, então
a <e< b, de modo que podemos escolher a E (a, e) e f3 E (e, b) tais que e= º'!-
8 . Ademais, sendo
a = (1 - s)a + sb e f3 = (1 - u)a + ub, com s, u E (O,1), mostre que e= ª!p ::::}t = siu. Agora,
use a condição do enunciado, juntamente com a definição de convexidade estrita e o resultado do
problema anterior, para mostrar que J((l - t)a + tb) < /(a);/(/3) ~ (1 - t)J(a) + tf(b).

10. Use as hipóteses sobre f para mostrar que, para x e y positivos e distintos, tem-se :r/(:rJ!:/(y) 2'.:
2
J ( :r;!~ ) 2'.:f (:r!y), com as duas igualdades não ocorrendo simultaneamente. A partir daí,
deduza que g(x);g(y) > g c=!Y)e aplique o resultado do problema anterior.

11. Para o item (a), use a desigualdade do item (b) do problema 2, página 416, para mostrar que
J(ª'tb) $ J(a);J(b) para todos a, b > O, com igualdade se, e só se, a = b. Para o item (b), use
indução sobre n, juntamente com o caráter crescente de /, para mostrar que (ª!bt $ ªn~b"
para todos a, b > O, com igualdade se, e só se, a = b. Quanto a (c), use Trigonometria para
mostrar que, em (O,1r), temos sena + sen b $ 2 sen ( ªt), com igualdade se, e só se, a = b.
Para o item (d), argumente de maneira análoga a (c), utilizando Trigonometria para mostrar que
2 tg ( ªIb) $ tg a + tg b para todos a, b E (O,~), com igualdade se, e só se, a = b.

12. Se x = x 0, nada há a fazer. Senão, aplicando o TVM de Lagrange duas vezes, concluilnos peln
existência de e entre xo e x tal que J(x) - f(xo) = f'(c)(.c - J'o), e d entre e e :ro, tnl que

515
f'(c) - J'(xo) = J"(d)(c - xo). Portanto,

f(x) - J(xo) - J'(xo)(x - xo) = J'(c)(x - xo) - J'(xo)(x - xo)


= (!'(e) - J'(xo))(x - xo)
= J"(d)(c - xo)(x - xo).
Mas, como fé convexa e duas vezes derivâvel, segue do corolário 65 que !"(d) ~ O. Então, uma
vez que (e - xo)(x - xo) > O, os câlculos acima garantem que

J(x) - J(xo) - J'(xo)(x - xo) ~ O.

13. Inicialmente, note que a função / : (O,+oo) -+ lR, dada por J(x) = xk, ê estritamente convexa.
Em seguida, aplique a desigualdade de Jensen (3.31).

14. Para o item (b), observe inicialmente que, pelo item (a), podemos supor que O pcrlcnce ao
interior de A1A2 ... An. Sendo a-1 = A,ÔA,+-1 para 1 ~ i ~ n (com An+l = A1), isso garante que
O< O.i < 1r para todo i. Agora, use (3 31), nos moldes do exemplo 72.

15. Sendo a-1 = A,ÔA 1 +1 para 1 ~ i $ n (com An+I = A 1), temo~ O < 0 1 < rr. l\Iostre, agora, que o
perímetro de A1A2 ... An é igual a r E~ 1 tg T; em seguida, note que a função tg: (O, ; ) -+ lR é
estritamente convexa, e aplique (3 31)

16. A lei dos senos fornece a+ b +e= 2R(sen  + sen Ê, sen ê) Agora, aplique a desigualdade de
Jensen (3.31) à função sen: (O,rr) -+ R.

17. Para a primeira desigualdade, aplique (3.31), juntamente com o resultado do problema 6.4, página
189. Para a segunda desigualdade, aplique (3.31) novamente, dessa vez utilizando o fato de que
a função raiz quadrada é estritamente côncava em (O,+oo).

SEÇÃO 3.7
1. Como senx = cos(; - x), temos cscx = sec(; - x). Use, agora o resultado do exemplo 73, em
conjunção com o problema 5.10, página 59.

2. Delineemos sugestões a alguns dos itens de (a) a (h), deixando a análise dos demais itens a cargo
do leitor. Para o item (b), calcule f'(x) = 1- :,re J"(x) = -:rpara concluir que/ é decrescente no
intervalo ( -1, 1), crescente em cada um dos intervalos ( -oo, -1) e (1, ;-oo), estritamente convexa
em (O,+oo) e estritamente côncava em (-oo, O). Também, o resultado do problema 1.11, página
2
137, garante que y = x ê a única assíntota oblíqua de f. Para o item (f), calcule f'(x) = (;,1-_:; ) 2
2
e f"(x) = {~~~ 0 ~) para concluir que x = ±l são os únicos pontos críticos de /, enquanto x = O
e x = ±vÍJ são os únicos pontos de inflexão de f Também, f'(x) > O se, e só se, lxl < 1, de
forma que/ é crescente no intervalo (-1, 1) e decrescente em cada um dos intervalos (-oo, -1)

516
e {l, + ); logo, x = -1 é ponto de mínimo local e x = l é ponto de máximo local. Mas, como
/(-1) = -½, ½
/(1) = e limlxl-++oo/(x) = O, segue que :r = -1 é o único ponto de mínimo
global para /, enquanto x = 1 é o único ponlo de máximo global para /. Agora, no ponto de
inflexão x = O, a tangente ao gráfico de / forma um ângulo trigonométrico de 1 com o eixo
das abscissas, enquanto nos pontos de inflexão x = ±\/'3 esse ângulo é de arctg (-1). Veja
também que /"(x) > O se, e só se, x E (-\/'3,0) U (v'3,+oo); logo,/ é estritamente convexa
em cada um desses intervalos. Também, J"(x) < O se, e só se, x E (-oo, -v'3) U (O,\1'3); logo,
J é estritamente côncava em cada um desses intervalos. Por fim, o eixo das abscissas é a (mica
1
assíntota horizontal ao gráfico de/. Para O item (h), calcule /'(x) = - (~I~=H:2
e f"(x) = 2[)..:irl)
para concluir que / é decrescente em todo seu domínio, estritamente convexa em cada um dos
intervalos (-1, O) e {1, +oo), e estritamente côncava em cada um dos intervalos (-oo, -1) e (O,1).
Também, x = O é o único ponto de inflexão de f e, nele, a tangente ao gráfico faz um ângulo
trigonométrico de - ¾com o eixo das abscissas. Por fim, o eixo das abscissas é a única assíntota
horizontal, com limx-++oof(x) = limx-+-oof(x) = O; as retas x = -1 e x = 1 são as únicas
assíntotas verticais, com liffix-+(-l)- /(x) = -oo, Wllx-+(-l)+ J(x) = +oo, limx-+1- J(x) = -oo e
limx-+1+ f(x) = +oo.

3. Inicialmente, note que, como J'(xo) = g'(xo), a reta r tangencia o gráfico de g (logo, r) em A.
+-+ +-+
Portanto, OA.lr. Para o ilem (a), se f'(xo) = O, então r é horizontal e, daí, OA é vertical. Se
+-+
J'(x 0 ) i- O, então a proposição 60 garante que o coeficiente angular de OA é - /'(~o). Para o item
(c), segue de (x - af + (g(x) - /3)2 = R 2 e da regra da cadeia que 2(x - a)+ 2(g(x) - {3)g'(x) = O
e, daí, 2 + 2g'(x) 2 + 2(g(x) - {3)g"(x) = O. Em seguida, faça x = xo na primeira igualdade, e use
o resultado da segunda parte do item (a), para obter a primeira igualdade do item (c); depois,
faça x = xo na segunda igualdade, e use o fato de que g(xo) = f(xo) e g'(xo) = f'(xo) para obter
a segunda igualdade do item (c). Para o item (e), observe que R 2 = (xo - a) 2 + (f (xo) - {3)2 e
substitua os valores de a- e /3 calculados em {d). Por fim, quanto à segunda parte de (f), observe
+-+
que a condição J'(xo) = g'(xo) garante quer .l AO.

4. Para o item (a), note que f é uma função para. Para o item (b), derive f para obter f'(x) =
sen ½- ¼cos ½-Para (c), observe que sen = O implica cos i- Oe, daí, tgy = Agora,
y-ycos y y y.

utilize o TVI (esboçar os gráficos de H tg e H


y pode ajudar) para garantir que o conjunto
y y y

dos > O tais que tg = é uma sequência (Yn)n~l, satisfazendo as condições elencadas. Para
y y y

(d), nole que


< < Y2k ~ tg < ~ sen
2k1r y cos < O,
y y y - y y

de forma que J'(x) < O para x E (x2k, 2~1r); em seguida, use um argumento análogo para mostnu·
1
que J'(x) > O para x E ( 2k7r+!. ,x2k), de forma que X2k ê ponto de máximo local para/. Então,
2
raciocine de forma análoga para concluir que x 2 k-l é ponto de mínimo local para/. Pt,rn o item
(f), note inicialmente que, como f'(xn) = sen ..L Xn
- ..L
.Cn
cos ..L
.Jn
~ O, temos lf (l·11)I= l:.t,.sen +I
;;i,,
=
1 = 1 cos Ynl; em seguida, aplique a última parte do item (e). Porn (f), conchm inicialmente
1 cos -.L

517
1
\ , 1,1, 1 I\ S1 c;1-:...,l<>I..., \<>..., flH<>l111 \f\-.;

que f'(x) > O para x > x 1 ; em seguida, aplique o limite trigonométrico fundamental para obter
limx-++oof(x) = limy-+O+se;u = 1. Para o item (g), calcule J"(x) = -~ sen ½-Em seguida, para
x > O, note que f"(x) > O (resp. f"(x) < O) se, e só se, sen ½< O (resp. sen ½> O, e resolva tais
inequações. Por fim, para (h), apoie-se nos resultados dos itens anteriores, notando também que
lf(x)I ~ lxl, com 1/(x)I = lxl se, e só se, x = mr~l!, para n E Z. Numa vizinhança suficientemente
2
pequena de x = O, você deve obter um esboço similar ao da Figura B.4. Em seguida, observe

y
0.24

0.20

0.16
0.12
y = xsen {1/x)

-0.2 0.2

-0.12
-0.16
-0.20
-0.24

Figura B.4: grâfico de xi--:-+xsen ½-

(cf. Figura B.5) que, para lxl > ¾,o gráfico não exibe o comportamento altamente oscilatório da
Figura B.4.

SEÇÃO 3.8

1. Use (3.36) para obter ti = ¼(v(ti) -vo) (i = 1, 2). Em seguida, escreva (3.35) para t = t1 e t = t2 ,
subtraia os resultados, substitua ti= ¼(v(t,) - vo) (i = 1, 2) e desenvolva.

2. Evidentemente, devemos ter 0 < o < ~. Adote um sistema cartesiano de coordenadas como no
exemplo 77, de forma que os pontos da colina sejam descritos pela equação y = (tg0)x. Segue
de (3.39) que, após o lançamento, o projétil atinge novamente a colina no ponto P(a, b), cujas

 518
- - - - - - - - - - - - - -,- - - -t.8 ------r------ , ------
1 1 1
1 1 1
1 1 1
1 1 1
1 1 1
1 1 1 1 1 1
,- - - - - - -,- - - - - - -1- - - - - -,- - - - - - -i- - - - - - -,
1 1 1 1 1 1
1 1 1 1 1 1
1 1 1 1 1 1
1 1 1 1 1 1
1 1 1 1 1 1
1 1 1 1 1 1

J
3
1
~2
1
~l i "1
1 1 1
1 1 1
1 1 1 1 1 1
1- - - - - - -'-
1 1
- - - - - -1- -
1
--G.5 - - - - - -1- - - - - - -1- - - - - - -1
1 1 1
1 1 1 1 1 1
1 1 1 1 1 1
1 1 1 1 1

Figura B.5: novamente o grâfico de xi--+ xsen ½·

coordenadas formam a solução diferente de (O,O) do sistema de equações y = (tg a)x- 2vo/ 062 0 x 2 ,
y = ( tg O)x. Por fim, calcule a = ~ cos2 a( tg o - tg 0) e veja que OP = cC:8 , de sorte que basta
maximizar a função f: [0,rr/2]--► IRdada por /(a)= cos a(tga-2
tgO). Então, utilize o teste
da primeira derivada para concluir que o valor procurado de a ê i + !.
3. Derive (3.41) para calcular v(t) e, em seguida, utilize a relação fundamental da Trigonometria.

4. Considere a Figura B.6 como representativa da situação em estudo. Argumentando de maneira


análoga ao exemplo 82, verifique que 111 c°oso + V2 :os/3ê o tempo que a luz leva para, partindo do

A
1
1

ª' 1
1
r
A' , B'
'b
1

B
Figura B.6: a lei de Snell-Descartes.

519
ponto A, incidir no ponto B após tocar a superfície de separação em P. Verifique também que
o tg a + b tg /3 = A' B' (*), qualquer que seja a posição de P sobre r, com P =I=A', B'. Agora.
note que /3 é uma função derivável de a, com /3(0- 1 ) = 0-2, e conclua que vi :osa + 112c~P =
J(a), para alguma função derivável f. Então, invoque o princípio de Fermat para deduzir que
J'(o- 1) = O i.e. que - asen 2°1 - bsenoa
' ' v1 cos 2
01
. f3'(o-
1 ) = O. Por fim, derive{*) em O'= 0-1 para obter
112cos 02
asec 2 0-1 + b{sec2 o-2 )/3'(o-1) = O, e deduza a lei de Snell-Descartes.

SEÇÃO 4.1

1. Adapte a demonstração de que S(f; Q) $ S(f; P) para mostrar o que se pede. Você precisará
utilizar o fato de que inf[x,-i,x') /, inf[x',x,)J 2::inf[x,_1 ,z,J/.

2. Se Pê uma partição do intervalo [a, b), mostre que s(f; P) = S(f; P) = c(b - a).

3. Releia a prova do teorema 5 para checar que limk-++oo(S(f;Pk)-s(f; Pk)) = O. Em seguida, use o
fato de que fé integrável, juntamente com as desigualdades s(f; P1.J$ J;f(x)dx $ S(f; Pk), para
mostrar que os limites limk-++ooS(f; Pk) e limk-++oos(J; Pk)) existem e são iguais a J;J(x)d:,:

4. Se Pk = { a = xo < x1 < · · · < Xk = b} é wna partição é equiespaçada, temos A(!; k) = s(J; P1.)
í
e A(!; k) = S(f; Pk), de forma que, por (6), temos s(J; Pk) < ~:; (1 - r+l < S(f; Pk)- Agora,
faça k ➔ +oo.

5. Para o item (a), faça u = x 2 , v = y2 e desenvolva as expressões assim obtidas. Para o item
(b), faça u = x 1 e v = X;-1 em (a) e some membro a membro as desigualdades assim obtidas,
observando que x 1 - x 1 -1 = b,t. Por fim, para o item (c), faça k ➔ +oo nas desigualdades do
item (b) e use o resultado do problema 1.3.

6. Para 1 $ k $ n, escreva ~
vn • "4-+
y,;:+n
= ¾· Jr:{
1
!+l . Em seguida, se f(x) = ~ e Pn =
vx+l
n

{O,¼,¾, ... , n~l, l}, veja que an = s(J; Pn)- Por fim, use o resultado do problema 1.3 para
concluir o que se pede.

7. Para 1 -< k -< n, escreva n • n2~k2 = ¾• l+ lkn )2. Em seguida, se f(x) = I7.=7 1
+:r e Pn -
{O,¾,¾, ... , n~l, l}, veja que an = s(J; Pn)- Por fim, use o resultado do problema 1.3.

8. Para a primeira parte, use a desigualdade do enunciado para mostrar que o critério de Cauchy
é satisfeito. Para a segunda parte, conclua, também a partir da desigualdade do enunciado,
que limk-Hoo s(J; Pk) e limk-++ooS(f; Pk) existem e são iguais. Em seguida, use o fato de que
s(/; Pk) $ J;f(x}dx $ S(J; Pk) para terminar a demonstração.

9. Para o item (a), adapte (4.6) ao presente caso, observando que, se At/1 = f(x ) em; = J(x'J), então
1
M1 - m; = J(xí) - f(x'J) $ clxí - x'JI $ c(x; - X;-1) = c(b;a>. Para o item (b), mostre primeiro
que E(/; Pk; {) e J:J(x)dx pertencem ambos ao intervalo de extremos s(J; Pk) e S(J; Pk).

520
10. Para o item (a), observe que 2sen (a+ 1h)sen ~ = cos(a + (j - l)h) - cos(a + jh), some de j = O
a J = k e transforme o resultado em produto. Para o item (b), o exemplo 37 mostra que a função
seno é lipschitziana, com constante de Lipschitz e= l. Seja Pk ={a= xo < xi < · · · < x1c =b}
uma partição equiespaçada de [a,b] e faça (nas notações do problema anterior) ç1c 1 = x 1 , para
1 :5j $ k. Sendo h = ~ = x 1 - x 1 _ 1 para 1 :5 j $ k, observe que

k h sen ( a -+ (k-;l)h) sen (k~l)h


E(sen; Pk; çk) = h L sen (a+ jh)
j=l
= h
sen 2
- hsen a.

Em seguida, aplique o limite trigonométrico fundamental, juntamente com o resultado do item


(b) do problema anterior, para concluir que

. ( hsen (a+ (k-;l)h) seo (k~l)h )


1b sen(x)dx = lim ------,------ - hsena
h-+O sen ~

= 2 lim - !12
- • sen (b+a
h-+O[ sen ~
--
2
- -h) sen (b-a
2
--
2
+- h)]
2

b+a)
= 2sen ( ~ sen (b-a)
- - = cosa - cosb.
2
11. Inicialmente. observe que, pela densidade dos irracionais na reta, temos s(J; P) = O, para toda
partição P. Agora, dado E > O. fixe no E N tal que no > ¾-Em seguida, se A= {x E [O,1]; x =
';:, com m, n E N e n < no}, mostre que A é finito e use este fato para garantir a existência de
uma partição Po do intervalo [O,l], tal que a soma dos comprimentos dos intervalos de P0 que
contêm algum elemento de A ê menor que ½·Por fim, mostre que S{/; Po) < E.

SEÇÃO 1.2

1. Seja J(x) = xsen(1rx). Se Pn = {O= xo < xi < · · · < Xn = 1} é uma partição equiespaçada,
mostre que an = 1r E(!; Pn;fJ, onde ç é o pontilhamento de P tal que ç1 = x 1, para 1 $ j $ n.
Aplique, agora, o teorema de Riemann.

2. Seja Q = {O = Yo < Yt < · · · < YL = 1} uma partição do contradomínio [O,1] de f e, para


1 $ j < k, seja Aja imagem inversa de [Y1 -1, Yí); seja também Ak a imagem inversa. de [YA:-1,
Yk]-
~Iostre que A 1 = 0, para 1 < j < k e recorde que, como Ak = Qn [O,l], temos m(A1t) = O. Então,
use (4.10) para concluir que Í[o,i) ÍQ = O.

3. Suponha, inicialmente, que fé crescente, e seja Q ={O= Yo < Y1 < · · · < Yl = L} urna partição
do intervalo [O,L]. Para 1 $ j < k, seja A 1 a imagem inversa de [y1_1, y1); seja também Ak "
imagem inversa de [Yk-1,Yk]- Sendo x 1 = 1- 1(y1), para 1 $ j $ k, mostre que A;= [x1-1,:r 1),
para 1 $ j < k, e Ak = [xk-1,xk]- Em seguida, se P ={a= xo < xi < · · · < x1.;= b}, mostre
que Íia,b) ÍQ = s(J; P). Agora, considere o caso em que/ é não decrescente, e ob erve que o caso
em que f ê não crescente é totalmente análogo.

521
4 Olhe para nmbas as integrais como áreas das regiões sob os gráficos dele 1- 1 , e relembre que o
gráfico de 1- 1 é simétrico ao grâ.fico de / com respeito à reta y = x do plano cartesiano. Então,
reflita a região sob o gráfico de 1- 1 em torno dessa reta e mostre que a união da região refletida
com aquela sob o gráfico de f contém um retângulo de lados a e b.
5 Comece mostrando que, dado t E [O,1], o fato de que f é não decrescente garante que A(R1) é
maior ou igual que a â.rea do retângulo de base [t, 1] e altura l(t), i.e., que

1 1
l(x)dx ~ (1 - t)J(t) ~ J(t) - t.

Então, fixes E [O,1] e faça t = g(s) na desigualdade acima para obter J(g(s))- g(s) ~ 101 f(x)dx.
Por fim, use o fato de que s E [O,1] foi escolhido arbitrariamente para concluir que

1 1
(f (g(s)) - g(s))ds ~ 1 1
f(x)dx.

SEÇÃO 4.3
1. Modifique a função do item (a) do problema 3.12, página 160, tomando, por exemplo, f(x) =
(x - a) 4l 3 sen -x-a'
1
- se x E (a' b]•

2. Fazendo xo = a e Xk+i = b, a proposição 20 garante que 1:f(x)dx = I:J=1 l~1_ 1 J(x)dx. Agora,
observe que, nessa última expressão, a soma das parcelas tais que f > Oem (x3 _ 1 , x3 ) corresponde
a A(Rj), ao passo que a soma das parcelas tais que f < O em (xj-l, x 1 ) corresponde a -A(R"j)

3. A hipótese do problema garante que fi(x) - g1(x) = h(x) - g2(x), para todo x E [a,b].

4. Utilize o resultado da proposição 18, juntamente com os resultados do problema 1.2, página 221,
e do lema 16.

5. Para o item (a), para j E Z+, seja 1, : [j,j + 1] ➔ IR a restrição da função parte fracionâ.ria ao
intervalo [j,j + 1]. Se P = {O= xo < X1 < •· · < Xk = 1} é uma partição do intervalo [O,1] e
y3 = x 3 + n para O~ j ~ k (de forma que Q = {n = Yo < Y1 < · · · < Yk = n + 1} é uma partição
de (n, n + 1)), mostre que s(/o; P) = s(/n; Q) e S(Jo;P) = S(Jn; Q). Para o item (b), utilize o
resultado da proposição 18 juntamente com os resultados do problema 1.4, página 221, e do lema
16.

6. Inicialmente, use a linearidade da integral para mostrar que é suficiente considerar o caso em
que / = O. Nesse caso, se existe xo E (a, b] tal que g(xo) > O, use o lema 48 para mostrar que
1:g(x )dx > O.
7. Use a linearidade da integral para obter - 1;ll(x)ldx ~ 1:f(x)dx ~ 1;11(x)ldx. Em seguida,
conclua o que se pede a partir daí.

522
SH,'.\O 1. 1

8 Adapte, a esse caso mais geral, a análise do caso em que J = g em [a, b), feita no texto.

9. Para a primeira parte, use o resultado do problema 1.4, página 221, juntamente com a proposição
20. Para a segunda parte, use o resultado da proposição 12.

11. Se A1 > O ê tal que 1/(x)I ~ Pvlpara todo x E [a, b], então

Logo, lim(-+0 (l; J(x)dx - 1;:;:J(x)dx) = O.


12. Observe que l:(J(x)-tg(x))2dx ~ O para todo t E IR. Desenvolvendo (J(x)-tg(x))2, conclua que
At 2 - 2Bt + C ~ O, para todo t E lR,onde A= 1;g(x) 2 dx, B = 1;f(x)g(x)dx e C = 1:f(x)2dx.
Em seguida, note que ó. = 4(B 2 - AC) ~ O. Para a última parte, use o resultado do problema
3.6.

13. Considere primeiro o caso em que f ê positiva, e interprete ( f(a)if(b)) (b - a) e 1:J(x)dx como
áreas. Para o caso geral, observe que trocar f por / + e equivale a somar e a ambos os membros
da desigualdade do enunciado.

SEÇÃO 4 4

1. Para o item (a), use que cos2 t = ½(1+ cos(2t)); para (b), use que cos t = 1- 2 sen2 ½;para o item
(c), integre por partes; para o item (d), use a substituição x = t- l, juntamente com o fato de que
7t = c 112 = -9t( w;);para (e), use a substituição X= sen t; para O item (f), use a substituição
X= ../i.

2. Para o item (a), integre por partes; para (b), integre por partes duas vezes; para (c), faça a
substituição y = 1 + x 3 ; para (d), use integração por partes, juntamente com o resultado do item
(c); para (e), faça a substituição x = sen t e, em seguida, integre por partes; por fim, para (f),
opere a substituição x = t 2 e, em seguida, integre por partes.

3. Para a primeira integral, sem# n, escreva sen(mx) sen(nx) = ½(cos(m - n)x - cos(m + n).r); se
m = n, use que sen 2nx = ½(1- cos(2nx)). Para a segunda integral, argumente de modo análogo.

523
4. Em ambos os casos, utilize (4.25). Adicionalmente, para o item (a), observe que x H jx 312 é
uma primitiva de x H vx
em [O,+oo); para o item (b), note que x H tgx é uma primitiva de
2
.i; H sec x, em(-;,;).

5. Em ambos os casos, escreva J\ J(x)dx = f~a J(x)dx + J; J(x)dx e, para a primeira integral, use
a substituição x = -t.

6. Para a primeira integral, observe que a função seno é ímpar e aplique o resultado do problema
anterior. Para a segunda, argumente de modo análogo.

7. Integre a igualdade x'(t) = v(t), de t = t 1 a t = t 2 , observando que


1
1
t2 lt2 1v(t2) 1.t=v(t2)
a v(t) dt = v(t)v' (t) dt = s ds = -2 s2 •
ti t1 v(ti) .t=v(ti)

8. Comece tomando uma primitiva F para/ e calculando a integral com o auxílio do TFC.

9. Se F(x) = J;+P f(t)dt, use o resultado do problema anterior para mostrar que F' é identicamente
nula.

10. Faça f(x) = mx + n, calcule J;(mxn)dx com o auxílio do TFC e, em seguida, verifique que o
resultado coincide com as fórmulas para a área de um triângulo retângulo (se / se anular em
x = a ou x = b) ou para a área de um trapézio retângulo (se / for positiva em [a, b]).

11. A(r) = 2A(R1) = 2J!:nf(x)dx = 2f~R JR 2 - x 2dx. Para calcular a última integral, use a
substituição trigonométrica x = R cos t, com O $ t $ 1T.

12. Se 0(0, O) é o centro da elipse e AA' é seu eixo maior, com A(-a, O) e A'(a, O), então a equação
da elipse é~+~= 1. Se f: (-a,a) ➔ IRé a função dada por /(x) = bJ1 - ~, mostre que a
área da elipse é dada por 2 f~a f(x)dx. Em seguida, use a substituição trigonométrica x = a cos t,
com O $ t $ 1T, para calculá-la.

13. Sendo F uma integral indefinida de / e a < x < y < b, use a interpretação de J(t) dt como JI
área e a monotonicidade de/ para comparar as diferenças F(y) - F (:z:1y) e F (:z:1y) - F(x).

14.. Note primeiro que /(a) = a e J(b) = b. Portanto, aplicando sucessivamente as fórmulas de
integração por substituição e por partes, obtemos

tla f- 1(x) dx = J,J(b} /- /(a)


1 (x) dx = fb /-
la
1 (/(x))J' (x) dx

b ,:r=b 1b
= xf'(x) dx = xf(x) _ - f(x) dx
1a :r-a a

= bf(b) - af(a) - 1b f(x) dx

= b2 - a2 -1b f(x)dx.

524
15. Suponha que g é crescente (o caso em que g ê decrescente é análogo). Sejam /,1 < 12 < ••• < t,c
os pontos do intervalo [g(c), g(d)] nos quais f ê descontínua, e e :5 si < s2 < • •• < Bk :5 d tais
que g(s 1 ) = t1 , para 1 :5 J ~ k. Se / 1 : [t1 , t 1+1J ➔ IR é tal que !1 = f em (x1 ,x 1 +1), / 1 (xF) =
limx--+x,+f(x) e J1 (x 1+i) = limx-+x,+i- f(x), então / 1 é contínua. Use o lema 16, juntamente
com o teorema 35, para concluir que J/ ,1 +1 J(t)dt = fa",+]
1
f(g(s))g'(s)ds, para 1 :5 j < k, e,
analogamente, J;(c) f(t)dt = J;1f(g(s))g'(s)ds e f,:<d)J(t)dt = J! f(g(s))g'(s)ds. Por fim, some
tais igualdades para obter o resultado desejado.

16. Aplique o TVM de Lagrange à integral indefinida de f baseada no ponto a.


17. Para o item (a), note que se 1r =!,com a,b EN, então 7r 2 =~,com a2 ,b2 EN. Parai.,
desenvolva (1 - xr com o auxílio da fórmula do binômio de Newton. Para item üi., observe
inicialmente que, pelo item i., temos J(k)(O) = O se k < n; por outro la.do, se k ~ n, então,
também pelo item i., temos J(k)(O) = k(k-l) •••~k-n+l) • ak = (k)ak
n. n
E Z. Por outro la.do, como
J(x) = /(1 - x), a regra da cadeia garante que j(k)(o) = (-l)k J(k)(l); logo, f(k)(l) E Z, para
todo k ~ 1. Para o item (c), observe que o primeiro membro ê igual a (g"(x) + 7r 2g(x))sen (1rx):
em seguida, substitua as fórmulas para g(x) e g"(x). Para o item (d), integre ambos os membros
da igualdade do item (c) e aplique o TFC. Por fim, para o item (e), use o resultado de ü. para
mostrar que, sendo 7ran< n!, temos

1ran
1
o
1 J(x) sen (1rx)dx < n! 11 f(x) sen (1rx)dx <
o
11
o
sen (7rx)dx = -2
1T"
< 1.

SEÇÃO 4.5

1 Para o item (a), observe (cf. Figura 4.9) que, após r rolar para a direita por t radianos a partir
de O, o ponto de tangência entre r e r serâ o ponto T(t, O) e o centro de r será o ponto A(t, 1).
Por outro lado, o ângulo LTOP, medido no sentido horário, será igual a t radianos. Como
BÂP = ~ - t (no sentido anti-horário), é imediato que
3 3
x(t)=t+cos( ;-t) =t-sent, y(t)=l+sen ( ;-t) =1-cost.

Para (b), utilizando a fórmula de integração por substituição, obtemos


2 2
A = fo 1r f(x)dx = h2-,r f(x(t))x'(t)dt = fo 1r y(t)x'(t)dt
2
= fo (1 - 1r cos t)2dt = 31r.

Por fim, quanto a (c), observe inicialmente que

2,r ____,,. 127r ,---~ 12,r '(t)2


J1 + f'(x) 2
dx = J1 + f'(x(t)) 2
x'(t)dt = 1 + y ( ),, x'(t)dt
10 o o x' t ..
/2,r ~ r27r
= lo Jx'(t) 2 + y'(t) 2 dt = h lo ✓1 - cos t dt.

525
Para o cálculo da última integral, veja o item (b) do problema 4.1, página 259.

2. Aplique (4.35), com f(x) = J R 2 - x 2 ex variando no intervalo fechado e limitado de comprimento


h e extremidades escolhidas entre os fmmeros ± J R 2 - r? e ± J R 2 - r~.

3. Adapte, ao presente caso, a discussão que levou à definição 45.

4. Aplique o resultado do problema anterior às funções /, g : [-R, R] ➔ IR, dadas por f(x) -
d + JR x e g( x) = d - J R
2

-
2

x2. 2

5. Dada uma partição P = {a = x 0 < x 1 < • • • < Xk = b} de [a, b], seja n 1 o retângulo de base
[Xj-1,xj] e altura /(ç1 ), com ç1 E (x1 _ 1 ,x 1 ), de sorte que podemos aproximar seu baricentro
pelo ponto G;(çj, ½f(ç1)). Sendo p a densidade superficial de massa de n (a qual, de acordo
com a discussão que precede o enunciado, supomos constante) e m 1 e m as massas de n 1 e
'R, respectivamente, temos m 1 = pf(ç 1 )(xi - x 1 _ 1) em = p J; f(x)dx. Segue de (4.40) que
r:;=lm;(ç;-xc, ½J({;)-yc) = (O,O)ou, ainda, r:;=m ç -r:;=m xa = Oe½ r:J=l m f(f,
1 1 1 1 1 1 1 )-
L;=lm yc = O. Nessas igualdades, substitua as expressões para mj, me use quem= r:;=m
3 1 1;
em seguida, faça IPI➔ O para concluir, com o auxílio do teorema de Riemann 7, o que se pede.

6. Inicialmente, recorde (cf. (A.35)) que o ponto de interseção das medianas de ABC ê (ªjb, ~).
Em seguida, mostre que J; f(x)dx = ª2h, f 0ª xf(x)dx = ª2h (ª1h) e ½f 0ª f(x) 2 dx = ª2h • ~-

7. Adapte, ao presente caso, a sugestão dada àquele problema. Você deve chegar às fórmulas
2
J; x(f (x) - g(x))dx ½J;U(x) - g(x)2)dx
xa - e YG = b
- J;U(x) - g(x))dx fa (J(x) - g(x))dx
2
rb ½I:U(x) -g(x) 2 )dx
8. Use que A = Ja (f(x) - g(x))dx, d= YG = J:(J(x)-g(x))dx e (de acordo com o problema 5.3)
2
V= 1r J;U(x) - g(x)2)dx.

9. Pelo teorema de Pappus, temos !1r R 3 = 21rAd, onde A = ½1rR 2 .

10. Pelo teorema de Pappus, o volume do toro em questão ê 21r• 1r R 2 • d = 21r2 Rd.

11. Como na sugestão ao problema 5.5, dados [c,d] e (a,b) e uma partição p ={e= xo < x 1 <
... < Xk = d} de [e, d], seja 91 a porção do gráfico situada entre os pontos de abscissas x 1 _ 1 e Xj,
de sorte que podemos aproximar seu baricentro pelo ponto (ç /(ç com ç E (x x ). Use
1
1
,

1
)),

1 1
_

1
,

(4.33), (4.40) e faça IPI➔ O para concluir o que se pede.

12. Adapte, ao presente caso, a discussão do problema anterior.

14. Aplicando a versão do teorema de Pappus dada pelo problema anterior, juntamente com O resul-
tado do exemplo 49, concluímos que 41rR2 = 21r• 1r Rd.

526
S1-:c;.\<
> 1.(i

=
15. Considere as funções J, g: [-R, R] , IR, tais que J(x) =d+ J R 2 - .r2 t• g(.r) d - J f(J. - x 2,
para x E [-R, R]. Em seguida, aplicando o resultado do problema 5.13, conclua. q,w a área. do
toro é 21r • 21r R • d.

SEÇÃO 4.6

1. Como y = log<=>x = aY, temos log0 x • log a = y log a = log aY = log e, daí, log0 x = I@:~;
0 x .e

por outro lado, o teorema 27 fornece log~ x = 1Jy) = alil~ga = xl~ga. Agora, o item (a) segue
da primeira parte de (4.50), uma vez que log e = 1. O item (b) segue da segunda parte de
(4.50), posto que log0 é crescente <=>log~ x > O para todo x > O <=>:tl~a > O, para todo
1
x > O<=>Ioga> O, para todo x > O<=>a> 1. Quanto a (c) e (d), temos loga(xy) = ,6~2;') =
Ioga =
logx+logy oga + ~
j°sx logO= loga x + loga y e loga e= /ogc
oga = Jog
0
s 6• /ogb
oga = logb e· loga b. Por fim ' (e)
segue imediatamente de (d).

2. Suponha que log 10 2 é racional e chegue a uma contradição ao teorema fundamental da aritmética.

3. Tomando logaritmos naturais, conclua que basta comparar os números ¼ e 1°!,r. Para tanto,
estude a primeira variação da função f : (O,+oo) ➔ IR dada por J(x) = l~x, mostrando que
x = e é seu único ponto de rnâximo global.
1
4. Tomando logaritmos naturais, mostre que basta analisar as soluções a,b E N da equação º!ª =
10
:b. Para tanto, proceda como na sugestão dada ao problema anterior.

5. Use a fórmula do corolário 33.

6. Se f(x) = xlog(l +
- comece mostrando que é crescente. Em seguida, escreva
x), f f(x) -

x ( 1 - log(~+x)) e use a regra de PHõpital para concluir que lim:r➔O+ J(x) = O.

7 Utilize o resultado dos problemas 4.8, página 259, e 4.1, página 166, para mostrar que a função
J: (O,+oo) ➔ IR, dada por J(x) = fi/x i~~~dt,é constante. Em seguida, observe que f(l) = O.
8. Basta observar que, como log é crescente e x!u ~ y'xy para todos x, y > O, temos log ( x!y) >
log y'xy = 1ºS x!log U, ocorrendo a igualdade se, e só se, x = y.

9. Mostre que J"(x) < O para x E (O,1r).

10. Aplique a desigualdade de Jensen (3.30) à expressão log ( 0; + b;), observando que, se t = },
então t E (O,1) e 1 - t = ¼·
11. Use a convexidade estrita da função J(x) = x log x, x > O, juntamente com a desigunklt\de de
Jensen (3.30).

12. Recorde que f;ex = ex.

527
13. Como f"(x) = ex+ 6ax, queremos encontrar os a < O tais que ex+ 6ax tenha uma única rajz
real. Raciocinando geometricamente, conclua que basta encontrar os a < O para os quais a reta
y = -6ax tangencia o grãfico de x i--+ ex. Por fim, use o resultado do problema 2.10, pâgina
149 (com (a, b) = (O,O)) para concluir que, sendo (x 0 , exº) o ponto de tangência, devemos ter
exo = -6axo e -6a = f;ex 1 = exo.
x=xo

14. Recorde que xº = eº logx e, em seguida, aplique a regra da cadeia.


15. Adapte, ao presente caso, a sugestão dada ao problema anterior para obter h'(x) -
(f(x)g'(x) log f(x) + J'(x)g(x))f(x)9(x)- 1.

16. Estude a primeira variação de / para concluir que x = a~b é seu único ponto crítico. Em seguida,
use o teorema de Weierstrass 53, capítulo 2, para concluir que tal ponto é de máximo global.

17. Faça a= b = ./2 e analise separadamente as possibilidades ab racional e ab irracional, observando,


neste último caso, que (abl E Q.

18. Certamente, x = 2 e x = 4 são raízes. Para mostrar que há exatamente três raízes reais, comece
esboçando os grãficos das funções x H x 2 e x i--+ 2x; em segmda, use o TVI para mostrar que
.., existe xo E (-2, O) tal que x~ = 2xo. Por fim, para mostrar que x 0 é irracional, argumente por
contradição.

19. Pela continuidade da função log, basta mostrar que limx-++oox log (1 + i) = a ou, ainda (fazendo
y = ¾), que limy-+O+log(IY+ay)= a. Para tanto, use a regra de l'Hôpital.

20. Como ( x+a ) x = exlog(~)


z-o , queremos os a E R tais que limx-++oox log ( x+a ) = 1 ou ainda
x-a x-a ' '
limx-++oox log ( l+~)
i-! = 1
1. Fazendo Y = i• conclua que queremos encontrar os a > O tais que
1 (.!.±g)
limy-+O+ og ~- 011
= 1. Por fim, aplique a regra de l'Hôpital.

21. Derive g(x) = e-x f(x) e utilize a proposição 53.

22. Para a segunda parte do item (a), use a regra da cadeia, juntamente com o fato de que fxex =
ex. Para o item (b), use que cosh" x = coshx > O, coshx = e"'+ 2
e-"' ~ ✓ez. e-x = 1 e
limz-++oocoshx = +oo. Para o item (c), mostre que senh'x = coshx > O e senh"x = senhx,
com senhx > O se, e só se, x > O. No item (d), você deve obter algo similar à Figura B.7.
Para o item (e), se k(xo) denota a curvatura em questão e /(x) = cosb x, conclua, a partir do
problema 7.3, página 198, que k(xo) = (1+s~C::2;;,)J/2 = co::>~-xo.
Por fim, para o item (f), pondo
/(x) = coshx, temos ~!x;;-
J'(x) 2 = coshx, de forma que, por (4.34), o comprimento desejado é
Jtºcosh x dx = senh J.'lr=O = senh xo.

 528
Su:.\o 1.,

coshx
~
~
\
\

''
''
X

Figura 8.7: gráficos do seno e cosseno hiperbólicos.

23. Como x 2 - y 2 = 1 ~ y = ±Jx 2 - 1, temos

r1 {eo:,ht
A = lo (tanh t)x dx + li {(tanh t)x - J x2 - l)dx

= 21 tanh t + (tanh t) • 21 (cosh2 t - 1) - Ío


t J cosh s - 2
1 senh s ds

1
= -senh t • cosh t - -11t (cosh{2s) - 1) ds t
= -.
2 2 o 2

2 J. Escreva x>-log x = - 1~\Y , onde y =½.Em seguida, adapte a prova do teorema 57 ao presente
caso para concluir que limy-++oo~
y = O.
25. hucíalmente, observe que /(1) = log(3e/8) > O, uma vez que {4.44)) 3e > 8. Agora, escreva
f(x) = log (r;~{
2
) - log (1+ ½r+ 1 para = O, e conclua que é
concluir que limx-t+oo/(x)
suficiente mostrar que J'(x) < O para x ~ 1. Para o que falta, calcule J'(x) = :z:+
1
172
+ log (x~i),
f"(x) = zl~:z:- z2+;+ 1; 4 e conclua que /' é crescente, com limx➔+oo f'(x) = O.

SEÇÃO -1.7

1. Para o item (a), proceda como no exemplo 61; para {b), observe que o integrando é da forma 7C:));
O mesmo em relação a (c), se notarmos inicialmente que 1!:z:2 =½ c~,l' + l~.r); para {d), integre

529
por partes duas vezes; para (e), escreva 1 )e"'= e-=~:
1 e observe que essa última expressão é, a
menos de sinal, da forma ~gl;
por fim, para os itens (f), (g) e (h), use a técnica de decomposição
em frações parciais, nos moldes do exemplo 66.

J
2. Para n E Z+, seja an = 01 xne-x dx, de sorte que a0 = 1 - e- 1 . Integre por partes para obter
ak = -e- 1 + kak-l, para k E lR. Em seguida, utilize essa recorrência, juntamente com o valor de
ao, para calcular sucessivamente a 1 = 1 - 2e- 1 , a 2 = 2 - 5e- 1 , a3 = 6 - l6e- 1 , a4 = 25 - 65e- 1
e as = 120 - 326e- 1 .

3. Para a segunda parte do item (a), use (4.52). Para o item (b), use o resultado de (a), juntamente
com o TFC. Por fim, para o item (c), temos J sec3 t dt = J sec t(l + tg 2 t)dt = J sec t dt + J( tg t •
sect)tgtdt = -loglsect- tgtl+ J sec't· tgtdt = -loglsect- tgtl+sect• tgt- Jsect· tg'tdt

4. Vimos, no exemplo 44, que f = J;1"ctan{ 2b) sec3 t dt, onde f o comprimento em questão. Pa.rn
calculá-lo, aplique a fórmula do item (c), observando que, se a- = arctg (2b), então tg a = 2b e,
daí, seca-= J1 + tg 2 a- = ✓1 + 4b2 .

5. O item (a) segue imediatamente de (4.33). Para o item (b), use o fato de que a 2 = b2 + c2 1
b2:r2
juntamente com a relação fundamental da Trigonometria., para escrever 1 + 02 (a'Lx2) = 1 +
2 2
b 2 cos t
a sen2t
=
2 2 2
a sen t+b cos t
a2sen2t
2
= 2 2 2
b ~c sen t
a sen2t
= (-b-)
ascn t
2
(1+ (c)2 sen2t) •
l)

6. Para o item (a), segue do corolârio 72 que


2 2
t t ( 1 - tg .t 1 - tg .t
tg 2-
t t ) 2
cost = cos2 - - sen 2 - = cos2 - 1- = 2 -
2 2 2 2 sec2 ½ - 1 + tg 2½
e
t t t t 2 tgi 2 tg½
sen t = 2 sen - cos - = 2 tg - • cos2 - = -- 2 2 = --~ 2
2 2 2 2 sec ½ 1 + tg ½•
Para o item (b), use (4.30).

7. Para a terceira igualdade, use integração por decomposição em frações parciais. Para a última
parte, observe que

1 - tg ½ cos ½- sen ½ t
( cos 2 - sen 2t )2 1 - sent
1 + tg ½ = cos ½+ sen ½ cos2 -2t - sen2-2t = cos t = sec t - tg t.

8. Para a primeira parte, comece operando a substituição x = a cos t; em seguida, opere as substi-
tuições utilizadas no problema 7.6. Para a segunda, observe que, nesse caso, temos R(x, y) = ~
e a = 2, de forma que, após o emprego das substituições de variáveis sugeridos, obtemos o inte-
grando
2 4s
R (2(1 - s ) 4s ) 4 _ ~ 4 2s
1 + s2 ' 1 + s 2 (1 + s 2) 2 - 2{1-s 2 )) 3 • (1 + s 2)2 = (1 - s2)3 •
( l+s 2

Agora, use decomposição em frações parciais.

530
SEÇÃO 4.8

l. Para o item (a), use o resultado do item (b) do problema 7.3, página 298 Para (b), aplique o
critério de comparação para integrais impróprias, juntamente com o fato de que le- 1senti ~ e-t,
para todo t E lR. Para o item (c), observe que l~t ➔ O quando t ➔ O+ e aplique o critério
de comparação para t ➔ 1-. Por fim, o item (d) segue do fato de que a função arctg é uma
primitiva de l~t 2 .

2. Aplique o teorema 35 para calcular J:,~~


f(t) dt, para x > B. Em seguida, faça x ➔ +oo.

3. Adapte, ao presente caso, o argumento do exemplo 69. Alternativamente, aplique o teorema de


mudança de variáveis para integrais impróprias (cf. problema anterior), com a substituição t ~ ¾,
que aplica (O,1) em (1, +oo).

4. Por definição, temos r(½) = J0+00 e-tt- 112 dt. Então, opere a mudança de variável s = t 112 e
aplique o resultado do problema 8.2.

5. É suficiente mostrar que a integral imprópria em questão converge absolutamente. Para tanto.
comece utilizando o resultado do problema 6.24, página 288, para concluir que existe A > O tal
que d j log ti $ A, para O < t $ 1. A partir dai, observe que

1
1
e-te:-l l log ti dt = 1
1
e-ttJ-l. d l logtl dt < A 11
e-tt½- 1 dt.

Agora, use o fato de que log t < t para t ~ 1 para escrever

Por fim, junte as estimativas anteriores para obter

6. Para o item (a), integre sobre I a desigualdade de Young (cf. problema 6.10, página 2 6),
obtida a partir de a = J(t) e b = g(t). Para o item (b}, comece observando que podemos
supor que Jt
f(t)P dt > O e J!
g(t)q dt > O. Em seguida, faça /i = / /A e g 1 = g/ B, onde
1 1
A= (J!
f(t)Pdt)1 P e B = (Jt
g(t)qdt)1 q; por fim, mostre que fi(t)Pdt = Jt
g 1 (t)qdt = 1 Jf
e aplique o resultado do item (a).

i,
7. Para o item (a), comece observando que, ses = então 1 - s = ¼; em seguida, tome logaritmo:s
em ambos os membros de (4.74}, utilizando o fato de que log ê crescente. Para o item (b), nplique

531
1
a de-igualdade de Holdcr a /(t) = e-tfpt r; e g(t) = e-tfqt9. Por fim, para o item (c), conclua
A partir dos itens (a) e (b) e da proposição 63 que log r é contínua; em seguida, observe que
f = elogr_

SEÇÃO 4.9

1. Faça M(t) = ½A-/


0 em (4.76).

2. Para o item (a) adapte, ao presente caso, a demonstração do lema 78. Para (b), adapte a
demonstração do teorema 79. Por fim, para (c), adapte, ao presente caso, a demonstração do
corolário 80.

3. Para o item (b), segue da segunda lei de Newton que F = ma = mv' Para o itens (c) e (d).
repasse a discussão que levou a (4.80). Por fim, para o item (e), faça t ➔ +oo na fórmula do item
(d).

4. Para os itens (b) e (c), revise a definição de fator integrante, assim como os cálculo que lcvanlln
a (4.79).

5. Para o item (c), substitua y = l 1v1 + l2v2 em (4.87). observando que, pelo lema 79, u1 e u2
satisfazem a equação y" + ay' + by = O. Para (d), observe que o sistema do enunciado é linear,
nas incógnitas l~ e l2. Resolva-o com o atLxílioda regra de Crnmer para sistemas lmeares e, cm
seguida, aplique o TFC para obter li e l2 a partir de t; de l2. Por fim, para a primeira parte
do item (e), desenvolva o determinante, derive a expre~ão assim obtida e, em seguida, utilize as
relações v~'+ av: + bv, = O para simplificar a expressão assim obtida.

6. Comece resolvendo a equação y" + 4y = O. Em seguida, aplique os resultados dos itens (b) a (e)
do problema anterior.

SEÇÃO 5.1

1. Inicialmente, mostre (por indução, por exemplo) que I::~:~ªkª 11c+1 = :;~!.para todo inteiro n > 1.
Em seguida, faça n ➔ +oo, observando que an = a1 + (n - l)r.

2. Faça Sn = "'"
L..,k=l or
2k-l e, em segw'da, most re que

(a - l)sn = asn - Sn = a+ 2 ( -1 + -;;-+


1 ••-+ -- 1 ) - __
2n -1_
a a- an-1 a"
=a+~(! _ __!_)_ 2n - 1 _
a- 1 a a" a"

Agora, use os resultados do exemplo 12 e do problema 24, página 85, para concluir que sn ➔
....!L. + ~ quando n ➔ +oo
a- 1 \a-,r 1

532
1
3 • Pvse que ªn+J 1 1
= ª" - an+l para todo n E N.

4. Inicialmente, note que Jk + Jk 2- 1 < /2f, de forma que Ek2:l Jk+~ > Ek2:l v1'k' Agora,
como 7n2: ¼para n 2: 1, use o teste da comparação para mostrar que a série dada diverge.
5. Mostre inicialmente que n 3 - lO00n2 > (n - 500)3 para todo ri suficientemente grande; em seguida,
use o teste da comparação, juntamente com a convergência da série Ek>SOO (k _r),o)3 72 •
1
6. Seja r > O a razão da PA. Para o item (a), temos 0 " = ai+(!-i)r > 2(n.:l)r se k > 7 + 1. Para o
•item (b) , temos _1
ª 2" -- ai+( 21"-l)r < (2"-l)r
l 1
$ 2n-lr· A p1"
1que, agora, o leste da comparaçao.
-

7. Observando que ak $ 9 para todo k 2: 1, use o teste da comparação para mostrar que a série
Ek2:l ~ converge. Agora, denotando por x o valor de sua soma, observe que, para n 2: no,
temos x - E~=l ~ = Ek2:no+l fã\ $ Í:k2:no+l ~ = 10~0•
8. Para a primeira parte, suponha que, para algum x E (O,1), tenhamos x = O,a1a2a3 ... =
O,b1 b2b3.. . , com an -:/=O para infinitos valores de n e bn f:. O para infinitos valores de n. En-
tão, rc;< Í:k2:l ~ = Ek2:l $!8r fli t
+ Í:k2: 2 ~ = b 11 , de forma que a1 $ b1. Analogamente,
1

mostre que a1 2: b1, de sorte que a 1 = b1. Em seguida, argumente de modo similar para esta-
belecer, por indução sobre n, que an = bn, para todo n 2: 1. Quanto à segunda parte, defina
f(x) = (y, z), com y = O,a1a3a5 ... e z = O,a2a4a6 ... se x = O,a1a2a3 .. .. Em seguida, use a
discussão do problema anterior para garantir que f está bem definida e que é uma sobrejeção.

9. Note que a~ + ~ 2: 2:r, para todo n E N. Em seguida, aplique o resultado do exemplo 6,


juntamente com o teste da comparação.

10. Use o teste da comparação, juntamente com o fato de que Janan+l < ½(an + an+i) para todo
n EN.

11. Impondo que Fj 2: a 1

para todo j $ k 1, deduza que Fk+2 2:


, + se 1 2: o A partir daí,
ak+
2

a+
2

mostre que Fn 2: an, para todo n 2: 3, onde a-= min{ .e,'2,víJ,1+2../s}. Por fim, note que o> 1 e
aplique o teste da comparação.

12. Aplique o teste da integral às funções (loglx)º, x(lo!x)º e x(logx)(l~glogx)ª.

13. Aplique o teorema 18.

14. Comece mostrando que Í:xeA ½ < E~rc=o


2ai6sc.Em seguida, escreva O segundo somatório
acima como o produto de três séries geométricas, aplique duas vezes o teorema 18 e use a fórmula
para a soma de uma série geométrica.

15. Para o item (a) (a prova do item (b) ê anãloga), seja q = ~ E (O,1) e tome no E N tnl que
v'an < q para n > no ou, o que é o mesmo, an < q11 para n > no. A partir dtú, adnpte n
argumentação do final da prova do teste da razão.

533
-

\ l 'I \ 11ft 1 1\ St < ;1-:---1 <)) ..__\( '"' l 'J{( 1111.F\I \"-

16. Parn o item (b), pela definição de série convergente é suficiente mostrar que a sequência (xn)n~I,
dada para n ~ l por Xn = a 1b1 + a 2 b2 + ... + anbn, é convergente. Como sabemos, isso é o
m~mo que mostrar que ela é uma sequência de Cauchy. Assim, sejam me n números natural,
com m > n, e tome A,/ > O tal que !skl < A,J para todo k ~ l. Segue da identidade de Abel e da
desigualdade triangular que
m-1
lxm - Xnl = 1 L S1(bibi+1)+ Smbm - Snbnl
i=n
m-1
~ L lsil(bibi+1) + lsmlbm+ lsnlbn
i=n

= M(bn -bm) + Mbm + Mbn


= 2Mbn ~ O.

Assim, (xn)n~l é, de fato, uma sequência de Cauchy.

17. Nas notações do problema anterior, troque ak por (-ll e bk por ªk·

18. Para o item (a), adapte, ao presente caso, a demonstração do teste da integral.

19. Para o item (a), argumente geometricamente para mostrar que, se 2l1r- ~ < n < n+ 1 < 2l7r+ i,
i
então 2l7r+ < n + 2 < 2l71'+ 5;, e se 2l71'+ s; < n < n + 1 < 2l71'+ 1;, então 2l71'+ 1; < n + 2 <
2l1r+ ll1r Para o item (b) 1 conclua a partir de (a) que jsennj + jsen(n+l)I + jsen(n+ 2 1 > 1 Em
6 • n n+l n+2 - 2(n+2) •
seguida, use a divergência da série harmônica para concluir o que se pede. Por fim, para o item
' Jk,r
I I
(c) para cada k E N mostre que r(k+l)7r sen t dt > rCk+l)r.-f sen t dt > ____lfl_
t -
. 2" Em seguida
Jk11"+{
I I t - ~ 6 • 1

conclua o que se pede a partir da divergência da série harmônica. (Por que não ê possível utilizar
o teste da integral?)

20. Para o item (a), estude a primeira variação de f(x) = (1 - x)e 2x no intervalo [O,½]-Para o item
(b), use o teorema fundamental da aritmética (cf. último parágrafo da introdução à seção A.l).
O item (c) segue dos itens (a) e (b), juntamente com o fato de que 1 + .l. + ::\-+ ... = (1 - ..1.)-l.
Pk Pk Pk
Por fim, para o item (d), use a divergência da série harmônica.

SEÇÃO 5.2

1. Para o item (b), use o resultado do item (a), juntamente com o teorema 20, para obter e -
E~=l ir,= Ek~lO ~ 1i>rir
E 1 ~0 1~; = 10 ~0! < 2.;11 < 10-
1 6
• Para o item (c), segue de (b) que
e ~ E2=l tr,
com cinco casas decimais corretas.

2. Comece observando que a série Lk~O (-N" é absolutamente convergente. Em seguida, faça a =
E1t~o~ e use que e = Lk~O ;h,juntamente com o teorema 18, para concluir que ae = 1.

3. UHea fórmula de Stirli11g.

534
4. Use a fórmula de Stirling para obter

(
(k+l)n) ~-1- ✓k+l(k+lt(l+~)kn_
n J2rn k k
Agora, observe que k > an ⇒ k --+ +oo quando n --+ +oo.

5. Para o item (a). por um lado, temos J'(x) = --:z- x~I + ½ = --:z- x(x~l) < O, de sorte que
J é monótona decrescente; por outro, limx➔+oo J(x) = limx➔+oo {½- log (1 + ½))= - log 1 = O.
Para (b), dado x > 1 e utilizando o resultado do problema 6.5, página 286, juntamente com o
teorema 59. obtemos

Jx J(t)dt = Jx (}- Iog(t + 1) + logt) dt

= log {:: - ((t + 1) log(t + 1) - (t + 1))1:::

+ {t log t - t) 1:::

= - log ( 1 + } ) - log ( 1 + l)x + 2 log 2 + 1

r➔+oo -log 1 - loge + 2log2 + 1 = 2log2.

6 Como O $ cos x $ 1 para x E [O,i], temos {cos x t+l $ (cos x r, para todo n 2::'.o. Portanto,
/ fc"12 (cosz)"+ 1d.:r
T = n
0,.72
fo (co x)"d.:r
$ 1, para todo n 2::'.O. Por outro lado, vimos no exemplo 34 que ln+l =
J 1 J /
....!LJ
n+l n-1,
para todo n -> 1. Portanto ,
1>
-
~
J,.
= ~ln 1

n-l ln-2
1
= n
n 2 -l
2
• ..!!.:::.!.>
ln-2 -
n-•
In-'l'
de sorte que
n

cada uma das sequências (~)'lk-2 k>l


e (~)2k-l k>l
é não decrescente e limitada superiormente,
logo convergente. Denotando por -eoe f 1, respectivamente, os limites dessas duas sequências,
temos e0 , f 1 > O Observe agora que, novamente por (a), temos 1
•t• = ~~n-t = n~l ( J~~i)- 1
.

Portanto, fazendo n = 2k --+ +oo, obtemos f1 = fõ 1 ou, ainda, f 0 f 1 = 1. Mas, como 11!• $ 1,
segue prontamente que fo, f1 $ 1. Logo, 1 = fof1 $ 1 • 1 = 1, o que nos dá lo = f 1 = 1. Por fim,
- • (/'lk-1)
como am bas as sequencias -r--
2 2
e (_fu..__)
12,,,,_, convergem para 1, cone 1mmos
• que a sequencia A •

' 1t- k~l .. -t k~l

( ~)n-1 n~l
converge para 1.

SEÇÃO 5.3
l. Use a versão (5.12) da fórmula de Taylor de J, centrada em xo, juntamente com o fato de que
J" 2::'.O (cf. item (a) do corolário 65).

2. Suponha que JCn)(xo) > O (o outro caso é anâlogo). Como JCn) é contínua e I ê aberto, podemos
tomar um intervalo aberto J C I, centrado em xo e tal que JCn)(x)> O, para todo x e J. Fixado
x E J \ {xo}, (5.11) garante a existência de e entre xo ex tal que f(x) = J(xo) + t<•;:fc)
(:t - :ro)".
Conclua que f(x) > f(xo)-

535
3. ~lostre que, se f(t) = Jl + k sen 2t para O $ t $ 1r, então f"(t) = kf~~k~;~)trr'lt). Em seguida,
observe (4.68), substitua a fórmula para / em ambos os membros da desigualdade do teorema 25,
observando que 1/"(t)I $ k(k + 1) para O $ t $ 1r.

4. A expansão da exponencial em série de Taylor. juntamente com a definição de senh x, fornece


sen h x -_ !( ~
x _ e -x) -_ 1 L-k~O kl x k -_ ~
1-(-1)" l 2J-l
. Ar gumen t e an alogamen t e para
2 e 2 L., ~ 1 ( _ 1)!x
1 21
desenvolver cosh x em série de Taylor.

5. Aplique o resultado da proposição 26, com n + l no lugar de n.

6. Seja p(x) = amxm + ªm-ixm-l + • • • + a 1x + ao e suponha que am > O (o outro caso pode sei
tratado de modo análogo). Pelo exemplo 16, podemos tomar A> O tal que p(x) > O para x > A.
Então, para x > A, aplique (5.16), com n = m + l, para obter o resultado desejado.

7. Para o item (a), como /(/3), f'(/3) > O, temos claramente , < /3, agora, use o resultado do
problema 3.1 para concluir que /(,) ~ O, de modo que , ~ o. Para (b), use o resultado
de (a) para mostrar que (an)n~l é monótona decrescente e ª" E [o, a1J, para todo n ~ 1, de
forma que existe e E [o, a 1] tal que ª" -t e quando n -t +oo; em seguida, faça n -t +oo
na recorrência que define (an)n~l para concluir que e = a. Para (i), use (5.12) para obter
/(o) = J(an) + J'(an)(cx - an) + ½J"(<n)(o - an) 2, para algum {n E (o, an)- Para a igualdade
em (ii), utilize a recorrência satisfeita pela sequência (an)n~ 1 ; para a desigualdade, use que /' é
crescente. Por fim, para a primeira parte do item {ili), use o resultado de (ii); para a segunda
parte, faça indução sobre n, observando que O$ an - o$ d - e

8. Verifique que/',/"> O em [2, ~] e que max 12.; 1{i,= J.


Nas notações do item (iii) do enunciado
do problema anterior, conclua que podemos fazer e = 2 e d = ~ para obter O $ an - a $ ( ~ J)".
para todo n ~ 1, onde a1 =~e an+l = ;:i~~-
Por fim, verifique que (~) .? < 10-
4 5.

SEÇÃO 5.4

1. Vimos, no exemplo 34, que ( 1 fl,


)n $ 2~, para todo x E IR. Portanto, fazendo Mk = e ir
observando que Lk>l
-
Nh < +oo, segue do 1\1-teste de \Veierstrass que Lk>i_ ( ~
l+x
)k converge
uniformemente em IR. Agora, segue da proposição 1 que

L (1 +xx2)k -
k~l
1 n:x:
- 1+~2
~
X
- --=----
x 2 - X+ 1 •

2. Utilizemos, em todos os casos, o M-teste de Weierstrass Para a série de Taylor da exponencial,


tem 08 1b,xk 1 $ ~ se !xi $ a. Portanto, pondo Ah = ~ para k ~ O, temos que Lk>o Ah =
Lk~O ~ = eª < +oo, e o M-teste ga.r;ante a convergência uniforme no intervalo [-a, a}~ Para a
série de Taylor do seno, lemos 1~;J~l)I x
21 1
- I $ (~;~-1\1 se lxl $ a. Pondo 1\,/21 = O para J ~ Oe

536
~ ~ ~
Af
11 21 -1
a2J-1 •
= c21 _ 1)' para J 2: 1, temos que wk'2:,0 Ah = w;'2:,l /1121 -1 = wj'2:,l {2;-J)I
a2J-I
= scnh a < +oo.
Novamente, o M-teste garante a convergência uniforme no intervalo [-a, a]. Por fim, parn a sfaie
de Taylor do cosseno, argumente de modo análogo.

3. Para a não uniformidade da convergência, mostre que, para todo n E N, o valor máximo de fn é
e- 1 Para calcular J; f n(x) dx, integre por partes.

4. Para o item (a), observe que, para O < x < 1, temos que xn --2:.+
O. Para (b), integre por
0
1
partes para obter J fn(x) dx = -e- + n 1
f 1
0 fn-i(x)
dx para todo n E N; em seguida, escreva
J 1
0 fn(x) dx = an - bne- , J; fn-1(x) dx = ªn-1 -bn-1e-
1 1
e utilize o fato de que e- 1 é irracional.
Para o item (c). faça indução sobre n, a partir das recorrências do item (b). Por fim, para (d),
use o fato de que e= I:k'2:.0 -tf·
5. Comece utilizando o T\Tl\I de Lagrange para concluir que 1/k(x)I $ 1/k(x) - Ík(xol + lfk(xo)I $
Mklx - xol + lfk(xo)I $ !lfk(b - a)+ 1/k(xo)I- Em seguida, aplique o M-teste de Weierstrass, com
M{ = /1,h(b- a)+ lfk(xo)1 no lugar de /Ih.
6. Para a primeira parte dos ilens (a) e (b), use integração por partes. Para a segunda parte do item
(a), use a desigualdade triangular para integrais para obter lak(f)I, lbk(J)I $ 2~ J;1r lf (x)ldx, para
lodo k 2'. 1. Para a primeira parte do item (b), integre por partes novamente. Para a segunda
parle de (b), utilize de novo a desigualdade triangular para integrais, juntamente com o M-teste
de Weier trass Por fim, para o item (c), u e o corolário 39, juntamente com o resultado do
problema 4.3, página 259.

7. Se ak(J) e bk(f) são como no enunciado do problema anterior, calcule as integrais que definem
2
tais números para obter ao(/) = 2 ~ , a1,: = 4 <d)k e bk = O, para todo k E N. Em seguida, use
o teorema de convergência de Fourier para escrever J(x) = ªº~/) + Lk>l ak(J) cos(kx). Por fim,
use lal expansão em série para calcular /(1r) = 1r2 . -

8 Use a condição do enunciado para concluir que J:f(x)pn(x)dx = O, para todo n 2'.1. Em seguida,
use a convergência uniforme de (Pn)n'2:,l para/ para mostrar que J:f(x)2dx = O.

9. Para o item (a), para x E (O,21r), use o resultado do item (a) problema 1.10, página 222, para
obter
n sen ( (n-;l):i:) sen (n-;l)x
I::sen (kx) = x
k=l sen 2
A partir dai, conclua que II:~=Isen (kx)I $ se~! e aplique o critério de Abel. Para o item (b),
2
observe que, para O < ô < 1r e x E [ô,21r - ô], temos lsen ~ 1 2: sen ô.

SEÇÃO 5.5
1. Para os itens (a) e (b), use o resultado do corolário 43. Para o item (c), observe que o raio de
convergência da série é menor ou igual a 1, uma vez que ela não converge quando x = l. Em

537
• \ 1·1 \ 111, 1 1: St e ;1-:s r< >I·..., \< ,..., l 'H< >Hl.l·.\l.\s

~cgnidn, nolc que, para O < x < 1, temos Lk~O x 2 k < EJ~O xJ < +oo, de forma que Lk~o x 2 k
converge.

2 e /(x) = Lk~l ~;~r~;


x 2k-l e g(x) = Lk~O ~~!~~ x 2k, é imediato que f e gestão definidas em
toda a rela. Por outro lado, pelo teorema 46, temos /'(x) = g(x) e g'(x) = - f(x). ~las, como
f(O) = O e g(O) = 1, segue do lema 78 que /(x) = sen x e g(x) = cos x.

3. Aplique k - 1 vezes o teorema 46, a partir da igualdade l~:i: - Ln~O xn. Alternativamente,
expanda (1 - x)-k com o auxílio da série binomial.

4. Expanda (1 - x 2 )- 112 em série de potências com o auxílio da série binomial. Em seguida, use o
item (b) da proposição 45, juntamente com o resultado do problema 2.3, página 148.

5. Para o item (a), some a PG do segundo membro. Para o item (b), após integrar de O a 1 ambos
os membros da igualdade do item (a), observe que
1 (-1)11·x2n 1 11 x2n 11 1
...;...._---dx < -~dx 2 < x 2 ndx = --·
11o 1 + x 2 - 0 1+ x - 0 2n + 1'
em seguida, faça n ~ +oo.

6. Para o item (a), observe que log ( ~!~)


= log(l + x) - log(l - x) e aplique o resultado do exemplo
49. Para o item (c), temos ~!~ = 3 para x = ½- Substituindo x = ½na fórmula do item (a), note
que 21 + 23.1 3 + 2:s.
1 + 1.
1 1 1 • • corre t as, uma vez
dâ 1 log 3 com quatro casas d ec1mrus
5 2 7 + 29. 9 + vTTi 2
que
1 1 1 1(1 1 1 )
213. 13 + 21s. 15 + 211. 17 + ••• < 13 21J + 21s + 211 + •••
1 1 1
11 < -30--2_0_0_0
= -3-.1-3---2~ = -60_0_0_0.

SEÇÃO 5.6

1. Queremos o coeficiente de xm no produto

com k fatores x + x2 + · · · = 1,::i:, para lxl < 1.

2. Queremos o coeficiente de x 20 no produto

(1 + x + x 2 + .. • )(x 2 + x 3 + .. • )(1 + x + x 2 + • • • )(1 + x + x 2 + ... + x 1).


Denotando a expressão acima por f(x), para lxl < 1, temos
f(x) = x2 (1 + x + x 2 + .. • )3 (1 + x + x 2 + .. • + x 1 )
= ( x 2 +x 3 +x 4 +· .. +x 9) 1
·-----=-.
(1 - x) 3
UHC,agora, o resultado do problema (5.3), página 379, para expandir (l-\:)l em série de potências

538
SlºBSl·:c;.\o .-\.1.1

3. Faça indução sobre k ~ 2. sendo o caso inicial k = 2 dado pela proposição 62.

4. Derive termo a termo ambos os membros de (5.32) e, em seguida, faça x = 1. Observamos que o
teorema 46 garante a licitude de um tal procedimento.

5. Inicialmente, mostre por indução que an ~ n 2, para n ~ 2. Fazendo J(x) = Ek2:l a1cxke
observando que o raio de convergência da série Lk2: 2 k 2xk é igual a 1, conclua que/ está definida
no intervalo ( -1, 1). Em seguida, escreva

J(x) =L akxk = 2x + L akxk = 2x + L(ªk-1 + k)xk


k2:l k2:2 k2:2
= 2x + x L ªk-1Xk-1 + x L kxk-1
k2:2 k2:2
= 2x + xf(x) + x L kxk-l = 2x + xf(x) + xg'(x),
k2:l

onde g(x) = Lk2:ixk = 1_::r,para lxl < 1. A partir daí, obtenha uma expressão fechada para
J(x), vâlida para lxl < 1, expanda tal expressão em série de potências e obtenha a expressão para
ak em função de k.

6. Para o item (b), temos

f(x) = 1+ L UnXn = 1 + L Un+1Xn+l= 1 + L(2an + n)xn+l


n2:l n2:0 n2:0
= 1 + 2xf(x) + Í:(n + 2)xn+l - 2L xn+l
n2:0 n2:0
= 1 + 2xf(x) + g'(x) - 2g(x),

com g(x) = 1 _::r.Para o item (c), reduza a soma do segundo membro ao mesmo denominador
comum e, em :;eguida, compare, nos numeradores do primeiro e do segundo membros, os coefici-
entes constantes, de x e x 2 , obtendo um sistema linear de equações nas incógnitas A, B e C. Por
fim, para o item (d), use o resultado do problema 5.3, pâgina 379.

7. Adapte, ao presente caso, as sugestões dadas aos vários itens do problema anterior.

SUBSEÇÃO A.l.1

1. Escreva i = ã= r, obtendo a= br, e= dr e, em seguida, a± e= r(b ± d).

2. Seja x = O,a1 a2a3 ... e suponha que a sequência (a1, a2, a3, .. .) é periódica, como em (A.3),
digamos. Se y E N é o inteiro com representação decimal b1~ ... bp, conclua, a partir daí, que
1Ql+Px = y + 101x, de sorte que x = 101+!_ 10P, um número racional. Reciprocamente, seja :.r= %,
com a, b E N e O < a ~ b. Se Yk E N é o inteiro com representação decimal a 1a2 ... ak, u~e o

539
• \1 1 ,1111 1 1: St (;f.:'-,1()1 '-, \()'-, l>B<>BI.1:\I.\'-,

algoritmo da divisão para concluir que lOka = byk + rk, com O ~ rk < b. Em seguida, use o fato
de que só há um número finito de possibilidades para rk para concluir pela existência de naturais
l e P, tais que r1+p = ri. A partir daí, conclua que a sequência (a 1, a2, a3, ... ) é da forma (A.3),
com b1 = a1+1, b2 = a1+2 , ... , bp = a1+p·
3. Imite a prova da unicidade do inverso aditivo, trocando + por · e O por 1.

SUBSEÇÃO A.1.2
1. Para os itens (b) e (c), utilize (7') e (a); o item (g) decorre dos itens (b) e (c).

2. Para os itens (a) e (b), comece observando que an < bn se, e só se, (tr < 1; em seguida, aplique

( a:b r r
o resultado do corolário 2. Quanto a (c), comece observando que an + bn < (a+ b)n se, e só se,
+ ( a!b < 1; em seguida, aplique o resultado do corolârio 2.

3. Adapte, ao presente caso, a demonstração do corolário em que::;tão.

4. Para o item (a), observe que (rst =~


(rs)···(rs) = (r~•r) • (s ··s)
~
= r"s". Para os demais
n n n
itens, argumente de modo análogo.

5. Considere separadamente os casos m < n, m =ne m > n, aplicando convenientemente o item


(b) do problema anterior em cada um deles.

6. Se n = 2k, com k E N, escreva x" = (x 2 )1-e, em seguida, aplique o item (g) da proposição 1. O
caso em que n é únpar pode ser tratado analogamente.

SUBSEÇÃO A.1.3
1. Por contraposição 1 , mostre que, se b =/O, então r é racional.

2. Argumente por contraposição. Por exemplo, se ifr = %, com a, b E N, então r = (??= ~, com
ak,bk EN.

3. Por contradição, suponha que /2 = f, com a e b naturais primos entre si, de maneira que 2b2 = a 2 .
A partir dessa igualdade, mostre que a é par e, daí. que b é par, chegando a uma contradição.
1Uma proposição da forma A =>B (i.e., Se A, então B) pode ser provada de forma direta, por contraposição
ou por contradição. No primeiro caso, assumimos a veracidade da asserção A e deduzimos a veracidade da
asserçãoB diretamente; no segundo caso, assumimos que a asserção Bê falsa e deduzimos, diretamente, que a
asserçãoA também é falsa; por fim, no terceiro caso, assumimos que a asserção A é verdadeira e a asserção B é
falsa e deduzimos diretamente, a partir daí, uma contradição (i.e., deduzi.mos que uma asserção obviamente falsa
deveria ser verdadeira, o que é impossível de ocorrer). Para uma revisão mais ampla sobre Lógica e métodos de
demonstração, sugerimos ao leitor uma das excelentes referências [171ou [29j.

540
s1·ns1-:c;.\o .\.1.1

4. Adapte, ao presente caso. a sugestão dada ao problema anterior, trocando 2 por p.

SUBSEÇÃO A. 1.4

L Primeiramente, note que ad = bc. Em seguida, mostre que o ponto associado às frações feâ
coincide com aquele associado à fração ~ = ~.

SUBSEÇÃO A.2.1

1. Para o item (a), basta desenvolver o segundo membro, obtendo

(x - y)(x + y) = x(x + y) - y(x + y) = (x 2 + xy) - (xy + y 2) = x 2 - 2


y .

Quanto a (b), temos

(x ± y) 2 = (x ± y)(x ± y) = x(x ± y) ± y(x ± y)


= (x 2 ± xy) ± (xy ± y 2 ) = x 2 ± 2xy + y2.
Por fim (c) é tamb m obtido desen\'oh·endo o segundo membro, o que deixamos a cargo do leitor.

2 Para o item (a), aplicando a dc::;igualdade(A.6) com a= x e b = ½,obtemos x+ ½~ 2;;-1 = 2,


com igualdade , e só se, x = ½,i.e., :;e, e ó e, x = 1 (wna vez que x >Opor hipótese). Quanto
a (b), basta desen\'Ol\'er o produto do enunciado e, em seguida, aplicar a desigualdade do item
(a), com ~ no lugru de x· (x + y) (¾ b)
+ = 2 + (~ + ~) ~ 2 + 2 = 4. Haverá igualdade se, e
só se, !.11 = Il,
:r
i e , se, e só se, x = y2. 11as como x, y > O, tal condição ê equivalente a termos
2

.r = y Outra p 1bilídade é aplicar a desigualdade (A.6) duas vezes, multiplicando os resultados:


(.r + y) ( ¾+ ~) ~ 2,/xy • 2 jf1 = 4. Segue diretamente de (A.6) que haverá igualdade se, e só
se, x = y.

3 Para obter a desigualdade, basta somar membro a membro as desigualdades parciais x2!y2 ~ xy,
.r t: ~ xz e V ~z ~ yz ote agora que, se x = y = z, então a desigualdade do enunciado
2 2 2 2

claramente se torna uma igualdade. Reciprocamente, se ao menos uma das desigualdades acima
2 2
for ~trita (i.e., não for uma igualdade), digamos x ~V > xy, então uma generalização óbvia
do item (e) da proposição 1 garante que, após somarmos as mesmas membro a membro (com
2 2
a primeira delas trocada por :z: ~V > xy), obteremos x 2 + y 2 + z 2 > xy + xz + yz. Portanto,
a fim de que ocorra a igualdade na desigualdade do enunciado, devemos ter igualdade nas três
desigualdades acima, de maneira que x = y = z.

4. Para o item (a), segue do item (a) da proposição 8, com vx e Jy no lugar de x e y, respectivamente,
que
1 _ vx=r=~ _ vx=r=Jv _vx=r=Jv
VX± /y - (VX± /y){VX=f /y) - (Jx) 2 - (/y) 2 - X -y

541
Quanto a (b), aplicando o item (c) da proposição 8, com rx
e ?fy no lugar de x e y, respectiva-
mente, e observando que ( rx)2
= ~ e ( ,VX) = X (e analogamente para y), obtemos
3

1 ~+~+W ~+~+W
rx± w = ( rx± ?!YH
~+ .yxy+ W) - x- y

Por fim, (c) segue, imediatamente, de (b).

5. Para o item (a), basta ver que

(a_ b)(an-1 + an-2b + an-3b2 + ... + abn-2 + bn-1) =


= a(an-1 + a"-2b + an-3b2 + ... + abn-2 + bn-1)
_ b(an-1 + an-2b + an-3b2 + ... + abn-2 + bn-1)
= (an + an-lb + an-2b2 + ... + a2bn-2 + abn-1)
_ (an-lb + an-2b2 + an-3b3 + ... + abn-1 + bn)

onde operamos uma série de cancelamentos na última passagem acima. Quanto a (b), observe
que a alternância de sinais faz sentido exatamente porque n é ímpar:

(a+ b)(an-1 - an-2b + a"-3b2 - ... - abn-2 + bn-1) =


= a(an-1 + a"-3b2 _ ... _ abn-2 + bn-1)
_ an-2b
+ b(an-1 _ an-2b + an 3b2 _ ... _ abn-2 + bn-1)
= (an _ an-lb + an-2b2 _ ... _ a2bn-2 + abn-1)
+ (an-lb _ an-2b2 + an-3b3 _ ... _ abn-1 + bn)
= an + bn.

(Na última passagem, mais uma vez executamos uma série de cancelamentos.)

SUBSEÇÃO A.2.2
2. Para a primeira desigualdade, interprete lx- aicomo a distância de x a a na reta numerada. Para
as outras três desigualdades, adapte a sugestão dada à primeira.

3. Analise separadamente os casos (i) x, y ~ O, (ü) x ~ O > y, (iii) y ~ O > x e (iv) x, y < O,
mostrando que a igualdade verifica-se em todos eles.

4. Para o item (a) consideremos dois casos: se x - 1 ~ O, então lx - li = x - 1 e a equação dada


2
resume-se a x 2 + 5(x - 1) + 11 = O, i.e., x + 5x + 6 = O. Suas raízes são x = -2 ou -3 , mas
nenhuma delas cumpre a condição de ser x ~ 1; logo, não há soluções neste caso. Se x - 1 < o,
2
então lx - li = 1 - x e a equação fica x - 5x + 16 = O. Como o discriminante dessa equação é

542
t1 = -39 < O, não há raízes reais. Portanto, a equação dada não possui soluçõc::i.Quanto a (b), há
dois casos a considerar x 2 - 3x ~ O ou x 2 - 3x < O. No primeiro caso, que equivale a :r:(x - 3) ~ O,
temos lx2 - 3xl = x 2 - 3x. A equação dada reduz-se, então, a x 2 - 3x = .e - 1, cujas raízCRsiio
x = 2 ± v'3. Destas, somente x = 2 + v'3satisfaz a condição x(x - 3) ~ O, sendo, portanto, a
única raiz válida. O segundo caso equivale a x(x - 3) < O, e nele temos lx2 - 3xl = -(x 2 - 3x).
Assim, a equação dada reduz-se a -(x 2 - 3x) = x - 1, ou ainda x 2 - 2x - 1 = O, cujas raízes são
x = l ± ./2. Somente x = 1 + ./2 cumpre a condição x(x - 3) < O, sendo, então, a única raiz
válida nesse caso. Para (c), podemos reescrever a equação dada na forma 2
,-;~!I
= 6 - x, ou
ainda l3x - 21 = (6 - x)lx + li. Para nos livrar dos módulos, teremos de considerar quatro casos,
resultantes dos cruzamentos das possibilidades de sinal para 3x - 2 e x + 1:

• 3x - 2 ~ O e x + 1 ~ O: nesse caso, l3x - 21 = 3x - 2 e lx + li = x + 1, e ficamos com a


equação 3x - 2 = (6- x)(x + l), de raízes x = -2 ou 4. Somente x = 4 satisfaz as condições
dadas.

• 3x - 2 ~ O e x + 1 < O: tais condições são o mesmo que x ~ ~ e x < -1, não havendo
número real que as satisfaça.
• 3x - 2 < O e x + 1 ~ O· aqui temos l3x - 21 = -(3x - 2) e lx + li = x + 1, de modo que
a equação dada se toma -(3.t - 2) = (6 - x)(x + l). Essa equação ê idêntica à do item
anterior, tendo portanto raízes x = 4 ± Js. ovamente nenhuma delas satisfaz as condições
dadas.
• 3x - 2 < O ex+ 1 < O: deixamos ao leitor a tarefa de terminar a análise desse caso.

Por fim, (d), (e) e (f} podem ser resolvidos com a utilização de argumentos anâlogos aos acima.

SLBSEÇ\O A 2.3
1. Para o item (a), observe que

y x y(l+x)-x(l+y) y-x
_1_+_y - -1
+-x = (1 + x)(l + y) - (1 + x}(l + y) ~ O.

Agora, como la+ bl ~ lal + lbl, segue do item (a) que

la + bl < lal + lbl _ lal + lbl < lal lbl


1 + la + bl - 1 + lal + lbl 1 + lal + lbl 1 + lal + lbl - 1 + lal + 1 + lbl •

2. Aplicando (A.14) às 50 primeiras e às 50 últimas parcelas, obtemos lx-ll+lx-21+· • •+ll·-501 ~


(x - 1) + (x - 2) + • • • + (x - 50} = 50x - {1 + 2 + .. • + 50) = 50x - 1275 e 11·- 511+ lx - 521+
• .. + lx -1001 ~ (51 - x) + (52 - x) + .. • + (100 - x) = {51 + 52 + · · · + 100) - 50J· = 3775 - 50l·.
Logo, lx - li+ lx - 21+ • • • + lx - 1001~ (50x - 1275) + (3775 - 50..c)= 2500 = 502 .

543
"'UB~li:Ç\O À 2.4

1. R' ·olva a equação equivalente x 2 - a2 = O, observando que x 2 - a2 = (x - a)(x + a).

2. B~ta ob-ervar que 6. = b2 - 4ac > O.


3. Compare os coeficientes de ambos os membros da igualdade

x 3 + ax 2 + bx +e= (x - a-)(x2 + ux + v)

para obter as igualdades u - a- = a, v - a-u = b e -a-v = e. Agora, use a primeira e a terceira


relações para obter u = a + a- e v = - ~, mostrando, em seguida, que tais valores para u
e v também satisfazem a relação v - a-u = b (nesse passo você precisará utilizar a igualdade
o 3 + ao 2 + bo +e= O). Os demais itens podem ser analisados de maneira semelhante.

11. Use o resultado do problema 3 para obter

Em seguida, desenvolva o produto do segundo membro e compare os coeficientes das expressões


de ambos os membros.

SLBSEÇÃO A.3.1
1 3
1. Basta observar que bk+l = 0- = ª01r+l = 3 + ..1..= 3 + bk.
k+l k ak

2. Para o item (a), temos a1 = a2 = a3 = 1 e ªk = ªk-1 + ªk-2 + ªk-3, para k 2::4. Para o item (b),
temos a1 = 1 e ak+l = 2ªk, para k 2::1.

3. Use a fórmula para o termo geral de uma PG no primeiro membro.

n n
(q-1)8 = qS- S = LbkCk+l - LbkCk
k=l k=l
n-1
= L(bk - bk+1)ck+1 + (bnCn+1- b1c1)
k-1
n
= -1' LCk + (r - b1)c1 + (b1 + (n - l)r)c 1qn
k-1

= -rc, qn -
( q _ 1
1)+ (r - b1)c1 + (b1 + (n - l)r)c 1qn.

SIJBBEÇ.ÃO A.:i.2
1. Escreva ªk+J - 111,.= :JJ..·
- 1 e uHc 1:,ornusLclcscópicus.

544
2. Escreva ak+I - ak = Bk e use somas telescópicas.
3 Observe que (k_!I)k = k,: 1 - ¼e use somas telescópicas.
4. Veja que. para k > 1:temos ,b < (k_!l)k: use. em seguida, o resultado do problema anterior.

5. Adapte. ao presente caso, o argumento do problema 3, escrevendo (4k-d( 4 k+J) = ¼( 4k:. 1 - 4 k~3) •

6. Ink1almente, observe que

g
"(
1-k
1)
2 =g n(
1-k l)n( g 1+k1) =g ~n(k l)"(k+l)
g ~ ·

Agora, faça ak = k. para k ~ 1 Segue da proposição 33 que

7. Imc1almente, obsen·e que o termo· da -.equencia são todos positivos. Em seguida, defina a
sequencia (bdk>J
-
pondo b1,= -1..,
u1
e lllebtre que bk+l = bk + k, para todo k E N. Aplique, agora,
a fórmula para somas tele.-.cópicns.

SUBSl~ÇÃO .3.3
Segue da hipóte:se de indução (implícita!) que 1+2+ • -+k+ (k+ 1) = k(k;-1>+(k+ 1) = (k+l~k+2).

2 Segu • da hipótese de indução que

3 Segue da hipótese de indução que

1
1 · 2 + 2 · 3 + • • • + (k - l)k + k(k + 1) =
3(k - l)k(k + 1) + k(k + 1)
1
=
3k(k + l)(k + 2) .
.J. Segue da hipótese de indução que

1
12 + 32 + ... + (2k - 1)2 + (2k + 1)2 = (2k - 1)2k(2k + 1) + (2k + 1)2
6
1
= 6(2k + 1)2(k + 1)(2k + 3).

545
5. Para o item (a), segue da hipótese de indução (novamente implícita!) que

Quanto a (b), segue novamente de uma hipótese de indução implícita que


m+l l
Lak
k=l

6. Para n E Z, seja an = xn + x-n. Mostremos primeiro, por indução, que an E Z para todo n EN.
A hipótese do problema fornece a 1 E Z. Então, a2 = x 2 + --1,i = (x + ½)2 - 2 = ar - 2 E Z.
Suponha, agora, por hipótese de indução, que a 1 , a 2 , ... , ak E Z para um certo k 2::2. Então,
ak+l = xk+l + xk~I = (xk +:,e) (x + ½) - (xk-I + ~) = akai
o qual é inteiro por - ªk-1,
hipótese de indução. Portanto, pelo segundo princípio de indução, segue que On E Z para todo
n E N. Para concluir, basta ver que a 0 = 2 e, se n < O é inteiro, então an = ª-n, o qual já.
provamos ser wn inteiro. Assim, an E Z para todo n E Z.

7. Se Xk > .fã, então Xk+l - .fã = ½( Xk - 2.,fõ.+ :. ) = i.!A:(xk - va) 2 > o. Para a outra
desigualdade, comecemos observando que, se x 2::y > ,lã, então x + i 2::y + ~; de fato,

(x + xª) - (y + yª) = (x - y) + a (x1 - y1) = (x xy- y) (xy - a) 2::O.

Agora, se Xk ~ ,lã+ 2,l_1 , então, a partir da desigualdade acima, obtemos

Xk+i -Hx•+ :.) H


~ ✓<i+ 2L + ✓,,: ,k)
< ~ ( y'a + 2kl-l + /ã)= y'a + 21k•

SUBSEÇÃO A.3.4
1. Façamos a prova por indução sobre k 2:: 1. O caso inicial k = 1 resume-se a verificar que
(:) = (:!D, sendo, portanto, imediato. Por hipótese de indução, suponha que, quando k = l 2::1,
a fórmula do enunciado é verdadeira para todo inteiro n 2::O. Então, para k = l + 1 e todo n 2::O
inteiro, temos

onde, na última igualdade, usamos a relação de Stifel. Portanto, segue por indução que a fórmuJa
do enunciado é verdadeira para todo k E N e todo n 2::O.

546
SUBSEÇ,\O A.-1.1 \ 1 1 , .1, 1.

2. Prim ir m ut apliqn' u t 'Or me das culu11ns pHrn os l,i110111ini


• (~). 1!.m • 11ida, ,,1, ·rvandn
qu, (~) = ½(j2 - j). , cr ,, L;, =! (~) = 1 ~1 I .J2- ½L,_~1=1 j.

3. pliqu o t or ma das oi una . obs rYa11do que ( 11 11


.1J) = ( : 1 ).

rva½=l o.coma>0. us of~tod•(l+o) 11


2:1+110 .

. -l. 1

1. '(-a, -b) O • a orig m, mo tr qu O ·ão colin ar s, com AO = A'O =

2. jam A' 11
o p ··d· perpendicular bai.xad d A - r tas y = Yo y = O. re p ctivam nt .
11
oord nad d A' no ·i ·t ma onginal ·ão. r ·pectivam nt , (x. Yo) (x, O). Em gmda
ob •rv qu . y > y0 > O. ntão AA 11 = A'+ 'A 11, d orte que y' = AA' = AA" - A' A"=
y - Yo- Por fim d mai • caso ( im como a igualdade x' = x - xo) pod m r anali ado d
forma s melhant .

1. hi • triz do quadrant ímpar •ar t d quaçào y = x. A r ta que p a pelo ponto A. e


B t •m quaçào , y = a +b d ~ort qu p rpendi ular à bi triz do quadrante ímpar a
qual int r ta uo ponto (ª/, ªib). Por fim • im diato v rifi ar qu AC= BC

2. Para qu'

li tinto , d v
s udo
B
-
jam
B
im ·trico
nr
••r o ponto m d10 d· AB.
em r lação a r, • n e· ário que
= {C}, tamb'm d v mo t r AC
omo a ab ci • a d C
+--+

= B.
Bl..r d forma qu b = d.

x0,
orno
gu qu :ro
B
= ªte.
;o

r dpro a mai rá d i.xada ao l itor.

3. Ad, pt caso, a ug tão dada ao probl ma ant rior.

l. ma I o ibilid d u ilizar o m ·todo intéti o para obt rê • coord na la· d B lt rnativ, m nte
+--+
obs •rY qu • quação dar ta OA • y - *:i·, d ort qu a quação dar •t OB y = -fr. Em
'guida impondo qu OB - OA, obt mos :i· - ±b , daí, y =Fa Ic , omo B p 1t n
gundo quadrant única po 1bilidad • q11 :i· - -b, y - a.

5. omo :r0 ,y 0 =/=O.tal r ta não pas a p la orig m. Portanto, a obscn·aç;o 3 ( m J..-= -l) !.?,ll nt
qu pod mo· r v r ua quação na forma a:i· + by - 1 = O. gorn, impou 1 ·u siv m ntt 1u
(:ro, O) (O,y0 ) p rt nçam à r ta, obt mo· a = }0 b = )0 •

6. • labor um pouco mai a discus ão da prova ela proposição fi2.


7 1 rt1 ,1 t 11tio /,=O I di t,111c.iad cl •Par·, o ompnm no do m nto honzon 11 PQ
il q111 ) r . . l,1 , rn1110u qua iio d ,. • .r - -;;-, l mo Q(-~ !Jo). d man ita qu

la.ro+ cl
d = .r.o- ( - ; )
1 =----
1°1
r • horiz ntal o ; r 111nnt o ·, ; nú logo m t ra que (A.3 ) continua v 1(1. d ira.

l. <li li q1w u d (A.19) mo tra q11 - 1- 2= '1~~


21 B1B2 = "Pi ond' B1B d no a

o i. • llH nor d, lip, '. ortanto, 8 81


A1A2
= Jll - r 2 1. Por fim, ,. j, qu lim 1- JI l - r !- O'
lim o Jl1-l 2 1= 1.

.1.4

l. Par it m (a). ob rv qu . 110 i lo trigonom 'tri o mid d füwi P Q <k ur u


Q-- im'tricê m r laç-o o i.·o d, . Pc11, (b) oi m (,) nrnt
m , trarm qu n (e - ; ) = - c e o ( - ; ) = n . P,1r, t, n o Y ritiqu qu
.·tr midad • finais P
Q d . ar P = e AQ = - E. iio tai qu P(-yo .ro) 'Q(.ro 110)
Por fim, ( ) u d (a) d (b).

'> D nvoh-a n (a - b) = n (a (-b)) com o u.·ilio la p, 1t J d mon r, d, <lo it m b) da


propo içã 71. Argum nt 11<l gam nt para g (a - b).

3. pliqu a r laçã fundam ntal d Thgonom tria , o it m (a do orolário 7 __

plique a r laçã fun Iam ntal da Tri on m tria junt, m nt com d finiçõ d
tang nt d um ar o.

Para a fórmula qu falta m (a), na- nb= na n (-b) pliqu fórmula d


rar forma ão d • 11111auma d doi n m produ o· rgum nt , n, lo 111 nt pr r d <luzir a
fórmula par g a - t, /.J. Quaulo a fónnul do it m (b) a co b = n C a)±
11(2 -IJ) , 1111 •g11ida apliqu •; fórmula do it m (a).

6. } •rv i11i ialuwute q11•, . <' OQ • obt icl, d O P p ,1, rota f o d II ulo tri 011om• ri 0 -O
•utão • coord1 11ad; d<' / 110 1 l 'IWI .r' y' uin icl 111 om , nada d Q no • na .r.Oy.
Ag<Jia, 11cJun o ,UI •ulu tligono111(•t1i o nlr o, mi 'i.·o po itivo dru l b m
P :.i·o y0 )(rc o n,,. 1•11 o), 011dl' ,. OP ,i Q = (.r(1 y~) = (r c o - O) r d
f, rm 4m :.i~1 - , <·o o c·o O ,. 1•11 o <'li O = .roco O y 11 < v:,= ,..
0 . 11 0 -

1 n
Yo< fJ o I fJ.)
St·BsEÇ1\0 A.-L-J \1·1 1,1, •.

1. \
1 l rtu• d x2 + p-
ã'l"
u2
= 1. t mo m ê:Hti
I ulm qu
:i
f,,-
• :5 1 ou,
1f, 1 :5
;iill(la, 1. fw,, 11 iu, 'XI t
,

o E [O 2,.) tal qu ! = e a ou, ainda, .r = o os n. Sul si it ui mio .r = 11co. n 11a ·qun.1.;aoda


l. . oJt
•1p l mo. vµ2 _-1 _ -(a co o}2
02 - -
_ -,
n·a, cl modo que y = ±b ·nn. Se!/ - b <•nn, pod mr
tomar0= y=b n(- ). • gu cl (-o)=cosn . n{-n)=-s nnq11•.r.=r1co -r.)
y = b n (- ) pod mo tomar 0 = - . Para m si rannos qu, 1a] ftngulo • único, upouha.
qu nhamo tamb'm 0' E [0.21r) tal qu .r = a s O', y os n 0'. nt ão, co 8 = co 0',
n0 = n8 1
• orno 8.8 E [0.2,.), ab m • qu 1 • unpli a n O - O'. O qu falta • im díato.
1

Você também pode gostar